GS4 VisionIAS Material

You might also like

Download as pdf or txt
Download as pdf or txt
You are on page 1of 350

ETHICS AND HUMAN INTERFACE Student Notes:

Contents
1. Introduction ............................................................................................................................... 2
2. Ethics and Human Interface – The Essence of Ethics ................................................................. 3
2.1. Key Terms: Beliefs, Values, Norms, Principles, Morals, Ethics ............................................ 4
2.2. Determinants of Ethics ........................................................................................................ 8
2.3. Consequences of Ethics..................................................................................................... 10
2.4. Dimensions of Ethics ......................................................................................................... 10
3. Applied Ethics: Specific Examples ............................................................................................ 12
3.1. Environmental Ethics......................................................................................................... 12
3.2. Business Ethics .................................................................................................................. 13
3.3. Ethical Conflicts in Cosmopolitan Culture and Urbanisation ............................................ 15
3.4. Ethical Management ......................................................................................................... 14
3.5. Bioethics ............................................................................................................................ 16
4. Ethics in Private and Public Relationships ................................................................................ 18
4.1. Ethics in Private Relationships........................................................................................... 18
4.2. Ethics in Public Relationships ............................................................................................ 19
4.3. Relation between Public and Private Ethics ...................................................................... 19
5. Resolving Ethical Conflicts: What is the way out? .................................................................... 20
6. Human Values .......................................................................................................................... 20
6.1. Role of Family, Society and Educational Institutions in inculcating values ....................... 21
6.1.1. Role of Family in inculcating values: .......................................................................... 22
6.1.2. Role of Society and Educational Institutions in inculcating values ............................ 23
7. Glossary of terms: .................................................................................................................... 24
8. Previous Years Questions of UPSC............................................................................................ 25
9. Previous Years Case Studies of UPSC........................................................................................ 26
10. Previous Years Questions of Vision IAS Test Series ................................................................ 27
11. Previous Year Case Studies of Vision IAS Test Series .............................................................. 46

Copyright © by Vision IAS


All rights are reserved. No part of this document may be reproduced, stored in a retrieval system or
transmitted in any form or by any means, electronic, mechanical, photocopying, recording or
otherwise, without prior permission of Vision IAS.

DELHI JAIPUR PUNE HYDERABAD AHMEDABAD LUCKNOW CHANDIGARH


1 www.visionias.in # 8468022022 ©Vision IAS
https://t.me/UPSC_PDF Download From > https://upscpdf.com https://t.me/UPSC_PDF

1. Introduction Student Notes:

Ethics plays an immensely important role in the human society. It helps us to identify moral
values, understand our behavior and shape our decisions, both in our personal and public life.
We study ethics as a subject to raise our awareness, resolve moral dilemmas and be a better
person. Following example highlights the relevance of ethics in the decision making-

We tend to hold opinions on a variety of subjects. How is that opinion shaped? How do we
define whether a particular event, decision or a fact, howsoever unconnected to us, is good or
bad, right or wrong?
Many people will not be concerned about how much money a cricketer in India earns. But how
do we co-relate their earnings with other equally demanding, but not as glamorous sports? Or
of a hardworking traffic policeman with a hardworking corporate employee working in an air-
conditioned office in Mumbai? Does the right to bear child also includes the right to terminate
the child’s life? Is saving several lives at the cost of one life justified?
These questions do not have simple yes-no answers. But the framework which we use to
evaluate these questions cannot be arbitrary. It has to be reasoned. What can be the moral
argument for or against such discrepancies?
Further, as a civil servant, not only does one have to take decisions which impact the lives of
others but has to make choices for oneself as well. For example, does accepting tenders from a
firm run by a relative amount to conflict of interest? What should be correct way to approach
such situations? In this chapter we will try to draw a basic framework of reasoning that can be
helpful for us to handle and answer such moral dilemmas.

DELHI JAIPUR PUNE HYDERABAD AHMEDABAD LUCKNOW CHANDIGARH


2 www.visionias.in # 8468022022 ©Vision IAS
Google it:- https://upscpdf.com
https://t.me/UPSC_PDF Download From > https://upscpdf.com https://t.me/UPSC_PDF

What is ‘Ethics’? Student Notes:


The term ‘ethics’ is derived from the Greek word ‘ethikos’ which can mean custom, habit,
character or disposition. At its simplest, ethics is a system of thinking about the moral
principles. It is the investigative study of ‘What is the right thing to do?’ They affect how people
make decisions and lead their lives.
Thus, ethics is a system of moral principles that helps us differentiate between right and wrong,
good and bad, fair and unfair. These can be said to be the guiding light for human conduct. The
change in human behavior and action due to application of ethical principles helps us to build a
humane society where everyone can live in peace and harmony.
Whether or not there are any universal moral principles that can be applied irrespective of
person or circumstance is a debate that philosophers have had throughout history. Every
society has tried to codify the principles of good or acceptable conduct many times in past. Bad
or unacceptable conduct has been discouraged or punished. Religious views on ethics, for
example, tend to be universal. The concept of Categorical Imperative as proposed by Immanuel
Kant gives a test of determining universality of an act.
What is not Ethics?
(a) Ethics is not religion: Many people are not religious, but ethics applies to everyone. Most
religions do advocate high ethical standards but sometimes do not address all the types of
problems we face.
Prof. M. V. Nadkarni explains, it is possible to have ethics without religion, but religion without
ethics is not worth its name. One can be very ethical without being religious, but cannot be
religious without being ethical. In spite of this a combination of ethics with religious can be
useful.
(b) Ethics is not following the law: A good system of law does incorporate many ethical
standards, but law can deviate from what is ethical. Law may have a difficult time designing or
enforcing standards in some important areas, and may be slow to address new problems. For
example: marital rape is not illegal, however it is considered unethical.
(c) Ethics is not following culturally accepted norms: Some cultures are quite ethical, but
others become corrupt or blind to certain ethical concerns. For example, the prevalence of
slavery in the United States before the Civil war or the caste system in India.

2. Ethics and Human Interface


Ethics operates in our lives through human interface. These interfaces are made of human
institutions like family, peer group, school, community, market and state etc. Understanding the
relation between our thinking and these interfaces brings out the essence i.e. underlining core
principles that influence our behavior.

2.1. Essence of Ethics


The essence of ethics lies in the requirement of common ethical principles to ensure our
collective goals of peace, harmony and stability in society. In public life, it can be best reflected
in values of accountability, empathy, honesty, integrity, probity, compassion etc.
The most basic need for ethics lies in the fact that we do not automatically know what is
beneficial for our lives, and what is detrimental. We constantly face complex choices that affect
the length and quality of our lives. We must choose our values, where to live, how to spend our
time, whom to associate with, whom to believe? We must choose what to think about, and
how to go about achieving our goals. Which character traits to acquire, and which to eliminate?
Which of our emotional responses are beneficial, and which, detrimental? By what criteria
should we judge others, and on what basis to interact with them? To the extent that we default

DELHI JAIPUR PUNE HYDERABAD AHMEDABAD LUCKNOW CHANDIGARH


3 www.visionias.in # 8468022022 ©Vision IAS
Google it:- https://upscpdf.com
https://t.me/UPSC_PDF Download From > https://upscpdf.com https://t.me/UPSC_PDF

on deliberation, we are at the mercy of social and emotional factors that may be far from Student Notes:
optimal.
Ethics is about the choices that we make or fail to make. We are aware of our conscious
thoughts and of our ability to make informed, intelligent choices - that is what we call free will.
We are aware that the choices that we make have consequences, both for ourselves and for
others. We are aware of the responsibility that we have for our actions. But, we do not have
reliable inherent knowledge or instincts that will automatically promote our survival and
flourishing. We may have an inherent emotional desire to survive and avoid pain, but we do not
have innate knowledge about how to achieve those objectives. A rational, non-contradictory
ethic can help us make better choices regarding our lives as well as social well-being.
The choices which individuals make may become rigid over a period of time into beliefs and
value system of a society. Legal system of a country greatly depends upon these values and
beliefs. Think of surrogacy laws: does the freedom to become a biological parent gives us the
authority to put monetary value on something as intrinsic as motherhood? The law on
surrogacy in India reflects the Indian value system whereas in other countries, it reflects theirs’.
Ethics or Moral philosophy contemplates what is wrong or right. As a discipline, it has three
branches – Meta-ethics, Normative ethics and Applied ethics.
• Meta-ethics investigates the broader questions, such as ‘how can morality be defined?’,
‘What is justice?’ etc.
• Normative ethics is concerned with what we ought to do. It provides a framework for
deciding what is right or wrong. Various philosophers have tried to give this framework
through reasoning, for e.g. deontological ethics of Kant, utilitarianism of Jeremy Bentham
and John Stuart Mill, virtue ethics of Aristotle.
• Applied Ethics deals with practical issues of moral importance such as capital punishment,
surrogacy, and dilemmas in day-to-day life, etc. (Further refer to section on dimensions of
ethics below)

2.2. Key Terms: Beliefs, Values, Norms, Principles, Morals, Ethics


The terms beliefs, norms, values, morality, ethics and principles are used interchangeably very
often but there are subtle differences between them. One can think of these terms in an
evolutionary framework to appreciate the differences – from an individual’s beliefs to a social
norm to a country’s law. A human being thrives on what he/she believes in. A community goes
by what they think is acceptable. A society acts based on how they are expected to act. These
are called, respectively, beliefs, norms and values.
Beliefs
• Beliefs are basis of our thinking. A coffee-mug lies on my table. I believe that a coffee mug
lies on my table. Ram killed Ravan according to Ramayana. Someone might believe that
Ram did in-fact kill Ravan.
• Beliefs come through our experiences and influence our ideas, viewpoints, knowledge and
attitudes.
• They create fables, myths, folklore, traditions, and superstition. They can also be true and
verifiable facts, history or legends.
• Beliefs lay the foundation of a cultural group, but they are often invisible to the group that
holds them.
• They are important because they give us hope. A human being thrives on what he/she
believes in. However, beliefs are not fixed, they can be changed. Peripheral beliefs can also
be changed.
• Two people might have different beliefs about a phenomenon – as simple as a glass being
half empty or half full, to complex theological questions such as how did earth or life come
to being?

DELHI JAIPUR PUNE HYDERABAD AHMEDABAD LUCKNOW CHANDIGARH


4 www.visionias.in # 8468022022 ©Vision IAS
Google it:- https://upscpdf.com
https://t.me/UPSC_PDF Download From > https://upscpdf.com https://t.me/UPSC_PDF

• Beliefs evoke emotions, but not-necessarily actions. Student Notes:


Values
• Values denote preference – for good or bad. They are important and lasting beliefs or ideas
about what is good or bad and desirable or undesirable.
• Values come through external environment, personal experiences and human interfaces
like family, school etc. They are expressed generally in the terms of ‘should’.
• Our interest in the stories like Ramayana and Mahabharata make us learn certain values
such as elderly respect, duty, honesty, integrity, etc. One can also assign negative values to
acts such as kidnapping and insulting. We make judgment about how desirable or
undesirable a thing is.
• Values thus act as guiding principles to move ahead. It has a major influence on person’s
behavior and attitude.
Norms
• Norms are generally accepted social practices that guides human behavior e.g. it is a norm
to say thank you to helping hands as a mark of expressing gratitude.
• Norms functions as mainly as an informal guidelines in a particular group or community
about right or wrong social conduct. They are a form of collective expectations of
community’s members from each other.
• Norms are a form of social control or social pressure on individual to conform, induce
uniformity and check deviant behavior. They are expressed through social customs,
folkways or mores.
• Norms provide order in a society. For e.g. in a traditional society, it is a norm that a son
must obey his father’s command and fulfill his wishes.
• Non-conformity to social norms can attract punishment. Punishment may be in form of
being looked down upon, derision, scolding, boycott, imposing penance, etc. Laws are a
later stage of evolution of norms, where the society has codified the terms of expected and
unexpected behavior from its members. Those who are deviant are tried in a court of law
and punished accordingly.
Norms, Values and Belief
• It is important to note that for an individual, norms are imposed externally whereas
beliefs and values are internal.
• Norms are a specific guide to behavior whereas values provide indirect guidance only.

Principles
Values, beliefs, morality vary from individual to individual. Ethics may also differ in different
communities and culture. However, Principles are moral rules that are considered as universal
in nature e.g. honesty is the best policy can be an individual principle to follow as it is
universally believed. Principles are about universal truths and standards such as fairness,
truthfulness, equality, justice etc.
Morals
• Morals are the desired values and beliefs about what is right or wrong, acceptable or
unacceptable in a particular situation.
• While they do prescribe what the right conduct is, morality is ultimately a personal
compass, a personal choice. Think of religion, say Jainism, which has its five principles
(Truth (Satya), Non-Violence (Ahimsa), Non-stealing (Asteya), Celibacy (Brhamacharya) and
Non-attachment (aparigriha)) that act as moral guides for people following Jainism.
However, it ultimately is the individual choice of which morals do they actually follow or use
to judge rightness or wrongness of an action. Those who are indifferent to right or wrong
are called amoral, while those who do evil acts are called immoral.

DELHI JAIPUR PUNE HYDERABAD AHMEDABAD LUCKNOW CHANDIGARH


5 www.visionias.in # 8468022022 ©Vision IAS
Google it:- https://upscpdf.com
https://t.me/UPSC_PDF Download From > https://upscpdf.com https://t.me/UPSC_PDF

• Morals may change over time. Historically, religion has been an important source of Student Notes:
morals. With new knowledge, a person’s morals may change. For example, homosexuality -
an act still considered unnatural and hence, immoral, is gaining more acceptance
throughout the world. Some morals transcend time and cultures. For example, selfishness
is considered as immoral, whereas loyalty and telling the truth are considered as moral.
Ethics
• Knowledge of ethics provides the logical framework for understanding and interpreting
right and wrong in a group or a society.
• Ethics (also understood as Moral Philosophy) is concerned with questions of how people
ought to act, and the search for a definition of right conduct and the good life.
• Here, ethics involves systematizing, defending, and recommending concepts of right and
wrong behavior. There may be different approaches to arriving at the right conduct i.e.
ethical behavior. However, all these approaches are reasoned frameworks. Major
approaches are virtue based, duty based and utility based.
• In a practical sense, ethics refers to rules concerning upright behavior. Many organizations
have rules concerning the standards of behavior expected of their employees. For example,
a hospital may require that doctors prescribe the treatment which balances the
requirement, effectiveness and cost to the patient. There is no legal requirement, but it
ought to be the way in which a doctor acts. Medical profession requires that doctors must
respect the autonomy of the patients (autonomy as in right to choose the type of treatment
depending on means available, confidentiality of personal information, etc.). A financial
company may require that investment details of its clients should not be revealed by its
employees. The so called ‘codes of ethics’ are there because a person’s own morals may be
silent on these aspects. These codes thus act as guides for right conduct.
• Ethics explains the guiding principles to decide what is right or wrong. They are the
standards which influence the decisions made by a person.
• Note the word ‘guiding principles’ - ethics are externally imposed. Since they come from
external source, they tend to be consistent and objective. They require the person to act in
specific ways. Digression from these standards is considered unethical and may be
informally disapproved or censured.
• Truthfulness, honesty, integrity, respect, fairness are examples of ethical principles.
Ethics and Morality
Ethics and morality seem to carry the same meaning and they are generally used
interchangeably. However, some philosophers make a distinction between the two.
Ethics differs from morality in that it denotes the theory of right action and the greater good,
while morality indicates their practice at the individual level. Ethics is the science of morals.
Ethics and morality is practiced through morals.
Elaboration: Consider science. Science is not there in nature in a systematized form. It has to be
derived based on observations and practice. Once scientific study has been done and results
arrived at, they are subjected to tests in the real world. Once it gets established, it is prescribed
as the correct way to go about doing things, such as construction of a multistory building based
on principles of physics.
• Ethics is where one studies about moral principles. Morals are the practice of this
knowledge.
• Ethics thus are rules which govern the conduct of all the members of a group (such as
doctors, lawyers, policemen, cultural group or society). Morals are those standards which
the individual sets up for himself/herself.
• Both ethics and morals may be embodied in conduct rules, which may have the force of
law. Otherwise, morality means individual propositions and is subjective by nature as it
varies from person to person while ethics are collective propositions of society and thus are
more objective compared to morality.

DELHI JAIPUR PUNE HYDERABAD AHMEDABAD LUCKNOW CHANDIGARH


6 www.visionias.in # 8468022022 ©Vision IAS
Google it:- https://upscpdf.com
https://t.me/UPSC_PDF Download From > https://upscpdf.com https://t.me/UPSC_PDF

• Morals are principles which help guide a person about rightness or wrongness of an act. Student Notes:
Ethics are principles of right conduct –i.e. what should be the right way in which a person
should act in a situation.
The difference clearly comes out in the classic case of a defense lawyer. A lawyer may
consider murder to be a reprehensible act, deserving severe punishment. But a defense
attorney has to be unbiased and her professional ethics require her to defend the client to
the best of her abilities, even if she knows that the client is guilty. Everyone, even a
terrorist, deserves a fair trial in a court of law, not a kangaroo court or mob justice.
• An individual’s morals can be informed by the broader rules of ethics. Believing that theft is
wrong may stem from the ethical principle of respect of private property of others.
Take example of a civil servant being involved in two different illegal acts. Taking bribe for
doing his/her duty is a crime under Prevention of Corruption Act. It is also a violation civil
service ethics. On the other hand, being involved in a relationship outside marriage is a
moral lapse. Similarly, consider a police officer who indulges in consumption of drugs.
Society considers such person to be of weak morals. Further, it is unethical to be inebriated
if one is responsible for maintaining law and order. Moreover, suppressing information from
the department about his involvement in such activities will also be a breach of
departmental ethics. Finally, it is also illegal to consume banned substances under the law.
Ethics and Laws
Ethics is a reasoned framework of moral principles, which is concerned with what is good for
individuals and society. However, law is structured system of rules and regulations enforced
through certain authority to govern individual’s behavior, actions and social relations.
Relationship between law and ethics
Similarities:
• Both laws and ethics are systems which maintain a set of moral values and prevent people
from violating them. They both provide guidelines to people of what they may or may not
do in certain situations. Most laws represent the minimum standards of ethical human
behavior.
• It is when certain ethical principles become widely
accepted, they are codified into laws. For example,
‘motherhood’ in our society is considered to be sacrosanct
and does not trade for monetary value. Therefore, the law
regulating surrogacy in India allows only altruistic
surrogacy.
Dissimilarities:
• Foremost, law provides for punishment for violation- a legally enforceable action by the
competent authority. Ethics on the other hand may invite social sanction, but no legally
enforceable punishment in a court of law.
• There may be many areas where law does not exist or is silent. But ethics and morals have
a wider scope. For example, the law will not bind a businessman to tell his competitor
about their new client that has a reputation of not paying dues, but ethics will still guide
judgment here.
• Ethics may vary from people to people because different people may have different
opinions on a certain issue, but laws describe clearly what is illegal no matter what
individual opinions people have of them. To some extent, ethics is not well defined, but
laws are defined and precise.
• Ethics as a code of conduct may evolve as a society matures, but laws need to be changed
through specific action of the legislature.

DELHI JAIPUR PUNE HYDERABAD AHMEDABAD LUCKNOW CHANDIGARH


7 www.visionias.in # 8468022022 ©Vision IAS
Google it:- https://upscpdf.com
https://t.me/UPSC_PDF Download From > https://upscpdf.com https://t.me/UPSC_PDF

The relationship between law and ethics is dynamic and a function of its time. Ethics guide Student Notes:
laws. Changing ethical and moral standards lead to amendment or invalidation of existing laws.
For example, The Right to Information was enacted to codify the ethics of transparency in
government functioning. Similarly, earlier homosexuality was widely perceived as unnatural and
immoral. But with the social progression, Section 377 has been declared unconstitutional in so
far as it criminalized “consensual sexual conduct between adults of the same sex”. Abrogation
of Section 66A of the IT Act, 2000 which curtailed free speech was a classic case where
progressive ethics of people helped reading down of a regressive law. As is clear from above
analysis, laws and ethics can be in conflict with each other or may reinforce each other.

2.3. Determinants of Ethics


Our ethics is largely shaped by influences that determines how we live our lives in a society.
These influences are called determinants. They inform the way in which an individual acts or
takes decision and tells us what society expects from its members in a behavior. These
determinants can be largely categorized as pychological, personal, religio-cultural and socio-
political.

1.Pychological Determinants
• Human Conscience and Intuition – This school believes that what is good is good because
it is good. Its goodness or badness does not need any justification. Accordingly, what is
good or not can be detected by the intuitive moral sense. Basic moral truths are self-
evident to a person who directs his/her mind towards moral issues. Please note that moral
truths according to an intuitionist are not derived from rational arguments or from having
a hunch or a feeling. They are simply realized by pondering over moral issues. It is like an
‘aaha!!’ moment of realization.
Conscience and Intuition
Human beings behave according the knowledge they have and experiences they go through. Our
conscious mind is what actively evaluates a situation and then takes decisions. However, our
subconscious mind is a repository of all our past experiences and decisions. Although we do not
actively summon our subconscious mind, there are, metaphorically, lightning fast calculations
that tell us what to do or what not to do.
• Intuition is the ability to understand something instinctively, without the need for conscious
reasoning. Our intuition, although seemingly totally random (as it is instinctive), is not
always so. For example: One sees a cricket match and sees that Sachin gets out when he

DELHI JAIPUR PUNE HYDERABAD AHMEDABAD LUCKNOW CHANDIGARH


8 www.visionias.in # 8468022022 ©Vision IAS
Google it:- https://upscpdf.com
https://t.me/UPSC_PDF Download From > https://upscpdf.com https://t.me/UPSC_PDF

smiles. Next time, if the person sees Sachin smiling while batting, he would probably think Student Notes:
that he will get out. This is intuition. Although it seems totally random, it is based on our
past experiences, observations or simply information given by others. However, intuition has
no moral choice involved. It is merely an expectation of an outcome.
• Conscience is a person's moral sense of right and wrong, viewed as acting as a guide to one's
behaviour. It also flows from our repository of experiences and knowledge. It is often
referred to as the inner voice of reason. This inner voice may or may not involve elaborate
reasoning, but it does concern with judging a situation as good or bad i.e. a moral choice is
involved. Conscience can also be suppressed willingly. A person may choose to do something
which his/her conscience opposes because there is some higher authority or a threat
prohibiting the desired action. This leads to crisis of conscience.
2.Personal determinants
• Leadership – The leadership of a society or an organization or nation also helps to
determine the conduct of their followers or admirers is ethical. For example – Indian
democratic, liberal, secular, tolerant tradition has been the gift of our forefathers and
makers of modern Indian society. However, leadership is also prone to unethical practices.
It depends on the virtue of the leaders themselves. Thus, being ethical cannot always be
equated to following the leader.
• Philosophies – Various philosophers and thinkers subscribe to different sets of ethics. For
example – for a consequentialist, ethics in action is determined by its outcome while for a
deontologist, it depends on the nature of the act itself. Thus, ethical values of an individual
will depend upon the philosophy to which it was closer to in formative years.
3.Religio-cultural determinants
• Culture – The ethical principles of an individual are also influenced by the culture and the
country in which she/he is based. For example - western culture seems to be individualistic
while Indian culture is based on values of universalism and multiplicity. Even literature,
scriptures etc. guide person’s conduct. For example – Vedic literature, Buddhist literature
etc.
• God and Religion - It advocates universal and ideal ethical standards. Religious textbooks
deal with questions of how an individual should behave and how the society should be. For
e.g. Christian Morality is one of the important sources of ideal individual behavior in west.

The conception of ‘Ram Rajya’ in India is more of an ethical society rather than a
governance model. Religion based ethics trace the source to God - something is good
because God says so, the way to live a good life is to do what the God wants. However, one
should not identify ethics with religion. Because then ethics would apply only to religious
people. But ethics applies to the behavior of the atheist as well. Also, the source of religion
can scientifically be only traced back to humans. As such, it is dependent upon context,
situation and experience and is subject to errors and revision.
4.Socio-political determinants
• Society - In any society, most people accept standards that are, in fact, ethical. However,
one should not equate being ethical to "whatever society accepts" as society can deviate
from what is ethical. An entire society or its large influential section can become ethically
corrupt. For example, Nazi Germany or Stalinist Russia imposed on their citizens particular
standards of living and tried to indoctrinate them. Caste system in India has continued
through millennia because of approval of influential members of the society. Further, there
are many issues where there is no social consensus. Thus, ethics cannot be equated with
whatever society accepts.
• Family, teachers and life experiences – Parents, family and teachers shape the value
system of a child. Their behavior, conduct and teachings influence the direction in which

DELHI JAIPUR PUNE HYDERABAD AHMEDABAD LUCKNOW CHANDIGARH


9 www.visionias.in # 8468022022 ©Vision IAS
Google it:- https://upscpdf.com
https://t.me/UPSC_PDF Download From > https://upscpdf.com https://t.me/UPSC_PDF

the child steers. In fact, affiliations to same religion, customs and tradition are seen in Student Notes:
same family. However, some also break free from the beliefs, notions and ideas of their
parents and choose their own set of principles according to own free will. For example – a
boy may consider girl child same as that of male child even if he and his sister was treated
unequally by their parents.
• Law - The law often incorporates ethical standards to which most citizens subscribe.
However, one should not equate being ethical to following the law. Laws, at times, can
deviate from what is ethical. For example – slavery laws in US did not make slavery ethical.
On the other hand, laws which banned slavery and gave equal rights reflect convergence of
laws and ethics. Relationship between law and ethics has been detailed above.
• Constitution – Constitution of various countries also is a way to establish moral disposition
of their society. For example – equality, accountability, democratic values are reflected in
some country’s constitution and autocratic, high-handedness, and inferiority, non-
participative values are reflected in the framework of other countries.
Because, law, society, religion can deviate from what is ethical, one needs to constantly examine
one's moral standards to ensure that they are reasonable and well-founded. Being ethical
requires continuous effort in studying our own moral beliefs and our moral conduct and striving
to ensure that we follow it in our routines. Thus, individual by oneself is also one of the most
important determinants of ethics – everyone has emotional convictions or deep intuitions
about the immorality of, say, murdering an innocent victim, about aborting a female fetus, or
regarding child abuse.
Skill in addressing ethical issues can be learned and cultivated if we recognize the importance
of doing so. The requirement is that we view our treatment of ethical problems as an ongoing
process of designing the best courses of action for specific situations we face. This demands
that we are able to establish a framework for understanding ethics in dynamic rather than static
terms.

2.4. Consequences of Ethics


Consequences of ethics mean the outcome of human actions which are guided by ethical
thinking. This means consequences also determine how good or bad an act is.
• It can be measured by happiness, suffering, pain, pleasure born out of human action.
• It motivates or prohibits one to do or not to do certain things. For example – If hard work
guided by ethical values results in desired success then action itself gets justification to be
good and effective but if action does not produce outcome then it will be treated as not
efficient.
• Humans in general tend to maximize pleasure from their actions. However, some people
prefer short term pleasure without consideration of long-term consequences while some
may prefer temporary pain to ensure long term pleasure in future. For example – one may
satisfy its sweet tooth or give in to their cravings while ignoring the health benefits which
may cause pain in future. Others may exercise daily and practice self-control to ensure
disease-free life in future.
• However, predicting consequences of human actions is difficult. There is whole theory of
ethics that is based on nature of consequences. It is called Consequentialism. However,
please note that ‘Consequentialism’ is just one part of ‘Consequences of Ethics’.
Consequentialism is an ethical theory which says that an act should be considered ethical if
it is able to produce good consequences.

2.5. Dimensions of Ethics


Different ethical theories that inform the ethical thinking create various dimensions of the
subject. Ethical theories are philosophical explanation to understand, explain and perhaps guide
us in practice of making moral judgments. An ethical theory is not merely description of moral

DELHI JAIPUR PUNE HYDERABAD AHMEDABAD LUCKNOW CHANDIGARH


10 www.visionias.in # 8468022022 ©Vision IAS
Google it:- https://upscpdf.com
https://t.me/UPSC_PDF Download From > https://upscpdf.com https://t.me/UPSC_PDF

beliefs of individuals, societies or cultures. A person pondering over ethics tries to answer the Student Notes:
following questions:
A. The very nature and status of morality: Is morality what god/religion says? Or is it the set
of rules which human beings have designed for themselves to further their mutual self-
interests? So what exactly is ethics, its definition and its scope?
B. The nature and meaning of moral judgments: Are moral judgments merely emotional in
nature i.e. do they express our emotions and desires rather than being rational and
reasoned? Can moral judgments be classified as true or false universally or are they
dependent on context? E.g. ‘Honesty is the best policy’- is honesty only a desirable trait in a
person or is this statement a universal truth? If true, is there a context which is associated
with it?
C. The fundamental rules, principles and values justifying moral judgments: Are
consequences the only thing that matter in a decision or are there certain other principles
which must be followed? Is human happiness more important than adhering to duty? Or is
being virtuous more important than both? Any of these principles may underlie a moral
judgment.
D. Practical application of ethics: Is stealing for the sake of feeding children justified? Should
advancement in biology continue unhindered or should there be certain control over
genetically modified babies? Should admissions in universities be based solely on merit or
should there be affirmative action for people with relatively backward socio-economic
background? What should be the parameters to decide merit?
The questions A-D are the different dimensions along which ethics is studied. These are
categorized more specifically as Meta-ethics, Normative-ethics and applied ethics.
1. Meta-ethics deals with the nature of moral judgments (questions in A&B). It looks at the
origins and meaning of ethical principles. For example, when we talk of bioethics, meta-
ethics will not answer the questions of right or wrong. Rather it tries to define the essential
meaning and nature of problem being discussed. So it will examine ‘what does it mean to
ask whether genetic research is morally permissible?’ ‘What are the sources which we
should consider in determining the ethical nature of genetic engineering? – is it god or the
human made laws?’
2. Normative ethics (questions in C) is concerned with the content of moral judgments and
the criteria for what is right or wrong. It involves arriving at moral standards that regulate
right and wrong conduct. In a sense, it is a search for an ideal litmus test of proper behavior.
It is the branch of ethics concerned with establishing how things should or ought to be, how
to value them, which things are good or bad, and which actions are right or wrong. It
attempts to develop a set of rules governing human conduct, or a set of norms for action.
Under normative ethics we study different approaches to ethical questions. A given
situation has not one ideal solution, rather different values to different principles will lead
us to different decisions.

Some of the important approaches of Normative ethics are:


• Consequentialist approach – According to this approach, ethics of an action is
determined by its consequences, result or outcomes. The more good consequences
an act produces, the better or righter that act was. Utilitarianism is one form. It states
that people should maximize utility. Utility can be measured in terms of human
welfare or well-being. In hedonism, it is said that people should maximize their
pleasure. Thus, according to these, actions which maximize pleasure or welfare and
minimize pain or suffering are ethical e.g. higher economic growth justifies the weak
environmental and labor laws, glowing face justifies the use of harmful chemicals in
beauty creams, and Growing revenue justifies the use of more commercials in TV
programs.

DELHI JAIPUR PUNE HYDERABAD AHMEDABAD LUCKNOW CHANDIGARH


11 www.visionias.in # 8468022022 ©Vision IAS
Google it:- https://upscpdf.com
https://t.me/UPSC_PDF Download From > https://upscpdf.com https://t.me/UPSC_PDF

The greatest happiness of the greatest number is the foundation of morals and Student Notes:
legislation.- “Jeremy Bentham

• Deontological approach – According to this approach, ethics is based on what people


do and not what was the outcome of that action. It is non-consequentialist as we
can’t show good outcomes to justify our actions. According to this, we need to do the
right thing because it is right and avoid the wrong thing because it is wrong. Certain
acts by themselves are unethical such as it is wrong to kill people, steal, tell lies etc.
Natural Rights Theory (such as that espoused by Thomas Hobbes and John Locke),
holds that humans have absolute, natural rights. These are in the sense of universal
rights that are inherent in the nature of ethics and not contingent on human actions
or beliefs. This eventually developed into what we today call human rights.
Immanuel Kant's Categorical Imperative roots morality in humanity's rational
capacity and asserts certain inviolable moral laws. Kant's formulation is deontological
in that he argues that to act in the morally right way, people must act according to
duty, and that it is the motives of the person who carries out the action that make
them right or wrong, not the consequences of the actions. Simply stated, the
Categorical Imperative states that one should only act in such a way that one could
want the maxim (or the principle) of one's action to become a universal law, and that
one should always treat humanity as an end rather than means.

“An action, to have moral worth, must be done from duty”- Immanuel Kant

• Virtue ethics – It is person rather than action based. It focuses on the inherent
character of a person rather than on the nature or consequences of specific actions
performed. According to this approach, a virtuous person always does the right thing.
He will do the same thing always in similar circumstances. Virtue ethics not only deals
with the rightness or wrongness of individual actions, it provides guidance as to the
sort of characteristics and behaviors a good person will seek to achieve. Thus,
according to it, to build a good society, people should be helped to be virtuous
people. Although list of virtues changes over time, yet certain virtues which more or
less remains in the list are – justice, fortitude, temperance, self-care, fidelity.
“An unexamined life is not worth living.”- Socrates
• Contextualist approach – It rejects prescriptive rules. According to this approach,
right and wrong depend upon the situation as there are no universal moral rules or
rights. Thus, each case is unique and deserves a unique solution e.g. At some places,
lockdown during pandemic is justified due to worsening situation, but in other places
may not be justified. U.S uses protectionism in global trade citing its situation.
• Super-naturalist approach – where ethics is attributed to God, that is, the only source
of moral rules is God – whatever the God says is ethical. Thus, being ethical in your life
means following what God says e.g. god made animals inferior to humans so animals
can be used for human purposes as commodities.
3. Applied ethics (questions in D) – it attempts to apply ethical theory to real life situations such
as it looks at controversial topics like war, animal rights and capital punishment. It has many
specialized fields, such as engineering ethics, bioethics, geoethics, public service ethics and
business ethics. It is used in determining public policy.

3. Applied Ethics: Specific Examples


3.1. Environmental Ethics
Environmental ethics is the branch of ethics that studies the relation of human actions and
natural environment. Environmental ethics treat environment as a part of society. It is about

DELHI JAIPUR PUNE HYDERABAD AHMEDABAD LUCKNOW CHANDIGARH


12 www.visionias.in # 8468022022 ©Vision IAS
Google it:- https://upscpdf.com
https://t.me/UPSC_PDF Download From > https://upscpdf.com https://t.me/UPSC_PDF

environmental values and societal attitudes related to protecting and sustaining biodiversity Student Notes:
and ecological systems.
Increasing pollution, depletion of natural resources, dwindling plant and animal biodiversity,
the loss of wilderness, the degradation of ecosystems, and climate change are all part of a raft
of “green” issues that have implanted themselves into both public consciousness and public
policy in recent years.
The job of environmental ethics is to outline our moral obligations in the face of such concerns.
The two fundamental questions that environmental ethics must address are: what duties do
humans have with respect to the environment, and why?
The latter question usually needs to be considered prior to the former. In order to tackle just
what our obligations are, it is usually thought necessary to consider first why we have them. For
example, do we have environmental obligations for the sake of human beings living in the world
today or for our future generations, or for the sake of entities within the environment itself,
irrespective of any human benefits in terms of ecosystem services? Different ethical approaches
give quite different answers to this fundamental question and has led to the emergence of
quite different environmental ethics.
Ecological values are part of Indian tradition where nature was revered for its services to
mankind. Various environmentalists such as Baba Amte have also spread awareness about
ecological balance and wildlife preservation. They believed that humans have to live in harmony
with nature, and not by exploiting nature and motivated people to adopt a model of
sustainable development that would be beneficial for both mankind and nature.

“Earth provides enough to satisfy every man's needs, but not every man's greed.”
Mahatma Gandhi

3.2. Business Ethics


Business ethics represents a set of ethical principles that are applied in a business environment.
It is applicable to all activities and individuals of an organization. Many firms develop detailed
codes of conduct to guide the actions of people employed in their organization. Thus, business
ethics can be termed as study of content and effectiveness of these codes of conduct.
Business entities should have the moral responsibility for what they do. For example –
responsibility of treating their employees well, responsibility of respecting the environment
from which they draw resources, responsibility for the impact of its product on consumers etc.

DELHI JAIPUR PUNE HYDERABAD AHMEDABAD LUCKNOW CHANDIGARH


13 www.visionias.in # 8468022022 ©Vision IAS
Google it:- https://upscpdf.com
https://t.me/UPSC_PDF Download From > https://upscpdf.com https://t.me/UPSC_PDF

Corporate reputation is dependent on the level of morality adopted in corporate operations. Student Notes:
Some examples of ethical practices in business can be:
• Not being tempted to cheat, deceive or manipulate others
• Following laws and regulations that structure market and organizations in letter as well
spirit.
However, laws cannot cover all the business practices. Thus, law leaves gaps which can be
exploited by businesses. This is where business ethics comes in. Businesses should not exploit
market when market is failing due to externalities or imperfect information.
Business ethics with respect to consumers
Firms, in their engagement with consumers, should follow certain ethical practices such as
• Maintaining standards in the production of goods and services such as healthcare, pharma
production etc. to ensure safety of the users consuming it
• Giving true picture of the product to consumers in advertisements
• Not undertaking sale of inappropriate products such as illegally obtained organs, drugs,
sexual services etc. which demean intrinsic value of humanity by treating it merely as a
means to generate profits.
Business ethics with respect to employees
Firms should take care of following issues while dealing with their employees
• Non-discrimination – The employees should be treated strictly according to the basis of the
merit they possess with respect to the job
• Pay commensurate to their efforts – Employee should be paid according to the
contribution he/she made towards success of the organization.
Note- Other aspects of business ethics will be covered in detail as part of the document “Corporate
Governance”.

3.3. Ethical Management


Ethical management is the incorporation of ethics in management i.e. one should refrain from
bad practices in management. Managerial ethics is the set of standards that dictate the
conduct of manager operating within a workplace.
• There are no legal rules or laws formulated for this purpose.
• Instead, the ethics code is assembled by the company to guide its managers. It typically
references shared values, principles and company policies about basic conduct and
outlines the duties of the manager towards employees, the company and its stakeholders.
• Although not enforceable, managers who consistently ignore certain company’s ethics may
be asked to step down. Examples include having a code of conduct to use company’s
equipment such as telephone or cab service for personal use, giving fair opportunity to
subordinates to express their grievances, or prior declaration of any conflict of interest
situation such as accepting gifts from vendors, etc.
Kindly note: Business ethics and Ethical Management (Managerial Ethics) are slightly different.
Business ethics affects the parties that are influenced by actions of the business. It is a standard
for enterprise decision making and action. Ethical management concerns more with standards
of personal behavior for managers to deal with employees and other stakeholders.
Also note: Management of ethics is different. To manage ethics means to create a set of
principles or code for all to comply with ethical behavior. It is how one deals with conflicts of
interests and dilemmas to make an ethically sound decision, how one manages to direct
his/her’s actions and satisfy his/her’s conscience to move towards ethical path.

DELHI JAIPUR PUNE HYDERABAD AHMEDABAD LUCKNOW CHANDIGARH


14 www.visionias.in # 8468022022 ©Vision IAS
Google it:- https://upscpdf.com
https://t.me/UPSC_PDF Download From > https://upscpdf.com https://t.me/UPSC_PDF

3.4. Ethical Conflicts in Cosmopolitan Culture and Urbanisation Student Notes:

Economic development and desire for wellbeing has made the phenomenon of rapid
urbanization and culture of cosmopolitanism a de-facto feature of the modern society. This
leads to enhanced job opportunities, rise in standards of living of people, economic progress of
a nation, rise in awareness, assertive demands regarding provision of services by government
etc.
Cosmopolitanism is the ideology that all human beings belong to a single community with
similar set of moral principles. Theoretically, here everyone can retain their individual and
cultural integrity while also keeping themselves open to the larger world. Cosmopolitanisation
and urbanization are associated with the following values:
• Inclusivity, oneness, human rights and dignity, cultural diversity, solidarity, equality
• Openness, good governance, responsibility and accountability, democracy
• Globalization, modernism, industrialization, consumerism, liberalization
• Opposition to chauvinism, social justice, peace
• Collective Intelligence through information

However, they also give rise to various ethical conflicts such as:
• Environmental sustainability versus use of resources for development projects – Large
projects such as mining, building dams, power projects etc. are required to meet rising
demands of people and improve standard of living. This is in direct conflict with the
question of sustainability i.e., optimum utilisation of resources by present generation so
that sufficient resources are available for the generations to come.
• Rising inequality and the condition of poor and vulnerable – The fruits of economic
growth have been appropriated by only a few and thus, it has led to immense inequality.
The conditions of people living in urban slums and squatters versus the gated communities
in metros reflect the divide that urbanization has created and is perpetuating.
• Conflict of resources – Although, people are migrating to urban areas, following a
cosmopolitan culture, yet the vast population growth in urban areas is leading to resource

DELHI JAIPUR PUNE HYDERABAD AHMEDABAD LUCKNOW CHANDIGARH


15 www.visionias.in # 8468022022 ©Vision IAS
Google it:- https://upscpdf.com
https://t.me/UPSC_PDF Download From > https://upscpdf.com https://t.me/UPSC_PDF

crunch in terms of land, water, infrastructure etc. Thus, many people living in small spaces, Student Notes:
with low water accessibility and congestion on roads etc.
• Salad bowl vs Melting pot model of society – Due to cosmopolitanisation, many cultures
have lost their identity and are increasingly being subsumed and consumed by dominant
cultures of the world.
• Global vs Regional Values – In opposition to melting pot model, regional and local cultures
are asserting themselves strongly as a reaction to globalization.
• Cultural diversity vs Cultural clashes – the opportunities in these areas brings people from
various backgrounds and culture here and enables them to get an exposure to them. But
this diversity at times leads to clashes when some people are not open to new culture.
• Individualisation vs Collectivism - increasing individualisation in urban areas results in new
social structures where the traditional family is not as dominant as before. Self-interest
seems to have taken over other values.
• Lifestyle Changes vs Healthy routine – People have begun to associate themselves more
with the Mcdonaldistion than their traditional diverse cuisines and habits.

3.5. Bioethics
The modern field of bioethics emerged in the 1950's and 1960's and represents a radical
transformation of the much older and traditional domain of medical ethics. The term
"bioethics" refers to the broad terrain of the moral problems of the life sciences, ordinarily
taken to encompass medicine, biology, and some important aspects of the environmental,
population and social sciences. The traditional domain of medical ethics would be included
within this array, accompanied now by many other topics and problems.
Four general areas of inquiry can be identified:
• Theoretical bioethics which deals with the intellectual foundations of the field.
• Clinical ethics which refers to the day-to-day moral decisions confronted in caring for
patients.
• Regulatory and policy bioethics which seeks legal and policy solutions for moral problems
concerning life and death. Examples of issues falling into this area would be use of foetal
tissue in research, defining death, guidelines for do-not-resuscitate (DNR) orders in
hospitals, euthanasia, cloning, rationing health care resources, and so on.
• Cultural bioethics which considers ethical questions in relation to the historical, ideological,
cultural, and social contexts in which they are expressed.
Bioethics Principles: Following are the four principles which form the framework for moral
reasoning.

DELHI JAIPUR PUNE HYDERABAD AHMEDABAD LUCKNOW CHANDIGARH


16 www.visionias.in # 8468022022 ©Vision IAS
Google it:- https://upscpdf.com
https://t.me/UPSC_PDF Download From > https://upscpdf.com https://t.me/UPSC_PDF

Student Notes:

The moral questions of bioethics can also be viewed within broader theories of ethics-
• A utilitarian approach asks which consequences of a choice or action or a policy would
promote the best outcome. In this view, the broader good might be deemed the greatest
good. The utilitarian view would, in the context of health care rationing, for example, look
for the collective social benefit rather than advantages to individuals.
• A deontological perspective, on the other hand, would argue that "good consequences may
have to be set aside to respect inalienable human rights". An example would be subjecting
individuals to medical research that may do harm to that individual, while providing the
potential to help others.
• Other moral theories, such as that of Aristotle, stress neither principles nor consequences
but see a combination of virtuous character and seasoned practical reason as the most
likely source of good moral judgement.
• The approach of casuistry is to carefully examine individual cases in the solving of practical
moral problems, and to let principles emerge from these over time.
Today, in the medical and health fields health practitioners are frequently called on to make
moral decisions as well as medical decisions. Indeed, it may be held that a good medical
decision should be tantamount to a good moral decision. It is in this context that the field of
bioethics has emerged as a vitally important field.
“What distinguishes ethics from science is not any special kind of knowledge but merely desire. The
knowledge required in ethics is exactly like the knowledge elsewhere; what is peculiar is that certain
ends are desired, and that right conduct is what conduces to them." -Russell, Bertrand.

DELHI JAIPUR PUNE HYDERABAD AHMEDABAD LUCKNOW CHANDIGARH


17 www.visionias.in # 8468022022 ©Vision IAS
Google it:- https://upscpdf.com
https://t.me/UPSC_PDF Download From > https://upscpdf.com https://t.me/UPSC_PDF

Student Notes:

4. Ethics in Private and Public Relationships


4.1. Ethics in Private Relationships
It refers to the ethics that a person identifies with in respect to people and situations that they
deal with in everyday life. It largely involves relations with family and friends. They are based on
emotional bonds rather than any formal procedure that regulates them and therefore, they are
informal in nature. Private relationships are often given or inherited. They are relatively
permanent with more tolerance for imperfections.
Ramayana is a classic example of ethics in personal relationships. Although ethics in private life
varies from person to person but there are some common underlying principles that are
accepted by the society. For example -
• Loyalty – loyalty to one’s partner and to the family members
• Love – loving all the members of family with their imperfections
• Affection – caring for needs of all the members as your own
Ethics in private relationship are generally directed by individual virtues, universal human
values, religion, social norms and law of land. Ethics in private relationship are also checked by

DELHI JAIPUR PUNE HYDERABAD AHMEDABAD LUCKNOW CHANDIGARH


18 www.visionias.in # 8468022022 ©Vision IAS
Google it:- https://upscpdf.com
https://t.me/UPSC_PDF Download From > https://upscpdf.com https://t.me/UPSC_PDF

private religious law. Individual familial and community obligations have long been written into Student Notes:
law and supported by serious sanctions from ancient time to today’s inheritance divorce,
marriage and other laws. In India, along with moral codes, religious institutions and
constitutional provisions govern ethical issues in private relationship

4.2. Ethics in Public Relationships


It refers to the ethics that a person may adhere to in respect of their interactions and business
dealings in their professional life. Public relationships are those that exist by the virtue of
profession or the position one holds in professional life.

The Committee for Standards in Public Life (Nolan Committee) defined seven principles. These
are: selflessness, integrity, objectivity, accountability, openness, honesty and leadership.
Similarly, the OECD countries have also come out with a set of core values to guide public
servants. These are impartiality, legality, integrity, transparency, efficiency, equality,
responsibility and justice.
Note: More details on this part will be covered under the document- Ethics in Administration.

4.3. Relation between Public and Private Ethics


The distinction between private and public ethics is a dubious one because both in public and
private lives, we have to live by same ethical values in general. There cannot be any
dichotomous relationship between the two. Since ethics promotes a virtuous life, it is equally
applicable to both.
A person who is unethical in his public life can hardly be expected to be ethical in his private
life and vice versa. The way a civil servant treats women in his family is reflected in how he
handles female co-workers or whether policies implemented by him have a gender bias or not.
If we take examples of great personalities, we will find that they had same yardsticks to
measure their public as well as personal lives, for example Gandhiji, former President Shri APJ
Abdul Kalam.
Sometimes public/professional relations may become personal relations. For example – a
teacher-student relationship - sometimes in class, teachers uses personal examples with
children, children also shares their weaknesses with the teachers so that he/she can help in
improvement. So, in this case, at some level, the relationship has become personal with the
kind of bond the two have started sharing.

DELHI JAIPUR PUNE HYDERABAD AHMEDABAD LUCKNOW CHANDIGARH


19 www.visionias.in # 8468022022 ©Vision IAS
Google it:- https://upscpdf.com
https://t.me/UPSC_PDF Download From > https://upscpdf.com https://t.me/UPSC_PDF

It is very important that there is no conflict between personal and professional ethics as it may Student Notes:
lead to frustration, guilt or confusion & dissonance in the mind of some persons. Both of them
shape and reinforce each other. For personal development, they need to be congruent to each
other. But too much of congruence may also lead to stagnation of ideas and changes – For
example – if no person in public service is reform oriented and is congruent with the present
society, then socio-cultural changes will be difficult to bring.
Ethics in public life places a greater responsibility and is more demanding as in public life you
cannot always follow your personal ethics. For example – personally you may feel abortion is
morally wrong, but if you are a doctor, you need to do abortion according to your professional
ethics. Such dilemmas are natural but we need to draw a line between personal and
professional role. When performing a role in public, we need to separate our personal lives and
follow professional code of conduct strictly.

5. Resolving Ethical Conflicts: What is the way out?


Every human being is an end in itself. Certainly, we use other human beings to achieve our
ends. A student uses the teacher to achieve his/her goals. However, there is a crucial aspect of
an informed consent that is involved here. Hence, it is important that we do not treat humanity
as merely as a means.
What is ethical or moral can be determined by whether it is just and fair. For example, if you
have one flute, whom should you give it to: a person who is good at playing flute (virtuous), a
person who has the maximum need for it (utility) or a person who has been assigned the task
to play flute (deontological)? The answer is not simple. However, by making the process of
distribution fair and just can one really claim that an ethical solution has been arrived at.
No one approach is universal or claim to be without any demerits. Aiming for maximizing total
happiness might lead to violation of basic rights of an individual. Aiming to adhere to duty
might lead to negative consequences; just that responsibility of those consequences will be
lessened on the individual who takes the decision. In the long run, general happiness can be
increased by rewarding the virtue and penalizing vice.
Justice means choosing what the right thing to do is without fear or favor. The process of
judgment must be free of any bias- not only bias towards an individual or a class should not be
there, but also the approach or the methods which one uses cannot be the same all time. This
means that if the utilitarian approach produced the best results in a case, it does not and
should not become the favored approach in all subsequent decision making. A wider, multi-
dimensional worldview equips a person with tools to make a well-informed and just decision.
With experience in diversity, one comes to realize more and more about circumstances and
problems of others. As such, effort should be made to incorporate interests in other cultures
and societies as well. This way, a reasoned and well-argued decision can be arrived at. This
decision should be able to recognize the benefits and aim for their maximization as well as
recognize the demerits and aim for their minimization.

6. Human Values
“Values” denote the value or importance we assign to different aspects of the world around us.
A value is a preference as well as conception of the preferable. We attribute values to every
human action, thus denoting its vastness.
Ethical decision-making often involves weighing values against each other and choosing which
values to elevate. Conflicts can result when people have different values, leading to a clash of
preferences and priorities.
Some values have intrinsic worth, such as love, truth, and freedom. Other values, such as
ambition, responsibility, and courage, describe traits or behaviors that are instrumental as
means to an end.

DELHI JAIPUR PUNE HYDERABAD AHMEDABAD LUCKNOW CHANDIGARH


20 www.visionias.in # 8468022022 ©Vision IAS
Google it:- https://upscpdf.com
https://t.me/UPSC_PDF Download From > https://upscpdf.com https://t.me/UPSC_PDF

Some values are considered sacred and are moral imperatives for those who believe in them. Student Notes:
Sacred values will seldom be compromised because they are perceived as duties rather than as
factors to be weighed in decision-making. For example, for some people, standing during the
National Anthem is a non-negotiable value. For others it may only be a matter of choice.
However, human values are of utmost importance. Human values are defined as those values
which help man to live in harmony with the world. They are at the core of us being humans.
Without a deep understanding of one’s relationships with nature, fellow human beings, society,
and a deep respect for all life, one is not really educated. Human values consists of wide range
of values related to personal, family, community, social, political, environmental and work life
of the individuals.
The sense of equality, mutual respect, the philosophy of live and let live are the cherished
results of human values. They can be thought of as socially desirable goals that are internalized
through the process of conditioning, learning or socialization. Our educational system is
naturally concerned with values. One of the important missions is to inculcate some universal
basic human values such as – happiness, fairness, love, peace, freedom, safety, respect,
responsibility, cooperation, self-reliance, equality etc.
So, whether values are sacred,
have intrinsic worth, or are a
means to an end, values vary
among individuals and across
cultures and time. However, values
are universally recognized as a
driving force in ethical decision-
making. Human values hold
importance due to the benefits it
has for society. They provide
criteria by which we judge people,
objects, actions, ideas and
situations. Human values may be
subjective or objective, intrinsic or
extrinsic, personal or community,
theoretical or practical, social,
political or economic etc.
One of the attempts made by researcher, Shalom H. Schwartz’s theory of basic values which
seeks to identify a core set of basic human values grounded in the motivational goals inherent
in
1. our individual, biological needs,
2. our need for smooth coordination and cooperation with others, and
3. the need of groups of people to survive and grow as groups.
The system of 10 basic values derived from these goals forms a continuum arranged in a closed
circle as in the above infographic.

6.1. Role of Family, Society and Educational Institutions in


inculcating values
Inculcation of values basically means teaching ethical standards to child or developing moral
system with respect to different domains of his life. Moral system can be described here as set
of internalized values. This responsibility of teaching morals has long been seen as the domain
of family where children spent the formative years being taught basic issues of conduct along
with what family believed was right and wrong. However, there is overwhelming external
influence that childhood is subjected to which parents finds difficult to stop. Thus the

DELHI JAIPUR PUNE HYDERABAD AHMEDABAD LUCKNOW CHANDIGARH


21 www.visionias.in # 8468022022 ©Vision IAS
Google it:- https://upscpdf.com
https://t.me/UPSC_PDF Download From > https://upscpdf.com https://t.me/UPSC_PDF

responsibility of ethical teaching has subtly shifted from parents to social institutions like Student Notes:
education institutes and media.
Socialization- Process by which an individual is indoctrinated into his culture- transfer of culture from
one generation to other. It’s a lifelong process to shape an individual- such that the person is useful and
productive member of society. Observation, Conditioning, Role playing, Trial and Error are some of the
techniques of socialization.
Family, society and educational institutions are three most important factors affecting the
values of a person. Large part of this process of inculcating values happens consciously in a
deliberate manner by socializing agents.
6.1.1. Role of Family in inculcating values:
• The family, the basic unit of society is also the first school of value learning through
examples set by the members, ethical teachings imparted by elderly. These can be through
stories, life lessons etc.
• The cultural values of harmony, equity, cooperation, democracy, peace are passed on to by
the family to child. It is family which imparts in an individual the value of sacrifice, love,
feelings, high morals and others thereby, shaping a child’s attitude towards other people in
society.
• Family members are the immediate role models of a child who models his/her behaviour
around them.
• Members of family inculcate moral values such as honesty, truthfulness, happiness, loyalty
and integrity in children, which are synonymous with societal values.
• How a child is reared also impacts the values of child in future. For example – an
authoritarian type of child rearing may develop greater value for authority. It is very much
possible that such children may have less concern for democratic values. In nuclear families
children learns more individualistic values that in the joint peaceful family. In India, children
in cities living only with parents tend to be more addictive to video games than those who
are living in bigger families.
Changing nature of family system and values
Family is and always the first value provider, but in recent years, its role has changed which can
be seen in the behavior patterns of the children. In a modern nuclear family, the value system
being imparted to a child has changed. Focus may be more on competition rather than
cooperation, on individualism rather than family and collectivism, on consumerism rather than
gratification and sacrifice. It is not necessary that the values being transmitted have degraded,
but they have certainly changed. Remember, values are preferences. Earlier, sharing or delayed
gratification may have been a preferred value. Now it is being replaced by consumerism and
instant fame. Some values over time have gained the status of being fundamental. Some on the
other hand are compromised because of human weakness.
Sometimes, family itself teaches one to be selfish in this fiercely competitive world, say by
preventing the sharing of notes or information with friends to maintain an edge over them in
various competitions. At times, it may be in the interest of the child, but ultimately, it inculcates
the value of self-interest and deters him from inculcating the values of cooperation and
sharing. This just shows how a modern family value is different from a traditional family
value. May be in future, this will become a traditional value.
However, it is not necessary that a child’s value system once he/she grows up will be similar to
that of parents. One may actively discard certain values through other influences such as
media, education system, friends, work, etc. and above all, self-evaluation.

DELHI JAIPUR PUNE HYDERABAD AHMEDABAD LUCKNOW CHANDIGARH


22 www.visionias.in # 8468022022 ©Vision IAS
Google it:- https://upscpdf.com
https://t.me/UPSC_PDF Download From > https://upscpdf.com https://t.me/UPSC_PDF

6.1.2. Role of Society and Educational Institutions in inculcating values Student Notes:
A. Role of Educational Institutions: After family, it is educational institutes in which a child
spends most of his time. Thus, they also have an important role in shaping the personality of
the child. Here child is introduced to the world away from the comfort zone of the family. As is
being witnessed today, a world evolved through narrow, exclusive and intolerant thought is full
of conflicts, violence, inner tensions and war. Therefore, there is a need for a world evolved
through harmony, tolerance, peace, and concern for achieving human sustenance. Value
education helps in achieving these goals. School provides a medium where the individual to
communicate, acts as a bridge between family and society,

Role of Teachers: Teachers are great role models and their actions leave a great impact on
children in their impressionable age. Teachers inculcates values in students via-
• Observational learning, conditioning to produce desired behavior.
• Exhibiting commitment, diligence, timeliness in their conduct, ensuring unbiased treatment
• Influence the self-attribution pattern of learners through the judicious use of rewards and
punishments.
• Role in positive self-appraisal and creating an inclusive environment
Role of Curriculum-
• Culture-specific inputs are required to facilitate adjustment of child in the social milieu
• The content needs to be driven by scientific temper without any ideological coloration
• The focus could be given to the right to learning rather than the right to education
• Emphasizing on extracurricular Activities which helps in building cooperation, team-spirit,
leadership, responsibility.
Education is a systematic attempt towards human learning. All education in essence develops
all the dimensions of a human personality – intellectual, physical, social and moral. In recent
years, due to crisis of values in educational system, the term ‘value education’ has become the
buzz word in the educational institutions and academia.
B. Media as an agent of inculcating values: Media is a tool for social management, social
diffusion, and social change. Children are easily influenced by the functional “parallel school” of
the media. The media influence the development and transformation of values and beliefs of
people in a variety of ways like social development, tolerance, positive attitude towards others
culture. They also contribute to the enhancement of knowledge, language and vocabulary and
influence a major part of daily life. It helps in framing new ideas, attitudes and lifestyle. Media
plays role in developing the social values in terms of consumerism, urban modeling,
restructuring of human relationships and the emergence of new ideas and policies
In contemporary times, the social media has emerged as a powerful tool, which can influence
the attitude and behavior both individually and collectively. Provides the platform to express
our opinion but also provides anonymity. It has the potential to promote public participation,
social engagement, and democratization of public life.

DELHI JAIPUR PUNE HYDERABAD AHMEDABAD LUCKNOW CHANDIGARH


23 www.visionias.in # 8468022022 ©Vision IAS
Google it:- https://upscpdf.com
https://t.me/UPSC_PDF Download From > https://upscpdf.com https://t.me/UPSC_PDF

Positive impact of media- Student Notes:


• Promoting social and gender awareness removed ignorance, led to challenging various
preconceived notions,
• Community radio, Bultoo experiment, act as a linking pin between government and the
common man
o Bultoo Radio, launched in Chattishgarh, is basically a radio program that is shareable
through Bluetooth, which is available on the most basic mobile sets – thus bringing an
end to the need for internet and mobile signals
• Strengthening democracy by ensuring people's participation, etc.
Negative impact of media- Commodification and objectification of women, consumerism,
relative deprivation, radicalization, given rise to various anti-social behavior, etc.
At last we can say that there is a strong relationship between the media and the common man
which further contributed for the development of values only if the media sources are wisely
used. The media has played an important role in positive developments like the fight against
racism, gender bias, unemployment, poverty, and spreading awareness about the need for
world peace.

7. Glossary of terms:
Morality The state or quality of being moral with particular application. It exists as social fact.
When particular tends to become universal, it may come in conflict with ethics
Ethics The state or quality of being moral with primacy of reason. It exists more as
cultural ideal. Both particular can be universalized and universal can be
particularized
Reason Human faculty to be used for right decisions. Process of synthesizing using logic to
draw inferences/conclusions. It can work in harmony or opposition to desire
Belief Our brain is the 'belief engine' and believing is a natural process of development
of mind. Our mind constantly needs beliefs to come in terms with reality. Beliefs
are cognitive constructs which exist in the form of statements/propositions which
are assumed to be true but could be true/false
Faith It is the state of unchanging belief towards something
Trust It is the state of developing beliefs towards something based on evidence
Knowledge It is the end product of process of testing beliefs based on scientific evidences
Values Values denote preference – for good or bad. Values are important and lasting
beliefs or ideas about what is good or bad and desirable or undesirable
Virtue They are the values that are part of human character and are universally believed
to have some goodness in it e.g. honesty, trust, gratitude etc
Principles Principles are the abstract concepts to guide the behaviour according to a
particular value e.g. impartiality as a principle to follow the value of ‘equality’
Norms As an established social practice in particular context which is considered as
normal. Norms as standards of behaviour that regulates individual actions
Freedom Is a political value and the state of being capable of making decisions without
external control. It is the basis of moral agency
Rights They are normative rules or claims to realize freedom in certain respect. e.g. right
to education, life, liberty
Duty It is the obligation tied to certain roles to behave in a certain way
Probity It is the condition in which prescribed ethical standards are upheld by governance
system. This condition is consisting of process and procedures
Rules They are set of instructions/ prescription which tells us the way things are to be
done. They are in conformity with laws or tradition to be followed voluntarily or
non-voluntarily for particular purpose

DELHI JAIPUR PUNE HYDERABAD AHMEDABAD LUCKNOW CHANDIGARH


24 www.visionias.in # 8468022022 ©Vision IAS
Google it:- https://upscpdf.com
https://t.me/UPSC_PDF Download From > https://upscpdf.com https://t.me/UPSC_PDF

Rule of Law Legal and political systems, structures and practices that condition a Student Notes:
government’s actions to protect citizens’ rights and liberties, maintain law and
order, and encourage the effective functioning of the country
Action It is a part of behaviour which are conscious, goal oriented leading to change in
existing behaviour
Behaviour It is range of human response to the environment, it includes conscious action
and unconscious reaction, thinking and feeling
Character It is the mental and moral qualities in totality that underlies the personality.
Character traits are revealed themselves only in specific and often uncommon
situations. They are based on beliefs. A person is said to have strong character if
her behavioural qualities are consistent across different situations. In cinema,
literature, comics, roles are also called characters as behavioural qualities of a
person are clearly distinguishable

8. Previous Years Questions of UPSC


2013
1. What do you understand by ‘Values’ and ‘Ethics’? In what way is it important to be ethical
along with being professionally competent?
2. Some people feel that values keep changing with time and situation, while others strongly
believe that there are certain universal and eternal human values. Give your perception in
this regard with due justification.
3. “The good of an individual is contained in the good of all.” what do you understand by this
statement? How can this principle be implemented in public life?
2014
1. All human beings aspire for happiness. Do you agree? What does happiness mean to you?
Explain with examples.
2. What does ethics seek to promote in human life? Why is it all the more important in public
administration?
3. In the context of defense services, 'patriotism' demands readiness to even lay down one's
life in protecting the nation. According to you, what does patriotism imply in everyday civil
life? Explain with illustrations and justify your answer.
4. "Human beings should always be treated as 'ends' in themselves and never as merely
'means'." Explain the meaning and significance of this statement, giving its implications in
the modern techno-economic society.
5. Which eminent personality has inspired you the most in the context of ethical conduct in
life? Give the gist of his/her teachings. Giving specific examples, describe how you have
been able to apply these teachings for your own ethical development.
6. The current society is plagued with widespread trust-deficit. What are the consequences of
this situation for personal well-being and for societal well-being? What can you do at the
personal level to make yourself trustworthy?
2015
1. What is meant by ‘environmental ethics’? Why is it important to study? Discuss any one
environmental issue from the viewpoint of environmental ethics.
2. Differentiate between the following
(a) Law and ethics
(b) Ethical management and management of ethics
(c) Discrimination and preferential treatment
(d) Personal Ethics and Professional Ethics
3. Social values are more important than economic values. Discuss the above statement with
examples in the context of inclusive growth of a nation.
2016
1. Explain how ethics contributes social and human well-being.

DELHI JAIPUR PUNE HYDERABAD AHMEDABAD LUCKNOW CHANDIGARH


25 www.visionias.in # 8468022022 ©Vision IAS
Google it:- https://upscpdf.com
https://t.me/UPSC_PDF Download From > https://upscpdf.com https://t.me/UPSC_PDF

2. Law and ethics are considered to be the two tools of controlling human conduct so as to Student Notes:
make it conducive to civilized social existence.
(a) Discuss how they achieve this objective.
(b) Giving examples, show how the two differ in their approaches.
2017
1. The crisis of ethical values in modern times is traced to a narrow perception of the good
life. Discuss
2. Without commonly shared and widely entrenched moral values and obligations, neither the
law, nor democratic government, nor even the market economy will function properly.
What do you understand by this statement? Explain with illustration in the contemporary
times.
2018
1. “In doing a good thing, everything is permitted which is not prohibited expressly or by clear
implication.” Examine the statement with suitable examples in the context of a public
servant discharging his/her duties.
2. With regard to morality of actions, one view is that means are of paramount importance
and the other view is that the ends justify the means. Which view do you think is more
appropriate? Justify your answer.
3. Suppose the Government of India is thinking of constructing a dam in a mountain valley
bound by forests and inhabited by ethnic communities. What rational policy should it resort
to in dealing with unforeseen contingencies?

9. Previous Years Case Studies of UPSC


1. Sivakasi in Tamil Nadu is known for its manufacturing clusters on firecrackers and matches.
The local economy of the area is largely dependent on firecrackers industry. It has led to
tangible economic development and improved standard of living in the area. So far as child
labour norms for hazardous industries like firecrackers industry are concerned,
International Labour Organization (ILO) has set the minimum age as 18 years. In India,
however, this age is 14 years.
The units in industrial clusters of firecrackers can be classified into registered and non-
registered entities. One typical unit is household-based work. Though the law is clear on
the use of child labour employment norms in registered/non-registered units, it does not
include household-based works. Household-based work means children working under the
supervision of their parents/relatives. To evade child labour norms, several units project
themselves as household-based works but employ children from outside. Needless to say
that employing children saves the costs for these units leading to higher profits to the
owners.
On your visit to one of the units at Sivakasi, the owner takes you around the unit which has
about 10-15 children below 14 years of age. The owner tells you that in his household-
based unit, the children are all his relatives. You notice that several children smirk, when
the owner tells you this. On deeper enquiry, you figure out that neither the owner nor the
children are able to satisfactorily establish their relationship with each other. (25 marks |
300 words)
(1) Bring out and discuss the ethical issues involved in the above case.
(2) What would be your reaction after your above visit?

2. You are working as an Executive Engineer in the construction cell of a Municipal


Corporation and are presently in-charge of the construction of a flyover. There are two
Junior Engineers under you who have the responsibility of day-to-day inspection of the site
and are reporting to you, while you are finally reporting to the Chief Engineer who heads
the cell. While the construction is heading towards completion, the Junior Engineer have
been regularly reporting that all construction is taking place as per design specifications.

DELHI JAIPUR PUNE HYDERABAD AHMEDABAD LUCKNOW CHANDIGARH


26 www.visionias.in # 8468022022 ©Vision IAS
Google it:- https://upscpdf.com
https://t.me/UPSC_PDF Download From > https://upscpdf.com https://t.me/UPSC_PDF

However, in one of your surprise inspections, you have noticed some serious deviations and Student Notes:
lacuna which, in your opinion, are likely to affect the safety of the flyover. Rectification of
these lacunae at this stage would require a substantial amount of demolition and rework
which will cause a tangible loss to the contractor and will also delay completion. There is a
lot of public pressure on the Corporation to get this construction completed because of
heavy traffic congestion in the area. When you brought this matter to the notice of the
Chief Engineer, he advised you that in his opinion it is not a very serious lapse and may be
ignored. He advised for further expediting the project for completion in time. However, you
are convinced that this was a serious matter which might affect public safety and should
not be left unaddressed. What will you do in such a situation? Some of the options are
given below. Evaluate the merits and demerits of each of these options and finally suggest
what course of action you would like to take, giving reasons.
(1) Follow the advice of the Chief Engineer and go ahead.
(2) Make an exhaustive report of the situation bringing out all facts and analysis along with
your own viewpoints stated clearly and seek for written orders from the chief Engineer.
(3) Call for explanation from the Junior Engineers and issue orders to the contractor for
necessary correction within targeted time.
(4) Highlight the issue so that it reaches superiors above the Chief Engineer.
(5) Considering the rigid attitude of the Chief Engineer, seek transfer from the project or
report sick.

10. Previous Years Questions of Vision IAS Test Series


1. Do you think that ethical principles are universal in nature? Justify your answer with
relevant examples.
Approach:
• Introduce by writing about the concept of ethical universalism.
• Discuss the concept of ethical relativism.
• Provide relevant examples for both – universalism and relativism.
• Conclude based on the arguments written in the answer.
Answer:
Ethical principles are used to decide the rightness or wrongness of an action. But what
is ethical under a given circumstance may itself be a subject of debate. In this respect,
there are two schools of thought regarding mutability of ethical principles:
Ethical Universalism
• It believes in universal and immutable ethical values i.e. there are some ethical
principles that are always true, that these principles can be discovered and that
these principles apply to everyone.
• Certain universal ethical principles are – love, universal declaration of human rights
etc. Similarly, principles of honesty, integrity, justice, accountability etc. are central
to the functioning of a democratic administrative system. These principles cannot
be traded for any kind of just or unjust objectives.
• However, this school overlooks the need to respect diversity and also the view that
consequence of an act is also a factor in deciding the ethicality of that act.
Ethical Relativism
• It believes that ethical principles keep changing depending on the circumstances
and socio-cultural factors. Ethicality of an act depends on the different cultures or
different periods in history. For example, homosexuality has been de-criminalized
recently in our country.

DELHI JAIPUR PUNE HYDERABAD AHMEDABAD LUCKNOW CHANDIGARH


27 www.visionias.in # 8468022022 ©Vision IAS
Google it:- https://upscpdf.com
https://t.me/UPSC_PDF Download From > https://upscpdf.com https://t.me/UPSC_PDF

• Ethical relativism was on display when India, irrespective of its commitment Student Notes:
towards non-violence, conducted surgical strikes across the border inside Pakistan
to destroy terrorists’ launch pads. Another example showing mutability of ethics is
of Bihar, which recently prohibited sale and consumption of liquor in the state.
• Similarly, while the principle of equality is an accepted principle, there can be
exceptions towards the betterment of marginal communities on the basis of
compassion and equity such as- mid day meals, various subsidies etc.
• However, this school seems to reduce ethical principles to the level of just being
“general agreement of a group of people”. It gives priority to a majoritarian view
even if the act in question is bad in morality, for example, moral sanction of honour
killing of couples opting for inter-caste marriage in certain pockets of India.
• Further, in this school, the only moral standards against which a society's practices
can be judged are its own. So, if ethical relativism is correct, there can be no
common framework for resolving moral disputes or for reaching agreement on
ethical matters among members of different societies.
Thus, though there are certain principles which remain immutable under all
circumstances, there are other principles which keep changing with time and situation
with socio-cultural practices playing a significant role in deciding the ethicality of an act.

2. What do you understand by global ethics in today's world? Highlight the role that
global ethics can play in achieving universally accepted goals.
Approach:
• Start by explaining global ethics in today’s world.
• Discuss the role of global ethics in achieving universally accepted goals.
• Conclude accordingly.
Answer:
Global ethics is an area of critical ethical enquiry into the nature and justification of
values and norms that are global in kind and into the various issues that arise such as
world poverty and international aid, environmental problems, food security, disaster
mitigation, peace and security, and human rights.
Without the acceptance of global ethics, a large number of these common global
problems cannot be resolved effectively. Here, global ethics can contribute in following
ways:
• Draws parallels between self-interest of nations and the universality of these
interests so a shared understanding of global problems such as disaster
management, biodiversity conservation can be developed.
• Creates moral pressure for countries to come forward and provide assistance when
conditions in other countries are such that governments either will not or cannot
address natural and human-made evils effectively. For instance, Common but
Differentiated Responsibilities and Respective Capacities (CBDR-RC) acknowledges
the different capabilities and differing responsibilities of individual countries in
addressing climate change.
• Establishes checks and balances to avoid unilateral actions such as exporting
environmental problems, economic aggression etc that may involve violation of the
interests of another country. For instance, Missile Technology Control Regime
(MTCR) adopts these principles to limit the proliferation of missiles and missile
technology.
• Provide space to countries to discuss and generate consensus on issues of global
importance such as fighting against terrorism or money laundering such as FATF
that sets international standards to prevent terror financing.

DELHI JAIPUR PUNE HYDERABAD AHMEDABAD LUCKNOW CHANDIGARH


28 www.visionias.in # 8468022022 ©Vision IAS
Google it:- https://upscpdf.com
https://t.me/UPSC_PDF Download From > https://upscpdf.com https://t.me/UPSC_PDF

• Holding governments responsible for domestic action which leads to violation of Student Notes:
human rights. For example, UNHCR visiting countries alleged rights violations.
• Developing an international institutional framework so that collective efforts can
be directed towards achievements of common goals for e.g. sustainable
development goals.
• Checking rise and growth of rogue nations and non-state actors by taking
collective actions against them for activities such as nuclear armament, terrorism,
human trafficking, organ trafficking etc.
None of the aforementioned responses to global problems can occur without a
commitment to global goals as an ethical requirement or acceptance of global
responsibilities. Considering the uncertain world scenario and differences among
nations, global ethics can mobilize the nations; channelize their efforts in the right
direction to deal with the emerging challenges and enable states to rise above the
narrow self-interest and move towards enlightened self-interest.

3. What do you understand by utilitarianism? Illustrate with examples the grounds on


which it has been criticized.
Approach:
• Write a brief explanatory note on Utilitarianism and give examples.
• Discuss its types and explain each of them briefly.
• Make use of examples to discuss various grounds on which Utilitarianism can be
criticized.
Answer:
Utilitarianism is fundamentally driven by the principle of utility i.e. that action is
morally right which produces the best overall consequences with regard to the utility or
welfare of all the affected parties. Utilitarian theories propound “greatest happiness of
the greatest number”. It suggests that the only reason for performing action X over
alternative action Y is that doing X will make mankind happier than doing Y. Jeremy
Bentham (1748–1832), John Stuart Mill (1806–73), and Henry Sidgwick (1838– 1900)
are some utilitarianism theorists. 

OBJECTION TO UTILITARIAN THEORIES
• In some situations, Utilitarian theories might require us to do morally problematic
or doubtful things in order to bring about a good result. For instance:
o If a judge can prevent riots that will cause many deaths only by convicting an
innocent person of a crime, utilitarianism implies that the judge should convict
and punish the innocent person.
o If a doctor can save five people from death by killing one healthy person and
using that person’s organs for life-saving transplants, then utilitarianism implies
that the doctor should kill the one person to save five.

• In a utilitarian society, people’s behaviour would lack the kind of predictability and
consistency that is required to sustain trust and social stability. For instance, in the
previous examples, if judges and doctors can do anything to maximize well-being,
then no one will be able to trust that judges will act according to the law or that
doctors will not use the organs of one patient to benefit others.
• By propounding happiness of the maximum number, Utilitarianism overlooks the
interests / needs / happiness of the groups that are numerically inferior. E.g.
Restrictions on consumption of beef to appeal to majoritarian voices, is
inconsiderate to the dietary preferences of minority.

DELHI JAIPUR PUNE HYDERABAD AHMEDABAD LUCKNOW CHANDIGARH


29 www.visionias.in # 8468022022 ©Vision IAS
Google it:- https://upscpdf.com
https://t.me/UPSC_PDF Download From > https://upscpdf.com https://t.me/UPSC_PDF

• Happiness in many cases cannot be quantified as is not measurable thus it is often Student Notes:
very difficult to apply the test of happiness.
• Predicting consequences is impossible that utilitarianism requires because
consequences are inherently unknowable. For example, people celebrate extra-
judicial killings, however, it disregards due process of law and may lead to
lawlessness.
Thus, there are various concerns associated with utilitarian ethics and it cannot be the
sole guiding light for human actions, societal goals and government’s programmes.

4. As interactive technologies such as Artificial Intelligence penetrate peoples’ lives, the


lack of understanding of their social influence carries a risk of them doing more harm
than good. Discuss.
Approach:
• Define and establish the increasing use of interactive technologies in today’s world.
• Highlight how that has made social influence inevitable.
• Discuss the underlying importance of social influence and principles governing the
same, especially amongst the creators and regulators of these technologies.
• Briefly conclude on the basis of aforementioned arguments.
Answer:
Interactive technology allows for a two-way flow of information through an interface
between the user and the technology. The user usually communicates a request for
data or action to the technology with the technology returning the requested data or
result of the action back to the user. These technologies work on real time basis and are
available in the form of software applications, games, internet of things including social
media sites such as facebook and twitter, etc.
Interactive and transformative technologies, such as artificial intelligence, augmented
reality, autonomous vehicles etc. are designed to make our lives better. They are of
immense potential in learning, entertainment, interpersonal communications,
etc. However, such technologies work along various principles of social influence, an
inadequate understanding of which causes more harm than good. This can be
demonstrated as under:
• In terms of Conformity: i.e. change in behaviour or attitude brought about by a
desire to follow beliefs and standards of other people.
o Some interactive technologies like Instagram, facebook, tiktok, etc. have the
capacity to create social comparisons i.e. people start using information about
others to evaluate themselves.
o The public and social leader boards in online games create unnecessary
competition among players and negatively affect those who are at the lower
ranking. They also generate unwanted peer pressure especially upon youth
with impressionable minds.
• In terms of Compliance: i.e. a social influence where an individual does what
someone else wants them to do following his/her request or suggestions.
o The ultimate goal of interactive technology is to capture and analyze the
different ways humans interact with each other, their environments, and
objects around them. Nefarious elements use this to fulfill their own agendas;
for e.g. ISIS used this technology to brainwash and recruit several Asian
muslims.

DELHI JAIPUR PUNE HYDERABAD AHMEDABAD LUCKNOW CHANDIGARH


30 www.visionias.in # 8468022022 ©Vision IAS
Google it:- https://upscpdf.com
https://t.me/UPSC_PDF Download From > https://upscpdf.com https://t.me/UPSC_PDF

o Several applications require access to personal data such as fingerprints, Student Notes:
contacts, videos, photos, etc. Scammers use this information to commit
financial and other data fraud.
o Inadequate importance is given towards privacy controls. People easily upload
their pictures, addresses, last locations, etc. This has resulted in several criminal
cases such as stalking and harassment.
• In terms of Obedience: i.e. giving in to the commands of others.
o Several online games such as the Blue Whale Challenge, Choking game etc.
instructed the participants to complete several tasks, which included self harm,
watching scary videos, and waking up at odd hours. This brings physical and
psychological harms to the participants especially teenagers.
o Fake news and videos on platforms trusted by people has resulted into mass
hysteria, panic and lynching as well.
Social influence is a complex area of behavioural science. While there can be no doubt
regarding the positive outcomes of interactive technologies, the rapid rate of their
acceptance in our daily lives has squeezed the amount of time required to assess their
negative outcomes. Adoption of any such new technologies must be well informed by
correctly assessing the amount of imbalance these can cause.

5. The nature of a business's operations has a major influence on the ethical issues with
which it must contend. Giving examples, discuss how business ethics is crucial in
today’s world.
Approach:
• Briefly explain the term business ethics.
• With the help of appropriate examples, explain how the nature of a business'
operations influence the ethical issues of an organization.
• Highlight how business ethics is crucial in the present-day world.
• Conclude on the basis of the above points.
Answer:
Business ethics is the system of moral and ethical beliefs that guides the values,
behaviors, and decisions of a business organization and the individuals within that
organization. It is the overarching framework that puts moral regulations on an
organization based on its business operations. The nature of a business' operations has
a major influence on the ethical issues, which need to be dealt by an organization. As a
result, the ethical issues also differ from industry to industry, which can be
understood from the following examples:
• An energy company produces energy by utilizing various natural resources like
fossil fuels, nuclear fuel etc. The nature of their operation may include price risks in
exploration, environmental concerns like oil spills as well as risks to human life in
the form of nuclear disaster. Here, business ethics would demand a company to be
transparent in its accounts as well as follow a clearly defined system of
environmental ethics.
• An e-commerce company like Amazon, Flipkart etc., which is involved in online
operations of goods and services must be prepared for ethical issues like protection
of data, customers' privacy and security.
• A pharmaceutical company is engaged in development and manufacturing of life
saving drugs. Such a business would require that the organization does not indulge
in unethical actions like improper clinical trials, misleading advertisements, nexus
with doctors, patent claims for non-innovations etc. They must follow business

DELHI JAIPUR PUNE HYDERABAD AHMEDABAD LUCKNOW CHANDIGARH


31 www.visionias.in # 8468022022 ©Vision IAS
Google it:- https://upscpdf.com
https://t.me/UPSC_PDF Download From > https://upscpdf.com https://t.me/UPSC_PDF

ethics like optimum pricing and quality of drugs, accurate information sharing and Student Notes:
informed consent during clinical trials.
Business ethics not only help an organization tackle the ethical issues, but it is also
crucial in today’s world as it helps:
• Builds trust among people: Ethical standards of operations followed by the
company helps in building a positive reputation of the company within the
community.
• Provides stability to the company: Running a business in an ethical manner from
top to bottom builds a stronger bond between employees and the management.
• Improves performance of organization: A high standard of business ethics in all
facets of operations makes people in an organization perform their job duties at a
higher level and also to stay loyal to that organization.
• Increase formal investment in the economy: A company with a foundation of
ethical behavior increases its potential of attracting more investors and
shareholders. With increasing number of investments, the company as well as the
economy develops potential to generate further employment opportunities in a
country.
In an era of increased ethical awareness of the people, adhering to business ethics has
not just remained a moral responsibility but also a legal obligation. In this direction,
many countries, including India have formulated many civil and criminal laws that a
business must follow to operate in an ethical manner.

6. Environmental ethics is about the moral relationship of human beings to, and also the
value and moral status of, the environment and its non-human contents. Elaborate.
Approach:
• Briefly explain environmental ethics.
• State how its domain goes beyond humans and encompasses non-human
components.
• Conclude appropriately.
Answer:
Environmental ethics is the branch of ethics that studies the moral and ethical aspects
of the relationship between humans and the natural environment. It focuses on the
environmental values and societal attitudes related to protecting and sustaining
biodiversity and ecological systems for the present as well as future generations.
The environmental ethics include both human and non-human content:
• Unlike traditional ethics, which is mainly concerned with human duties,
environmental ethics is all-encompassing and extends the scope of ethical
concerns to include non-living natural resources like rivers, grasslands, mountains
etc. For instance, the Whanganui River in New Zealand was given the status of a
‘legal person’ with its own rights and values as it has special and spiritual
importance for the Maori tribe.
• Environmental ethics moves ahead of the instrumental value of the natural
environment to further include the intrinsic value that is the value of things as ends
in themselves regardless of whether they are also useful as means to other ends.
For example, a certain wild plant may have instrumental value because it provides
the ingredients for some medicine or as an aesthetic object for human observers.
But the plant also has some value in itself independently of its prospects for
furthering some other ends such as human health, or the pleasure from aesthetic
experience.

DELHI JAIPUR PUNE HYDERABAD AHMEDABAD LUCKNOW CHANDIGARH


32 www.visionias.in # 8468022022 ©Vision IAS
Google it:- https://upscpdf.com
https://t.me/UPSC_PDF Download From > https://upscpdf.com https://t.me/UPSC_PDF

• Environmental ethics questions the assumed moral superiority of human beings to Student Notes:
members of other species on earth. It questions the dictum that nature has made
all things specifically for the sake of man.
• Environmental ethics is global as environmental issues transcend individual
identities and national boundaries and there must be value consensus and
cooperation among nations to cope with them. These include concerns such as
climate change and associated problems that have led to increase in environmental
refugees, melting glaciers, increasing marine pollution, sinking islands etc. In this
context, islands in the Maldives and Tuvalu are rapidly shrinking and it is the
responsibility of the global community to prevent them from disappearing.
• This branch of ethics critiques consumerism accompanying modern capitalism and
anthropocentrism (human-centered point of view). It calls for a ‘green lifestyle’
that is harmonious with nature and espouses an economic arrangement that is
sustainable and sensitive to nature’s limits.
Environmental ethics has been an integral part of the Indian tradition where nature has
been revered for centuries for its services to mankind. Various environmentalists such
as Baba Amte and Anna Hazare, movements such as the Chipko movement, Narmada
Bachao Andolan etc. have also spread awareness about ecological balance and
preservation of nature. It is important for humans to live in harmony with nature, and
adopt a model of sustainable development that would be beneficial for both mankind
and nature. In this context, Gandhiji has rightly said,’ the world has enough for
everyone’s needs but not for everyone’s greed’.

7. What do you mean by the term “Conflict of Interest”. Identify some situations that
may fall within its definition and analyze each one of them.
Approach:
Straight forward question. Explain the meaning of the term and identify some
situations that fall within its ambit.
Answer:
Conflict of interest may be defined as a situation in which a public employee has a
private or personal interest sufficient to influence or appear to influence the objective
exercise of his official duties. Here personal interest includes more than economic
matters. It is considered an indicator, a precursor, and a result of corruption.
Transparency International understands a conflict of interest as a situation where an
individual or an entity for which they work, whether a government, business, media
outlet or civil society organization, is confronted with choosing between the duties and
demands of their position and their own private interests.
The OECD guidelines distinguish between:
• Actual conflict of interest: a direct conflict between a public official’s current duties
and responsibilities and his/her private interests.
• Apparent conflict of interest: where it appears that a public official’s private
interests could improperly influence the performance of their duties but this is not
in fact the case.
• Potential conflict of interest: where a public official has private interests which are
such that a conflict of interest would arise if the official were to become involved in
the relevant official responsibilities in the future.

DELHI JAIPUR PUNE HYDERABAD AHMEDABAD LUCKNOW CHANDIGARH


33 www.visionias.in # 8468022022 ©Vision IAS
Google it:- https://upscpdf.com
https://t.me/UPSC_PDF Download From > https://upscpdf.com https://t.me/UPSC_PDF

The following situations can be said to fall within the definition of conflict of interest. Student Notes:
1. Bribery
Bribery is the illegal acceptance of money or other valuable considerations in exchange
for special favours from public servants having to do with their official duties. The
critical condition here is that the bribe giver clearly intends to distort the objective,
even-handed conduct of the official, and the receiver intends to willingly comply. Thus
the official, faced with personal interests, pecuniary or otherwise, in conflict with the
laws, policies, and procedures for the conduct of his office, is induced to resolve the
conflict in favour of himself. Although bribery usually involves money, it may include
other rewards, such as sexual favours, promises of favourable publicity, or offers of
access to exclusive social circles.
2. Influence Peddling
Influence peddling occurs when a public employee attempts to influence a government
decision so that it favours a third party in which the employee has an interest.
Understood in typical terms, this might include such cases as policy decisions regulating
a business in which the employee holds shares or developing a general plan affecting
the value of land owned by the employee. This becomes an actual conflict of interest
situation whenever the employee stands a high chance of significant gain.
3. Information Peddling
Officials who are privy to information not available to the general public and who use
that information to their own advantage, monetary or otherwise, are guilty of
information peddling. The key factors are the power of the information and privileged
access to it. Actual conflict of interest is present when the information is highly
confidential and the official in question is responsible for maintaining the confidence.
4. Financial Transactions
Financial transactions become conflicts when a public servant has direct or indirect
financial interests that directly conflict with the responsible performance of the job.
Actual conflict of interest is present to the extent that an official is in direct personal
control over a decision that will produce significant personal gain. This differs from
influence peddling in that the official in question effectively controls the outcome – as
for example a secretary does when he can influence the location of a new airport near
undeveloped land that he owns.
5. Gifts and Entertainment
Seeking or accepting gifts and hospitality creates a conflict if these items influence a
public employee’s impartial discharge of his or her duties. This category of conflict of
interest amounts to a broadening of understanding of bribery. It includes such things as
discounts on purchases, offers of theater tickets, sex, or vacation trips, use of vehicles,
and gifts of lavish meals, recreational equipment, and liquor. Typically gifts of this kind
differ from other bribery in that they are given with no specific favours requested, but
they are nevertheless intended to create a generally positive predisposition towards
the donor.
6. Outside Employment
Part-time employment, consulting, retainers, and self-employment may cause a conflict
of interest with official duties. Conflict situations include the use of public employment
status to enhance a private employer (or oneself), the draining away of effort and
energy required for official duties, and the use of government services and equipment
in outside work.

DELHI JAIPUR PUNE HYDERABAD AHMEDABAD LUCKNOW CHANDIGARH


34 www.visionias.in # 8468022022 ©Vision IAS
Google it:- https://upscpdf.com
https://t.me/UPSC_PDF Download From > https://upscpdf.com https://t.me/UPSC_PDF

7. Future Employment Student Notes:


If a public employee intends to seek employment in the future with a firm he now
transacts official business with, the tendency may be to give favoured treatment to this
prospective employer in hopes of encouraging a job offer. Also, the government
employee may present her inside contacts and knowledge of agency procedure as an
attractive package for a firm that does regular business with his public organization. As
people move back and forth between public and private sector service, intertwined
interests resulting from past work and future employment expectations can become
extremely complex.
8. Dealing with Relatives
Situations in which a public administrator may be in a position to do favours for a
relative can also create conflict. We might think of this form of conflict of interest,
sometimes called nepotism, as a special class of influence peddling because the
motivation is similar to other such cases. Essentially it involves using influence to gain
preferential treatment for a relative in hiring, promoting, awarding contracts, or any
other business practice. The public administrator who engages in such practices gains
not directly, but indirectly, by reinforcing family bonds and mutual support.

8. What do you understand by Administrative Ethics? Why is it needed for efficient


functioning of the administrative machinery?
Answer:
There are many definitions of administrative ethics.
• Administrative ethics are considered as ‘a set of moral norms and requirements for
those in public administration to aim their professional activity at attainment of
common wealth and effective use of moral values’.
• Administrative ethics are based upon moral norms. These norms are approved by
society, in its role as an important regulator of collective activities and existence as
well as upon professional values, which are closely connected to public
administration.
• Its tasks are to regulate employee relations by means of norms, behaviour and
actions, and to form an ethical component in the consciousness of public
administration employees.
• Normally, it is stressed that governmental employees set up ethical standards for
managerial decisions, analyze these standards and bear personal and professional
responsibility for the decisions made.
Administrative ethics studies all moral aspects of public administration employees' and
senior managers' activities. It includes three basic components.
• Values: individual, group and social statements, opinions and attitudes towards
concepts like freedom, justice, honesty, loyalty, neutrality, responsibility, etc.
• Standards and norms: the principles that guide the actions of people and
employees and help lead and control their behaviour (laws, codes, rules).
• Behaviour: different forms of employee activity limited by certain standards and
norms corresponding with social values.
Importance of administrative ethics:
• They reflect the concept and goals of the civil service as well as special tasks of
different governmental institutions.

DELHI JAIPUR PUNE HYDERABAD AHMEDABAD LUCKNOW CHANDIGARH


35 www.visionias.in # 8468022022 ©Vision IAS
Google it:- https://upscpdf.com
https://t.me/UPSC_PDF Download From > https://upscpdf.com https://t.me/UPSC_PDF

• These principles are influenced by a conception of an ideal or target public Student Notes:
administration model adopted in society.
• They aid in defining the ethical requirements for civil servants, to understand
clearly the aim of the civil service and the mission of the civil servant.
• To participate in the regulation of the relationship between government and
citizens.
• To promote public and state concerns in government activities as much as possible.
• To provide public administration staff with certain behavioral standards based on
morality.
The main task of administrative ethics is participation in regulation of the relationship
between the government and people. A system of ethical standards and requirements
would assist:
1. in putting into practice social constitutional values, such as freedom, social justice,
equal opportunities, essential civil rights;
2. in overcoming differences between public groups by means of development and
performing reasonable and constructive policies, compromising ethics and social
integration; ethical aspects of working out and making decisions in the area of
public administration are of primary importance in this case;
3. in distributing public wealth, goods, benefits and compensations made by
governmental bodies; furthermore, in the course of governmental institutions
performing these tasks, the problems of justice, equality, etc. arise;
4. in increasing authority and prestige, since any state and regional government
institution employee represents power in general, and ordinary people considers
his/her activities as actions of the state; this imposes a special responsibility for
those employed in the area of public administration.
Why administrative ethics are needed for efficient functioning of the administrative
machinery:
• An increasing need to define ethical criteria for every public servant, and to impose
responsibility for his/her professional activities.
• Developing a unified approach to set up easily interpreted moral principles and
standards and to work out a unified system of values and ideals.
• Administrative ethics require professional groups and specialists working for these
groups to serve social, professional, group and single customer concerns.
• Special ethical requirements for administrators are necessary in order to realize in
society such constitutional values as freedom, social justice, equal opportunities
and essential human rights. Therefore, there is a strong relationship between
ethical requirements and constitutional values. Special requirements for
administrators should cover the following standards: requirements (how
professional morality directs civil servant behaviour in certain conditions);
prohibitions (what is prohibited in professional behaviour); recommendations (how
civil servants should behave in certain moral conditions).
• It is extremely important to develop high moral responsibility for public
administration employees who are dealing with individuals representing different
social groups. It should be mentioned that from the citizen, professional group or
institution point of view it is public administration employee who represents the
state and acts on behalf or according to the order of the government body. His/her
behaviour, style of work, ways to resolve problems and to talk to ordinary people
finally forms the impression upon government machinery.
• All mentioned above oblige to develop measures to improve style of ministries and
departments work as well as to control the performance of government

DELHI JAIPUR PUNE HYDERABAD AHMEDABAD LUCKNOW CHANDIGARH


36 www.visionias.in # 8468022022 ©Vision IAS
Google it:- https://upscpdf.com
https://t.me/UPSC_PDF Download From > https://upscpdf.com https://t.me/UPSC_PDF

administrative machinery. It is necessary to pay attention to high responsibility for Student Notes:
one's duties in the employee education, to inculcate in servant the features like
efficiency, orderliness, irreproachable and honest duty performance as well as
ability to foresee and to work prospectively.

9. Colleges and schools are a microcosm of the real world. Elaborate as to how these are
instrumental in character-building. Also suggest certain measures through which
these institutions can play a greater role in the current scenario.
Approach:
Establish how colleges and school represents the real world. Then discuss the ways in
which they play a role in character formation, the way they influence an individual and
the degree of influence. Finally suggest some measure to enhance the quality, impact
and form of the values imparted by these institutions. These measures should be
mainly focused upon enhancing the efficiency, role and scope of these institutions in
character building.
Answer:
Family acts as a primary teacher in child’s life but it is school, which broadens his
horizon and allows others (teachers, staff, peers, etc.) to help him develop into the
smart, confident and respectful adult. Schools and colleges represent a small sample of
real world to the child in a way that there are individuals senior to him (teachers, staff)
whom he is supposed to obey, there are colleagues from different class, caste and race,
religion, sex, nationalities, cultures and traditions with whom he interacts and it
provides him the first experience of future life in which he has to be among such
persons belonging to various groups. It also helps them to shape their viewpoint of the
world around them. Schools provide a world view in a sense that there are deadlines to
complete the tasks, examination are manifestation of performance oriented life,
gradation/marking signifies that in real life one will be evaluated and rewarded based
upon his performance. Thus in schools/colleges different kinds of processes and
interactions come to a child for the first time and thus it makes them very important
institutions for character building and inculcating values.
Role of Schools and Colleges:
School is primarily an educational institution, which fosters systematic learning in a
more or less prescribed manner. School’s role is to bring each student to his individual,
maximum academic potential. A significant amount of schoolwork throughout the
grades is dedicated to helping children become expert problem solvers and solution
seekers, skills that will come in just about every personal and professional aspect of
child’s adult life. Therefore, in addition to academic skills, problem solving is an
essential life skill.
The school plays an important role in helping children learn to interact positively with
their peers and teachers. They learn about healthy relationship skills and develop them
further through interactions, both in classroom and on the playgrounds. Thus schools
provide them an opportunity to connect with other children and adults in a caring and
emphatic manner. Schools not only provide lessons on reading, writing and arithmetic
rather from the very beginning it also focuses on helping kids learn about compassion,
respect, empathy and integrity. At first it begins with basic lessons like “raise your hand
to speak” and “keep your hands to yourself”.
Later it develops into argumentative debates about moral and social issues. By the time
one graduates high school he should have a fairly firm grasp on his own beliefs, values
and how he see himself. All this helps him to be a responsible and confident human

DELHI JAIPUR PUNE HYDERABAD AHMEDABAD LUCKNOW CHANDIGARH


37 www.visionias.in # 8468022022 ©Vision IAS
Google it:- https://upscpdf.com
https://t.me/UPSC_PDF Download From > https://upscpdf.com https://t.me/UPSC_PDF

being. Team spirit, leadership, obedience and discipline are some other important Student Notes:
personality traits, which are imparted by schools through various activities.
Measures, which can improve role of Schools/Colleges:
• The way in which teachers interact with children and encourage interaction
between children, affects each child’s development in important ways. So it is very
important to ensure quality interaction at each level. For this it is expected that
schools publish policies for building the character of their pupils and regularly
assessing pupils’ strength of character. This can also include general behavioural
expectation from the teachers and staff.
• Moreover, there should be a part of curriculum dedicated to education of values.
Lectures and events based upon lives of great leaders, social reformers and noble
personalities should be promoted and rather than keeping them superficial in
nature they should be so designed that it can create maximum impact on students’
life.
• Students with good character should be rewarded and appreciated; this will
promote others to behave like that.
• Teachers should present themselves as a role model to students and stay
completely honest and dedicated in their responsibility.
• Students should be taught about emerging ethical dilemmas in the times of
growing use of technology with respect to social media, internet, mobile phones
etc.
• Cordial and constructive relation between seniors and juniors is an important
element for strong and swift character building process of an individual while lack
of this results in many problems like clashes (in form of ragging etc.), stress and
exploitation.

10. Is the morality of an action dependent upon the circumstances of the act or is it
independent of it? Examine. Can an action be unethical yet moral?
Answer:
Morality is individual's own sense of right or wrong which is subject to change
according to their changing social milieu and associated social, cultural, religious and
political conditionings. Besides this there is an existence of transcendental, objective
moral truths which functions regardless of geographic location, place in history or form
of culture. Humans have historically recognized some objective moral absolutes; these
principles transcend culture, location and history. The doctrine that says that morality is
depended on circumstance sees morality in relative terms and it has important
implications for how we conduct our lives, organize our societies, and deal with others.

Suppose you have a moral disagreement with someone, for example, a disagreement
about whether it is okay to live in a society where the amount of money you are born
with is the primary determinant of how wealthy you will end up. In pursuing this
debate, you assume that you are correct about the issue and that your conversation
partner is mistaken. Your conversation partner assumes that you are making the
blunder. In other words, you both assume that only one of you can be correct. However,
both conflicting moral beliefs can be true.
However some moral actions are often dependent upon the circumstances in which
they are made. For e.g. moral actions (such as killing) are sometimes justified under
certain circumstances (i.e. when protecting the life of an innocent child) . Similar is the
case with attitudes towards violence which is paralleled by variation in attitudes

DELHI JAIPUR PUNE HYDERABAD AHMEDABAD LUCKNOW CHANDIGARH


38 www.visionias.in # 8468022022 ©Vision IAS
Google it:- https://upscpdf.com
https://t.me/UPSC_PDF Download From > https://upscpdf.com https://t.me/UPSC_PDF

towards sex and marriage. When studying culturally independent societies, Student Notes:
anthropologists have found that over 80% permit polygamy. Some cultures marry off
girls while they are still pubescent or even younger. In parts of Ethiopia, half the girls
are married before their 15th birthday.
Some scholars say moral variation is greatly exaggerated – people really agree about
values but have different factual beliefs or life circumstances that lead them to behave
differently.
With morals, unlike science, there is no well-recognized standard that can be used to
test, confirm, or correct when disagreements arise.
Therefore what we require is to address the multifariousness of moral considerations
that arise in particular cases and the need and possibility for employing moral
principles in sound moral reasoning. We require moral judgment, not simply a
deductive application of principles or a particularist bottom-line intuition about what
we should doour moral judgment is greatly aided if it is able to rest on the sort of
heuristic support that casuistry offers.
Our principal interest is in ways that we need to structure or think about conflicting
considerations in order to negotiate well our reasoning involving them.Some influential
building-block for thinking about moral conflicts can be W. D. Ross's notion of a “prima
facie duty” ( if there is a conflict between prima facie duties, the one that is strongest in
the circumstances should be taken) etc.
But relativism has been widely criticized as well. It is attacked as being sophomoric,
pernicious, and even incoherent. Moral philosophers, theologians, and social scientists
try to identify objective values so as to forestall the relativist menace.
ii) Morals and ethics are two sides of the same coin. Morals are 'rules' (if you will) that
individuals are personally bound by through our experiences, knowledge, upbringing,
environment, etc. Ethics on the other hand are 'rules' that we subscribe to based on
any particular social group with which we associate ourselves. For the better part of the
population, morals and ethics are in line with each other. Simply put, most people
choose to associate themselves in groups with which they can 'see themselves' in.
Telling lies is unethical, but to do so to save a life is certainly moral. For example, there
are numerous examples of very ethical people who lied to the Nazi's in order to save
the lives of Jews.Many professions are put in ethical dilemmas all the time, for example
police officers. If their higher purpose is to save lives, then their action is moral even if
sometimes the means is unethical. Many times those same police officers are willing to
delude themselves into thinking the action is moral when it is not.
A dilemma arises when a groups 'morals' (i.e. ethics) are not in line with an individuals
morals. Whether you choose to analyze such a dilemma from a psychiatric,
physiological or sociological stance is a matter of personal preference. However, any
act, belief, behavior, thought, etc. can be moral but unethical. That is, if you are
personally bound to believing that a particular act is moral, it does not necessarily have
to be ethical. Take any 'hot political topic', such as war, abortion or human rights, or if
you prefer more personal acts/behaviors such a monogamy or suicide.
Essentially, any act that an individual chooses to pursue, but is frowned on by the social
group serves as an example . The only pre-requisite is that the individual who performs
the act believes it to be moral. Extreme examples; Hitler and Napoleon. They each
(most likely) believed their respective actions were morally justified, but the world
marginalized them as unethical leaders.

DELHI JAIPUR PUNE HYDERABAD AHMEDABAD LUCKNOW CHANDIGARH


39 www.visionias.in # 8468022022 ©Vision IAS
Google it:- https://upscpdf.com
https://t.me/UPSC_PDF Download From > https://upscpdf.com https://t.me/UPSC_PDF

11. Should circumstances be the sole criterion for judging the morality of human action or Student Notes:
the nature of the action and its purpose must also be considered? Justify your stand
with examples.
Approach:
The answer should contain the following parts:
• Identify the elements of human action (nature/object, circumstances, and purpose)
that are analyzed for judging the morality of human action.
• Further, explain that though circumstances are a criterion but they are not the sole
criterion since the object, as well as, purpose of human action should also be
considered in deciding the morality of human action.
• To justify your point, give examples/illustrations to show that circumstances can’t
make an action, whose object is bad, ethical.
Answer:
Some philosophers (teleologists or interpretivists) subscribe to the view that
circumstances are the sole criterion for judging the morality of human action. Joseph
Fletcher maintained that action becomes specific through circumstances. Without
specificity ethical elements in action can’t be examined. Circumstances make an
abstract action to be specific based on time, place, agent, and manner. For example, to
strike in self defence is one thing and to strike without any provocation is another
matter. Thus, circumstances decide the morality of human action.
However, there are two other elements to every action that decides the morality of
human action - nature/object and purpose/end of human action- not just the
circumstances.
An action whose object is bad by its very nature will remain bad and nothing can
improve it- neither circumstances nor purpose. For example, a lie remains a lie despite
the purpose or circumstances. Circumstances can only make it less bad but never good.
Here another issue with circumstances as the sole criterion becomes apparent - that it
makes morality subjective and relative (telling lie can be bad or less bad depending on
circumstances).
Similarly, an action whose purpose/intention is bad is unethical and nothing can
improve it- neither object nor circumstances. For example, giving donation to a poor
helpless person is good because of its object and the circumstances but can be termed
as unethical if you give donation with an intention to lure a homeless person into doing
something evil.
Thus, circumstances are not the sole criterion ratherobject, circumstances, and
purpose together decide the morality of human action.

12. The legal framework in modern society has both anomalous and complementary
relationship with its ethical framework. Discuss with examples.
Approach:
The answer should contain the following part:
• A brief introduction discussing the relationship between law and ethics.
• How law and ethics complement each other, with the help of examples.
• When do both stand in contradiction to each other, again with the help of
appropriate examples.

DELHI JAIPUR PUNE HYDERABAD AHMEDABAD LUCKNOW CHANDIGARH


40 www.visionias.in # 8468022022 ©Vision IAS
Google it:- https://upscpdf.com
https://t.me/UPSC_PDF Download From > https://upscpdf.com https://t.me/UPSC_PDF

• The answer should also address why such a variation in relationship is seen in the Student Notes:
modern society.
Answer:
The old adage “If it isn’t illegal, it must be ethical” is deeply flawed in the context of
modern society. Ethics and law are as different as the unenforceable from the
enforceable, and as complementary to each other in the ultimate aim of creating a
citizenry, which does the ‘right’ thing when confronted with dilemmas.
Law are codified norms of the society, backed by the legitimacy of the state. It reflects
areas of moral agreement so broad that the society comes together and says, “This
ethical behaviour shall be mandated”. To a broad extent, when ethics collapses, the
law rushes in to fill the void.
An example helps here. Earlier you didn’t throw litter on the road simply because
“people don’t do those things”—because it was the “wrong” thing to do. Now you
don’t toss litter because there are fines for doing so. What was once a second-domain
issue of ethics has shifted to a first-domain issue of law. In such examples, legal
framework and ethics play a complementary role.
But there’s another side to it too, when legal framework can possibly have an
anomalous relationship with the ethical framework. A case in point is the “Ethics of civil
disobedience”. The standard of civil disobedience urges that unjust laws be disobeyed.
Mahatma Gandhi effectively used it in the 1920s against an unjust but lawful
government. Later on, civil rights movement in the United States in 1960s and anti-
apartheid movement in South Africa also were directed against unjust laws present in
those times. Civil disobedience demands moral reasons to disobey the law.
Such an anomaly is striking in the modern society because of the growing acceptance
of importance of delivering human rights to everyone, which many a times come in
conflict with the prevailing conservative laws. Gay rights movement is a relevant
example.

13. "Morality is based neither on the principle of utility, nor on a law of nature, but on
human reason. But human reason can be fallible." Comment. What does morality
mean to you?
Approach:
• Explain the statement giving examples where human reason cannot decisively
judge between what is good or bad.
• Give definition of morality, explain what it means to you by giving examples.
Answer:
It is through reasoning and logical thinking that man has been able to separate unjust
from just and ethical from unethical; not on the basis of utility or law of nature. Human
actions are based on reason, which guides us what we ought to do. The source from
where we derive our reasoning can be religion, law, internal beliefs and values, etc.
As such, human reason is conditioned by a number of factors, such as existing political
and social order. This leads to an act being decided as immoral by some while moral by
others. For example, today, capital punishment is held as immoral by many countries
and activists while there are many who reason for it as a retributive justice to some
heinous crimes. Thus, depending upon the reasoning an act can be moral or immoral.
We face number of dilemmas daily where reasoning creates conflict with the societal
norms and ethics. Depending on new information, reasoning can thus change, making it

DELHI JAIPUR PUNE HYDERABAD AHMEDABAD LUCKNOW CHANDIGARH


41 www.visionias.in # 8468022022 ©Vision IAS
Google it:- https://upscpdf.com
https://t.me/UPSC_PDF Download From > https://upscpdf.com https://t.me/UPSC_PDF

fallible. Hence, it is true that morality if left to pure reason can be susceptible to fault. Student Notes:
Morality may be termed as principles concerning the distinction between right and
wrong or good and bad behavior. It is a particular system of values and principles of
conduct. Generally these principles are accepted by society at large, yet, they may be
personal standards of an individual that he thinks are virtuous.
To me also, morality means taking a reasoned stand to differentiate good from bad.
However, it should be open to new facts and alternate viewpoints which broaden the
horizon of human reason. For example, speaking truth is considered moral universally.
It may also be considered moral by an individual. However, for somebody who
considers helping someone in need as greater moral principle may do so even if he
needs to tell a lie for same.

14. What are the various sources through which humans can judge the correctness of
their actions? In the context of public life discuss how these sources are important in
offering a clear and practical guidance.
Approach:
• Discuss rules, regulations and conscience as sources by which humans can judge
the correctness of their actions.
• Discuss their role in public life.
• Conclude by emphasizing the desirability of these sources in ethical decision
making.
Answer:
Laws and conscience are the two sources of guidance by which human beings can judge
the morality of their actions. These sources are particularly important to public
administrators in offering a clear and practical guidance.
1) Law, rules and regulations
Laws have a moral connotation. It induces people to act or restrains them from acting
and imposes an obligation. It must not only be just, but also burdens equally. Also, it is
for common, not private good. Regulations often help clarify laws. Unlike laws, rules
need not be for the common good and can be for the private good. For example, rules
made within an organization for efficient utilization of resources etc.
2) Conscience
It is a person's moral sense of right and wrong, viewed as acting as a guide to one's
behavior. Law states a general rule concerning actions; conscience lays down a practical
rule for specific action. Conscience applies the law or rule to specific actions; therefore
it is wider than law.
Importance in offering a clear and practical guidance
In considering right and wrong, public administrators have at their disposal information
on the nature of the action performed or about to be performed, the circumstances
surrounding the action and the purpose of the action. Laws, rules and regulations
provide additional guidance to these.
Sometimes rules circumvent what the civil law clearly states. While a superior can
punish a subordinate for violating such rules, if the rule is contrary to civil or natural
law, the violator may have acted ethically. Hence, rules violating natural or civil laws
should be avoided.

DELHI JAIPUR PUNE HYDERABAD AHMEDABAD LUCKNOW CHANDIGARH


42 www.visionias.in # 8468022022 ©Vision IAS
Google it:- https://upscpdf.com
https://t.me/UPSC_PDF Download From > https://upscpdf.com https://t.me/UPSC_PDF

It is often recognized that rules and regulations alone are insufficient for public Student Notes:
administrators. Without a conscience to apply those laws and rules to particular
actions, public administrators miss a critical element. Conscience helps in applying the
laws, rules and other criteria of morality to specific actions.
All public administrators have to make discretionary decisions where conscience plays
an important role.

15. What do you understand by ‘civic virtue’ in public life? What are the challenges in
practicing it in today’s time? How can these challenges be overcome?
Approach:
• Define the civic virtue and discuss different dimensions of civic virtue.
• Discuss the challenges faced by public while practicing it.
• Also list some solutions.
Answer:
Civic virtue is morality or a standard of righteous behavior in relation to a citizen’s
involvement in a society. It is an underpinning of how a citizen relates to the society. As
such, civic virtue has different notions in different societies. A liberal society makes
minimum demands on its citizens, whereas a republican tradition demands that citizens
be active, on the assumption that high levels of civic engagement is necessary to
protect against government excesses or to create shared public goods. Conservatism
emphasized family values and obedience to the father and the state. Nationalism
carried by masses of people made patriotism an important civic virtue.
A citizen may exhibit it simply by voting, by paying taxes, volunteering for social cause,
maintaining cleanliness, etc. Civic virtue lays stress on communitarianism rather than
individualism. Robert Putnam has defined three civic virtues: active participation in
public life, trustworthiness and reciprocity that is required through social
connectedness.
Challenges in practicing
• Growing individualism due to changes in society and its organization. Most
importantly, with conflict between traditional and modern values, the notions of
civic virtues are changing leaving a void in its actual practice.
• Competition for limited resources, with everyone trying to extract maximum for
self. In such an environment, people tend to forget their obligations towards society
and commitment to values. For example, people evade taxes to save money for
personal expenses.
• Lack of trust between public organizations and people and subordination of public
interest over personal interest.
• Difficulties in bringing about changes in behaviour and attitude, which stand in the
way of practicing desirable civic behaviour.
Solutions
• Promote civic education: School education is necessary for the upbringing of a
child. But it is not sufficient to generate a sense of responsibility towards society.
Hence we need to promote civility through value education.
• Lead by Example: Presently, people have cynical views regarding their leaders. The
public justify their action by assuming that leaders preach the ideas but not follow
them. So leaders need to be torchbearers. For example, many political leaders
came forward and tried to set an example for the society to promote cleanliness in
India.

DELHI JAIPUR PUNE HYDERABAD AHMEDABAD LUCKNOW CHANDIGARH


43 www.visionias.in # 8468022022 ©Vision IAS
Google it:- https://upscpdf.com
https://t.me/UPSC_PDF Download From > https://upscpdf.com https://t.me/UPSC_PDF

• Influential People: People have blind following for actors, entrepreneurs, speakers Student Notes:
etc. These people can easily generate sense of a responsibility towards the society.
For example, the government makes use of actors to fight against the problem of
open defecation and to encourage tax payments.
• Awareness about rights and duties: It is high time that people also give due
importance to their duties. For instance, recently SC has held that if a person does
not cast his vote, then he has no right to question the government.

16. The lives of great leaders inspire us in many ways. Which leader has had a lasting
impact on you and in what way? Mention one outstanding value that you have
imbibed from him/her and its importance to being a civil servant.
Approach:
• Describe which leader had a lasting impact on you and why.
• Mention the outstanding value of the leader that you have imbibed.
• Also explain the importance of those values in civil services.
Answer:
Dr. APJ Abdul Kalam is known to the world as the first scientist who went on to become
the President of India. But he was so much more than the labels that can describe. He
inspired others to dream big, and led through example. He did not limit his thinking
only to individual growth but expanded it to his country, and the world.
Dr. APJ Abdul Kalam embraced his unique traits, and strengths. He dared us to think
differently, innovate, and experiment. He reminded us again and again, not be afraid
of failures or what people think. Because without failed experiments, there will not be
new learning.
He displayed the qualities of devotion, dedication and determination towards his
works. He had a vision of developed and progressive India and inspired us all around his
vision. He had remarkable qualities of leadership. As a leader of his team, he used to be
the first to accept failures but gives first credit to his team on success.
The most outstanding value of Dr. Kalam which had lasting impact on our minds is His
unparalleled humility came with deep compassion. Such quality should also be
imbibed by the civil servants.
He wanted each and every Indian to realize their full potential, and contribute to the
well-being of the country and the world.He said - “Look at the sky. We are not alone.
The whole universe is friendly to us and conspires only to give the best to those who
dream and work.”
Humility is important in civil servants. It means the quality of having a modest or low
view of one's importance. A humble civil servant seek input from others to ensure they
have all the facts and are making decisions that are in the best interest of the different
sections of society. It becomes more significant in Indian context where owing to
colonial legacy and the miniscule proportion of its elite cadre, civil servants often
display ivory tower mentality, supremacy, individualism and disdain for public
participation in decision making.
A great way to show humility is accepting and admitting to past mistakes and using
these mistakes as teachable moments for constituents. When institutions and leaders
admit their mistakes, they make it acceptable for others to make their own mistakes
and learn from them.
Each civil servant has to place the people at the centre, be responsive to their needs,
respectful of their wishes and accountable to them. This requires civil servants to be
selfless in their contribution, inclusive in their decisions, humble in their behaviour and
inspiring in their actions.

DELHI JAIPUR PUNE HYDERABAD AHMEDABAD LUCKNOW CHANDIGARH


44 www.visionias.in # 8468022022 ©Vision IAS
Google it:- https://upscpdf.com
https://t.me/UPSC_PDF Download From > https://upscpdf.com https://t.me/UPSC_PDF

17. What is the relationship between Ethics and Law? Explain with examples. Student Notes:
Approach:
• Briefly, define Ethics and Law.
• Discuss the various aspects of Ethics and Laws to explain the relationship between
both.
• Provide an appropriate conclusion.
Answer:
Ethics is a reasoned framework of moral principles, which is concerned with what is
good for individuals and society. However, law is structured system of rules and
regulations enforced through certain authority to govern individual’s behavior, actions
and social relations.
Relationship between Law and Ethics
Similarities:
• Both laws and ethics are systems which maintain a set of moral values and prevent
people from violating them. They both provide guidelines to people of what they
may or may not do in certain situations. Most laws represent the minimum
standards of ethical human behavior.
• It is when certain ethical principles become widely accepted, they are codified into
laws. For example, ‘motherhood’ in our society is considered to be sacrosanct and
does not trade for monetary value. Therefore, the law regulating surrogacy in India
allows only altruistic surrogacy.
Dissimilarities:
• Foremost, law provides for punishment for violation- a legally enforceable action by
the competent authority. Ethics on the other hand may invite social sanction, but
no legally enforceable punishment in a court of law.
• There may be many areas where law does not exist or is silent. But ethics and
morals have a wider scope. For example, the law will not bind a businessman to tell
his competitor about their new client that has a reputation of not paying dues, but
ethics will still guide judgment here.
• Ethics may vary from people to people because different people may have different
opinions on a certain issue, but laws describe clearly what is illegal no matter what
individual opinions people have of them.
• To some extent, ethics is not well defined but laws are defined and precise.
• Ethics as a code of conduct may evolve as a society matures, but laws need to be
changed through specific action of the legislature.
The relationship between law and ethics is dynamic and a function of its time. Ethics
guide laws. Changing ethical and moral standards lead to amendment or invalidation of
existing laws. For example, The Right to Information was enacted to codify the ethics of
transparency in government functioning. Similarly, earlier homosexuality was widely
perceived as unnatural and immoral. But with the social progression, Section 377 has
been declared unconstitutional in so far as it criminalized “consensual sexual conduct
between adults of the same sex”.

DELHI JAIPUR PUNE HYDERABAD AHMEDABAD LUCKNOW CHANDIGARH


45 www.visionias.in # 8468022022 ©Vision IAS
Google it:- https://upscpdf.com
https://t.me/UPSC_PDF Download From > https://upscpdf.com https://t.me/UPSC_PDF

11. Previous Year Case Studies of Vision IAS Test Series Student Notes:

1. The global toll of the COVID-19 pandemic is enormous: more than a half-million lives
lost, hundreds of millions out of work, and trillions of dollars of wealth destroyed. And
the disease has by no means run its course. There is tremendous interest in the
development of a vaccine, with more than a hundred initiatives under way around
the world. Even if one or more vaccines emerge that promise to make people less
susceptible to COVID-19, the public-health problem will not be eliminated. But
policymakers can avert some foreseeable problems by starting to address key
questions about financing and distribution now. In view of the above scenario, answer
the following questions: (a) Identify the different stakeholders involved in this
scenario. (b) Identify some of the ethical questions and issues that are likely to
emerge as the vaccine becomes available. (c) Who, in your opinion, should be
amongst the first recipients of the vaccine? Give reasons for your answer.
Approach:
• Introduce by providing the gist of the given case study.
• Enlist the various stakeholders involved in the given case study.
• Bring out some of the ethical issues that are likely to emerge as the vaccine
becomes available.
• Discuss who should receive the vaccine first. Support your argument with valid
reasons.
Answer:
COVID-19 emerged in China and quickly became a worldwide problem. Just within the
first few months, the pandemic has adversely affected the lives of hundreds of millions
and more than a half-million have lost their lives. In light of this, various efforts are
underway for developing a suitable vaccine.
a) Following are the different stakeholders involved in this scenario:
• Common public at large: Their lives and livelihood depend upon the availability
of vaccines, especially of those who cannot afford masks, sanitisers and other
precautionary measures.
• Health-care workers and other essential workers involved in providing services
to people: They are in direct contact with the virus and are overworked due to
large number of cases coming. Every day they are putting themselves, as well
as their families at risks. Therefore, they would be willing to have a cure as early
as possible.
• Health organizations around the world like the World Health Organisation etc.:
They are responsible for research, collaboration, providing accurate
information, etc. Their efficacy as well as reputation is at stake.
• Governments around the world: It is their moral responsibility to control the
pandemic, provide adequate fund for the research and production of vaccine
and work towards its equitable access.
• Non-governmental institutions like Bill & Melinda Gates Foundation: They
invest in the research and also work with the governments in distributing the
vaccines and making them available to the vulnerable sections.
b) As the vaccine becomes available then following are some of the ethical issues that
are likely to emerge:
• Concern about safety of the developed vaccine: In a hurry to develop a
vaccine, various organizations are rushing through trial phase raising concerns
about efficacy and side-effects of the vaccine. In the process, they are
neglecting one of the principles of medical ethics- ‘to do no harm’.

DELHI JAIPUR PUNE HYDERABAD AHMEDABAD LUCKNOW CHANDIGARH


46 www.visionias.in # 8468022022 ©Vision IAS
Google it:- https://upscpdf.com
https://t.me/UPSC_PDF Download From > https://upscpdf.com https://t.me/UPSC_PDF

• Issues related to vaccine producing companies: Here the related question is of Student Notes:
how the companies that develop a vaccine will be compensated if they are
required to license the patents and know-how to producers elsewhere.
• Issues with capacity of vaccine production and distribution: There are nearly
eight billion people on the earth. Manufacturing eight billion doses (or
multiples of that if more than one dose is needed) of one or more vaccines and
distributing them around the globe could require years, not months.
• Political concerns: These include questions like who should receive the initial
doses of any vaccine, who should pay for the cost of vaccine production and
distribution, which authority will determine with regard to the above
questions, should a vaccine be given for free to some or all etc.
• International concerns: At the international level, the questions are even more
complex like what special advantages should be accrued to the country where a
vaccine is developed, to what extent the wealthier countries should help the
poorer countries.
c) Considering the large population and limited capacity of production and
distribution of vaccine, it will not be possible to provide everyone around the world
with the vaccine at the same time. There is need to develop a strategy for the same
which will guide us in deciding who should receive the vaccine first.
• One idea in this regard would be to administer the initial doses of vaccine first
to health-care workers, followed by police, firefighters, the military, teachers,
and other essential workers. This is done by analyzing the risk one faces due to
their occupation. The frontline workers who are at higher risk should be given a
vaccine first. This will help them in performing their role fearlessly and
efficiently. There have been several cases where frontline workers have lost
their lives while performing their duty. Also, there have been instances of
discrimination against them.
• Second idea is that the governments must consider the risk of COVID-19
disease progressing into serious complications. As per this approach, priority
should be given to those who are at higher risk of developing serious
complications from COVID-19, such as the elderly and those with pre-existing
medical conditions. This will not only help us in halting the progression of
disease into severe form but will also help in reducing the mortality.
In this context, any effective vaccine that is developed should be treated as a global
public good and should be distributed equally around the world, regardless of where it
was invented or of a country’s ability to pay. The WHO for this has put forward a global
allocation framework that seeks to ensure priority for the most vulnerable populations
and frontline workers.

2. Various studies have found out that cases of depression and mental illness have
increased exponentially in the recent past. Also, in the age group of 15-30 years, this
problem is further pronounced. Furthermore, the rising trend of suicides in this age
group has been attributed to depression.
Given the situation, answer the following questions:
(a) Present an ethical critique of the prevalent societal attitude towards mental
illness.
(b) Given the magnitude of the problem among younger generation/young adults,
analyse the role of the following:
i. Parenting
ii. Social Media
iii. Video Games

DELHI JAIPUR PUNE HYDERABAD AHMEDABAD LUCKNOW CHANDIGARH


47 www.visionias.in # 8468022022 ©Vision IAS
Google it:- https://upscpdf.com
https://t.me/UPSC_PDF Download From > https://upscpdf.com https://t.me/UPSC_PDF

Approach: Student Notes:


• Introduce the answer with statistics around mental health problems.
• Discuss the prevalent societal attitudes toward mental health issues and highlight
an ethical critique towards it.
• Briefly mention the perceptional shifts needed to tackle these issues.
• Analyse the role of parenting, social media, and video games in causing the mental
health issues.
• Conclude briefly.
Answer:
According to WHO, stress and depression cases increased by 18% in the last decade but
the expenditure on mental health in India is merely 0.06% of GDP. In the age group of
15-30, increase in mental health issues and eventual suicide rates can be attributed
toward cultural changes, and behavioural changes in the wake of increasing
urbanisation, employment and emotional uncertainty, social media generated online
bullying and peer pressure.
a) Attitudes and beliefs about mental illness can be shaped by multiple factors:
• Knowledge acquired through keenness
• Experience/interaction with someone living with mental illness
• Cultural stereotypes
• Media stories
• Familiarity with institutional practices and past restrictions (e.g., employment
restrictions)
Hence, the prevalent societal attitudes towards mental illness vary from empathy and
caring to that of neglect, pity and abhorrent treatment like a less-worthy human.
Although the societal attitude towards mental illness though has slowly been
evolving, however, following ethical issues continue to remain. For instance:
• Trivialisation: It is a minimising behaviour where an illness is conceptualised as
being easier to acquire, suffer with, or treat. Attitudes towards mental health are
still not on equal terms with those towards physical health.
• Stigmatisation: It is a significant barrier for the early diagnosis and treatment of
various mental health conditions. People with mental health problems are less
likely to seek help if they feel their condition is stigmatised.
• Social Labelling: In a lot of cases depression is stereotyped as a ‘first world
problem’ or the ‘problem of luxury’ and not an issue in the third world – which is
untrue, as large number of students, female, and farmer suicides take place in
developing countries like India.

DELHI JAIPUR PUNE HYDERABAD AHMEDABAD LUCKNOW CHANDIGARH


48 www.visionias.in # 8468022022 ©Vision IAS
Google it:- https://upscpdf.com
https://t.me/UPSC_PDF Download From > https://upscpdf.com https://t.me/UPSC_PDF

• Treatment issues and perceptional biases: Researching the diagnosis and Student Notes:
treatment of mental disorders also presents special ethical issues. For instance,
while dementia is perceived as quite straightforwardly ‘medical', other ailments
like personality disorders are sometimes treated and looked at differently, with a
stigma.
The society at large, beginning from the family, needs to be open about discussing
mental health issues. Consulting psychiatrist or medications should not be considered
taboo and immediate assistance and familial support must be provided, for which
district administrations can play a big role by creating sensitisation campaigns.
Interventions like happiness curriculum as introduced by the Delhi government that
included meditation; sports activities; reconnecting with nature should be undertaken
so as to hone young minds.
b) Mental health problem is a scientifically treatable disease. A disease has a cause, an
aggravating condition, and finally a treatment. Excessive pressure and competition is a
common cause of occurrence and its ignorance, a cause of aggravation. The role of
parenting, social media and video games for the younger generation vis-a-vis mental
health problems is as under:
• Parenting: Parental support and a nurturing family are key to addressing mental
health issues. The first step begins with a liberal environment in the family where
young adults, teenagers, can have free discussions with their parents. Moreover,
parents need to also understand that peer pressure, and creating pressure over
exam results, performance in competitive exams may impact young minds and
increase student suicide rates. Thus, a conducive environment where the younger
generation is encouraged to discuss issues should be promoted, over and above
dismissing beliefs related to mental health treatments.
• Social Media: Social media addiction is one of the key factors responsible in
changing our behaviour towards others and alienating ourselves. While social
media is a good platform towards connecting with people, and gather information,
over-dependence and over-consumption of it amount to addiction, which
influences the cognitive ability of adults, teenagers. Moreover, things such as
trolling, cyber bullying, perceptions about others, the need to constantly post or
upload – further isolate minds, while also limiting good habits like reading, playing,
or spending time with nature.
o Social media behaviour should be restricted and sensitisation campaigns on
limiting their usage must be undertaken by all schools, colleges, and work
environments.
• Video Games: According to the WHO, video-gaming disorder is a "pattern of
persistent or recurrent gaming behaviour" in which people lose control over their
gaming behaviour, resulting in impairments in their family relationships and social
lives and has recommended to classify gaming disorder as a separate addiction.
The Blue Whale Challenge game posed serious mental health risks in the form of
self-harm and attempted suicides.
It is therefore important to not dismiss mental health issues based on ill-conceived
notions and understand that stigmatisation will only worsen the silent epidemic of
depression. Additionally, ensuring proper implementation of the National Mental
Health Policy 2014, which puts renewed focus on depression, suicides and attitudes
towards mental health, shall also help in addressing these challenges.

DELHI JAIPUR PUNE HYDERABAD AHMEDABAD LUCKNOW CHANDIGARH


49 www.visionias.in # 8468022022 ©Vision IAS
Google it:- https://upscpdf.com
https://t.me/UPSC_PDF Download From > https://upscpdf.com https://t.me/UPSC_PDF

3. You, a reporter currently working as an intern, accompanied a small group of Student Notes:
reporters to a political event organized by the ruling party's state unit. The party was
followed by dinner where you were also asked to join. Here, you noticed that alcohol
was being served to some members of the ruling party. Consumption of alcohol had
been declared illegal in the state a few months ago and the ban is being enforced
strictly across the state. The other reporters in your group ignored the issue and asked
you to ignore it as well. However, it was clear to you that the law was not being
followed.
(a) Identify the issues involved in this scenario.
(b) What are your duties in such a scenario?
(c) What course of action would you follow and why?
Approach:
• Discuss issues such as political apathy, insensitivity and lack of values.
• Highlight the duties as an ethical journalist and a responsible citizen.
• Discuss the action which you would take.
Answer:
a) Facts of the case:
I as a reporter have witnessed a clear violation of law regarding prohibition of alcohol
by the same party who is in power and being asked to keep quiet about it by my
colleagues.
The issues related to the party members involved include:
• Breach of the law banning alcohol consumption.
• Political apathy.
• Dishonest and inappropriate behaviour.
• Double standards while implementing the laws v/s following them in letter and
spirit.
• Possible misuse of authority and influence to gain access to banned substance.
The issues related to reporters who turned a blind eye towards the incidence are:
• Breach of journalistic ethics.
• Indirect collusion with law breakers.
• Possible vested interests.
b) My duty as a journalist is to uphold the core values and cardinal principles of
journalism - truth, accuracy, fairness, impartiality and accountability.

As a responsible citizen, it is my duty to abide by the law of the land and to help the
authorities in its implementation. I should report any visible breach of law which may
help the authorities to implement the rule of law in letter and spirit.
c) I have two alternatives, either to keep quiet or report the incident to concerned
authority. By choosing the former I will not only be violating journalistic ethics, it also
shows lack of courage and an attitude of indifference towards illegality.
• Thus, I will report the issue to the police as it is a blatant violation of the law. As a
journalist I would collect relevant evidence (photograph/video etc.) which may be
useful in reporting this issue and aiding the police in filing a case.
• The fact that other group members want me to ignore the issue may be due to fear
or due to some vested interest. I would also try to convince the other members in
my group to come forward and bring the law breakers to justice.

DELHI JAIPUR PUNE HYDERABAD AHMEDABAD LUCKNOW CHANDIGARH


50 www.visionias.in # 8468022022 ©Vision IAS
Google it:- https://upscpdf.com
https://t.me/UPSC_PDF Download From > https://upscpdf.com https://t.me/UPSC_PDF

• I will use the journalistic platform and social media to bring the issue to the notice Student Notes:
of the general public so that true picture of those involved may be brought to fore.
Reason behind the course of action:
Reporting the illegal consumption of banned substances will help the administration
curb smuggling and in better implementation of the law. The journalistic principles of
truth and fairness and duty as a citizen of not remaining merely a spectator after
witnessing an illegal act but raising my voice against it will be upheld. Consumption of
alcohol by party members shows a clear case of hypocrisy. Such hypocrisy must be
checked to have a government with integrity.

4. Ramesh, a very hardworking person, is the sole bread earner in his family. He has
worked with an oil company's local affiliate for several years, and has established a
strong, trustworthy relationship with Suresh, manager of the local facility. Suresh has
recently recommended Ramesh to be recruited as the corporate consulting engineer
for the company, which would be a position of greater responsibility along with a
stable income. During a casual conversation, Suresh mentions an incident in the 1960s
wherein 10,000 gallons of a petrochemical was leaked into the local environment by
the company due to negligence, though at the time no damage was found, and no
mention of this leak was made to the press. When Ramesh mentions that the state
law requires him to report all spills, Suresh reminds him that no harm had been done
and reminds him that the company can't have a consulting engineer who does not
value loyalty and respect confidentiality.
(a) Identify the ethical issues involved in the given case.
(b) What are the options available to Ramesh in this situation? Evaluate each of
them.
(c) Had you been at Ramesh’s place, what would have been your course of action?
Give reasons for the same.
Approach:
• Analyse the case and identify the key ethical issues involved.
• List the options available to Ramesh in a given situation and bring out the positives
and negatives of each available option.
• Give the course of action and give arguments to justify your course of action.
Answer:
Involved Stake holders
• Ramesh, Suresh, the oil company, the government and the general public.
(a) Ethical Issue involved
1. Self-interest vs public interest
Provided that Ramesh is the only earning person in his family and his source of
income is not stable; he is not in position to lose his job. But, if he considers his job,
he is showing apathy towards public interest which lies in reporting the incident.
2. Moral values vs organisational ethics
Ramesh will be having the conflict between his belief in personal values and the
organisational ethics. Even if, he follows any one of them he will be doing injustice
with the other.
3. Negligence to state laws vs responsible citizen
Being a responsible citizen of the country, it is his duty to show respect for the
public laws by reporting the issue. But, to save his job he will neglect the state laws.

DELHI JAIPUR PUNE HYDERABAD AHMEDABAD LUCKNOW CHANDIGARH


51 www.visionias.in # 8468022022 ©Vision IAS
Google it:- https://upscpdf.com
https://t.me/UPSC_PDF Download From > https://upscpdf.com https://t.me/UPSC_PDF

(b) Options Available Student Notes:


1. Remain silent on the issue
Merits
• Since it happened long back and nobody was affected, it may be argued that raising
the issue now would be pointless.
• Following the advice of Suresh would mean that Ramesh’s relationship with him,
both personal & professional, would not be affected.
• It may also pave the way for further promotions.
Demerits
• Such inaction shows lack of respect for state laws which require reporting all such
incidents.
• Also such action may pave the way for future where such leaks are not reported.
2. Convince Suresh to report to the Government
Merits
• Reporting such incidents shows respect for state laws.
• Ramesh’s personal relationship with Suresh will remain unaffected.
• It would lead to detailed investigation for assessing the impact of the leakage.
Demerits
• It may jeopardize the professional career of both Ramesh and Suresh.
3. Talk to his senior or director
Merits
• It will also give a chance to senior or director to take suitable actions to correct the
earlier wrong happenings.
• It shows that Ramesh is trying to exhaust all available avenues where he can justify
his own values and responsibilities as a citizen.
Demerits
• He may face resistance in the company against him as the company has not
reported this incident till now.
4. To become whistle blower and reveal the incident
Merits
• He will ensure his actions towards abiding by the state laws as responsible citizen.
• He will be able to save environment and risks to people’s lives, if there were any
adverse effects later.
Demerits
• He may lose his job and destroy his relationship with Suresh permanently.
(c) Best Course of Action
Considering the gravity of the situation, I would have tried to convince the seniors for
reporting the incident to the government which then can assess the damages done to
the environment and public. This would also mean that wrongdoers would be punished
for their actions. This will also set an example for others to work following the rule of
law. However, it may mean that I might lose my job. At the same time, it must be noted
that in such cases public interest is more important which cannot be compromised at
any cost.

DELHI JAIPUR PUNE HYDERABAD AHMEDABAD LUCKNOW CHANDIGARH


52 www.visionias.in # 8468022022 ©Vision IAS
Google it:- https://upscpdf.com
https://t.me/UPSC_PDF Download From > https://upscpdf.com https://t.me/UPSC_PDF

5. Regulation and procedure of human clinical trials vary from nation to nation. Stem Student Notes:
cell research, as an emerging biomedical field, requires approval for human trials and
encounters multiple challenges. You are the head of a team of scientists who
developed a new Tissue Engineering system, which appears to be a promising means
of regenerating heart tissue. Trials of the system have already been conducted on
animals and yielded good results. Millions of people suffering from critical heart
diseases would benefit immensely if this medication is immediately made available to
them. However, you need to conduct human clinical trials before it could be
commercialized. It is also known that the stringent regulatory environment in the
country will mean that human trials and final approval will take many years before it
is made commercially available. On the other hand, regulation of clinical trials in
many poor countries is weak and quick approval is possible. Many of your
competitors also resort to human trials in these countries, often bribing the officials
for getting quick approvals. Given this situation, answer the following questions:
(a) Identify the ethical issues which arise during clinical trials.
(b) Given the above situation, would you prefer to shift human trials to a third
country where regulations are lax? Give reasons in support of your choice.
(c) Suggest a framework of standard procedure to minimize ethical conflicts and
speed-up the approval process of new medicines.
Approach:
• Give a brief introduction about human clinical trials and identify the ethical issues
in clinical trials.
• Discuss the merits and demerits of shifting lab location and state your final stand
on shifting the lab from the home country.
• Give a framework of standard procedure to minimise ethical conflicts and speed-up
the approval process of new medicines.
Answer:
Case Summary: I am the head of a team, which developed a new system which will
help people suffering from heart diseases. Before being made available commercially, it
has to undergo clinical trials on humans, for which strict regulations have to be
adhered. Alternatively, weak regulation in poor countries will result in quick human
trials and approval there and many competitors are doing the same including use of
bribes.
(a) Clinical trials are the tests on human volunteers to see whether new medical
treatments should be approved for wider use in the general population. Various
ethical issues associated are:
• Moral duty to perform honest clinical trials for welfare of a large population.
• Humanity is an end in itself (Gandhiji). Using people from the bottom strata of
society for undertaking considerable risks in lieu of minor payments degrades
humanity. The end result may improve general population’s health, but the costs in
form of exploitation, which may even cause death is wrong.
• Ethical concerns regarding informed consent in poor countries.
• Medicine is governed by certain ethos, which includes that first consideration of
physician should be health and life of patient. She should not do anything, which
impairs patient’s health. Clinical trials may weaken the normative foundations of
doctor-patient relationship in the long run.
(b) In the current case, the scientists have developed a new technology to regenerate
heart tissues, which will provide new lives to millions of people who are suffering
from critical heart disease. However, fewer regulations and corrupt governance in

DELHI JAIPUR PUNE HYDERABAD AHMEDABAD LUCKNOW CHANDIGARH


53 www.visionias.in # 8468022022 ©Vision IAS
Google it:- https://upscpdf.com
https://t.me/UPSC_PDF Download From > https://upscpdf.com https://t.me/UPSC_PDF

poor and developing countries pull scientists to open their labs and conduct trials in Student Notes:
these countries. The merits and demerits of such practice are as follows:
Merits
• The costs are lower in poor countries. So, the developed medical treatment would
be cheaper. For example, clinical trials in India could cost one-tenth of that in US.
• Availability of "treatment-naive" patients i.e. drug-free bodies in third world
countries. They are much less likely to have been previously exposed to drugs or
trials.
• The developing countries get benefits of advanced medical science and access to
the latest medications.
• The process of clinical trials is quicker in developing countries because getting
regulatory approvals is easier. It shortens the development time of medical
treatments.
• It would make this cure quickly available to people suffering from critical heart
diseases.
Demerits
• Consent: Most of the time, people in poor countries give their consent without
understanding the contracts as they are written in foreign languages, without
understanding the risks.
• Economic Compulsion: Sometimes poverty is responsible for the poor selling their
body for clinical trials. The blind faith for western medicine also plays a role in their
decision-making.
• Poor healthcare system: The risks are too high in clinical trials. However, poor
healthcare system in the third world increases their vulnerability in cases of
complications and side effects arising out of experiments on their bodies.
• The reliability of data obtained in third world countries is also questionable because
of lax monitoring.
• Fewer regulatory safeguards, high levels of poverty and illiteracy encourage
misconducts and substandard approach by foreign drugs companies.
• Bypassing professional ethics and an established procedure raises doubts on
integrity. Competition may tempt to opt for unfair means like bribes to get
approval.
As can be seen, there are a number of advantages of shifting the trials to countries
where costs are cheaper and regulations less stringent. However, laxity in procedures
should not be the motive for shifting trials. It would save lives of millions of people and
improve accessibility of cure through cheaper cost. So, it is prudent to shift laboratory
to country with easier regulations.
However, I will use the same safeguards and precautions which are used in developed
countries, encompassing every aspect from informed consent, side-effects’ treatment,
compensation in cases of errors, etc. I would make sure that no illegal practice is done
and ensure strict action for any breach.
Thus, I would be able to ensure greater benefits to people at large without
compromising with the safety and well being of people who have put trust in me.
(c) Framework of standard procedure
• Social Value: Study should help researchers determine how to improve people’s
health or well-being.
• Scientific Validity: Research should be expected to produce useful results and
increase knowledge. Researchers should design their experiments to be as good as
possible.

DELHI JAIPUR PUNE HYDERABAD AHMEDABAD LUCKNOW CHANDIGARH


54 www.visionias.in # 8468022022 ©Vision IAS
Google it:- https://upscpdf.com
https://t.me/UPSC_PDF Download From > https://upscpdf.com https://t.me/UPSC_PDF

• Fair Subject selection: Researchers should be fair in both recruiting and deciding Student Notes:
which people can be in the study.
• Favourable risk benefit ratio: For research to be ethical, any risks must be balanced
by the benefits to subjects, and/or the important new knowledge society will gain.
• Independent review: Researchers sometimes overlook ways they could improve
their research results. To avoid such problems, a group of people who are not
connected to the research are required to give it an independent review
• Informed Consent: Subjects must be told about the details of the study. They
should voluntarily agree to participate and give informed consent.
• Respect for subjects: Regular health monitoring along with maintaining
confidentiality of information would impart trust.
Data Sharing: Data from past researches should be made available to others. It would
exclude the need for new trials for similar drugs thus speeding the process of drug
approval. This clause must be ensured in WTO negotiations on data exclusivity.

Copyright © by Vision IAS


All rights are reserved. No part of this document may be reproduced, stored in a retrieval system or
transmitted in any form or by any means, electronic, mechanical, photocopying, recording or
otherwise, without prior permission of Vision IAS.

DELHI JAIPUR PUNE HYDERABAD AHMEDABAD LUCKNOW CHANDIGARH


55 www.visionias.in # 8468022022 ©Vision IAS
Google it:- https://upscpdf.com
ATTITUDE Student Notes:

Contents
1. Attitude –Definition and overview ............................................................................................. 2
1.1. Meaning of Attitude ............................................................................................................ 2
1.2. Structure of Attitude ........................................................................................................... 3
1.3. Functions of Attitude .......................................................................................................... 4
1.4. Broad Classification of Attitude........................................................................................... 4
2. Some Other Specific Types of Attitudes ..................................................................................... 5
2.1. Attitude Towards Weaker Sections ..................................................................................... 5
2.2. Moral Attitude ..................................................................................................................... 6
2.3. Political Attitude .................................................................................................................. 7
2.4 Prejudice .............................................................................................................................. 8
3. Attitude Formation..................................................................................................................... 9
4. Attitude Change ....................................................................................................................... 10
4.1. Persuasion ......................................................................................................................... 10
4.2. Social Influence ................................................................................................................. 13
4.3. Emotions and Attitude Change ......................................................................................... 17
4.4. Consequences of Social Influence or Persuasion .............................................................. 19
5. Attitude’s Relation with Thought and Behaviour ..................................................................... 21
5.1. How does Attitude Influence our Thought and Behaviour? ............................................. 21
5.2. When Does Attitude Predict Behaviour? .......................................................................... 22
5.3 Why it is difficult to change behaviour in Indian society ................................................... 23
5.4. Future applications of behaviour change.......................................................................... 24
5.5. Let’s apply what we have learnt!!!!!!!!!!........................................................................... 24
6. UPSC Previous Years Questions ................................................................................................ 25
7. UPSC Previous Years Question Paper: Case Studies ................................................................. 26
8. Vision IAS Test Series Questions............................................................................................... 27
9. Vision IAS Test Series: Case Studies.......................................................................................... 35

Copyright © by Vision IAS


All rights are reserved. No part of this document may be reproduced, stored in a retrieval system or
transmitted in any form or by any means, electronic, mechanical, photocopying, recording or
otherwise, without prior permission of Vision IAS.

DELHI JAIPUR PUNE HYDERABAD AHMEDABAD LUCKNOW CHANDIGARH


1 www.visionias.in # 8468022022 ©Vision IAS
https://t.me/UPSC_PDF Download From > https://upscpdf.com https://t.me/UPSC_PDF

1. Attitude –Definition and overview Student Notes:

1.1. Meaning of Attitude


Attitude is the learned and enduring predisposition to behave, either favourably or
unfavourably, towards something. This something can be an event, a person, an object or
classes of such events, persons, or objects, respectively.
Few examples - Explanation of “half-full glass” i.e. a person’s attitude determines whether it is
half-full or half-empty. Another example when population of India was exploding, one set of
people looked at it as burden, while another set of people looked it as expansion of human
resource that can be capitalized for development.
Two different individuals make two opposite senses of the same target object. This is because
of the differences in their attitude.
Characteristics of attitude
• Attitudes are abstract constructs, not something we can directly observe. We are only able
to infer them from behaviour.
• It is lasting evaluation with respect to various social significant issue and events. It is always
serving some purpose.
• Attitude is acquired through the socialisation process. It may involve individual or groups.
• Attitude can be expressed both verbally and non-verbally.
• Attitude is important because it shapes people’s perceptions of the social and physical
world and influence overt behaviour. For example, attitude influences friendship and
animosity towards others, giving and receiving help, hiring of ethnic minority job
candidates, etc. More dramatically, attitudes are at the heart of many violent attacks,
including crimes against humanity.
• Attitudes change with experiences and knowledge as man is a ‘rational’ animal.
• A rigid or dogmatic attitude is not the same as a strong attitude. A strong attitude may
have been formed with knowledge and experience and be grounded in rationality, hitherto
oblivious to others

DELHI JAIPUR PUNE HYDERABAD AHMEDABAD LUCKNOW CHANDIGARH


2 www.visionias.in # 8468022022 ©Vision IAS
Google it:- https://upscpdf.com
https://t.me/UPSC_PDF Download From > https://upscpdf.com https://t.me/UPSC_PDF

Attitude is ambivalent Student Notes:


• People can also be conflicted or ambivalent toward an object by simultaneously holding
both positive and negative attitudes toward the same object.
• There is no guarantee that attitude will be the same for similar objects or events.
For someone who likes chocolates, he may at times eat them to satisfy the craving and at
other times choose not to eat for health concerns.

1.2. Structure of Attitude


The structure of attitude has primarily been understood through two models i.e. CAB/ABC and
MODE model, which can be seen as-

Criticism of CAB/ABC Model


• This model is not validated by empirical research. Clear distinctions between thoughts,
emotions, and behavioural intentions associated with a particular attitude cannot be
established.
• A criticism of this ABC view of attitudes is that it requires cognitive, affective, and
behavioural associations of an attitude to be consistent, but this may be implausible. For
example, despite having negative attitude towards people from some caste, our behaviour
is mostly driven by the situation.
Importance of CAB /ABC Model
• It reflects something more than just negative and positive evaluation of a particular object.
Thus, this ABC structure of attitude acts as a good basic model to understand attitude.
• Importantly, it helps to identify the source from where attitude is originating.
• This will be helpful in persuading someone to change attitude towards something.

DELHI JAIPUR PUNE HYDERABAD AHMEDABAD LUCKNOW CHANDIGARH


3 www.visionias.in # 8468022022 ©Vision IAS
Google it:- https://upscpdf.com
https://t.me/UPSC_PDF Download From > https://upscpdf.com https://t.me/UPSC_PDF

1.3. Functions of Attitude Student Notes:

Attitudes serve various functions for the individual. These are broadly classified as:

1.4. Broad Classification of Attitude


When we talk of attitude, it is broadly in two manners- ‘attitude of’ and ‘attitude towards’.
Attitude of is the attitude of the subject in question- individual, society, institution,
organisation, etc. Attitude towards is the subject’s attitude towards any object- a person,
thing, institution, environment, etc. Hence, there can be many combinations of such types of
attitudes. For e.g.:
Civil Services as an institution may have democratic or bureaucratic attitude. Democratic
attitude is characterized by focus on goals/ends and treating citizens as stakeholders in the
governance process.
Bureaucratic attitude is characterized by focus on rules and processes rather than goals
(because elaborate and multiple rules make achievement of goals difficult, if not impossible).
Also, it is based on strict adherence to rules and hierarchy of the institution. As such, it treats
citizens as recipients of government services rather than as stakeholders. Environment,
Corporate and Political attitude can be other examples
As can be seen, attitude itself is broadly of three types: Positive, Negative and Neutral. These
are detailed in the infographic below-

DELHI JAIPUR PUNE HYDERABAD AHMEDABAD LUCKNOW CHANDIGARH


4 www.visionias.in # 8468022022 ©Vision IAS
Google it:- https://upscpdf.com
https://t.me/UPSC_PDF Download From > https://upscpdf.com https://t.me/UPSC_PDF

Student Notes:

2. Some Other Specific Types of Attitudes


2.1. Attitude Towards Weaker Sections
A man’s character is most evident by how he treats those who are not in a position either to
retaliate or reciprocate. The way we treat people who are disadvantaged, poor or marginalised
is determined by the attitude we have towards poverty, deprivation, discrimination and other
factors that perpetuate such conditions.
A positive attitude towards such factors, counter-intuitively, implies empathy and caring nature
of the person. It signifies compassion towards weaker and marginalised sections. This attitude
is utmost desirable for public servants because of the immense powers they exercise for the
inclusion or exclusion of the marginalised.
Significance for Civil servant
• A civil servant is duty bound to enforce laws and implement policies, rules and regulations
in a manner which is not only ‘just and fair’ but also seen to be so by maintaining
transparency and acting without fear and favour. This should be done in the true spirit of
‘rule of law’ on which rests the democratic polity.

DELHI JAIPUR PUNE HYDERABAD AHMEDABAD LUCKNOW CHANDIGARH


5 www.visionias.in # 8468022022 ©Vision IAS
Google it:- https://upscpdf.com
https://t.me/UPSC_PDF Download From > https://upscpdf.com https://t.me/UPSC_PDF

• He must be aware of the fundamental rights of the citizens enshrined in the constitution Student Notes:
and develop a proactive attitude of empathy to promote all round upliftment and
development of the weaker and downtrodden in the true spirit of ‘Sarvodaya through
Antyodaya’.
• Tolerance and compassion make a civil servant to lead with not only head but also heart.
They are the fundamental components of one’s character and of a positive relationship
which will is essential to meet the needs of weaker sections (who are most dependent on
public services).
For example, if a grievance redressal meeting was to be held for the differently abled in a
collectorate, an officer who has an empathetic attitude will ensure that proper arrangements
are made for accessibility such as having a ramp/elevator, etc.
Otherwise, there is no purpose of having such a meeting. Lack of this essential understanding
that develops through a positive attitude towards the weaker and marginalised sections leads
to inefficient and exclusionary public administration and service delivery, which may ultimately
lead to chaos and collapse.

2.2. Moral Attitude


Attitude, as defined earlier, is the enduring predisposition to behave, either favourably or
unfavourably, towards something. However, not every attitude is concerned with questions or
situations involving morality.
For e.g. a person’s liking or disliking towards apples or oranges has no question of morality
involved. But one may have moral considerations as far as being a vegetarian or a non-
vegetarian is concerned.
Similarly, a person may have a favourable attitude towards, say, transacting in cash rather than
electronically. There is nothing moral or immoral about it. However, if his motive to transact in
cash arises from his desire to hide his income from the government, then it has moral
connotation. Similarly, attitude towards democracy or say towards weaker sections will have
moral undertones.
Moral attitudes can, thus, be defined as those which are based on moral convictions of what is
“right” and what is “wrong”. It implies one’s reasoning about morality, his attitude toward
moral lapses (both on his part and by others too), and his behaviour when faced with moral
issues. Family, society, religion and education play important role in framing those moral
convictions.
How are moral attitudes shaped?
The determinants of the attitude are the same- Cognitive, emotional (affective) and
behavioural:
• Cognitive: It is the knowledge of ethical rules and judgments of what is good and what is
bad.
• Behavioural: It is the person’s actual behaviour, his response to situations involving ethical
considerations.
• Emotional: it involves the person’s feelings and conduct in reaction to situations that need
moral and ethical decisions.
Moral attitudes are strongly influenced by the society and culture. Religious beliefs, traditions,
folklore, myths, legends – all have an implicit messaging in them- about what is good and what
is bad. As such, they shape moral attitudes of people.
As such, moral attitudes vary over time and space. Similarly, they can also vary with
gender. Men, for example, may have a less negative attitude towards bribery than women.
Similarly, women may have more open attitude towards freedom to wear clothes of choice.

DELHI JAIPUR PUNE HYDERABAD AHMEDABAD LUCKNOW CHANDIGARH


6 www.visionias.in # 8468022022 ©Vision IAS
Google it:- https://upscpdf.com
https://t.me/UPSC_PDF Download From > https://upscpdf.com https://t.me/UPSC_PDF

There are four qualities that are generally associated with having a moral attitude: Student Notes:
1. Reverence: It means deep respect. Having respect for others, their opinions and behaviour
is considered as a hallmark of a moral person.
2. Faithfulness: It means remaining loyal and upholding trust that someone poses in a person.
3. Veracity or truthfulness: Being truthful and honest is another quality associated with being
moral.
4. Goodness: It is a broadly encompassing character of a person consisting of qualities like
generosity, love, care, empathy, etc.
Significance or impacts of Moral attitudes
Human beings have a desire to be right and a desire to be liked. As such, attitudes towards
moral issues tend to be strong and are also expressed strongly. For example, a person may have
a very strongly negative attitude towards dishonesty or telling lies, so much so that he/she is
ready to risk personal relations for sake of being truthful.
Moral attitudes can be both facilitative and prohibitive. They facilitate actions such as helping
someone in need (altruism), social service, etc. Also, actions that are considered immoral are
discouraged such as adultery, stalking, cheating, etc.
As attitudes are closely linked to a person’s behaviour, moral attitudes help in facilitation of
moral behaviour as they help in arriving at morally correct decisions.

2.3. Political Attitude


Political attitude is the predisposition or liking/disliking towards a political issue. The way in
which we define an issue to be political can be diverse.
1) In a simpler sense, political attitude refers to attitude of people towards the political
system, parties or their ideology. A person may identify himself to be conservative, liberal,
centrist, or so on. Similarly, a political party may identify itself as such. However, these
attitudes are too vague to be analysed. Having a positive or negative attitude towards, say
presidential system or parliamentary system or dictatorship is too broad a category, and
thus vague.
2) In a wider sense, political attitude means attitude towards specific issues of public life. It
is better to delineate attitudes towards specific issues such as economy, employment,
women, inequality, caste system, voting pattern, etc. rather than combining them into one
broad category.
For example, a person associating himself/herself with a so called conservative party may
well have a tolerant attitude towards people of differing ideology. In fact, political parties,
especially in India, cannot be compartmentalised into right or left based on western
construct. No political party in India can take the position which is, even perceptibly, anti-
farmer or anti-labour. Hence, it is better to study political attitudes with regards to specific
issues rather than broad categories.
Various ways through which political attitudes are manifested– voting, social media posts,
newspapers articles, sloganeering.
Significance of Political Attitude
• It is political attitudes that determine how people participate in the political process, whom
they vote for, and what political parties they support. Many factors including - family,
religion, caste, ethnicity, and region – all contribute to the political attitudes and
behaviour.
• It has been argued that the development of political judgment represents a part of moral
development and that political and moral education is largely identical, especially when
seen from wider perspective.
• From this perspective, political culture determines the system of values in the society.
Whereas, from narrow connotation, political culture is a subjective psychological

DELHI JAIPUR PUNE HYDERABAD AHMEDABAD LUCKNOW CHANDIGARH


7 www.visionias.in # 8468022022 ©Vision IAS
Google it:- https://upscpdf.com
https://t.me/UPSC_PDF Download From > https://upscpdf.com https://t.me/UPSC_PDF

phenomenon that appears in the process of interaction between individuals and the Student Notes:
political system.

2.4 Prejudice
Prejudice is a baseless and often negative preconception or Attitude toward members of a
group. it involves prejudgments that are usually negative about members of a group.
Prejudice can have a strong influence on how people behave and interact with others,
particularly with those who are different from them, even unconsciously or without the person
realizing they are under the influence of their internalized prejudices.
When people hold prejudicial attitudes toward others, they tend to view everyone who fits
into a certain group as being "all the same." They paint every individual who holds particular
characteristics or beliefs with a very broad brush and fail to really look at each person as a
unique individual.
Prejudice can translate into discrimination which has behavioral component –discrimination
in context of race caste class Gender are the most prevalent.

Some of prejudicial attitude and examples


Gender-Prejudicial attitude against women India
• The social stigma around working women, especially when the family does not desperately need
the money.
• Another area where discrimination and prejudice against women is evident is during meals.
Generally, women eat at last or in rural areas they eat whatever Is left. This discrimination result
into problem of malnutrition. Which again is root cause of many other social evils
• Third kind of prejudicial attitude with respect to women is the practice of ghunghat, or a Hindu
woman covering her head and face when in public or in the presence of men.
Racism –Many incidents of violence against the North eastern citizen and the Black especially from the
African countries are reported. Which is manifestation of Racial discrimination.
Caste discrimination –
• Many Incidents of Dalit students eating mid-day meals separately from other students at a primary
school.
• A Dalit groom in Rajasthan’s Madhopura village was prevented by members of the Jat community
from sitting on the mare, citing religious sentiments.
• Two Dalit men in the southern state of Tamil Nadu were killed by upper-caste Hindus after a Dalit
man sat cross-legged in front of them during a temple ritual

How the discriminatory behavior be curbed?


• Discriminatory behavior can be curbed by the law, but the cognitive and emotional
component are difficult to change. Hence, specific strategies required
• Knowing the cause and the source of the prejudice is important to curb the discriminatory
nature of prejudice
• Strategies will be effective if they aim at minimizing opportunities for learning prejudices
change such attitudes.
• De-emphasing narrow social identity. Individuals rather than considering themselves as
part of one social group must consider them as part of greater social identity like an Indian.
• Discouraging the tendency towards negative behavior among the victims of prejudice
which reinforces the Prejudice.
Way Forward
• Education and information dissemination for correcting stereotypes related to specific
target group.
• Increasing the inter group contact, which allows for direct communication.

DELHI JAIPUR PUNE HYDERABAD AHMEDABAD LUCKNOW CHANDIGARH


8 www.visionias.in # 8468022022 ©Vision IAS
Google it:- https://upscpdf.com
https://t.me/UPSC_PDF Download From > https://upscpdf.com https://t.me/UPSC_PDF

• Removal of mistrust that might lead to discovering positive qualities of the group. Student Notes:
• Highlighting the individual identity rather than group identity, thus, weakening the
importance of the Group as the basis of evaluating an individual.

3. Attitude Formation
Attitude is learned, formed, can be changed, and be reformed. Learning can account for most
of the attitudes we hold. The study of attitude formation is the study of how people form
evaluations of persons, places, things, objects, matters and issues.
Unlike personality, attitudes are expected to change as a function of experience. In addition,
exposure to the 'attitude' objects may have an effect on how a person forms his or her attitude.
This concept was seen as the "Mere-Exposure Effect" (The mere-exposure effect means that
people prefer things that they are most familiar with).

According to some experts, people are more likely to have a positive attitude on 'attitude
objects' when they were exposed to it frequently than if they were not. Mere repeated
exposure of the individual to a stimulus is a sufficient condition for the enhancement of his
attitude toward it.
Attitude formation or learning is a lifelong process, as it is based on experiences we gather or
the lessons we learn from people around us. These people are the agencies of attitude
formation-

DELHI JAIPUR PUNE HYDERABAD AHMEDABAD LUCKNOW CHANDIGARH


9 www.visionias.in # 8468022022 ©Vision IAS
Google it:- https://upscpdf.com
https://t.me/UPSC_PDF Download From > https://upscpdf.com https://t.me/UPSC_PDF

Student Notes:

4. Attitude Change
It refers to the process of modifying the undesirable attitude into a desired one. Here it must be
clear that desirable and undesirable are not value judgements about right or wrong. Desirable
for someone is good for them but may be considered bad by others.
Attitudinal change means changing someone else’s perception of what is right or wrong
according to our will. For example, Person X likes Windows operating system in laptop more
than Linux. Person Y wants to change the attitude of X because Y himself prefers Linux. This
does not say anything about Linux being better than Windows (i.e. a value judgement). This is
only a desire of Y. For him preference to Windows is undesirable, and therefore he desires to
change the attitude of X to his own liking. Since attitudes are learned, they can be unlearned,
relearned or changed too. Attitudes can be changed through persuasion or social influence.

4.1. Persuasion
It means the action or process of persuading someone or of being persuaded to do or believe
something. Persuasion is an umbrella term of influence, which can influence a person's beliefs,
attitudes, intentions, motivations, or behaviours.
It is a process aimed at changing a person's (or a group's) attitude or behaviour towards some
event, idea, object, or other person(s), by using written or spoken words to convey
information, feelings, or reasoning, or a combination thereof.

DELHI JAIPUR PUNE HYDERABAD AHMEDABAD LUCKNOW CHANDIGARH


10 www.visionias.in # 8468022022 ©Vision IAS
Google it:- https://upscpdf.com
https://t.me/UPSC_PDF Download From > https://upscpdf.com https://t.me/UPSC_PDF

It is also an often used tool in the pursuit of personal gain, such as election campaigning, giving Student Notes:
a sales pitch, or in trial advocacy. It can also be interpreted as using one's personal or positional
resources to change people's behaviours or attitudes.
Persuasion can be classified as-
• Systematic persuasion is the process through which attitudes or beliefs are leveraged by
appeals to logic and reason.
• Heuristic persuasion on the other hand is the process through which attitudes or beliefs
are leveraged by appeals to habit or emotion.
Formation and change of attitude are not two separate things - they are interwoven. People
are always adopting, modifying or relinquishing attitudes to fit their ever changing needs and
interests.
Acceptance of new attitudes depends on who is the communicator, how the communication is
presented, how the communication is perceived by the message receiver, the credibility of the
communicator, and the conditions under which the knowledge was received.
Attitudes change can manifest itself as:
• A person receiving new information from others or media - Cognitive change
• Through direct experience with the attitude object - Affective change
• Force a person to behave in a way different than normal - Behavioural change

4.1.1. Functions that can be performed with Persuasion


The persuader needs to select a purpose that is realistic for his/her audience. Five general
purposes of persuasion are listed below.
1. Create uncertainty: When an audience is strongly opposed to the persuader’s view, the
best that may be possible for the persuader is to make the audience a little less certain
they are right, a little less comfortable with their current attitude.
2. Reduce resistance: If the audience is moderately opposed to the persuader’s position
but not closed-minded, the persuader may be able to reduce opposition to his/her view
and move the audience toward neutrality. While not expecting a reversal of views this
goal asks the audience to recognize the validity of opinions different from their own.
3. Change attitude: If the audience is not committed, especially strongly, to any attitude on
the topic this goal is appropriate.
4. Amplify attitude: If the audience is already moderately favorable to the persuader’s
view, he/she can design a message which will reinforce current attitudes in the audience,
help the audience resist appeals from opponents, and motivate the members of the
audience to become strongly committed to his/her position.
5. Gain behavior: When an audience strongly favors the persuader’s position, the logical
goal is to get them to act on their convictions.

DELHI JAIPUR PUNE HYDERABAD AHMEDABAD LUCKNOW CHANDIGARH


11 www.visionias.in # 8468022022 ©Vision IAS
Google it:- https://upscpdf.com
https://t.me/UPSC_PDF Download From > https://upscpdf.com https://t.me/UPSC_PDF

We can further see the importance of message characteristics in the following way- Student Notes:
The success of any programme depends upon the level of involvement of all the relevant
stakeholders. Use of socially and culturally identifiable names gives a clear message of the
objectives of the programme – one of the principles of behavioural economics is that the
messaging needs to be clear and simple and aligned to a mental model.
This message is also manifested by appropriately using the power of name, as follows-

4.1.2. Steps in Persuasion Process


1. Establish credibility: Credibility grows out of expertise and relationships. A persuader needs
strong emotional characteristics and integrity. The need to listen carefully to other people’s
suggestions and establish an atmosphere where their opinions are valued.
2. Framing common goal with colleagues: Effective persuader must be adept at describing
the position in terms that illuminate the person’s point advantages that he/ she is trying
to persuade. It is a process of identifying shared benefits. This requires conversations to
collect essential information by asking thoughtful questions. This process will often prompt
to alter the initial argument or include compromises.
3. Reinforce positions with vivid language and compelling evidence: Persuasion requires
presentation of evidence — strong data in multiple forms (stories, graphs, images,
metaphors and examples). Persuaders need to make positions come alive by using vivid
language that complements graphics.
4. Connecting emotionally with audiences: Good persuaders are aware of primacy of
emotions and are responsive to them. They know how to maintain a balance between
professionalism and their own emotional commitment to the position they are advocating.
Their connection to their audience demonstrates both intellectual and emotional commitment
to their position. Successful persuaders cultivate an accurate sense of their audience’s
emotional state, and they adjust their arguments accordingly. Whatever their position, they
must match their emotional fervor to their audience’s ability to receive their message.
4.1.3. Effective Persuasion
Everyone is susceptible to being persuaded; persuasion is a process whose objective is to
change a person's attitude and/or behaviour towards an idea, event, person or an object.
Broadly speaking effective persuasion should have desirable source (having credibility),

DELHI JAIPUR PUNE HYDERABAD AHMEDABAD LUCKNOW CHANDIGARH


12 www.visionias.in # 8468022022 ©Vision IAS
Google it:- https://upscpdf.com
https://t.me/UPSC_PDF Download From > https://upscpdf.com https://t.me/UPSC_PDF

desirable message characteristics (having fear, rational, and emotional appeals). In more detail, Student Notes:
persuasion to be effective must have following things:
1. Establish a common ground: The persuader should establish positive rapport with target
people. For example-Sabka Sath Sabka Vikas. Slogan used by Honorable prime minter to
Unite and establish Common ground
2. Point out the benefits: Persuader should highlight the major benefits of changed behavior
or attitude. However, the persuader should avoid trying to push for the change, as it will
make him look desperate.
3. Turn objections into strengths: Objections to change are natural but the persuader should
turn them into opportunities. For this he may agree with the prospect's objection and then
illustrate how it is easily overcome by the proposed change.
4. Commitment and consistency: Persuader should try to get the target section (prospect) to
believe in something small or take a small action first. Once committed, the prospect will
most likely agree to a larger idea later. This technique employs the fact that people tend to
behave in a consistent manner, once they take a stand, they will act in ways consistent to
the decision as a means of defending and justifying it. Rationality commands an innate
appeal to the human mind.
5. Use the reciprocity principle: The principle implies that when someone does something for
us, we feel obliged to return the favour. This may include appropriately rewarding the target
population for the changed behaviour/attitude. This helps in strengthening and sustaining
the change. For example –Just before election many political parties Before the end of
tenure release a lot of vacancy thus inciting the principle of reciprocity. Believing that youth
will vote for another tenure. Banning of liquor in several states to collect women votes.
6. Social proof technique: People tend to follow others (bandwagon effect) more so when
they don't have sufficient information to make the decision on their own. This technique
will involve you telling the target population that other people are getting benefits from the
suggested change, with empirical evidence. For this the persuader may invoke the examples
of some well-known personality. For example, in campaigning against female feticide in
Haryana we may invoke the
examples of some female
sportspersons who have won
laurels, S. Nehwal in Badminton,
or Kangana Ranaut in Bollywood
etc.
7. Scarcity: This involves letting
people know that they stand to
lose on a chance to get the
benefits out of the proposed
change. For example we often
see end of season or hoardings
like Hurry!!Limited offer.

4.2. Social Influence


Social influence occurs when one's emotions, opinions, or behaviours are affected by others.
Social influence takes many forms and can be seen in conformity, socialization, peer pressure,
obedience, leadership, persuasion.
Social influence can be defined as a change in behaviour caused by real or imagined influence
from others. The most effective social influence attempts to succeed in changing a person’s
attitudes and behaviour. But changing someone’s attitude is not necessary for social influence
to occur; all that’s required is behaviour change. Following are the three broad varieties of
social influence.

DELHI JAIPUR PUNE HYDERABAD AHMEDABAD LUCKNOW CHANDIGARH


13 www.visionias.in # 8468022022 ©Vision IAS
Google it:- https://upscpdf.com
https://t.me/UPSC_PDF Download From > https://upscpdf.com https://t.me/UPSC_PDF

• Compliance is when people appear to agree with others, but actually keep their dissenting Student Notes:
opinions private. It is a change in behaviour but not necessarily attitude.
• Identification is when people are influenced by someone who is liked and respected, such
as a politician, guru, celebrity.
• Internalization is when people accept a belief or behaviour and agree both publicly and
privately.
Conformation
Why do we conform or attempt to conform to the expectation of others? There are two
psychological needs of human at play – our need to be right and our need to be liked. The
former is also called informational social influence and the latter is referred to as normative
social influence.
Informational influence (or social proof)-
• When a person is in a situation where they are unsure of the correct way to behave, they
will often look to others for clues concerning the correct behaviour.
• We conform because we believe that others' interpretation of an ambiguous situation is
more accurate than ours and will help us choose an appropriate course of action. It is an
influence to accept information from another as evidence about reality.
Social proof often leads not only to public compliance (conforming to the behaviour of others
publicly without Necessarily believing it is correct) but also private acceptance (conforming out
of a genuine belief that others are correct).
Social proof is more powerful when being accurate is more important and when others are
perceived as especially knowledgeable. For example The Swachh Bharat Mission (SBM) and
the Beti Bachao, Beti Padhao (BBBP) scheme are cases in point..People are more likely to stop
defecating in the open if their neighbours stop or are more likely to value their girl children if
that is touted as the social norm.
Normative influence
• Normative Influence is related with the need of an individual to be liked by others.
Human beings, being inherently social, desire companionship or associations. A group or an
association consists of people with some common interest. For a successful and healthy
atmosphere in the group, people try to blend in.
• They change their behaviour somewhat so that they are liked. This is a normative social
influence- normative meaning that how things ‘should be’, for e.g. Parents desire that
children should stay away from mobile phones. Hence, it is an influence to conform to the
positive expectations of others.
4.2.1. Summary of goals of social influence
To Choose Correctly
People often rely on two principles to help them choose correctly: authority and social
validation. Thus, they are more willing to be influenced by authority figures, on the one hand,
and similar peers on the other.
• Authority -One reason authorities are influential is that they are often experts, and, by
following an authority’s directives, people can usually choose correctly without having to
think hard about the issue themselves.
Social Validation
• Just as following an authority is normally a shortcut to choosing correctly, so is following
the lead of most of one’s peers. The choices of these others provide social validation for the
correctness of that choice.

DELHI JAIPUR PUNE HYDERABAD AHMEDABAD LUCKNOW CHANDIGARH


14 www.visionias.in # 8468022022 ©Vision IAS
Google it:- https://upscpdf.com
https://t.me/UPSC_PDF Download From > https://upscpdf.com https://t.me/UPSC_PDF

• People are most likely to allow themselves to be influenced by others when they are Student Notes:
uncertain about how to respond in the situation—because when uncertainty and
ambiguity reign, people lose confidence in their own ability to choose well.
• When others share a consensus about the correct way to act, they are especially influential
to observers.
• In addition, observers are more likely to be influenced by others who are similar to them
and who, therefore, provide better evidence about what the observers should do.
• When choosing accurately is important, only uncertain individuals are more likely to follow
the crowd; those who are already sure of the validity of their judgments are less willing to
conform.
To Gain Social Approval
• People change in order to be more accepted and approved by their groups and to avoid
the social rejection that often comes from resisting group pressure for change.
• Injunctive standards of a group or culture inform people as to the behaviours that are
likely to get them accepted or rejected there.
• One such norm is that for reciprocity, which obligates people to give back to those who
have given first. Anyone who violates this norm risks social disapproval and rejection, which
makes people more willing to comply with requests of those who have provided an initial
favour or concession.
• The desire for social approval and a collective self-definition both increase one’s willingness
to submit to social influence in order to gain acceptance. But a tendency to go against
conventional norms of behaviour or even rebelliousness decreases one’s susceptibility to
social influence, especially when the influence is seen as threatening one’s freedom to
decide.
• Two features of a person’s social situation increase the motivation to go along to get along:
the appeal of the group or individual pressing for change and the public observability of
the person’s actions.
• Even strong group norms can be resisted when members feel that they have the ability to
withstand group influence or when members don’t feel highly identified with the group.

DELHI JAIPUR PUNE HYDERABAD AHMEDABAD LUCKNOW CHANDIGARH


15 www.visionias.in # 8468022022 ©Vision IAS
Google it:- https://upscpdf.com
https://t.me/UPSC_PDF Download From > https://upscpdf.com https://t.me/UPSC_PDF

To Manage Self Image Student Notes:


• People can manage their self-images by yielding to requests for action that fits or enhances
their identities.
• Influence professionals can increase compliance by linking their requests to the values to
which people feel committed, especially when these values are prominent in
consciousness.
4.2.2 Successful case of the Beti Bachao Beti Padhao Scheme
The success of the BBBP Scheme demonstrates a powerful use of the insight on ‘social norm’ in
its 'Selfie with Daughter' initiative. This scheme was launched to address a highly imbalanced
child sex ratio in India. People’s attitude towards the girl child needed to change – people
needed to stop viewing girls as burdens and start celebrating them instead.
#SelfieWithDaughter- The selfie campaign showcased examples of parents around the country
who were doing exactly that. The celebration of the girl child quickly became the norm. Most
people wanted to conform, and more and more parents posted selfies with their girls. Started
by one proud father in a village in Haryana, the campaign went viral and #SelfieWithDaughter
became a worldwide hit.
Two elements enabled the campaign’s success: First, telling people what the norm is and
second, showcasing the thousands of other people who were acting in line with that norm.

The strategy addresses a cognitive bias called ‘failure bias’. The failure bias is the tendency to
focus on failures rather than successes, mostly because failures have greater visibility.
Because failures get the spotlight, people tend to think that failing is the norm, or at least that
failing is more prevalent than it really is. Therefore, in the context of BBBP, focus must be on

DELHI JAIPUR PUNE HYDERABAD AHMEDABAD LUCKNOW CHANDIGARH


16 www.visionias.in # 8468022022 ©Vision IAS
Google it:- https://upscpdf.com
https://t.me/UPSC_PDF Download From > https://upscpdf.com https://t.me/UPSC_PDF

people who treat their girls fairly; this corrects the failure bias and makes the social norm of Student Notes:
fair treatment of girls unequivocally clear.
BBBP’s work is far from over, of course, as posting a selfie is not tantamount to subverting
entrenched orthodox mind-sets overnight, but its leverage of social norms is certainly a step in
the right direction

4.3. Emotions and Attitude Change


Appeal to emotional aspect of person is also used as a tool for attitudinal change. In fact,
emotion is a common component in persuasion and social influence. Research on attitude has
also highlighted the importance of affective or emotive components of the message.
The ABC model of attitude emphasises the three components- cognitive (i.e. what we perceive),
affective (how we connect emotionally) and behavioural (how we act). Emotion works hand-in-
hand with the cognitive process, or the way we think, about an issue or situation. Emotional
appeals are commonly found in advertising, health campaigns and political messages.
Recent examples include No-smoking health campaigns and political campaign advertising
emphasizing the fear of terrorism. By activating an affective or emotion node, attitude
change may be possible, though affective and cognitive components tend to be intertwined.
Please note that emotional attitude is different from appealing to emotion for attitudinal
change. Emotional attitude is merely an attitude developed primarily by emotion, for e.g.
attitude of parents towards children.
Appeal to emotion is the method to develop a desirable attitude. Emotions such as fear, joy,
anger, empathy, ridicule, etc. Instead of facts, persuasive language is used to develop the
foundation of an appeal to emotion-based argument.
For example, to motivate someone to stay fit or to quit smoking, one should not only cite
scientific evidence to prove the point but can also convince using the fear of deadly diseases
or the joy of a healthy life.
However, if appeal to emotion is made using wishful thinking (i.e. something that is pleasing
to imagine but not based on evidence or fact), then it becomes a logical fallacy. Only a
temporary change in attitude can be achieved with wishful thinking or appealing to flattery or
to hatred.
In future this can also lead to development of attitudes counter to what was desired. Hence,
appeal to emotion alone cannot be a sustainable basis for attitudinal change. Appreciation of
facts gives legitimacy to message and thus is a more enduring way to change attitude.
4.3.1. Successful case of Swachh Bharat Mission
The main elements of Swachh Bharat Mission which led to attitude change are-
Choice of date and symbol
• SBM, as a nation-wide cleanliness drive, was launched on 2nd October, 2014, the birthday
of India’s most revered ‘role model’ Mahatma Gandhi.
• The day was chosen to leverage the values propagated by him and thereby create a mass
movement on the lines of ‘satyagraha’ for a cleaner India.
• The symbol used for SBM invokes Gandhiji’s ideas.Behavioural Science emphasises the role
of context in influencing choices and decisions, which has been effectively adopted by the
SBM campaign.
Local Swachagrahi
• To initiate behavioral change in usage of toilets, more than five lakh swachhagrahis, foot
soldiers of the SBM, were recruited; the similarity with satyagrahis is intentional to

DELHI JAIPUR PUNE HYDERABAD AHMEDABAD LUCKNOW CHANDIGARH


17 www.visionias.in # 8468022022 ©Vision IAS
Google it:- https://upscpdf.com
https://t.me/UPSC_PDF Download From > https://upscpdf.com https://t.me/UPSC_PDF

reinforce the message.As each village has at least one swachhagrahi, who is a local, these Student Notes:
swachhagrahis were able to leverage their social ties within their villages to effect change.
• People are more likely to listen to and emulate someone they know, which is why local
ambassadors of change are more effective in getting through to people than mass media
campaigns.
Community Based approach
• SBM relies on community-based approaches to sanitation. Behavior change techniques
such as Participatory Rural Appraisal and Community-led Total Sanitation induce people to
come together, appraise their community’s open defecation situation and plan the next
course of action.
• This makes sanitation a community-level concern rather than an obscure campaign of a
distant government. Non-conformers, therefore, find their act more visible to their
community. The fear of community scorn, or a desire to fit in, or both, have led many to
renounce open defecation.

Appealing to the Emotions


• SBM used yet another behavioural insight – that people internalize messages better when
these messages make them feel a certain way. Arcane concerns about hygiene and
disease appeal to few; it is natural that those who have defecated in the open all their lives
without consequence would fail to absorb the message that open defecation can have
deleterious effects.
• On the other hand, appealing to people’s emotions, for example by attaching a sense of
disgust to open defecation, has a better chance of moving people to change.

DELHI JAIPUR PUNE HYDERABAD AHMEDABAD LUCKNOW CHANDIGARH


18 www.visionias.in # 8468022022 ©Vision IAS
Google it:- https://upscpdf.com
https://t.me/UPSC_PDF Download From > https://upscpdf.com https://t.me/UPSC_PDF

Student Notes:

4.4. Consequences of Social Influence or Persuasion


The outcomes of persuasion or social influence could be good, bad or ugly depending on the
modes of tactics, motives, and contexts in which they are employed. These are discussed
below:
4.4.1. Ugly Face of Persuasion
Ugly influencers Push and shove others into decisions. Their style leaves others feeling
powerless and resistant to innovation or change. It refers to situation where the motive of
persuasion may be utterly selfish. For example, the miss-selling of financial instruments, or
duping the customers by making false claims.
4.4.2. Bad Persuasion
Bad influencers might work hard to achieve legitimate and desirable goals, but lack the skills
to influence effectively. Their style causes people to feel they are being punished or cutting
through red tape, all to please someone who appears ineffective. In this case the motive is
genuine but the means are ineffective. For example, promoting family planning with forced
sterilisation, as happened during emergency period.

DELHI JAIPUR PUNE HYDERABAD AHMEDABAD LUCKNOW CHANDIGARH


19 www.visionias.in # 8468022022 ©Vision IAS
Google it:- https://upscpdf.com
https://t.me/UPSC_PDF Download From > https://upscpdf.com https://t.me/UPSC_PDF

4.4.3. Good Persuasion Student Notes:


Good influencers get people to focus on an issue that is clearly and simply stated, finds out
what the emotional value of the issue is to the people involved, and seeks solutions that
satisfy the people who are needed to make the solution work.
Good influencers are effective because they create trust, which enables others to take risks.
Their habit of communicating, informing and including others builds loyalty among the target
population. They effectively use various kinds of appeal- rational, emotional, and fear.
For example, changing attitude against Untouchability should include invoking reason,
emotional appeal and fear of law.

4.4.4. Persuasion vs. Manipulation


The difference between persuasion and manipulation lies largely in underlying intent and desire
to create genuine benefit. The difference between Persuasion and Manipulation lies in:
• The Intent behind your desire to persuade that person,
• The Truthfulness and transparency of the process, and
• The Net benefit or impact on that person
Manipulation has negative connotation. It implies Persuasion with the intent to fool, control or
contrives the person on the other side of the conversation into doing something, believing
something, or buying into something that leaves them either harmed or without benefit.
It may also imply that you are concealing a desire to move them to your point of view in a way
that will benefit only you. And if this benefit were disclosed, that revelation would make the
other person far less receptive to your message.

DELHI JAIPUR PUNE HYDERABAD AHMEDABAD LUCKNOW CHANDIGARH


20 www.visionias.in # 8468022022 ©Vision IAS
Google it:- https://upscpdf.com
https://t.me/UPSC_PDF Download From > https://upscpdf.com https://t.me/UPSC_PDF

5. Attitude’s Relation with Thought and Behaviour Student Notes:

5.1. How does Attitude Influence our Thought and Behaviour?


An attitude is "a relatively enduring organization of beliefs, feelings, and behavioural tendencies
towards socially significant objects, groups, events or symbols". Since, attitude also includes
pre-disposition to behave in certain ways, therefore our attitude can be regarded as the
predictor of our behaviour in relation to the attitude object.
This kind of enquiry includes to what extent, and under what conditions, do attitudes drive our
outward actions? What is the relationship between what we are (on the inside) and what we do
(on the outside)?
In the early days of attitude research, most investigators accepted as a given that human
behaviour is guided by social attitudes. It was assumed that attitude was the key to
understanding human behaviour. Moreover, early work with the attitude gave no reason to
doubt this assumption. Yet some researchers challenged this view. They argued that the
attitude–behaviour relation works the other way around, with our behaviour as the horse and
our attitudes as the cart and that people’s expressed attitudes hardly predicted their varying
behaviour.

Key Study: La-Pierre (1934)


Aim: To investigate the relationship between attitudes and behavior.
Method: La-Pierre travelled round American hotels with a Chinese couple, expecting to meet
discriminatory behavior, because of anti-Chinese feeling prevailing at that time. At the time prejudice
against Asians was widespread and there were no laws against racial discrimination. They visited 67
hotels and 184 restaurants. Six months later, after their return, all the establishments they had visited
were sent a letter, asking whether they would accept Chinese guests.
Results: They were only refused at one of the establishments they visited, and were generally treated
very politely. Of the 128 establishments which responded to the letter, 91% said they were not willing to
accept Chinese guests.
Conclusion: Attitudes do not always predict behavior. Cognitive and affective components of attitudes
are not necessarily expressed in behavior. Thus, the La-Pierre’s study shows that the cognitive and
affective components of attitudes (e.g. disliking Chinese people) do not necessarily reflect in their actual
behavior (e.g. serving them).
Thus, it is not true that attitude will always predict the behavior. According to some experts, it depends
on the strength of the attitude held.

Attitude Strength
The strength with which an attitude is held is often a good predictor of behavior. The stronger
the attitude the more likely it should affect behavior. Attitude strength involves:
Importance / personal relevance refers to how significant the attitude is for the person and
relates to self-interest, social identification and value. If an attitude has a high self-interest for a
person (i.e. it is held by a group the person is a member of or would like to be a member of, and
is related to a person's values), it is going to be extremely important. As a consequence, the
attitude will have a very strong influence upon a person's behavior. By contrast, an attitude
will not be important to a person if it does not relate significantly to their life.
The knowledge aspect of attitude strength covers how much a person knows about the attitude
object. People are generally more knowledgeable about topics that interest them and are likely
to hold strong attitudes (positive or negative) as a consequence.
Attitudes based on direct experience are more strongly held and influence behavior more
than attitudes formed indirectly (for example, through hear-say, reading or watching
television).

DELHI JAIPUR PUNE HYDERABAD AHMEDABAD LUCKNOW CHANDIGARH


21 www.visionias.in # 8468022022 ©Vision IAS
Google it:- https://upscpdf.com
https://t.me/UPSC_PDF Download From > https://upscpdf.com https://t.me/UPSC_PDF

However, even strength of attitude alone can’t be the accurate predictor of our behavior, Student Notes:
therefore some experts have suggested the following factors responsible for determining the
attitude-behavior link. These are:
Accessibility:
It refers to the easy availability of the attitude to the mind of the attitude holder. It means
that the attitude holder is conscious of the “ACB” components
(Affection|Cognitive|Behavioural) of the attitude and therefore they immediately come to the
mind of the attitude holder.
Specificity:
Most people are not sure about their likings/dislikings about many things. If we do not have a
specific attitude towards something, it is prone to change, i.e. it is as yet weakly formed and
can be changed with little effort such as availability of new knowledge. Specificity here means
how specific an attitude do we have towards something. Having hazy or multiple attitudes
towards the something reduces their specificity. For example, political attitude towards an issue
can be fuzzy and change according to the group that we are in. This leads to a somewhat
confused behavior in that situation.
Congruence:
Continuing from specificity, congruence refers to the consistency among different components
of our attitude. If the knowledge component and affective component are inconsistent then our
behaviour would depend more on the situation than on attitude per se. The Mahabharata talks
about ‘Manasa, vacha, karmana’. The word manasa refers to the mind, vaachaa refers to
speech, and karmanaa refers to actions. These are used to describe the state of consistency in
behaviour of an individual. The motto ‘Manasa, Vaacha, Karmana’ is usually invoked to imply
that one should strive to achieve the state where one's thoughts, speech and the actions
coincide.

5.2. When Does Attitude Predict Behaviour?


In the earlier section we looked at how attitude influences our behaviour. Here we look at how
does attitude predict behaviour, i.e. what are the situations where we can predict a person’s
behaviour having the knowledge about his/her attitude. Researchers have discovered that
people are more likely to behave according to their attitudes under certain conditions. This is
particularly so when:
• Attitudes are a result of personal experience
• when one expects a favorable outcome
• when the attitudes are repeatedly expressed
• when one stands to win or lose something due to the issue under consideration
• when situational factors that contradict our attitudes are weak
• when we are aware of them, and
• When they are strongly held.
Our behaviour and our expressed attitudes differ because both are subject to other influences.
But if we could just neutralize the other influences on behaviour—make all other things equal—
then attitudes can predict behaviour fairly accurately. The following points explain in detail the
conditions under which people are more likely to behave according to their attitudes.
5.2.1. Strength of Attitude
One important condition for high attitude-behaviour consistency is that the attitude be strong
and clear. Strength of attitudes depends upon: personal implications; formed through direct
experience; highly embedded attitudes, that is, they are tied to other beliefs that people hold.

DELHI JAIPUR PUNE HYDERABAD AHMEDABAD LUCKNOW CHANDIGARH


22 www.visionias.in # 8468022022 ©Vision IAS
Google it:- https://upscpdf.com
https://t.me/UPSC_PDF Download From > https://upscpdf.com https://t.me/UPSC_PDF

5.2.2. Stability of the Attitude Student Notes:


Stable attitudes that are easily remembered are more likely to predict behaviour than attitudes
that are less stable and not accessible in memory.
5.2.3. Accessibility of the Attitude
Attitudes that are more accessible to memory influence behaviour more strongly. A primary
factor that determines whether an attitude is accessible in memory is how frequently it is
expressed. Attitudes also become more extreme when they are expressed more frequently.
That is, the more opportunities you have to express an attitude, the more you come to regard
that attitude as important to you.
5.2.4. Salience of the attitude
Salience is the quality of being particularly noticeable or important, i.e. prominent. In most
situations, several different attitudes may be relevant to behaviour. Salience is particularly
crucial when the attitude is not a very strong one.
Saliency refers to the fact that not all of a person's beliefs stand out with equal prominence in
his cognitive field. He may be more acutely aware of certain of his beliefs than others, they may
enter his thoughts more readily, they may be more frequently verbalized--they are, in a word,
salient.
Suppose a person is asked to name the things that "come to mind when you think about
buying…”. The order in which a person names those things reflects the salience. Please note
that salience is not perfectly synonymous with importance. But salience of attitude towards a
particular object is a considered a decent predictor of behaviour.
5.2.5. Affective vs. Cognitive Aspects of an Attitude
Some attitudes depend heavily on cognitions to back them up, that is, beliefs about the attitude
object. Other attitudes are more affectively based, dependent on the positive or negative
feelings or emotions that a person associates with an attitude object.
Making the affective component of the attitude more salient increases the influence of the
affective component over behaviour, whereas making the cognitive component more salient
makes the cognitive component the stronger determinant of behaviour.
However, when the cognitive and affective components of an attitude are consistent with each
other, it does not matter which is made more salient: both will be highly correlated with the
behaviour when either is made salient.

5.3 Why it is difficult to change behaviour in Indian society


• India has one of the oldest cultural traditions in the world. Over centuries, Indians have
developed many strong beliefs and practices. For Example, due to a deep-rooted affinity
for cleanliness, Indians are largely uncomfortable about having a place of defecation close
to the kitchen.
• Hierarchy is an integral facet of Indian society. Age-old caste systems to present-day factors,
such as income and education levels, have reinforced the hierarchical structure.
• In many cases, wrong behavior is now the social norm. The normal human tendency is to
follow the behavior of the majority. So, most behavioral change initiatives in India, which
will involve establishing a minority behavior as a social norm, is not going to be an easy
task.
• The biggest barrier to behavioral changes in India is that the common citizen does not have
an emotional connection with the chief change agent—the government. Governments are
considered corrupt and inefficient. Many citizens do not consider cheating the government
a mistake because in their minds, cheating a corrupt person is justified.

DELHI JAIPUR PUNE HYDERABAD AHMEDABAD LUCKNOW CHANDIGARH


23 www.visionias.in # 8468022022 ©Vision IAS
Google it:- https://upscpdf.com
https://t.me/UPSC_PDF Download From > https://upscpdf.com https://t.me/UPSC_PDF

5.4. Future applications of behaviour change Student Notes:

5.4.1. Transforming Gender Equations: From BBBP to BADLAV (Beti Aapki Dhan
Lakshmi Aur Vijay Lakshmi)
• While the BBBP campaign has helped, gender inequality needs a revolutionary campaign.
Our scriptures worship women as the embodiment of Shakti and exhort, that societies
where women are respected prosper.
• Campaign must draw on cultural and social norms because they affect behaviour so
crucially in India.
• Therefore, this campaign can be labeled BADLAV (Beti Aapki Dhan Lakshmi Aur Vijay
Lakshmi) to represent the 'change' towards gender equality. By drawing on the imagery of
the forms of Goddess Lakshmi that symbolises wealth (Dhan Lakshmi) and victory (Vijay
Lakshmi), the message of treating women as the forms of Lakshmi needs to be emphasized
5.4.2. From Swachh and Ayushman Bharat to Sundar Bharath
A strong way to reinforce behaviour is by getting people to pre-commit to a certain course of
action.
Reflection session
• SBM swachhagrahis may use this strategy to make people pre-commit to sanitation goals.
Further, the swachhagrahis may also assist people in assessing themselves periodically, say
once a month.
• An assisted Rflection session in the community or on a one-on-one basis with the local
swachhagrahi can prompt people to think about whether they acted as planned and how
many times they detracted.These reflection sessions should culminate in a commitment
about how people plan to act in the near-term future – whether they will refrain from open
efecation next month or not.
Making people to realize Tangible outcome
• A swachhagrahi may disseminate information about the incidence of sanitation-related
illness in her village and how this rate has improved after the adoption of sanitation
practices.
• When people realize the tangible outcomes of their actions, they are more likely to
sustain their behaviour. This is of tremendous importance for SBM because it relies on a
sustained change in behaviour, not a one-time change. To drive the point
home,swachhagrahis may also help individual households reflect on the incidence of illness
in their own families in, say, the last six months or since the time they quit open
defecation.
• If people find that their health outcomes are better after adopting the new practice, they
are likely to persist in that practice.

5.5. Let’s apply what we have learnt!!!!!!!!!!


5.5.1. Case 1
You are the engineering manager for the Road Commission with a primary responsibility for
district road safety. At least 1 person has died along a given stretch of road each of the last 7
years by crashing into a roadside tree. Many other accidents have occurred. Two law suits
were filed regarding the unsafe segment of the road, but were dismissed because the drivers
were exceeding the 45 mph speed limit. You recommend that the road be widened, resulting
in cutting down ~30 old trees. Environmental groups protest, and file a petition signed to 150
people to save the trees. Public debate falls on both sides of the issue.
“Discuss how you will proceed at this point.”

DELHI JAIPUR PUNE HYDERABAD AHMEDABAD LUCKNOW CHANDIGARH


24 www.visionias.in # 8468022022 ©Vision IAS
Google it:- https://upscpdf.com
https://t.me/UPSC_PDF Download From > https://upscpdf.com https://t.me/UPSC_PDF

Societal values: A variety of societal values pertain to this situation. Society values human life, Student Notes:
so this would favour widening the road and sacrificing the trees. Society also values following
the law. Since it can be argued that people in the accidents were speeding, and therefore
violating the law, perhaps leaving the road is fine.
Steps that I will follow
1. First, I would hold a public meeting to inform public. It should be noted that minimum no
of people on the petition is a small fraction of persons affected (up to 60,000 in town, so
there could be a silent majority in favor of widening the road). Explain why technically the
best solution is to widen the road.
2. The district should widen the road and remove trees from the right-of-way for optimal
safety.
3. The district should consider relocation of the displaced trees to other public property (a
park, etc).if possible and plant double the number of trees.
This solution accommodates both public safety which is a human ethics and environmental
preservation, part of environmental ethics.
5.5.2. Case 2
In the past month one of your employees has shown a major decline in performance when
dealing with citizen redressal. Though this decline has been on-going for the past six months it
was particularly sharp during the course of the last month and you receive a lot of complaints In
addition, she started coming in late, seeming very frustrated with her work. Her frustration is
influencing the atmosphere in the office as she is a popular employee and has been working for
the company for the past two years.
The following are the course of Action that will be taken by anyone depends on his
personality/character and values he holds and organisational Values. What will you choose
and justify the answer.
1. I call her for a talk and try to get down to the source of the problem. I explain that this
behavior is bad for all who are involved, including her. I express my true will to help her
with this rough patch in the condition that she works with me and not against me.
2. Demotion is the most appropriate solution. I replace her and assign her with back office
tasks. Employees are being measured by accomplishments and if she doesn't make the
requirements then I don't have a choice. In addition she is a respected worker in the staff
and I have to stop her from deteriorating all the other employees.
3. I call for a staff meeting to talk about the negative atmosphere and make sure that the
employee is not there so that there won't be any rejections.
4. She has been working for the past two years. Everyone has periods that are not the best
and part of my job is to be sensitive when it is called for and "cut her some slack" on this
one.

6. UPSC Previous Years Questions


1. What factors affect the formation of a person's attitude towards social problems? In our
society, contrasting attitudes are prevalent about many social problems. What contrasting
attitudes do you notice about the caste system in our society? How do you explain the
existence of these contrasting attitudes? (2014)
2. Two different kinds of attitudes exhibited by public servants towards their work have been
identified as bureaucratic attitude and the democratic attitude. (2015)
3. Distinguish between these two terms and write their merits and demerits. (2015)
4. Is it possible to balance the two too create a better administration for the faster
development of our country? (2015)

DELHI JAIPUR PUNE HYDERABAD AHMEDABAD LUCKNOW CHANDIGARH


25 www.visionias.in # 8468022022 ©Vision IAS
Google it:- https://upscpdf.com
https://t.me/UPSC_PDF Download From > https://upscpdf.com https://t.me/UPSC_PDF

5. How could social influence and persuasion contribute to the success of Swachh Bharat Student Notes:
Abhiyan? (2016)
6. Our attitudes towards life, work, other people and society are generally shaped
unconsciously by the family and the social surroundings in which we grow up. Some of
these unconsciously acquired attitudes and values are often undesirable in the citizens of a
modern democratic and egalitarian society. (a) Discuss such undesirable values prevalent in
today’s educated Indians. (b) How can such undesirable attitudes be changed and socio-
ethical values considered necessary in public services be cultivated in the aspiring and
serving civil servants? (2016)
7. Young people with ethical conduct are not willing to come forward to join active politics.
Suggest steps to motivate them to come forward. (2017)

7. UPSC Previous Years Question Paper: Case Studies


1. You are aspiring to become an IAS officer and you have cleared various stages and now you
have been selected for the personal interview. On the day of the interview, on the way to
the venue you saw an accident where a mother and child who happen to be your relatives
were badly injured. They needed immediate help. What would you have done in such a
situation? Justify your action.

2. In our country, the migration of rural people to towns and cities is increasing drastically.
This is causing serious problems both in the rural as well as in the urban areas. In fact,
things are becoming really unmanageable. Can you analyze this problem in detail and
indicate not only the socio-economic but also the emotional and attitudinal factors
responsible for this problem? Also, distinctly bring out why— Educated rural youth are
trying to shift to urban areas; Landless poor people are migrating to urban slums; Even
some farmers are selling off their land and trying to settle in urban areas taking up petty
jobs. What feasible steps can you suggest which will be effective in controlling this serious
problem of our country?

3. You are the Sarpanch of a Panchayat. There is a primary school run by the government in
your area. Midday meals are provided to children attending the school. The headmaster has
now appointed a new cook in the school to prepare the meals. However, when it is found
that cook is from Dalit community, almost half of the children belonging to higher castes
are not allowed to take meals by their parents. Consequently, the attendance in the schools
falls sharply. This could result in the possibility of discontinuation of midday meal scheme,
thereafter of teaching staff and subsequent closing down the school. (20 Marks) (250
Words) (a) Discuss some feasible strategies to overcome the conflict and to create right
ambiance. (b) What should be the responsibilities of different social segments and agencies
to create positive social ambiance for accepting such changes?

4. You are recently posted as district development officer of a district. Shortly thereafter you
found that there is considerable tension in the rural areas of your district on the issue of
sending girls to schools. The elders of the village feel that many problems have come up
because girls are being educated and they are stepping out of the safe environment of the
household. They are the view that the girls should be quickly married off with minimum
education. The girls are also competing for jobs after education, which have traditionally
remained in boys’ exclusive domain, adding to unemployment amongst male population.
The younger generation feels that in the present era, girls should have equal opportunities
for education and employment, and other means of livelihood. The entire locality is divided
between sexes in both generations. You come to know that in Panchayat or in other local
bodies or even in busy crosswords, the issue is being acrimoniously debated. One day you
are informed that an unpleasant incident has taken place. Some girls were molested, when

DELHI JAIPUR PUNE HYDERABAD AHMEDABAD LUCKNOW CHANDIGARH


26 www.visionias.in # 8468022022 ©Vision IAS
Google it:- https://upscpdf.com
https://t.me/UPSC_PDF Download From > https://upscpdf.com https://t.me/UPSC_PDF

they were en route to schools. The incident led to clashes between several groups and a Student Notes:
law and order problem has arisen. The elder after heated discussion have taken a joint
decision not to allow girls to go to school and to socially boycott all such families, which do
not follow their dictate. (250 words) (25 Marks)
(a) What steps would you take to ensure girls’ safety without disrupting their education?
(b) How would you manage and mould patriarchic attitude of the village elders to ensure
harmony in the inter- generational relations?
5. You are the heading the rescue operations in a area affected by severe natural calamity,
thousands of people are rendered homeless and deprived of food, drinking water and other
basic amenities. Rescue work has been disrupted by heavy rainfall and damaged to supply
routes. The local people are seeding with anger against the delayed limited rescue
operations. When your team reaches the affected area, the people there heckle and even
assault some of the team members. One of your team member is even severely injured.
Faced with this crisis some team member plead with you to call off the operations freeing
threats to their life.
In such trying circumstances, what will be your response? Examine the qualities of a public
servant which will be required to manage the situations.
Suppose the Government of India is thinking of constructing a dam in a mountain valley
bond by forests and inhabited by ethnic communities. What rational policy should it resort
to in dealing with unforeseen contingencies?
6. It is a State where prohibition is in force. You are recently appointed as the Superintendent
of Police of a district notorious for illicit distillation of liquor. The illicit liquor leads to many
deaths, reported and unreported, and causes a major problem for the district authorities.
The approach till now had been to view it as a law and order problem and tackle it
accordingly. Raids, arrest, police cases, and criminal trials – all these had only limited
impact. The problem remains as serious as ever.
Your inspections show that the parts of the district where the distillation flourishes are
economically, industrially and educationally backward. Agriculture is badly affected by poor
irrigation facilities. Frequent clashes among communities gave boost to illicit distillation. No
major initiatives had taken place in the past either from the government’s side or from
social organizations to improve the lot of the people.
Which new approach will you adopt to bring the problem under control?

8. Vision IAS Test Series Questions


1. Explore the reasons behind morality having little effect in regulating the attitude of
nations towards one another?
Answer:
Ethical decision-making can be more challenging for international relations than local
operations. It means morality has little effect in regulating the attitude of nations.
Some of the reasons are:
• Formulating foreign policies is the responsibility of governments. Thus moral
consideration in the foreign policy depends on the government, not on an
individual or the citizenry at large (though in well functioning democracies the will
of the people is usually expressed by the government).
• The government is an agent, not a principal. Its primary obligation is to ensure the
interests of the society it represents, not to the moral impulses that individual
elements of that society may experience.
• Nation states are usually concerned with their military security, territorial unity and
integrity, and the well-being of their people. With these and other priorities,
morality usually takes a backseat.

DELHI JAIPUR PUNE HYDERABAD AHMEDABAD LUCKNOW CHANDIGARH


27 www.visionias.in # 8468022022 ©Vision IAS
Google it:- https://upscpdf.com
https://t.me/UPSC_PDF Download From > https://upscpdf.com https://t.me/UPSC_PDF

• The unavoidable necessities of national existence are rarely subject to classification Student Notes:
in terms of "good" or "bad".
• There are no internationally accepted standards of morality to which any
government could appeal if it wished to act in the name of moral principles.
• Culture-driven ethics vary between nations, making it difficult for policy makers of
one nation to adhere to a strict code of ethics for itself, while dealing in the
international arena.
• Ethical dilemma occurs when foreign policy makers must decide whether to
commit an act that is ethically unacceptable in the home country, but expected and
necessary in the other country.
• If the ethics behind foreign policies will have diminishing or no resonance in the
societies they are applied, in or to, their effect will be perverse.

2. "Those who say religion has nothing to do with politics do not know what religion is".
Explain. Further examine the role of religion in formation of political attitude in the
Indian context.
Approach:
This is the quote given by Mahatma Gandhi, which underlines the importance of
religion in shaping political and moral attitude of people. The answer can be framed in
the following manner:
• Elaborate the statement and its assumptions in detail.
• Discuss how religion played an important role in shaping political attitude in India.
Answer:
Partition of India on communal lines and existence of political parties adhering to a
particular religious ideology are examples and validation of the above quote by
Mahatma Gandhi. In a developing society, like India, where tradition coexists with
modern practices, religion still plays a very important role in the distribution of power
in society. Hence, the idea that politics is immune from religious ideas is tantamount to
ignorance about the powerful role religion plays in the socio-political life of Indian
society. It can be said that religion still fundamentally shapes political attitude of a
large section of people in India.
Though there are various factors like family, caste, ethnicity, region, occupation which
determine general political attitude of people, in the Indian context religion has
historically played a very fundamental role. Various underlying causes for this condition
are:
• Inspite of diverse social composition, numerical dominance of one religion with
respect to various minority religious communities made religious identity important
in socio-political life.
• During colonial times religious identities were reinforced to divide the society on
religious-political lines, since then religion has become important social force in
political landscape of India.
• Religion provides the ethical framework of the society which also shapes the
political actions e.g. banning of beef, role of women in society.
• Religious identities are easier to mobilize for political ends as they are primordial
and resonates with masses.

DELHI JAIPUR PUNE HYDERABAD AHMEDABAD LUCKNOW CHANDIGARH


28 www.visionias.in # 8468022022 ©Vision IAS
Google it:- https://upscpdf.com
https://t.me/UPSC_PDF Download From > https://upscpdf.com https://t.me/UPSC_PDF

3. “Persuasion makes society work smoothly while physical coercion grinds it to a halt”. Student Notes:
Giving examples, compare the effectiveness of persuasion as an influence tactic vis-a-
vis coercion in bringing change in society. In what ways persuasion can be used by
civil servants to remove social evils existing in society?
Approach:
The basic theme of the question is to compare the importance of persuasion vis-à-vis
coercion to influence others and bring change in society. The answer can be framed in
the following manner:
• In brief define persuasion and coercion.
• Citing examples examine the conditions under which persuasion proves to be
better tactic vs coercion.
• Thirdly, suggest some ways/measures through which civil servants can use
persuasion to remove certain social evils.
Answer:
Our thoughts and actions are influenced by other people, whether we are passively
observing their behavior or actively complying with their requests. In everyday life we
use persuasion to convince people to either fulfill our needs or to alter their thoughts
and beliefs. Thus, Persuasion is a conscious attempt to change the attitude/belief of
others. It attempts to win ‘the heart and mind’ of the target and influences a person’s
attitude, intention, motivation or behavior. Coercion on the other hand is the action or
practice of persuading someone to do something by using force or threats. It implies
that the act is committed against the will of the person.
Effectiveness of Persuasion over Coercion:
• Persuasion relies on understanding and changing the hearts and mind. It focuses
on the need voluntarily change peoples’ belief or action. Coercion on the other
hand, employs threat power, punishment so that a person feel compelled to act
accordingly/involuntarily.
• Persuasive tactics include inspirational appeals, consultation and collaboration
which are democratic ways to bring change in society. Those being persuaded feel
involved and intrinsically motivated to take a particular action. Whereas, Coercion
makes a person accept change in a forced, imposed and undemocratic way.
• Although the process of persuasion may take more time, it is less likely to lead to a
cycle of retaliation or revenge. Persuasive means are likely to remain more durable
and result in greater productivity. Whereas, when bullied into submission, it is
human nature to fight back and break away at the earliest opportunity.
Example: To change peoples’ mind to adopt family planning measures in order to
control population, persuasion is a more effective technique than coercion because
adopting family planning methods affects socio-cultural and religious sentiments of
different communities in the society. If coercion is used, many would retaliate (as was
evident in 1976-77, when Family control measures were forced onto people, ignoring
their socio-cultural and religious sentiments).
Ways in which Persuasion can be used by Civil Servants:
• The civil servants at all levels of government exercise substantial discretion and
authority, and often have a significant impact on the lives of individuals. They
influence how policies are developed, taxes are spent and services are delivered.
• If Rational/Logical Persuasion is used by presenting the facts and laying out an
argument which generally includes emphasizing the positive benefits of a course of
action by providing data, facts, etc., then it would serve a better strategy to bring

DELHI JAIPUR PUNE HYDERABAD AHMEDABAD LUCKNOW CHANDIGARH


29 www.visionias.in # 8468022022 ©Vision IAS
Google it:- https://upscpdf.com
https://t.me/UPSC_PDF Download From > https://upscpdf.com https://t.me/UPSC_PDF

change in a person’s attitude- Example-‘Swachch Bharat Abhiyan’– use of Student Notes:


awareness to bring about a collective community behavioural change. Also, to
increase sensitivity towards dwindling sex ratio, if its implications on society overall
is shown through facts, datas and reports and its effect on future prospects of
marriage and security of girls per se then it would have a more long lasting effect
than coercive tactics.
• If all the stakeholders of society are consulted like Civil Society, NGOs, Religious
groups, Youth, elderly, women organisations, etc in a district/state, the civil
servants can reach to a more mutually consented decision on a particular course of
action to bring change in society- eg. Promoting innovative techniques like “Selfie
with daughter” to change peoples’/ family’s attitude whenever a girl child is born.

4. Social media has played a key role in influencing political opinions and social
attitudes in India. Comment.
Approach:
• Briefly define the term 'social media' and it’s current relevance in day to day lives.
• Highlight the positive/negative role played by social media in influencing political
opinions and social attitudes in India.
• Way forward.
Answer:
• Social media denotes a group of Internet-based applications which allow people to
create, share or exchange information, career interests, ideas, and pictures/videos
in virtual communities and networks. Social media platforms such as Facebook,
Twitter, WhatsApp and Instagram are being accepted as the fifth pillar of media
after print, TV, digital and radio and tend to provide news faster than most news
channels today.
Role of social media in influencing political opinions
• Outreach: Social media allows politicians and political parties a method to connect
directly with people at a reduced cost and greater reach than traditional media. For
instance, during 2014 Delhi Legislative Assembly elections, Delhi had 13 million
registered voters this election, out of whom 12.15 million were online, making
digital platforms the most effective engagement tool for party workers.
• Communication: It helps politicians interact with citizens, seek feedback and weigh
public opinion. Example, Twitter gives political organisations the ability to broadcast
information on a worldwide stream, join any ongoing debates and discussions and
have a two-way interaction with the public during political processes and
campaigns.
• Campaign management: Political campaigns leverage a wealth of information or
analytics about the people who are following them on social media, and customize
their messages based on selected demographics to manage the image of the
candidate. According to Internet and Mobile Association of India (IAMAI) report,
Facebook had a tremendous impact over the results of the polls in 160 of India‘s
543 constituencies in the last General elections.
• Polarisation: Social media has also become a new battleground, where political
parties leverage the pervasive nature of these platforms for their own advantage.
Many anti-social elements turn to social media for spreading hate campaigns and
support communal events, thus polarising the elections.

DELHI JAIPUR PUNE HYDERABAD AHMEDABAD LUCKNOW CHANDIGARH


30 www.visionias.in # 8468022022 ©Vision IAS
Google it:- https://upscpdf.com
https://t.me/UPSC_PDF Download From > https://upscpdf.com https://t.me/UPSC_PDF

Role of social media in influencing social attitudes in India Student Notes:


• Opinion building- It has eliminated the spatial difference between people &
allowed faster & more equitable flow of information about different socio-
economic topics. For instance, the role played by 'Facebook reporters'- living in the
enclaves of India in Bangladesh fighting for their cause.
• Social Reforms- Eminent personalities taking to social media platforms to broadcast
the importance of Swachh Bharat, kick started a new revolution for achieving Clean
India. Various online surveys, audio-visual campaigns and ideas infused new energy
among the masses and moulded their attitudes.
• Humanitarian campaigns- Twitter and Facebook proved an invaluable tool for
those involved in disaster relief efforts, posting up to the minute reports on
tsunami alerts, altered train schedules, emergency numbers and shelters. This
changed the attitudes of people in positive way and created a bandwagon effect
among people who wanted to reach out to the survivors.
• Misuse of Social Media-On the other hand, social media has also been used a
means to achieve wrong gains. The increased instances of misinformation (mass
exodus of people in Bengaluru hailing from North East), false opinion building,
virtual information wars among countries, hate and bully campaigns etc. depict the
narrow mindedness of some of the social media users.
Social media is here to stay. Given the reach, frequency, interactivity, usability,
immediacy, and permanence of social media, it becomes imperative to check its use for
malicious activity & ensure it serves the purpose of equitable access to correct
information.

5. It does not take long for conflict to turn violent when deep seeded prejudices and
discriminatory attitudes are not addressed. Discuss in the context of communal and
caste-based violence in India. What role should the state play in this context?
Approach:
• Briefly discuss about the caste and communal based violence and discrimination in
society and explain the meaning of prejudice.
• Highlight how it develops and its role in such violence and discrimination.
• Discuss the steps that must be taken to reduce prejudices.
Answer:
The very strength of Indian society which lies in its diversity, sometimes become its
main fault line dividing the society, ultimately leading to discrimination and even
violence. Though it was expected that with the urbanisation and increasing
technological and economic development, communal and caste identities would be
diluted, but this does not seem to be the case. Evidence can be provided for both
dilution of such ascriptive identities as well as their strengthening.
The instances of caste and communal violence are generally based on strong prejudice,
irrational beliefs and preconceived notion about other group. As a result, members of
one community perceive threat, harassment, fear and danger from the members of the
other community. Consequently, the response to the threat is mostly ignorance of the
other and sometimes hatred leading to violence.
In India these prejudices get strengthened because of the following factors:
• Legitimising discrimination through history, folklore and perception. For e.g. the
event of partition still incites deep emotions among many people; discrimination of
people from lower castes has many references in historical texts and thus derives
some sort of sanction.

DELHI JAIPUR PUNE HYDERABAD AHMEDABAD LUCKNOW CHANDIGARH


31 www.visionias.in # 8468022022 ©Vision IAS
Google it:- https://upscpdf.com
https://t.me/UPSC_PDF Download From > https://upscpdf.com https://t.me/UPSC_PDF

• Separatesettlements result in lack of inter-group contact and communication,that Student Notes:


strengthens in-group and out-group classification.
• Fomenting of insecurities and fear by vested interests by providing misinformation
about the aims of other group.
• As multiple groups compete for same resources, so in order to maintain legitimacy
and their dominance, socio-economically powerful group brandishes other as
incompetent.
• To prevent ascendancy of other groups, opportunists try to incite the negative
emotions in the people so as to serve their narrow motives by orchestrating
communal incidents, thus ultimately strengthening the prejudices.
Although the constitution of India explicitly bars discrimination by the state on grounds
such as race, religion, caste, place of birth and sex, as well as prohibits untouchability,
still discrimination is practices at a personal and community level. The state should thus
take appropriate steps to minimise conflicts that may arise due to cultural separation by
bridging the cultural divide. These include:
• At the preventive level, the state should mitigate long held prejudices by promoting
meeting of members of different sections in a free environment. For example,
cultural meets and community dinners on the occasions of different festivals. This
corrects the misinformation that one group holds towards the other group.
• In cases of discrimination or violence, state should not only do justice but ensure
that justice has been done.
• Caste and communal based politics must be checked through strict enforcement of
section 123 of RPA, 1951 and strengthening Model Code of Conduct.
• Motivating people to fight against their common enemy like poverty, illiteracy,
unemployment and pollution etc.
• Education system should be oriented towards inculcation of constitutional and
human values amongst children.
• Review of the training programs for the police forces with a view to inculcating in
them, the attitudes of secularism and communal harmony.
Yet, state alone can’t ensure social harmony; the responsibility lies with the public and
their individual conscience. It is the duty of every citizen to inculcate the feeling of
bonhomie towards other sections and rise above narrow interests and short term goals
and build a strong and united country.

6. While on the one hand, some state governments have implemented alcohol
consumption prohibition laws, it is permitted in other states. Debates around this
issue often involves aspects such as individual rights, cultural attitudes and social
welfare. As a teacher you need to explain the key issues involved to a young audience.
What are these? How would you conclude the lecture?
Approach:
• Discuss reasons for introduction of alcohol prohibition.
• The question mentions three aspects i.e. individual rights, cultural attitudes and
social welfare. Build on these while explaining the key issues associated with
alcohol prohibition.
• Conclude persuasively to emphasize on the points made above. One’s stand may
differ from that presented in this model answer.

DELHI JAIPUR PUNE HYDERABAD AHMEDABAD LUCKNOW CHANDIGARH


32 www.visionias.in # 8468022022 ©Vision IAS
Google it:- https://upscpdf.com
https://t.me/UPSC_PDF Download From > https://upscpdf.com https://t.me/UPSC_PDF

Answer: Student Notes:


The ill effects of consumption of alcohol are well documented. Deterioration in health
of the person concerned is just one aspect. It is well established through studies that
incidents of domestic violence, road rage and accidents, gambling, indebtedness and
theft are more common in an inebriated state. This has led various state governments
in India to prohibit alcohol from time to time with varied level of success. However
owing to vast socio-economic differences across states and federal nature of our polity,
a pan India ban on alcohol seems difficult to implement. Moreover, with people
ascertaining consumption of alcohol as a right when we are moving towards a rights
based framework governing individual-state relationship, it becomes even more
difficult to convince people for a complete ban on alcohol.
A teacher, being a strong agent of influence, should present various dimensions of
alcohol prohibition in a structured way, which may help the young audience to form a
well informed opinion about the subject. The key issues include:
A. Individual rights
Prohibition restricts dietary habits of citizens and their right to choose. Prohibition
should not be justified citing its possibility of being misused alone. In the times
when authority to make decisions is being decentralised, it becomes natural to
question the logic behind government prescribing a blanket ban. To a certain
extent, it does undermine the citizen’s ability to make a well-informed decision.
B. Cultural attitudes
In some cultures, alcohol is a means to socialize and celebrate . For example,
alcohol drink made from rice is popular in certain parts of Assam as is palm based
drinks in various parts of the country. tribes in central India also use alcohol brewed
from locally available grains in their celebrations. Blanket prohibition overlooks
these societal aspects. However even a wide acceptance alone cannot be the
reason to legitimize something and overlook some of its associated fallouts.
C. Social welfare
A developing country like India needs interventionist policies for the social and
economic well-being of its citizens. Article 47 of Indian Constitution directs State to
prohibit consumption of intoxicating substances, except for medicinal reasons.
Mahatma Gandhi was also one of the vocal votaries of prohibition.
Restrictions/bans become easy and preferable tool to implement such
Constitutional directives though their effectiveness is debatable. Given these
arguments, it is difficult to find a black and white answer to this debate. As a
teacher, it is reasonable to conclude with certain ingredients which any solution to
this debate must essentially incorporate:
• Sustainability- It depends on administrative capacity to implement effective
ban (preventing inter-state smuggling and black-marketing), generating
alternative revenue source to compensate the revenue loss due to prohibition
etc. Long term solutions include- citizen awareness and a well informed policy
on prohibition which takes into account the current socio-economic aspects of
our society.
• Role of various stake holders –Family, educational institutions, media and role
models have an important role in shaping the perception of youth and thus
they must be a part of the solution and must act responsibly in the best interest
of the society. Similarly political parties must not politicize the issue just for the
sake of political gains and must frame policies in line with constitutional and
global imperatives.

DELHI JAIPUR PUNE HYDERABAD AHMEDABAD LUCKNOW CHANDIGARH


33 www.visionias.in # 8468022022 ©Vision IAS
Google it:- https://upscpdf.com
https://t.me/UPSC_PDF Download From > https://upscpdf.com https://t.me/UPSC_PDF

Speaking from a utilitarian perspective, it is clear that the overall welfare of the society Student Notes:
is negatively hampered with alcohol consumption. A violation of right to consume of a
few cannot be used as pretext to overlook the violation of human rights of many more.
Moreover, from deontological perspective the right to consume alcohol can be tested
universality and humanity. It turns out that even in extreme case if everyone would like
to extend consumption as universally acceptable, it is unclear whether it treats
humanity as an end or a means. What it definitely does it is that it neglects the rights of
others at cost of rights of a miniscule minority. As such, it cannot be termed as a right
rather can be better classified as a desire or a luxury- a means to achieve self-
satisfaction.

7. What do you understand by cognitive dissonance? Giving examples, discuss how it


influences one's behavior and attitude.
Approach:
• Define Cognitive dissonance.
• Discuss in brief what you understand by Attitude and Behaviour.
• Discuss how Cognitive dissonance influences one's behaviour and attitude.
• Give appropriate examples wherever necessary to support your arguments.
Answer:
Cognitive dissonance is a phenomenon in which a person experiences psychological
agony due to conflicting thoughts or beliefs. It is the feeling of inconsistency in feelings,
beliefs and behaviour (the three components of attitude). For example, when people
smoke (behavior) and are aware that smoking causes cancer (cognition), they are in a
state of cognitive dissonance.
Attitude is a favorable or unfavorable evaluative reaction towards something or
someone found in one’s beliefs, feelings, or intended behavior. Behaviour is actual
expression of these feelings. Attitude is internal and Behaviour is external.
The Principle of Cognitive consistency states that human beings have an inner drive to
hold their attitudes and behavior in harmony and avoid disharmony. So, when there is
an inconsistency between attitudes or behaviors (dissonance), something must change
to eliminate the dissonance.
Change in behavior and attitude may occur by reducing the dissonance in following
three ways:
• When one of the dissonant elements is a behavior, the individual can change or
eliminate the behavior. For e.g.- giving up smoking.
• Acquire new information that outweighs the dissonant beliefs. For e.g. new
information such as “research has not proved definitely that smoking causes lung
cancer” may reduce the dissonance.
• Reduce the importance of the cognitions (i.e. beliefs, attitudes). For e.g. a person
could convince himself that it is better to "live for today" than to "save for
tomorrow", thus decreasing the importance of the dissonant cognition (i.e.
smoking is bad for one's health).

DELHI JAIPUR PUNE HYDERABAD AHMEDABAD LUCKNOW CHANDIGARH


34 www.visionias.in # 8468022022 ©Vision IAS
Google it:- https://upscpdf.com
https://t.me/UPSC_PDF Download From > https://upscpdf.com https://t.me/UPSC_PDF

Student Notes:

Though it is difficult to objectively measure cognitive dissonance, it plays a major role in


shaping attitudes and behaviours that influence judgments, decisions and evaluation of
different people.

9. Vision IAS Test Series: Case Studies


1. You are the District Magistrate in a district where a significant number of
transgenders reside. While discrimination against the community is well known,
commuters increasingly complain of harassment at their hands, especially at traffic
junctions where transgenders are mostly involved in begging. This, at times, also
leads to traffic management issues. You have received a number of complaints in this
regard and have to act quickly to resolve it. However, a group of transgender
associations argue that begging is their only source of livelihood. Given the situation,
answer the following questions:
(a) Describe the ethical issues involved in this case. Discuss the attitude of people
towards transgenders in general and reasons for the same.
(b) What possible courses of action can be undertaken in such situations? Discuss their
merits and demerits.
Approach:
• Discuss the ethical issues involved and the reasons for differential attitude of
people towards transgenders.
• Discuss possible course of action with merits and demerits. You may conclude by
suggesting long-term solution/elaborating on your solution.
Answer:
The given case involves the issues of vulnerable section of population as well as the
comfort of the community. The issue relates to taking into consideration competing
interests of discrimination, which transgender faces, be it education, employment,
social participation as well as the means they use to earn their living.
(a) Ethical issues involved in this case are as follows:
• Convenience versus livelihood: Begging by transgenders is causing
inconvenience to commuters and is leading to additional problems like traffic
congestion. While, on the one hand, poor socio-economic condition of
transgenders force them to make a living by begging, this at times becomes
inconvenient to general public.
• Discrimination of transgenders versus harassment of the commuters: The
society in general can’t have both ways, that is, restricting the options of a
section of population to earn decent income and at the same time have
problem with whatever little means they are using to earn their livelihood.

DELHI JAIPUR PUNE HYDERABAD AHMEDABAD LUCKNOW CHANDIGARH


35 www.visionias.in # 8468022022 ©Vision IAS
Google it:- https://upscpdf.com
https://t.me/UPSC_PDF Download From > https://upscpdf.com https://t.me/UPSC_PDF

• DM’s commitment to safeguard interests of the vulnerable sections versus Student Notes:
taking action on the registered complaints by citizens: Since a large number of
complaints have been registered, it is the DM’s duty to take up a call to reign in
the menace while balancing the right of transgenders to earn a living.
General attitude of people towards transgenders
• Discriminatory - based on their class and gender. This makes the transgender
community one of the most disempowered and deprived groups in the Indian
society.
• Non-inclusionary: They are treated as unnatural and as objects of ridicule and
even fear on account of superstition.
• Inequality and that of Seclusion: A long neglect in terms of rights (civil rights
like the right to a dignified living) and development (reservation in educational
institutions and public employment) has completely isolated the community.
• Negligence and indifference: Their numerical minority makes them politically
less significant as a vote bank and paves the way for their legislative and
administrative neglect.
Reasons for such attitude
• They are different from what has been considered ‘normal’ in the society
• Gender based discrimination has been the norm everywhere and ever since.
They have been treated as objects of entertainment rather than as human
beings.
• Discarded by their families and society, they turned to petty means to earn
livelihoods. Now they are looked down upon for that, without realizing the root
cause. Society misunderstands the changing concept of gender identity.
• Many superstitions have also built up around the community because of their
closed nature, without realizing that they have been discarded, not that they
chose to remain secluded.
(b) Possible course of action
• Ignore the issue as begging is a matter of livelihood for transgenders
Merit: Transgenders will have their way in collecting money at traffic junctions.
Demerit: This will amount to dereliction of duty and will neither help
transgenders nor the general public in the long run. Also, begging is a criminal
offence. Knowingly allowing begging amounts to collusion. Also, it does not
solve the issue of genuine inconvenience which the commuters face.
• Issue a stern warning to transgenders indulging in harassment of commuters.
Additionally deploy squads who may check the menace at traffic junctions.
Merit: It may rein in the menace of harassment at the hands of transgenders
and may prevent traffic problems.
Demerit: It may affect the livelihood of transgenders adversely who mainly rely on
begging and are discriminated by the general public. As the area has high population of
transgenders, it may also create law and order problems.
• Issue guidelines for traffic junctions along with establishing of committee for
chalking out measures to sensitise public about transgender issues as well as
implement skill development and vocational training schemes along with
rehabilitation measures.
Merit: It will provide alternate avenues of employment for transgenders. In fact,
they would happily switch their jobs given the opportunity. It redresses grievance of
citizens as well and may also lead to transgender community getting a respectable
place in society in the long run.

DELHI JAIPUR PUNE HYDERABAD AHMEDABAD LUCKNOW CHANDIGARH


36 www.visionias.in # 8468022022 ©Vision IAS
Google it:- https://upscpdf.com
https://t.me/UPSC_PDF Download From > https://upscpdf.com https://t.me/UPSC_PDF

Demerit: This is a step with a long gestational period. It may hamper the income Student Notes:
earning source of transgenders in the short term. This may reinforce the notion
that general society’s rights are valued above those of the vulnerable sections.
Though it is important to provide relief to general public, the perspective of
transgender community must be empathized, else the solution will not be sustainable
in the long run and the status quo will rebound. Implementing the Supreme Court
directions of giving them special treatment in certain matters will go a long way in
integrating them in the society.

2. You are a civil servant posted in a state where elections were recently held. The newly
elected Chief Minister had promised to ban alcohol in several of his election
campaigns as well as his election manifesto, which was widely praised and supported
by women of the state. Fulfilling his electoral promise, the Chief Minister has ordered
a blanket ban on the sale of alcohol in the state. Following the ban, concerns have
been raised about the feasibility of the ban and whether the government should
interfere in what is considered by many to be a matter of personal choice.
(a) Who are the stakeholders in this case and how are they affected by the ban?
(b) Is blanket ban on alcohol a feasible action?
(c) Identify the issues that may arise while enforcing the ban and the steps you will
take to handle them, as a civil servant.
Approach:
• Identify the stakeholders in the case and the effect of alcohol ban on them.
• Comment on whether blanket ban is a feasible action or a coercive action taken by
the government.
• State the issues that will arise after the enforcement and suggest measures to
handle them.
Answer:
In this scenario, blanket ban of alcohol has been imposed by the new CM in a state,
which affects various sections of the society in different ways.
(a) Following are the stakeholders and the effect of the ban endured by them:
• People who consume alcohol, as they will have to face withdrawal symptoms,
post ban.
• Their relatives and friends, especially women, who have faced problems like
domestic violence, family disputes. However, they will have to ensure that
alcoholic addicts are promptly taken to de-addiction centres.
• The government as the state exchequer will face massive revenue loss. Social
expenditure may have to be cut down. Moreover, law and order situation has
to be maintained.
• Alcohol suppliers and traders as they will have to find alternate means of
livelihood.
• The society at large, as it is adversely affected by alcoholism.
(b) While assessing whether blanket ban is feasible, both advantages and
disadvantages have to be analyzed:
Advantages:
• Fewer cases of domestic violence and relatively safe environment for children.
• Improvement in health of people leading to decreased burden on government
services such as hospitals, medicine stores etc.
• Increase in household savings.

DELHI JAIPUR PUNE HYDERABAD AHMEDABAD LUCKNOW CHANDIGARH


37 www.visionias.in # 8468022022 ©Vision IAS
Google it:- https://upscpdf.com
https://t.me/UPSC_PDF Download From > https://upscpdf.com https://t.me/UPSC_PDF

• It can lead to fewer road accidents as drivers will not drink and drive. Student Notes:
• It is in consonance with Article 47 of the Constitution, which emphasizes on
public health.
Disadvantages:
• Total ban violates individual’s freedom to choose.
• It will cause a heavy revenue loss on state exchequer. The money could have
been used for welfare purposes.
• The state might see a rise in alcohol bootlegging owing to emergence of ‘black
markets’. Since the markets will not be regulated, spurious liquor may be sold,
which can have adverse health impacts.
• Alcohol is used in certain religious functions e.g. use of wine during baptism by
Christians. Hence, it can interfere with religious practice of minorities.
• Unemployment for those involved in liquor manufacturing and trade.
• Immediate rehabilitation costs for alcohol addicts.
While total ban has several positive aspects, it is not entirely feasible as adults
should be given freedom of choice and decision-making. Alcohol production, sale
and distribution should be regulated rather than imposing a total ban, else it might
be deemed as a unilateral coercive action by the government. Alcoholism is a social
issue, therefore it can’t be banned merely by legislative means. Hence, we have to
bring in corresponding social change as well to make the ban effective.
Note: Arguments can be given in favour of ban also with proper justification.
Alcoholism has been a menace in the society with disproportionate suffering of
women in lower socio-economic strata. Just because some individual’s right to
choose their dietary habits is curtailed, it cannot be a pretext to prevent active
intervention by the government to correct a societal ill. Bans based on sentiments
are questionable, but bans based on empirical evidence of domestic violence (most
of which goes unreported), traffic violations and consumption of harmful country
made liquor have scientific substance to them. The government being the
representative of the people has the authority to initiate a change, if it does not
come from within.
(c) Issues that may arise in this case, and the ways in which a civil servant should deal
with them:
• Withdrawal symptoms of addicts: It should be ensured that there are
adequate de-addiction and rehabilitation centres with competent
professionals.
• Emergence of black markets and distribution of spurious liquor: checking at
regular intervals, especially at border check posts. Further, restaurants and bars
will also be frequently monitored.
• Livelihood of traders and alcohol producers at stake: Policy focus should be on
job creating programmes and poverty alleviation programmes.
• Further, work on organizing awareness programmes for promotion of healthy
lifestyle and disseminate negative effects of alcohol consumption in
collaboration with primary health care centres, NGOs, media etc., should be
emphasised.

DELHI JAIPUR PUNE HYDERABAD AHMEDABAD LUCKNOW CHANDIGARH


38 www.visionias.in # 8468022022 ©Vision IAS
Google it:- https://upscpdf.com
https://t.me/UPSC_PDF Download From > https://upscpdf.com https://t.me/UPSC_PDF

3. You, a manager in one of the top IT firms in the country, are tasked with hiring new Student Notes:
recruits for an upcoming project. You find that the company has given tacit
instructions of not hiring female candidates in view of the new maternity law passed
by the Government. You find this highly objectionable and lodge a protest with people
in the higher management but they are firm as they want to cut down all the
unnecessary costs. Based on this information, answer the following questions:
(a) Identify the stakeholders and their interests in the situation.
(b) What are the dilemmas that a recruiting manager may face in such a scenario?
(c) What are the different options available to you? Which one will you pursue and
why?
Approach:
• List stakeholders like the hiring manager, the company, female candidates,
Government and the society List their interests.
• Discuss the dilemma(s) you are facing.
• List the available options, analyze each in the light of given circumstances and
ethical conduct. Choose the one which you may pursue.
Answer:
(a)
Stakeholder Interest
Company/Higher Company’s interest is to maximize profit by lowering down cost
Management incurred per employee. In the case of female employee the cost of
maternity leave is to be borne by the company. Hence, the higher
management wants to avoid hiring female candidates.
Hiring My first interest is to hire suitable candidates for the listed positions
Manager(Myself) irrespective of the gender. Avoiding confrontation with management
while standing up to unjust policies is the dilemma faced in such
situation.
Female Candidates Their interest lies in seeking a fair chance to get the job in the
company. In the long run, they expect equity in hiring and promotion
and a safe working environment.
Government and These stakeholders seek gender equality at workplace. For this it is
Society important that issues arising out of maternity be redressed
adequately via legislation(s) and social change. Though various
maternity legislation may ensure assured maternity leave, flow of
regular income and job security, it alone cannot bring out the societal
change required for a gender vibrant culture at workplace.
(b) Dilemma: Accepting the higher management’s dictum and avoiding any female
candidate v/s being fair and appointing a suitable candidate irrespective of the gender.
Additionally, I may also face the dilemma of forgoing a better candidate just because of
gender. This will lead to loss of productivity and will incur a long term cost to the
company. The associated dilemma is the larger issue of gender biasness in society due
to patriarchal attitude conflicting with the idea of equality and progress, which
demands change. It’s important to promote gender diversity at the workplace;
however, the current instructions by the company go against this principle.
(c) Different options available:
• Rejecting female candidates: Though with this option I will avoid confrontation
with the higher management, this would be contrary to basic human rights and
the constitutional ethos of equality. This would filter out many genuine
candidates and will constrict the candidate pool. This smacks of short-
sightedness and narrow-mindedness.

DELHI JAIPUR PUNE HYDERABAD AHMEDABAD LUCKNOW CHANDIGARH


39 www.visionias.in # 8468022022 ©Vision IAS
Google it:- https://upscpdf.com
https://t.me/UPSC_PDF Download From > https://upscpdf.com https://t.me/UPSC_PDF

• Hiring a suitable candidate objectively and without discrimination: This will Student Notes:
ensure that the most competent and deserving candidate are hired for the job.
Option which I will pursue: Confrontation is not the way out; persuasion is. The most
appropriate way to persuade will be through attitudinal change, which however, is very
difficult in the immediate term. But if I can objectively demonstrate that costs
associated with hiring women employees are not significantly higher as compared to
men and that productivity is independent of gender, I will be able to start the process of
attitudinal change. I will take the help of HR department, and if required, the governing
board of the company in the matter.
Regarding recruitment, I have to be objective in my assessment of candidates, not
biased w.r.t. gender. They will be assessed based on their ability as well as job
requirement. If the job profile does not demand a particular gender requirement, the
tacit instructions should not carry much weight and may in fact be contrary to the
policies of the company. I will report such instructions to the HR as gender
discrimination is not only illegal but also vitiates the organizational culture and social
image of the company. Additionally I will undertake gender-sensitivity drive by involving
my peers especially the working women in my organization. By doing so I would have
upheld the dictum of ‘Be the change you want to see’.

4. Rapid growth of information and communication technology, with all its benefits, has
associated risks and far-reaching consequences. The government has constituted a
committee to frame guidelines for an inclusive and safe cyberspace in India. The
committee has solicited public opinion in this regard. As a concerned citizen, you have
to give your suggestions on the following themes:
(a) Why do you think some people or a set of people are more vulnerable to cyber
threats with special emphasis on cyber-bullying.
(b) Do you think the experiences and exposure in cyberspace are an important
influence in a person's attitude and behaviour?
(c) What reasonable restrictions can be applied to make cyberspace more safe and
friendly to all citizens?
Approach:
• Briefly define benefits and risks posed by ICT.
• State the groups/set of people who you think are vulnerable to cyber-threats
especially cyber-bullying with reasons.
• Discuss whether experiences and exposure in cyberspace are an important
influence in a person's attitude and behaviour.
• List reasonable restrictions that can be applied to make cyberspace more safe and
friendly to all citizens.
Answer:
ICT has a significant impact on the economic, political and social dimensions of
development. It has enhanced economic activity, empowered individuals by ensuring
their participation in decision-making processes at various levels and exposed diversity
of views. However global nature, speed, convenience and anonymity offered by ICT is
being misused to create numerous threats such as cyber-bullying, cyber-grooming,
hacking, pornography, radicalization, intimidation, breach of privacy, reputational loss,
identity theft etc.
a) Cyber-bullying which refers to use of internet or mobile technology to intentionally
harass or bully someone by sending rude, mean or hurtful messages, comments
and images/videos. A cyber bully can use text messages, email, social media

DELHI JAIPUR PUNE HYDERABAD AHMEDABAD LUCKNOW CHANDIGARH


40 www.visionias.in # 8468022022 ©Vision IAS
Google it:- https://upscpdf.com
https://t.me/UPSC_PDF Download From > https://upscpdf.com https://t.me/UPSC_PDF

platforms, web pages, chat rooms etc. to bully others. While all ICT users are at risk Student Notes:
of facing cyber threats, there are some social groups that are more vulnerable to
cyber threats like:
• Women: Cyber crimes against women are rooted in stereotypes about gender
roles, sexuality and sexual norms for women. Women are victims of cyber
bullying that includes non-consensual sharing of intimate images, unsolicited
sending of sexual and pornographic images and other forms of cyber bullying
entailing sexualised behaviour.
• Children and teenagers: Due to their experimental mind-set and limited
understanding of cyber threats, they are particularly vulnerable to cyber
threats like cyber bullying. Those with learning and attention issues are at a
greater risk. It may have physical, emotional and psychological consequences
and may lead to school dropout and depression.
• Youth: The youth are vulnerable to online radicalization, allurement to online
high paying jobs etc. A study in the Journal of Medical Internet Research
revealed that children and young people under the age of 25 who are victims of
cyber-bullying are twice as likely to commit suicide.
• Celebrities: Celebrities especially female ones are targeted victims of body
shaming, racial slurs, stalking, cyber-bullying for their appearance, work, social
life etc. Consequences include forceful exit from social media, mental and
psychological stress and even suicide in some instances.
Besides, late introduction to technology, information asymmetry and behavioural traits
such as unwillingness to use complex passwords etc makes the elderly and digitally
illiterate sections of the society are also increasingly vulnerable to cyber threats.
b) The development of services and products such as social networks, text
messengers, gaming sites etc. in the cyberspace affects attitude and behaviour by
harbouring flow of both desirable and undesirable things.
Such services have the ability to exert credible influential power on their users. For
instance, information provided to individuals over a group on social media is
personalized, timely and usually comes from a trusted/known source. People in
such a closed group may share similar worldviews and thus the information shared
gains legitimacy. Thus, a form of ‘Groupthink’ can develop and strongly influence
the group’s perception of events and opinions (social, political, economic).
The emergence of online communities/groups have led to the normal process of
information verification (i.e. identifying reliable sources) being replaced by
‘network wisdom’ (i.e. wisdom of the group network whose legitimacy is perceived
to be higher). Radical groups and even political parties use their innate flexibility to
exploit situations, misrepresent and change perceptions. For instance, circulation of
morphed images related to violence against Muslims in Assam in 2014 when in fact
there had been no such instance.
The concept of “Digital bedroom” has pervaded the mind space of children;
wherein childhood is significantly mediated through the internet and social media
and leads to increased isolation in real life. Online games are known to normalise
violent behaviour and specifically impact behaviour of children.
c) Reasonable restrictions that can be applied to make cyberspace more safe and
friendly
• Providing technological options to restrict access to social media sites to
vulnerable groups like children.
• Restrictions on broadcasting fake/unverified news especially when law and
order situation warrants so.

DELHI JAIPUR PUNE HYDERABAD AHMEDABAD LUCKNOW CHANDIGARH


41 www.visionias.in # 8468022022 ©Vision IAS
Google it:- https://upscpdf.com
https://t.me/UPSC_PDF Download From > https://upscpdf.com https://t.me/UPSC_PDF

• Strict compliance of anti-piracy laws and mandatory compliance audit of major Student Notes:
social networking sites.
• Cyber authorities should use the same medium to provide correct information
and nip rumors in the bud itself.
• Mandatory online content rating system to classify content with regard to
suitability for audiences in terms of issues such as sex, violence, substance
abuse, profanity, impudence or other types of mature content.

5. Various studies have found out that cases of depression and mental illness have
increased exponentially in the recent past. Also, in the age group of 15-30 years, this
problem is further pronounced. Furthermore, the rising trend of suicides in this age
group has been attributed to depression.
Given the situation, answer the following questions:
(a) Present an ethical critique of the prevalent societal attitude towards mental
illness.
(b) Given the magnitude of the problem among younger generation/young adults,
analyse the role of the following: i. Parenting, ii Social Media, iii. Video Games
Approach:
• Introduce the answer with statistics around mental health problems.
• Discuss the prevalent societal attitudes toward mental health issues and highlight
an ethical critique towards it.
• Briefly mention the perceptional shifts needed to tackle these issues.
• Analyse the role of parenting, social media, and video games in causing the mental
health issues.
• Conclude briefly.
Answer:
According to WHO, stress and depression cases increased by 18% in the last decade but
the expenditure on mental health in India is merely 0.06% of GDP. In the age group of
15-30, increase in mental health issues and eventual suicide rates can be attributed
toward cultural changes, and behavioural changes in the wake of increasing
urbanisation, employment and emotional uncertainty, social media generated online
bullying and peer pressure.
a) Attitudes and beliefs about mental illness can be shaped by multiple factors:
• Knowledge acquired through keenness
• Experience/interaction with someone living with mental illness
• Cultural stereotypes
• Media stories
• Familiarity with institutional practices and past restrictions (e.g., employment
restrictions)
Hence, the prevalent societal attitudes towards mental illness vary from empathy
and caring to that of neglect, pity and abhorrent treatment like a less-worthy
human. Although the societal attitude towards mental illness though has slowly
been evolving, however, following ethical issues continue to remain. For
instance:
• Trivialisation: It is a minimising behaviour where an illness is conceptualised as
being easier to acquire, suffer with, or treat. Attitudes towards mental health
are still not on equal terms with those towards physical health.

DELHI JAIPUR PUNE HYDERABAD AHMEDABAD LUCKNOW CHANDIGARH


42 www.visionias.in # 8468022022 ©Vision IAS
Google it:- https://upscpdf.com
https://t.me/UPSC_PDF Download From > https://upscpdf.com https://t.me/UPSC_PDF

• Stigmatisation: It is a significant barrier for the early diagnosis and treatment Student Notes:
of various mental health conditions. People with mental health problems are
less likely to seek help if they feel their condition is stigmatised.
• Social Labelling: In a lot of cases depression is stereotyped as a ‘first world
problem’ or the ‘problem of luxury’ and not an issue in the third world – which
is untrue, as large number of students, female, and farmer suicides take place
in developing countries like India.

• Treatment issues and perceptional biases: Researching the diagnosis and


treatment of mental disorders also presents special ethical issues. For instance,
while dementia is perceived as quite straightforwardly ‘medical', other
ailments like personality disorders are sometimes treated and looked at
differently, with a stigma.
The society at large, beginning from the family, needs to be open about discussing
mental health issues. Consulting psychiatrist or medications should not be
considered taboo and immediate assistance and familial support must be provided,
for which district administrations can play a big role by creating sensitisation
campaigns. Interventions like happiness curriculum as introduced by the Delhi
government that included meditation; sports activities; reconnecting with nature
should be undertaken so as to hone young minds.
b) Mental health problem is a scientifically treatable disease. A disease has a cause,
an aggravating condition, and finally a treatment. Excessive pressure and
competition is a common cause of occurrence and its ignorance, a cause of
aggravation. The role of parenting, social media and video games for the younger
generation vis-a-vis mental health problems is as under:
• Parenting: Parental support and a nurturing family are key to addressing
mental health issues. The first step begins with a liberal environment in the
family where young adults, teenagers, can have free discussions with their
parents. Moreover, parents need to also understand that peer pressure, and
creating pressure over exam results, performance in competitive exams may
impact young minds and increase student suicide rates. Thus, a conducive
environment where the younger generation is encouraged to discuss issues
should be promoted, over and above dismissing beliefs related to mental
health treatments.
• Social Media: Social media addiction is one of the key factors responsible in
changing our behaviour towards others and alienating ourselves. While social

DELHI JAIPUR PUNE HYDERABAD AHMEDABAD LUCKNOW CHANDIGARH


43 www.visionias.in # 8468022022 ©Vision IAS
Google it:- https://upscpdf.com
https://t.me/UPSC_PDF Download From > https://upscpdf.com https://t.me/UPSC_PDF

media is a good platform towards connecting with people, and gather Student Notes:
information, over-dependence and over-consumption of it amount to
addiction, which influences the cognitive ability of adults, teenagers.
Moreover, things such as trolling, cyber bullying, perceptions about others, the
need to constantly post or upload – further isolate minds, while also limiting
good habits like reading, playing, or spending time with nature.
o Social media behaviour should be restricted and sensitisation campaigns on
limiting their usage must be undertaken by all schools, colleges, and work
environments.
• Video Games: According to the WHO, video-gaming disorder is a "pattern of
persistent or recurrent gaming behaviour" in which people lose control over
their gaming behaviour, resulting in impairments in their family relationships
and social lives and has recommended to classify gaming disorder as a separate
addiction. The Blue Whale Challenge game posed serious mental health risks in
the form of self-harm and attempted suicides.
It is therefore important to not dismiss mental health issues based on ill-conceived
notions and understand that stigmatisation will only worsen the silent epidemic of
depression. Additionally, ensuring proper implementation of the National Mental
Health Policy 2014, which puts renewed focus on depression, suicides and attitudes
towards mental health, shall also help in addressing these challenges.

Copyright © by Vision IAS


All rights are reserved. No part of this document may be reproduced, stored in a retrieval system or
transmitted in any form or by any means, electronic, mechanical, photocopying, recording or
otherwise, without prior permission of Vision IAS.

DELHI JAIPUR PUNE HYDERABAD AHMEDABAD LUCKNOW CHANDIGARH


44 www.visionias.in # 8468022022 ©Vision IAS
Google it:- https://upscpdf.com
ETHICS IN ADMINISTRATION- SOURCES, Student Notes:

APTITUDE AND FOUNDATIONAL VALUES

Table of Contents
1. Introduction ...........................................................................................................3
2. Source of Ethical guidance in Administration ..........................................................3
2.1 Indian sources of ethical guidance ............................................................................... 3
2.2. Western sources of ethical guidance ........................................................................... 4
2.3. Laws, Rules, Regulations as sources of ethical guidance .............................................. 4
2.3.1. How Laws, Rules and Regulation are a source of ethical guidance: ........................................... 5
2.3.2. Limitations of Laws, Rules and Regulations: ............................................................................... 5
2.3.3. Constitution of India as the supreme law of the land ................................................................ 6
2.4. Conscience as a source of ethical guidance ................................................................. 6
2.4.1. Types of Conscience ................................................................................................................... 7
2.4.2. Ethical Principles Governing Conscience .................................................................................... 8
2.4.3. Are public administrators bound to educate their consciences according to the responsibilities
they have? ............................................................................................................................................ 8
2.4.4 Crisis of Conscience: .................................................................................................................... 8

3. Need of Ethics for Public Administrators .................................................................9


4. Determinants of Ethics in Public Administration .....................................................9
5. Concept of Public Service ...................................................................................... 10
5.1. Measuring Publicness of a Service ............................................................................ 11
5.2. Principles Guiding Public Service .............................................................................. 11
6. Aptitude .............................................................................................................. 12
6.1. Definition and Explanation: ...................................................................................... 12
6.2. Relationship of Aptitude with other qualities ........................................................... 13
6.2.1. Aptitude and Interest ............................................................................................................... 13
6.2.2. Aptitude & Ability ..................................................................................................................... 14
6.2.3. Aptitude and Intelligence ......................................................................................................... 14
6.2.4. Aptitude and Attitude .............................................................................................................. 15
6.2.5. Aptitude and Values ................................................................................................................. 15
6.2.6. Aptitude, Proficiency & Achievement ...................................................................................... 16
6.3. Aptitude for Civil Services ........................................................................................ 16
7. Foundational Values for Civil Services ................................................................... 16
7.1. Definition and explanation ....................................................................................... 16
7.2. Need of foundational values .................................................................................... 17
7.3. Types of Foundational Values ................................................................................... 17
7.4. Major Foundational Values for Civil Services ............................................................. 17
7.4.1. Integrity .................................................................................................................................... 19
7.4.2. Impartiality ............................................................................................................................... 23
7.4.3. Non-Partisanship ...................................................................................................................... 24
7.4.4. Neutrality ................................................................................................................................. 25

DELHI JAIPUR PUNE HYDERABAD AHMEDABAD LUCKNOW CHANDIGARH


1 www.visionias.in # 8468022022 ©Vision IAS
https://t.me/UPSC_PDF Download From > https://upscpdf.com https://t.me/UPSC_PDF

7.4.5. Anonymity ................................................................................................................................ 27 Student Notes:


7.4.6. Objectivity ................................................................................................................................ 27
7.4.7. Dedication to Public Service ..................................................................................................... 29
7.4.8. Empathy, Tolerance and Compassion towards the weaker section.......................................... 30
7.4.9. Miscellaneous Values ............................................................................................................... 33
7.5. Consequences of Erosion of Values ........................................................................... 34
8. Previous Years UPSC GS Mains Questions ............................................................. 34
9. Previous Years UPSC Mains Questions: Case Studies ............................................. 36
10. Previous Years Vision IAS GS Mains Questions .................................................... 36
11. Previous Years Vision IAS GS Mains Questions: Case Studies ............................... 41

Copyright © by Vision IAS


All rights are reserved. No part of this document may be reproduced, stored in a retrieval
system or transmitted in any form or by any means, electronic, mechanical, photocopying,
recording or otherwise, without prior permission of Vision IAS

DELHI JAIPUR PUNE HYDERABAD AHMEDABAD LUCKNOW CHANDIGARH


2 www.visionias.in # 8468022022 ©Vision IAS
Google it:- https://upscpdf.com
https://t.me/UPSC_PDF Download From > https://upscpdf.com https://t.me/UPSC_PDF

Note- This document shall cover the following topics from the UPSC syllabus- Student Notes:
• Ethics in Public Administration; Public/Civil Service Values; Laws, Rules, Regulations and
Conscience as Sources of Ethical Guidance
• Aptitude and Foundational Values for Civil Service, Integrity, Impartiality and Non-
partisanship, Objectivity, Dedication to Public Service, Empathy, Tolerance and Compassion
towards the weaker sections.

1. Introduction
Professional ethics of a public administrator encompasses the standards of behaviour and
conduct that are crucial for the performance of his/her public duties. It includes ideas and
actions to make things right and a behaviour that reflects a sense of public administration.
Ethics in public administration traditionally has been divided into two schools, the normative
and the structural.
• Normative perspective: Drawing upon organizational development, examines how ethical
values are inculcated and put into operation in organizations.
• Structural approach: It probes formal-legal arrangements, primarily regulatory and legal
prescriptions and prohibitions through which governments seek to channel and control
administrators' behavior.
The object of ethics in public administration is to instill a sense of professional morality in the
administrator to perform his duty in a manner that conforms to the highest ethical standards.

2. Source of Ethical guidance in Administration


There are various sources which show us the light in terms of what constitutes ethics in
administration. Firstly, we shall discuss some contributions from India and West. After that, we
see how laws, rules, regulations and lastly conscience can act as a source of ethical guidance.

2.1 Indian sources of ethical guidance


There are various contributions from India, where one can draw important facets for ethical
guidance in administration such as-
• Concept of Dharma: The idea of the State was based on dharma as it laid emphasis on
duty and righteousness.
o The main purpose of the State is welfare of the nation, protection for the vulnerable
sections of the society.
o Indian theorists stressed upon the prime necessity for the ruler and his ministers of
conquering personal desires for pleasure and power.
▪ The Mahabharata says self-discipline and the conquest of the self is greatest of
dharma.
o Rather, the state should aspire to achieve the following-
▪ Prabhava — for promoting advancement and growth of all beings;
▪ Sarankshan — for maintaining and giving security to all beings;
▪ Ahimsa — Freedom from violence.
• Kautilya- In his treatise on statecraft i.e. Arthashastra, he has discussed various aspects of
ethical governance and elaborated on the roles and duty of every stakeholder in the state.
Some these can be seen as-
o Code of conduct- It prescribes a detailed code of conduct for the king.
o Traits of an ideal king- The king should have self-control, open mindedness and seeks
advice from his council of ministers.
▪ He should stay ever active in promoting the security and welfare of the people.
▪ He ensures the observance of dharma by the people, by authority and by example.

DELHI JAIPUR PUNE HYDERABAD AHMEDABAD LUCKNOW CHANDIGARH


3 www.visionias.in # 8468022022 ©Vision IAS
Google it:- https://upscpdf.com
https://t.me/UPSC_PDF Download From > https://upscpdf.com https://t.me/UPSC_PDF

▪ He improves his own discipline by continuing his learning in all branches of Student Notes:
knowledge and endears himself to his people by enriching them and doing good to
them
o Importance of civil service- in providing good administration. Only persons with
unimpeachable character, possessing highest personal qualities of leadership, intellect
and energy, will seek service with the king.
o Equity in Taxation: While emphasising need for wealth collection, Kautilya underlines
the principle of equity in taxation.
• Mahatma Gandhi- Mahatma Gandhi’s philosophy, applied in its true spirit, provides
answers to the problem of social and economic crisis which India as well as the entire
global community is currently facing. Some critical guidance can also be taken from his
principles such as-
o Decentralized Governance/Self Governance- Gandhi believed in a decentralized
development model as it helps consequences of development reach everyone and
promotes equality and social harmony
o Social Control of Business- Gandhi was not opposed to private ownership of industries
and business, he advocated social control of business so that profit generated are
equitably distributed and prevents concentration of wealth in the hands of few people,
which leads to exploitation
o Rural-centric governance: Agriculture and small-scale industries being the prime
employer in the rural area should be focused in development agendas.

2.2. Western sources of ethical guidance


Some important contributions were made by western thinkers and theories, such as-
• Virtue Ethics- It talks about external and internal goods.
o The creation of internal good by virtue ethics may at time hinder the development of
external goods. For example, cultivation of justice and fairness within the institutions
may harm the private interests of the stakeholders.
o The external good here can be riches, power or fame. In such circumstances, political
rulers and others would punish public administrators for acting with virtue.
o Virtue Ethics, however has internal rewards, internal good such as contentment,
happiness, job satisfaction etc that no one can take away from the public administrator.
• Paul Appleby’s Idea of Democratic Morality- As per him, a free state, where the various
private interest think only about maximizing their own good and refuse to consider
common good, is likely to self-destruct over time unless the nation’s leaders and the
bureaucracy act to focus public policy on the public interest.
o For a free state to remain a free state, virtuous public servants, who shared common
social ethics of concern for the public interest, were essential.
o To avoid the moral corruption of the society out of inherent conflicts of interests, public
servants must maintain democratic morality.
• Weber’s Idea of Bureaucratic Morality- As per him, bureaucracy should be a goal-oriented
institution designed according to a rational principle. The bureaucrats should be guided by
explicit rules and responsibilities which should override moral and ethical norms of
personal conscience.
• United Nations Code of Conduct for Public Officials categorically states that a public office
is ‘a position of trust, implying a duty to act in public interest’ and ‘public officials shall
ensure that they perform their duties and functions efficiently, effectively and with
integrity, in accordance with laws and administrative policies.

2.3. Laws, Rules, Regulations as sources of ethical guidance


Law as used in ethics is different from the notion of law in physics, which implies a common or
constant way of action. In ethics, law has a moral connotation. For instance, it has been

DELHI JAIPUR PUNE HYDERABAD AHMEDABAD LUCKNOW CHANDIGARH


4 www.visionias.in # 8468022022 ©Vision IAS
Google it:- https://upscpdf.com
https://t.me/UPSC_PDF Download From > https://upscpdf.com https://t.me/UPSC_PDF

defined as "an ordinance of reason for the common good, promulgated by him who has care Student Notes:
of the community." (St. Thomas Aquinas).
Laws in modern context are synonymous with positive human laws. Laws are those basic
ethical standards that the society expects everyone to comply to. There are sanctions against
their violation, which are generally in the form of duly enforceable punishments.
Purpose of a law is to promote the common good and societal welfare as well as protect
individual rights.
Difference between law and regulation or ordinary rule:
• Although law is an ordinance or a rule resulting from human reason, it is not the same as a
regulation or ordinary rule. Regulations and rules often help clarify the intent of laws in
more specific terms.
• Unlike laws, individuals, organizations or groups can make rules. It must be clarified again
that rules are made under laws. Rules need not be for the common good; they can be for
the private good and they usually bind persons wherever they go. But rules, too, must not
violate natural law. Rules or regulations should declare or clarify civil laws, just as civil laws
declare or clarify natural law. Rules and regulations can be extra guidance to public
administrators as to what is right and wrong.
2.3.1. How Laws, Rules and Regulation are a source of ethical guidance:
• Regulate discretionary use of resources to fulfill personal needs.
• Laws command both action and inaction: Some laws lay down what should not be done,
for e.g. murder, whereas others lay down what should be done, for e.g. registration of
motor vehicles.
• Help clarify the values to be used and procedure to be followed.
• Modern laws are founded on the libertarian principles and therefore foster, liberty, equity,
equality and social justice.
• Rules and regulation for example communitarian laws have legal sanction and therefore
are embodiment of social values which deserves preservation.
2.3.2. Limitations of Laws, Rules and Regulations:
• Teleology says that we do not need standards to govern human behavior; the human
intellect alone is capable of knowing and judging what is right and what is wrong.
• There are so many laws, rules and regulations that govern human behavior that it is
virtually impossible for any human being to know all of them.
o Information, reflection, judgment, decision and action are the criteria for determining
morality. This approach makes sense and is a valid response to the fact that knowledge
of all laws and rules is virtually impossible.
• The presumption is that rule makers have not violated natural law or civil laws, but
sometimes they circumvent what the civil law clearly states.
o While a superior can punish a subordinate for violating the rules, if the rule is contrary
to civil or natural law, the violator may have acted ethically. As with laws, a person has
no obligation to obey an immoral rule
While deontologists are content with focusing on laws and rules as the principal guidance for
public administrators on morality, even they recognize that laws and regulations are
insufficient. Without conscience to apply those laws and rules to particular actions, public
administrators are missing a critical element. So, now we examine conscience as a mechanism
for deciding what is right and what is wrong.

DELHI JAIPUR PUNE HYDERABAD AHMEDABAD LUCKNOW CHANDIGARH


5 www.visionias.in # 8468022022 ©Vision IAS
Google it:- https://upscpdf.com
https://t.me/UPSC_PDF Download From > https://upscpdf.com https://t.me/UPSC_PDF

When is a law doubtful? Student Notes:


There are four principles that apply and the actor is at liberty to follow the principle that
appeals most.
• A law is doubtful and does not bind when there is more probable evidence on the side of
liberty than against it. This is probabilism. For example, a person in doubt about what day it
is observes four calendars. Three indicate it is one day and the fourth indicates that it is a
different day. The person may follow the date indicated by or deduced from the three
calendars if that ensures more liberty.
• A second version of probabilism states that the person may follow an option in favor of
liberty, provided the evidence in favor of liberty is solidly probable, even though the
evidence against liberty is more probable. In the same example, the person may follow the
time indicated by the fourth calendar even though the other three numerically appear to
offer more probable evidence.
• Another version of probabilism, equiprobabilism states that the person may follow an
opinion in favor of liberty if the evidence on both sides is equally balanced. In the above
example, if two calendars show that it is one day and the other two indicate that it is a
different day, the person may follow either option.
• Compensationalism says that the person should consider the evidence not only favoring
and opposing liberty but also the gravity of the law, the reason for acting against the law,
the inconvenience arising from following the strict interpretation of the law and the
justness of the cause for selecting the option offering most liberty.
Some laws may be doubtful, i.e. not clearly formulated with scope of misinterpretation, and
provide options for people. These serve as additional guidelines to the principles of conscience.
2.3.3. Constitution of India as the supreme law of the land
The Preamble to the Indian Constitution promises securing to all citizens: Justice, social,
economic, and political; Liberty of thought, expression and belief, faith and worship; Equality
of status and opportunity; and promote Fraternity, assuring dignity of the individual.
These ideals can be secured only if holders of public office, both the elected representatives as
well as public servants, entrusted with the task of running the government have ethical
moorings.
Constitutional Morality
The term ‘constitutional morality’ has often been invoked by Supreme court in India for striking
down laws which could be termed as manifestations of popular morality. But this term is not
found in our Constitution. Nevertheless, we find mention of the word ‘morality’ in the Indian
constitution at various places (Article 19, 25 and 26). Dr. B R Ambedkar used ‘Constitutional
Morality’ multiple times in ‘Parliamentary debates.
Constitutional morality is the morality of the constitution. It is the substantive content
of a constitution in the form of ethical ideas that underlies formal provisions. To be governed by
constitutional morality for a society is to be guided by ethical ideas carried by constitution e.g.
justice, liberty, equality and fraternity can be taken as element of our modern constitutional
morality.

2.4. Conscience as a source of ethical guidance


Conscience is a special act of the mind that comes into being when the intellect passes
judgment on the goodness or badness of a particular act. It is a practical judgment on
particular, concrete, human actions.
Deontological perspective: Conscience is a judgment—an act of the intellect. It is not a feeling
or an emotion, but, rather, an intellectual decision. It is also a decision with a view to a
particular action. Conscience can make a practical judgment on the morality of either a past

DELHI JAIPUR PUNE HYDERABAD AHMEDABAD LUCKNOW CHANDIGARH


6 www.visionias.in # 8468022022 ©Vision IAS
Google it:- https://upscpdf.com
https://t.me/UPSC_PDF Download From > https://upscpdf.com https://t.me/UPSC_PDF

action or an action about to occur. Student Notes:


Teleological viewpoint: Conscience is quite similar to completion of the ego identity, whereby
Every ego is in some sense a code of ethics. If ego and conscience are similar or identical,
people have the capacity of determining the meaning of a particular action, past or present,
and at the same time assess the morality of that action.
Difference between Conscience and Law:
• While law focuses on principles of morality outside human beings, conscience is something
within human beings that determines the morality of human actions
• Law states a general rule concerning actions; conscience lays down a practical rule for
specific action. Conscience applies the law or rule to specific actions, therefore it is wider
than law. Some have said that conscience is to law as a brush is to paint.

2.4.1. Types of Conscience


Human beings can have different kinds of conscience.
(a) True conscience: It means that judgment is in accordance with fact. The judgment is a
correct or accurate application of law to the action.

DELHI JAIPUR PUNE HYDERABAD AHMEDABAD LUCKNOW CHANDIGARH


7 www.visionias.in # 8468022022 ©Vision IAS
Google it:- https://upscpdf.com
https://t.me/UPSC_PDF Download From > https://upscpdf.com https://t.me/UPSC_PDF

(b) Erroneous Conscience: when the judgment is false—the practical judgment incorrectly Student Notes:
applies law to the action. The erroneous judgment can be vincibly or invincibly false.
(Vincibly false means that it can be corrected i.e. it is not invincible)
(c) Certain Conscious: A conscience is certain when the judgment on the morality of an action
is without prudent fear of error. Prudent fear of error does not involve metaphysical
certainty, but generally any normal person has no doubts about the judgment. That
certainty can apply to both a correct and an erroneous conscience.
(d) Doubtful Conscience: Conscience is doubtful when the judgment does not exclude all
prudent fear of error. The person is aware of some doubts about the practical judgment to
be made.
(e) Both Doubtful and Erroneous- A conscience can be both doubtful and erroneous at the
same time. A conscience is probable when the judgment "almost" excludes all prudent fear
of error. A normal person is almost certain the judgment is correct, even though it may be
erroneous.
2.4.2. Ethical Principles Governing Conscience
The discussion of conscience leads to the following principles governing conscience:
• A person must take reasonable care to ensure a correct conscience.
• A person is bound to follow a certain conscience even if that conscience is false. For
example, if I am certain that it is morally right to lie to save another's life, I am bound to lie.
• It is never ethically correct to act on a doubtful conscience. Vincible ignorance does not
excuse—the person must make some effort to resolve the doubt. If efforts to resolve the
doubt fail, the principle lexdubianonobligat ("a doubtful law does not bind") comes into
play.
2.4.3. Are public administrators bound to educate their consciences according
to the responsibilities they have?
If public managers must not only do things right but also do what is right, they have an
obligation to educate their consciences according to their state in life. This includes not only
management theory and practice but also ethical theory and practice.
• If managers do not do both, they run the risk of not only being outdated but also of
neglecting true managerial responsibility. If managers are educators and teachers, surely
they must learn both aspects of the job if they are to fulfill their role of teaching and
coaching others.
• In educating and updating the conscience, there are two extremes to be avoided. One is not
caring about conscience at all—making no effort to learn what is right or what is wrong, or
perhaps showing no interest in right and wrong. Some public managers exhibit this
characteristic.
• The other extreme is the person unable to distinguish serious actions from those that are
not, whether getting things done right or doing the right thing. There are some public
managers who fit this description. Neither extreme is in accord with the concept of
conscience, which involves a practical judgment on the morality of human action.
2.4.4 Crisis of Conscience:
When conscience fails to provide moral guidance to an individual in any specific situation as a
result of a particular experience, it can be called as Crisis of Conscience. Individuals loses the
ability to determine right and wrong.
It quite happens in a situation which is ambiguous in terms of values involved and its
consequences. As a result, individual cannot resolve an ethical dilemma using his moral sense.
For instance, an IPS officer is given the duty to regulate the peaceful protest by farmers against
some corporate bodies. Things are under control as situation is clearly understood. Now

DELHI JAIPUR PUNE HYDERABAD AHMEDABAD LUCKNOW CHANDIGARH


8 www.visionias.in # 8468022022 ©Vision IAS
Google it:- https://upscpdf.com
https://t.me/UPSC_PDF Download From > https://upscpdf.com https://t.me/UPSC_PDF

suddenly officer is asked to resort strict measures irrespective of nature of protest. Officer now Student Notes:
faces the ethical dilemma as situation is now ambiguous both in terms of the motive and the
consequence of actions. This can create crisis of conscience for an IPS officer as he/she could
not solely rely on his sense of right/wrong.
It is a situation in which someone feels worried or uncomfortable because they have done
something which they think is wrong or immoral.

3. Need of Ethics for Public Administrators


Compared to the political, legal, technical and financial dimensions of public administration, the
ethical dimension has been sorely neglected in Indian Administration. Further, much of the
recent public and media concern about public service ethics has centered on conflict of
interests and, to a lesser extent, on issues of political partisanship, public comment and
confidentiality.
The need of ethics for public administrators can be highlighted in the following points-
• Benevolence to all: Our constitution provides equality and equity as fundamental virtues to
be followed by administrators. This ensures that vulnerable get the protection and might
shall not be right. Ethical administrators are important to achieve these goals.
• To stay responsive and responsible- Ethics in public administration will help bureaucrats
use power in a responsive and responsible manner. This will improve sensitivity of the
administration. It will eventually increase/enhance the public trust in the administration.
• To resolve dilemmas: which relate less to the use of public office for private, personal, or
partisan gain and more to ethical and value conflicts and dilemmas that arise in the
performance of administrative duties.
• To fight against populist and majoritarian tendencies: Public administrator must have the
courage to face down the expert opinion that the expert cannot defend rigorously, and
sometimes resist that clamoring public opinion, powerful interest groups or the media. This
courage is the subject of Ethics in Public administration.
• To be optimistic: Public administrator must be able to face the ambiguity and the
paradoxical nature of ethics without being immobilized by them. They must be purposive in
their behavior and rather than reactive and most important, they must remain ever hopeful
in their outlook.
• Recognition of paradox of procedures: Those procedures which facilitate deliberations,
fairness, accountability, openness, equity and greater depth of thoughtful analysis to a
public decision are also at times the enemy of progress and dispatch. For example
Environment Impact Assessment are seen as just deliberation by the environmentalists but
are considered the enemy of progress and the means to slow proper actions of the
industrialist. Ethics help bureaucrats identify such paradoxes. Public servant who cannot
recognize the paradoxes of procedures will be trapped by them

4. Determinants of Ethics in Public Administration


The major determinants of administrative conduct in the public sector include:
1) Political construct: The public administrators are a subset of the political construct. They
contribute at both, the executive level and at the policy formulation level. Political ethics
therefore is intrinsically woven with the bureaucratic or administrative ethics. For example
Criminalization of politics adversely impacts the administrative behavior.
2) Legal Rational Framework: Weber gave a legal rational framework to the bureaucracy. The
role of hierarchy is important in fixing the accountability, impersonal order was important
for the bureaucrats to do their work without any fear and favor. Rules and regulations to
regulate administrative behavior and to impart, rationality, impartiality, honesty and
integrity in the administration.

DELHI JAIPUR PUNE HYDERABAD AHMEDABAD LUCKNOW CHANDIGARH


9 www.visionias.in # 8468022022 ©Vision IAS
Google it:- https://upscpdf.com
https://t.me/UPSC_PDF Download From > https://upscpdf.com https://t.me/UPSC_PDF

3) Public servant’s Intention: Public servants’ behavior is the key to the ethical nature of Student Notes:
administration. No matter how good the intentions of the policy makers be, if the
mechanism to implement it is corrupt, politicized, partisan, prejudiced or stereotypical in
nature, the intention will cease to matter. For example, Using Aadhar for direct benefit
transfer is an innovative idea, however not allowing a needy person ration without Aadhar
will jeopardize the whole effort and bureaucracy will be termed as insensitive.
4) Citizenry and Civil society: Citizenry hold administration accountable for its acts of omission
and commission. It provides the much-needed feedback which helps in evolution of
bureaucratic and political response. Role of civil society is to scrutinize the government and
its actions and inactions plus sensitize the citizenry about the same. It helps in Ethical
decision making.
The minor and discreet elements of administrative conduct are guided by-
• Individual attributes of public/civil servants which guide ethical decision-making skills,
mental attitude, virtues, and professional values.
• Organizational structure which is explained by clear accountability, collaborative
arrangements, dissent channels, and participation procedures.
• Organizational culture includes artefacts, beliefs and values, and assumptions. Ethical
behaviour is encouraged when organizations have a climate where personal standards and
employee education are emphasized, where supervisors stress the truth, and where
employees regularly come together to discuss ethical problems.
• Leadership is important in the development, maintenance, and adaptation of
organizational culture.
• Societal expectation includes public participation, laws, and policies.
OECD has developed an ethical infrastructure (refer the infographic below) which can be taken
as a framework to include all the above determinants.

5. Concept of Public Service


Public service is a public trust. Public service according to the modern definition goes beyond
the administrative functions of the governments and incorporates non-governmental
organization and institutions as well.
Citizens are the source of sovereignty and importantly, are taxpayers demanding accountability.
They should be treated as such and not merely as consumers or customers in the marketplace.
Markets are not alternatives to public service. Rather, sound governance, public service, and
responsible citizenship are sine qua non of a business-friendly environment, the smooth
operation of markets, effective democracy, and social peace.

DELHI JAIPUR PUNE HYDERABAD AHMEDABAD LUCKNOW CHANDIGARH


10 www.visionias.in # 8468022022 ©Vision IAS
Google it:- https://upscpdf.com
https://t.me/UPSC_PDF Download From > https://upscpdf.com https://t.me/UPSC_PDF

Primary responsibility of civil servants is enforcement of law, provision for various public Student Notes:
services such as food, health, education, relief during disasters, etc. Also, they are responsible
for settlement of disputes and enforcement of contractual obligations. These duties boil down
to exercise of power over other citizens, conferred under the statutes, which civil servants
exercise in their own right under the law.
They also implement programmes and policies of the government. This requires a certain level
of character, a sense of justice and fairplay, transparency and accountability. To be able to
deliver administrative justice, capacity for critical thinking and fairness and respect for
procedures are essential qualities which must be cultivated for efficient delivery of all type of
public services.
Public service is concerned with provisioning for the needs of public as per the expectations of
the public. It represents one of the oldest, if not the noblest, most widespread and celebrated
values in the history of humankind. In fact, civilization and public service and administration
are intimately related, one reinforcing and contributing to the other.
In traditional societies of the East and West, little distinction was made between voluntary
service for the good of the community and remunerated work performed on a professional
contractual basis. However, with development of the concept of State, a structured civil
service has been central to its (State’s) idea. From the times of Mauryan Kingdom, we have had
an elaborate public service performing public duties.

5.1. Measuring Publicness of a Service


The expectations of public i.e. ordinary citizens from the government have been changing.
Moreover, many private entities are providing services which were traditionally considered as
‘public’. According to M. Shamsul Haque of the University of Singapore, following are the
major dimensions along which ‘Publicness’ of a service can be measured-
• The extent of its distinction from the private sector: Haque identifies impartiality,
openness, equality and representation as being distinctly public characteristics.
• The scope and composition of service recipients: the greater the number and broader the
scope of service recipients, Haque writes, the higher the degree of publicness, and he refers
to a 'shared and universally accessible domain involving the interest of all citizens'.
• The magnitude and intensity of its socioeconomic role: the wider a service's societal
impact, the greater the degree of its publicness.
• The degree of its public accountability: this goes beyond the existence of institutions to the
extent to which those institutions are influenced by particular classes or sections of society.
• The level of public trust: that is, how much people trust the credibility, leadership or
responsiveness of a service.

5.2. Principles Guiding Public Service


Today, public sector is employing number of ways of private sector to enhance their efficiency.
Does it mean that the spirit behind the public service may get weakened? In this regard, it is
essential that the following principles should guide all the civil service institutions and their
personnel:

DELHI JAIPUR PUNE HYDERABAD AHMEDABAD LUCKNOW CHANDIGARH


11 www.visionias.in # 8468022022 ©Vision IAS
Google it:- https://upscpdf.com
https://t.me/UPSC_PDF Download From > https://upscpdf.com https://t.me/UPSC_PDF

Student Notes:

There are values which are supposed to take care in public life and administration and that is
why correct and appropriate aptitude and foundational values are required the self, according
to those values. In every administration, aptitude will play a very important role.

6. Aptitude
6.1. Definition and Explanation:
Our experiences shape our thinking in unique ways and this subconscious thinking shapes our
preferences, ultimately influencing (but not determining) how we will respond to an event in
future. Innumerable characteristics influence a person's behavior. The concept of aptitude is
applied to those which are considered as useful for particular task.
In a simpler way, Aptitude is the natural ability to do something, a natural tendency. ‘Natural’
here refers more towards the sum total of experiences that a person has had rather than simply
meaning ‘by-birth’. A person with an aptitude for boxing, if given necessary training, can
develop the ability to a great extent. On the other hand, if a person has no aptitude for boxing,
no amount of training can bring about any appreciable change in his performance.

DELHI JAIPUR PUNE HYDERABAD AHMEDABAD LUCKNOW CHANDIGARH


12 www.visionias.in # 8468022022 ©Vision IAS
Google it:- https://upscpdf.com
https://t.me/UPSC_PDF Download From > https://upscpdf.com https://t.me/UPSC_PDF

Thus, Aptitude is the combination of abilities and other Student Notes:


characteristics, whether native or acquired, known to
be indicative of an individual’s ability to develop
proficiency in certain particular area.
Aptitude is different from skill
Aptitude is a group of characteristics deemed to be
symptomatic of an individual’s ability to acquire
proficiency in a given area. It thus has a futuristic reference. Skill, on the other hand, is
acquired through training. Aptitude is not skill. Skill is acquired and constantly upgraded
through training and learning. Aptitude is what is there but could be nurtured.
To become an officer in defense or police services, you need to have both physical and mental
aptitude; for being a good sportsman one needs aptitude of psycho-motor coordination.
Problem arises when one is in a profession not suited by his/her aptitude. A person becoming a
cricketer because his father was a great may not be able to deliver best results unless he
himself has the aptitude required for the sport. Remember carefully – aptitude looks at the
future potential. It, however, is the present condition – a pattern of traits which are there
presently and are deemed to be indicative of potentialities.
Aptitude is generally categorized as physical or mental.
• Physical aptitude means the physical characteristics for performing some task successfully.
For example, armed forces require a specific set physical features, like height, strength etc.
• Mental aptitude means certain specific set of mental qualities needed to perform some
tasks successfully. This is further characterized as general mental ability and value
orientation. The former implies an ability to think rationally, while the latter also includes
certain value-based behavior, like the one guided by empathy, compassion, integrity,
accountability, responsibility etc.
This difference can be understood with some examples. For example, a banker should have the
ability to deal in numbers, while an effective leader is one who is empathetic and honest. A
thief, on the other hand, probably needs to be a good liar.

6.2. Relationship of Aptitude with other qualities


6.2.1. Aptitude and Interest
People have interests in many things, but this does not imply they have the aptitude for that.
One may like cricket very much – but playing it on field is much different that a watching it on
television. One may have the aptitude for good commentary or writing skills and then one may
choose a profession where his/her interests and aptitude match- such as becoming a
commentator or a sports journalist.

DELHI JAIPUR PUNE HYDERABAD AHMEDABAD LUCKNOW CHANDIGARH


13 www.visionias.in # 8468022022 ©Vision IAS
Google it:- https://upscpdf.com
https://t.me/UPSC_PDF Download From > https://upscpdf.com https://t.me/UPSC_PDF

Student Notes:

6.2.2. Aptitude & Ability


Aptitude is often considered to represent specific subsets of mental ability which provides
useful information on an individual’s potential, particularly with regards to education and
employment. Ability is much closer to intelligence. It is concerned with present. It is the
combination of skills, habits and powers that an individual now has which enable him to do
something.
Aptitude indicates what an individual will be able to learn/do and ability presents evidence of
what the individual is able to do now (or in future without additional training). From a practical
standpoint, it is impossible to measure aptitude without the degree of training which an
individual has already had. It is assumed that individuals have been exposed to certain
experiences which enhance their aptitude when it is measured.
For example, the CSAT (Civil Services Aptitude Test) exam which is meant to test the aptitude
of a candidate for Civil Services will invariably show better scores for candidates who have
practiced rather than those who have not. It tests aptitude by subjecting the candidates to
pressure situation (solving many questions in limited time), which they are likely to experience
once they become Civil Servants. Moreover, a person who has practiced the tests demonstrates
readiness and labour, which is again a trait desirable in a Civil Servant. It therefore, although
intended to measure aptitude, also measures ability.
Thus, according to some psychologists, ability includes aptitude and achievement. Ability refers
to what a person can do today. It is the power, at a given time, to perform.
6.2.3. Aptitude and Intelligence
Aptitude is innate potential to do certain task in future, provided appropriate training is
provided to the person. Intelligence’s key component is the ability to think rationally, act
purposefully and deal effectively in one’s present environment.
Intelligence has a wider scope as in it refers to general mental ability. Aptitude on the other
hand has narrow scope- it is specific to work. It breaks down intelligence into several
characteristics, referring to a specific ability of a person to do a task in future. Thus, aptitude is
not same as intelligence. Two people with same intelligence quotation (IQ) may have different
aptitudes e.g. one to become scientist and another novelist.

DELHI JAIPUR PUNE HYDERABAD AHMEDABAD LUCKNOW CHANDIGARH


14 www.visionias.in # 8468022022 ©Vision IAS
Google it:- https://upscpdf.com
https://t.me/UPSC_PDF Download From > https://upscpdf.com https://t.me/UPSC_PDF

6.2.4. Aptitude and Attitude Student Notes:

Example: In public service, aptitude may help a civil servant achieve career positions. For
instance, someone working very hard on sanitation best practices, who has the natural aptitude
to understanding issues of sanitation may get a posting with the central government, purely on
the basis of their aptitude and acquired experience.
Attitude in times of say disaster management exercises being undertaken in a district, may
determine the leadership abilities of an individual facilitating stress management and overall
composure of people in times of distress.
6.2.5. Aptitude and Values
Value denotes the degree of importance of some thing or action, with the aim of determining
what actions are best to do or what way is best to live. Though values may be “positive” or
“negative”, like empathy towards weaker section is positive while regarding one’s own caste to
be highest is negative. But generally, they are taken for positive and desirable values.
Values tend to influence attitudes and behavior. Types of values include ethical/moral values,
doctrinal/ideological (religious, political) values, social values, and aesthetic values. In the next
section we will discuss “Foundational Values for Civil Services.”
Relationship between Aptitude and values:
From the point of view of ethics, aptitude concretely exist in the form of desired values in a
person i.e. what is important in person's behaviour with respect to specific requirement e.g.
aptitude for civil service can be measured in terms of ability to perform, deliver, empathize and
exhibit integrity, emotional intelligence in performance of public services.

DELHI JAIPUR PUNE HYDERABAD AHMEDABAD LUCKNOW CHANDIGARH


15 www.visionias.in # 8468022022 ©Vision IAS
Google it:- https://upscpdf.com
https://t.me/UPSC_PDF Download From > https://upscpdf.com https://t.me/UPSC_PDF

6.2.6. Aptitude, Proficiency & Achievement Student Notes:


Proficiency is the ability to perform an activity with ease and precision. Achievement looks at
the past - what has been done, accomplished

6.3. Aptitude for Civil Services


While Aptitude tells us about the concrete values that can be realized (as we have discussed in
above section such as intellectual aptitude via CSAT etc), there are certain ideal values that
underlies civil service. Civil servants need to demonstrate moral aptitude for them.
Moral aptitude for ethical behaviour is the precondition for a public functionary to understand
the normative requirement of the job.
It helps to understand the role, its associated responsibilities and institutional environment
easily e.g. an individual who is honest, rational and believes in fairness of conduct will more
tended to understand the normative goals of integrity and accountability in the service.
Following section will deal with foundational values for civil services to understand the same.

7. Foundational Values for Civil Services


7.1. Definition and explanation
Value are a set of standards, on the basis of which, we judge things as right or wrong. There are
many values, and these may be arranged in hierarchy. For example, Gandhi regarded truth as
the highest value followed by non-violence.

DELHI JAIPUR PUNE HYDERABAD AHMEDABAD LUCKNOW CHANDIGARH


16 www.visionias.in # 8468022022 ©Vision IAS
Google it:- https://upscpdf.com
https://t.me/UPSC_PDF Download From > https://upscpdf.com https://t.me/UPSC_PDF

Civil service values are contextual to a culture. Democratic cultures, for example, are based on Student Notes:
the doctrine of public trust- that the sovereign power lies with the public, who has entrusted it
to the State, which must exercise that power through officials (appointed and elected) in the
best interest of the public.
Values are also contextual to time, i.e. they evolve. For example, civil services have now started
focusing on improving business environment, which may not have been the case earlier.

7.2. Need of foundational values


• To Uphold the Public Interest: Civil servants hold the public positions with wide-ranging
powers. They come with the special obligations because as they are responsible for
managing resources entrusted to them by the nation. They provide and deliver services to
the public and take important decisions that affect all aspects of a nation’s life.
Consequently, the public has a right to expect that the civil service functions fairly,
impartially and efficiently. It is essential that the people must be able to trust and have
confidence in the integrity of the civil service decision-making process.
• To stand up to the expectations reposed in the office: It ensures that the decisions and
actions of civil servants reflect the policies of the government of the day and the
governance standards. The expectation that the civil servants should maintain the
standards of professionalism, responsiveness and impartiality while serving successive
political governments is a key element of the way our democratic polity functions.
• To uphold the constitution, its laws and underlying values: To fulfill this, Civil Servants
must uphold the constitution, its laws and underlying values. To fulfill this, civil servants
must inculcate such values to guide their conduct. Particularly in the situations where
discretionary decisions have to be taken, it is expected that civil servants should be guided
by those values. In absence of such values, there are chances of corruption as well as
possibilities of abuse of power. These concerns are also raised by various Judicial
pronouncements as in Vineet Narain case by Justice J.S Verma.

7.3. Types of Foundational Values


Foundational values for civil services may be categorized into two main groups:
• End-oriented values: These are concerned with the goals which civil servants should aim to
achieve while performing their duties. They relate to the end product, and may best be
visualized as those covered by Directive Principles of State Policy, Fundamental Rights etc.
• Means-oriented values: They are concerned with the ways in which civil servants behave or
act while performing their duty. They relate to the means and include such values as
transparency, responsiveness, efficiency etc.

7.4. Major Foundational Values for Civil Services


In India, the current sets of ethical norms are –
• the Conduct Rules, contained in the Central Services (Conduct) Rules, 1964 and
• analogous rules applicable to members of the All India Services or employees of various
State Governments.

DELHI JAIPUR PUNE HYDERABAD AHMEDABAD LUCKNOW CHANDIGARH


17 www.visionias.in # 8468022022 ©Vision IAS
Google it:- https://upscpdf.com
https://t.me/UPSC_PDF Download From > https://upscpdf.com https://t.me/UPSC_PDF

Student Notes:

The code of behaviour as enunciated in the Conduct Rules, while containing some general
norms like ‘maintaining integrity and absolute devotion to duty’ and not indulging in ‘conduct
unbecoming of a government servant’ is generally directed towards cataloguing specific
activities deemed undesirable for government servants. These conduct rules do not constitute a
code of ethics. The Draft Public Services Bill, 2007, proposed a necessary first step in evolving a
code of ethics. It states Values of Public Service as a set of values which will guide the Public
Servants in the discharge of their functions. These are:
• Patriotism and upholding national interest
• Allegiance to the Constitution and the law of the nation
• Objectivity, impartiality, honesty, diligence, courtesy and transparency
• Maintain absolute integrity
Despite not having a specific code of ethics in public service, there are various sources which
contain- explicitly or implicitly, the values expected in a civil servant. The foremost source is the
Constitution of India

DELHI JAIPUR PUNE HYDERABAD AHMEDABAD LUCKNOW CHANDIGARH


18 www.visionias.in # 8468022022 ©Vision IAS
Google it:- https://upscpdf.com
https://t.me/UPSC_PDF Download From > https://upscpdf.com https://t.me/UPSC_PDF

A general overview of the Civil Service Values Student Notes:


2nd ARC in its report ‘Ethics in Governance’ has elaborated on values expected in Civil Servants.
It also makes references to Civil Services in other countries like the U.K. and Australia.
One of the most comprehensive statements of what constitutes ethical standards for holders of
public office came from the Committee on Standards in Public Life in the United Kingdom,
popularly known as the Nolan Committee, which outlined the following seven principles of
public life: Selflessness, Integrity, Objectivity, Accountability, Openness, Honesty, and
Leadership.
Note- For further details, please refer the document on Probity in Governance.

7.4.1. Integrity
The word ‘Integrity’ originates from the word integer (Latin). It means being whole or
complete. In ethics, being a person with integrity means having the quality of being honest and
having strong moral principles that one refuses to change. It means having strong moral
standards and the determination not to lower them.
Integrity is regarded as
• a concept of consistency of actions, values, methods, measures, principles, expectations,
and outcomes.
• the honesty and truthfulness or accuracy of one’s actions.
• the integration of ideals, conviction, standards, belief and behavior.
Integrity in essence means adherence to principles. It is a three-step process:
• choosing the right course of conduct;
• acting consistently with the choice—even when it is inconvenient or unprofitable to do so;
• openly declaring where one stands.
All three are equally important – for example a thief can be consistent in his thoughts and
actions but has low moral standards and has not chosen the right course of action. It doesn’t
matter whether he himself believes it to be correct or not. Has he done the hard work of
ascertaining what is right or wrong? No. In any case, he fails the test of trustworthiness as well.
Integrity is the foundation of public trust that is essential for a person holding a public office.
Civil Servant should not act or take decisions in order to gain financial or other material benefits
for themselves, their family, or their friends. They must declare and resolve any interests and
relationships.

DELHI JAIPUR PUNE HYDERABAD AHMEDABAD LUCKNOW CHANDIGARH


19 www.visionias.in # 8468022022 ©Vision IAS
Google it:- https://upscpdf.com
https://t.me/UPSC_PDF Download From > https://upscpdf.com https://t.me/UPSC_PDF

Student Notes:

How is it Different from Honesty?


Honesty Integrity
• It simply means holding the facts • It means thinking and doing what is right at all times, no
as they are, i.e. upholding the matter what the consequences. When you have integrity,
truth. you are willing to live by your standards and beliefs even
• If you tell the truth, you possess when no one is watching, to live so that your thoughts
the quality of honesty. Being and behavior are always in harmony.
honest means choosing not to lie, • Therefore, it not only requires truthfulness (honesty) but
steal, cheat, or deceive in any way also the quality to do what is right in all circumstances
Honesty as a subset of Integrity:
Integrity implies reliability; consistency in one’s principles and values. Honesty then becomes a
necessary, though not a sufficient, criteria for integrity.
To be integral one has to be honest, but not vice-versa. Put simply, one can be honest in an act,
but one needs to be integral in his/her whole conduct.
Honesty without integrity:
• Integrity requires one to face challenging situations. For example, in the battle of
Mahabharata Arjun was confused whether to follow dharma/righteousness/duty or to
follow kinship/inclination. Such conflict between duty (the right thing to do) and inclination
is a critical test to determine whether a person has high or low integrity.

DELHI JAIPUR PUNE HYDERABAD AHMEDABAD LUCKNOW CHANDIGARH


20 www.visionias.in # 8468022022 ©Vision IAS
Google it:- https://upscpdf.com
https://t.me/UPSC_PDF Download From > https://upscpdf.com https://t.me/UPSC_PDF

• Honesty on the other hand does not require such a context. One can be honest in day to Student Notes:
day life without necessarily showing integrity. Integrity means soundness of moral
character, as well as honesty – there's more to integrity than honesty alone.
• The major difference between honesty and integrity is that one may be entirely honest
without engaging in the thought and reflection integrity demands.
• The honest person may truthfully tell what he or she believes without the advance
determination of whether it is right or wrong. For example, a person may say “all athletes
cheat” and really believe in this statement -that’s honesty. But, not taking the effort to
determine whether this assertion is correct or not shows lack of integrity. Thus, one can be
honest without being integral.
Types of integrity
The view of integrity that has been described above is integrity as a moral purpose and a
wholesome virtue. Virtues such as benevolence dispose us towards a characteristic thought.
However, there is nothing in particular that integrity leads those who possess it to attend to.
However, integrity is an anomalous virtue – it is not associated with any characteristic
motivations and thoughts. It is a special kind of virtue and a cluster concept and thus involves
multiple motivations and thoughts.
Given human beings’ capacity and need for compartmentalization, or psychologically separating
out different parts of their lives, we can recognize different types of integrity.
1. Moral Integrity:
• It means consistency and honesty in standards that one uses in judging others as well
as oneself as right of wrong.
• This is the most comprehensive type of integrity, also the personal integrity that has
been described above.
2. Professional integrity:
• It means upholding the code of ethics, standards, norms and values of the respective
profession.
• It thus implies a professional who consistently and willingly practices within the
guidelines of the chosen profession.
• Professional integrity is weaker than moral integrity. It can be argued that the
responsibilities of performers, for example, are quasi-moral; they are not truly moral
because they are internal to the profession. However, it seems plausible to maintain
that professional integrity is better understood as an important contribution to the
living of a moral life. Professional integrity is specific to the sphere of a profession, but
not entirely independent of morality.
3. Intellectual Integrity:
• The term ‘intellectual integrity’ is ambiguous between integrity of the intellect and the
integrity of the intellectual.
• Construed broadly, as integrity of the intellect, it would be applicable to anyone who
thinks, and thus becomes too general.
• In a more specific understanding, intellectual integrity is the integrity as an academic’s
virtue.
• Intellectuals may differ in the extent to which they exemplify intellectual virtues such as
honesty, impartiality, respect for facts, and openness to the views of criticism.
• Intellectual integrity may then be thought of as the over-arching virtue that enables
and enhances these individual virtues by maintaining a proper balance between
them. For example, Socrates may be an outstanding example of a person of intellectual
integrity. He had a commitment to the pursuit of truth and knowledge, and he
demonstrated his intellectual integrity in the face of attacks on it.

DELHI JAIPUR PUNE HYDERABAD AHMEDABAD LUCKNOW CHANDIGARH


21 www.visionias.in # 8468022022 ©Vision IAS
Google it:- https://upscpdf.com
https://t.me/UPSC_PDF Download From > https://upscpdf.com https://t.me/UPSC_PDF

• Some commonly cited intellectual virtues central to the conception of intellectual Student Notes:
integrity are honesty, courage, fairness, sensitivity, perceptiveness or insightfulness,
intellectual humility, perseverance, adaptability and communicativeness.
• Possession of these virtues is part of what it means for a person to have intellectual
integrity, although they may exist in varying degrees without undermining a person’s
overall intellectual integrity.
• There are a range of kinds of actions one might expect from a person of intellectual
integrity as well: for example, being against plagiarism, refusing to suppress
counterarguments, and consistently acknowledging help.
• Thus, it can be said that intellectual integrity is that quality that enables a person to
balance the various demands of intellectual work and to manifest intellectual virtues in
a proper order.

4. Artistic integrity:
• It is essentially the integrity displayed by an artist towards his/her work. The
wholesomeness of the work – accuracy, careful depiction, placing no component out of
place, etc. are hallmarks of artistic integrity.
• Consider the example of a painter. He leaves his family, home, country, etc. and goes to
an exotic country so that he can find breathtaking views that can be put on the canvas.
Is he behaving as man of integrity? Should his commitment to his chosen profession
supersede his responsibility towards his family/country? There is no clear answer here –
if his artistic project fails, we might judge him as morally deficient, otherwise we are
likely to see his actions in a more favourable moral light. However, there is no reason to
think that he will succeed only if he leaves and not by remaining in his home. In any
case, our judgment about his artistic integrity does not absolve him for his lack of
personal/moral integrity.
Moral values which artworks suggest or promote are relevant to considerations of artistic
integrity. As such, artistic and moral integrity can overlap, particularly if the standards of
artistic integrity are high. On the other hand, artistic and moral integrity can come in conflict in
situations of great pressure. Circumstances also vary, and with them both the difficulty of
pursuing integrity, and our assessment of its merit. A journalist reporting from a totalitarian

DELHI JAIPUR PUNE HYDERABAD AHMEDABAD LUCKNOW CHANDIGARH


22 www.visionias.in # 8468022022 ©Vision IAS
Google it:- https://upscpdf.com
https://t.me/UPSC_PDF Download From > https://upscpdf.com https://t.me/UPSC_PDF

state is under considerable pressure. He may have to make certain compromises in his works. Student Notes:
One might rate his moral integrity as higher than artistic integrity – he may have made
compromises which affected his integrity as an artist (journalist).

7.4.2. Impartiality
• Impartiality is a broad concept founded on law that is commonly understood as a principle
of justice. It refers to the fact of not supporting one person or a group more than the other.
It simply means not taking any sides.
• Synonymous with fair-mindedness, impartiality holds that decisions should be based on
objective standards, instead of on the basis of bias, prejudice, or preferring the advantage
to one person over another for unsuitable reasons.
• Being impartial, though, is very difficult. People have their biases about most of the things.
For civil servants, politicians and judiciary, who most commonly deal with taking decisions
which may benefit one person or group of persons, being impartial becomes very
important.
For example, a judge cannot presume a person to be guilty only because he/she belongs to a
certain community. That would compromise the whole due process of law. Similarly, a civil
servant cannot prioritize one person over another just because of his personal preference.
There has to be a reasonable, intelligent criterion. Consider acquisition of land for ‘public
purpose’. Who are the people whose land will be acquired? They can be farmers cultivating
land on the outskirts of the city, or a hamlet comprising of scheduled caste population, or land
lying with a religious body, etc. The District Magistrate may have his/her own concerns or
preference towards any of the group, but the decision will have to be taken based on objective
criteria and most importantly, project requirements – he/she cannot be partial in evaluation of
facts or application of rules based on the owner only. The views of the people affected will be
ascertained and their objections disposed-off as per the procedure established under the Land
Acquisition Act.

DELHI JAIPUR PUNE HYDERABAD AHMEDABAD LUCKNOW CHANDIGARH


23 www.visionias.in # 8468022022 ©Vision IAS
Google it:- https://upscpdf.com
https://t.me/UPSC_PDF Download From > https://upscpdf.com https://t.me/UPSC_PDF

Student Notes:

For civil servants, impartiality works at two different levels:


Political Impartiality
• Since, impartiality is acting solely according to the merits of the case, therefore it implies
serving equally well governments of different political persuasions, irrespective of the civil
servant’s own personal opinion.
• A civil servant has to act in a way which deserves and retains the confidence of ministers.
• It also means that civil servants will comply with any restrictions that have been laid down
on him in regard to political activity.
• At the same time, civil servants are under no obligation to defend the ‘views’ of the
ministers but they must implement the ‘decisions’ with utmost objectivity, impartiality and
to the best of their abilities.
Public Impartiality
• Impartiality vis-à-vis public means that, a civil servant carries out his responsibilities in fair,
just, objective and equitable manner.
• He must not act in a way which is unjustifiably favors or discriminate against a particular
individuals or interest.
• The principle of public impartiality can be seen to be derived from the values of merit,
equality, justice, which are enshrined in the Indian Constitution.
• In other words, impartiality means that civil servants in carrying out their official work,
including functions like procurement, recruitment, delivery of services etc., should take
decisions based on merit alone.
7.4.3. Non-Partisanship
Non-partisanship can also be called political neutrality (explained in more details next). Non-
partisanship infers that the officer is to do his task without any fear of, or favor to any political
party, even if he has strong faith in any political thought. The values of the administrator will
flow from the constitution not from the philosophy of any political party. It would ensure that
civil servants would serve equally well to the political executives, irrespective of political
changes.

DELHI JAIPUR PUNE HYDERABAD AHMEDABAD LUCKNOW CHANDIGARH


24 www.visionias.in # 8468022022 ©Vision IAS
Google it:- https://upscpdf.com
https://t.me/UPSC_PDF Download From > https://upscpdf.com https://t.me/UPSC_PDF

Non-partisanship Impartiality Student Notes:


• Non-partisanship is a narrow concept, as it • Impartiality is a broader term and deals with
deals with relation of public officials only public officials’ relation with their entire ecology,
with the political executive, and is also called consisting not only of political executive but also
as neutrality the stakeholders i.e. the people.
• Practically, it may be said that non- • Impartiality has more to do with behavior in a
partisanship is a kind of attitude particular situation.
Why are they needed?
1. It brings credibility and trust in the public with respect to the functioning of the public
service.
2. It makes the civil servants gutsy, rather than merely capable, so that they can ask relevant
questions with respect to the policy, law etc. The difference between the two is that,
capability ensures that a civil servant does things correctly, but gutsiness would ensure that
he does the right things.
3. It ensures equality, justice among different sections of the society.
4. It ensures the morale, effectiveness and efficiency of civil services, as the transfer, posting
etc. are expected to be based on merit alone rather than any extraneous factors.
How are they ensured?
1. The Central Civil Services Conduct Rules, 1964 and The All India Services Conduct Rules
1968: To maintain integrity in public service the conduct rules were notified in 1964, which
stipulate certain guidelines for the Civil servant to perform the duty with full devotion; and
shall not adopt dilatory tactics in their dealings with the public & the likes that are uncalled
for like accepting bribe etc. from a civil servant.
2. Though we have these Conduct Rules for the Civil Servants in the form of certain guidelines
to follow while discharging of one’s duty, however they do not lay down the values &
morals in the form of a Code of Ethics that they should imbibe in their persona to do justice
with public service.
3. Code of Ethics, 1997: It was the first initiative to introduce the code of ethics for public
servants in India, which was considered a step towards better governance. The salient
features of the code were as follows:
• Uphold the rule of law & respect human rights
• Maintain objectivity & transparency in their conduct with the public & in discharge of
their duties
• Maintain utmost impartiality in respect of service matters
7.4.4. Neutrality
• Neutrality is with specific reference to relationship between the civil servants and the
government/politician.
• Neutrality is not being biased in providing facts, feedbacks, opinions, etc. to the political
executive or diligently carrying out tasks ordered by the government, irrespective of which
political party’s government is in power.
• It implies that a civil servant would remain politically impartial and non-partisan while
serving as public officials. He has to function with a kind of political sterilization, i.e.
bureaucrats should remain unaffected by the changes in politics. Conversely, he has to
implement the given policy as it is without any personal considerations.
The policy making is the prerogative of the political executive, and civil servants are meant to
implement that policy without questioning. In administrative jargon, it is called as politics-
administration dichotomy. At the same time, the bureaucrat must not hesitate to correct the
politician or provide honest opinion. His/her job is not be a ‘Yes-man’ or a ‘committed
bureaucrat’

DELHI JAIPUR PUNE HYDERABAD AHMEDABAD LUCKNOW CHANDIGARH


25 www.visionias.in # 8468022022 ©Vision IAS
Google it:- https://upscpdf.com
https://t.me/UPSC_PDF Download From > https://upscpdf.com https://t.me/UPSC_PDF

Student Notes:

Committed Bureaucracy
It can be understood from two perspectives:
1. Positive: It implies that civil servants should be committed to the objective of the state, the
constitution, and should have faith in the program and intention of the political executive, if they
genuinely seek to bring social change for the good.
Such committed bureaucrat needs not to be partisan in his working, he only needs to have empathy
to understand the mind of his political master and be political responsive. Therefore, he needs to
give technical advice in accordance to the political philosophy of the ruling party.
2. Negative: It implies politicized bureaucracy, where the administrative system functions only to serve
the narrow interest of the political party in power. For example, the functioning of administrative
system in Nazi Germany.
Generally, the term ‘committed bureaucracy’ is used with the negative connotation – a bureaucracy
committed to the goals of a political party/leader rather than that of the people
Types of neutrality
Sometimes, it is classified into two types.
1. Passive neutrality: Here the public official will do anything that the political executive
orders him to do. But then he may end up violating some legal/constitutional provisions. It
was the neutrality that was actualized by Nazi bureaucracy. Hence passive neutrality is
undesirable.
2. Active neutrality: The Officer will do what the constitution, rules, laws and office manual
says, without following any particular party. Sometimes, it leads to civil services activism.
Areas of conflict between political executive and civil servants:
The concept of civil servants being neutral to political dispensation has been debated in context
of India. Demands of a committed bureaucracy have been implicit on change of governments, if
not explicit. Further, what is the extent of neutrality that can be observed in practice? The
reasons for a potential conflict between civil servants and political executive can arise because
of the following reasons:
1. Societal consensus about the goals to be pursued varies among people. In West, there is a
certain consensus on goals of development. There is also a certain degree of homogeneity
in societal formation. Lack of consensus on the goals and path of development creates
ambiguity in the policy preferences. This in turn leads to adhocism, which cannot provide
clear direction to the permanent executive. On the contrary, political process start
occupying the space (i.e. more interference) meant for civil servants. This can strain the
relationship.

DELHI JAIPUR PUNE HYDERABAD AHMEDABAD LUCKNOW CHANDIGARH


26 www.visionias.in # 8468022022 ©Vision IAS
Google it:- https://upscpdf.com
https://t.me/UPSC_PDF Download From > https://upscpdf.com https://t.me/UPSC_PDF

2. Political executive may pressurize the permanent executive to violate the very rules that Student Notes:
they themselves formulated. Civil Services, being rooted in a rule-based system resist this
pressure. This may again lead to conflicts between the two.
3. Presence of shared belief system: It means there is common belief system with respect to
vital social issues. Belief system is a product of various factors, important amongst them the
culture, society, surroundings that one has grown up in, the education system one was
exposed to, etc.
In India, the civil services have mostly come from a urban middle class where as political
executive is more diverse. Although the character of bureaucracy is changing, it has been
changing rather slowly. While a bulk of the members of the political executive, particularly
at the state level, have been drawn from the rural and agricultural background, the top and
middle level administrators are from the urban middle and upper middle classes- most that
qualify for civil services are from urban towns, whereas political representation is spread
equally, and since roughly only 32 percent of population is urban, the distribution of seats
would also be similar. These differences are manifest in their style of living, mode of
communication, ways of looking at things and their mannerisms. The relationship
between the two is partly shaped by this factor of value system.
Further, it is argued that, in developing societies, there are weaker sections that must be
supported by the administration if the devised policies are ill-suited to their needs. In this
context, one of our former Prime Ministers said, “In developing countries, civil servants, to be
genuinely neutral, has to take side of the poor.”
However, it must be remembered that neutrality is an ideal, like any other ideal such as
democracy, which can’t exist in its perfect form, simply because it is an ‘ideal’. But it doesn’t
mean that we should discard them for this reason only. Just as the absence of democracy would
lead destruction of individual rights, similarly absence of neutrality would cause chaos in
functioning of public administration. Therefore, this ideal is worth pursuing forever in the
appropriate cultural context.
7.4.5. Anonymity
The principle of anonymity emphasizes that permanent executive works from behind the
screen. In other words, they should avoid public gaze.
The anonymity of the civil service is linked to two concepts: permanence and neutrality. Civil
servants, many of whom remain in their jobs whilst serving numerous governments, are thus
likely to have to give advice to governments of different political parties, who may have
different attitudes to policy. The advice they give needs to be given to ministers both freely and
also without fear of adverse public or political reactions and without fear of future career
damage. This is tied into the concept of ministerial responsibility, whereby the convention is for
the minister to accept responsibility for their actions and decisions and those of their
departments.
Anonymity and Neutrality: Both values have to be followed simultaneously. The technical and
managerial skills are not political. The tools of administration – i.e. the practices that make
carrying out of tasks more efficient and effective are politically neutral. It is only the diligence in
their application that incorporates the human bias. Anonymity, and thus accountability is
necessary to get rid of this bias. The skills and the technical knowledge which are assumed to
be non-political can be used by the government of any political ideology. For example, Lenin
incorporated many strands of efficiency of industrial capitalism in USSR’s economy. Or, China
embracing tenets of capitalism in its economy while maintaining communism in its polity.
7.4.6. Objectivity
Objectivity is sticking to the facts – the evidence. It means being guided by the evidence and
considering an event will be closer to the truth the more supporting evidence it has. It is that

DELHI JAIPUR PUNE HYDERABAD AHMEDABAD LUCKNOW CHANDIGARH


27 www.visionias.in # 8468022022 ©Vision IAS
Google it:- https://upscpdf.com
https://t.me/UPSC_PDF Download From > https://upscpdf.com https://t.me/UPSC_PDF

quality in which a person or his/her judgement is not influenced by personal feelings or Student Notes:
opinions in considering and representing facts.

It means evaluating a situation fairly, based on facts, without any bias, personal belief, feelings
or external influence. Therefore, it is rational and most of the times, empirical in nature. It is
antithetical to subjectivity, i.e. feeling, values, emotions etc. In practice, civil servants should
make choices on merit while carrying out their duties, for example making public
appointments, awarding contracts, or recommending individuals for rewards and benefits.

Why is it needed?
Civil servants discharge their duties based on clearly defined rules. These rules lay out their
responsibilities as well as give them required authority to carry them out. A public office enjoys
legitimacy and public trust because the responsibility and authority it carries are not arbitrary
(Max Webber’s Legal-Rational Authority). Hence, it is necessary for the individual who occupies
a public position to rational in his/her decisions rather than being arbitrary. Rationality comes
from facts rather than biases- a mind open to assessment of facts uninfluenced by any pre-
conceived notions. Biases blind our view and act as hindrance in taking a decision and its
execution.

DELHI JAIPUR PUNE HYDERABAD AHMEDABAD LUCKNOW CHANDIGARH


28 www.visionias.in # 8468022022 ©Vision IAS
Google it:- https://upscpdf.com
https://t.me/UPSC_PDF Download From > https://upscpdf.com https://t.me/UPSC_PDF

Student Notes:

How it is ensured?
To ensure objectivity, the civil servants are expected to function based on some written rules,
regulations, and laws, so as to eliminate or at least reduce the discretions. Even in situations
where discretionary decisions have to be taken, they must be based on facts and not on
personal beliefs or any other considerations.
How to develop it?
• Transparency: If the decisions and reasons behind them are made public, then civil servant
would be more careful in taking decisions according to the facts rather than whims and
fancies. For example, we have seen this trend with the enactment of RTI Act.
• Information Management System (IMS): If the organization doesn’t record and document
the incidents, information and statistics, then it won’t be able to make decisions based on
proper information.
7.4.7. Dedication to Public Service
Dedication is the quality of being able to give or apply one’s time, attention, or self, entirely to a
particular activity, person or cause.
It is commitment with passion and personal urge to do something, without any external formal
instrument to enforce that urge. Strictly speaking, dedication is different from commitment, in
that, commitment is formally obligated/bound, whereas dedication is guided by sense of duty,
inspired by some ideals, like that of state or society.
Thus, dedication ensures that one is devoted to the task even in the absence of a formal
commitment or external reward.
In civil services, dedication integrates the person with the idea of public good/interest. This
dedication would ensure that civil servant’s sense of duty is integrated with his official
responsibility. As a result, the will keep doing his job even if it boring, unwanting, tedious, or in
hostile environment etc. It is because his duty itself becomes his end, and he realizes Kant’s
maxim of “Duty for the sake of duty.”

DELHI JAIPUR PUNE HYDERABAD AHMEDABAD LUCKNOW CHANDIGARH


29 www.visionias.in # 8468022022 ©Vision IAS
Google it:- https://upscpdf.com
https://t.me/UPSC_PDF Download From > https://upscpdf.com https://t.me/UPSC_PDF

Spirit of sacrifice and service to the public & to the nation are the essential ingredients of Student Notes:
public service and civil servants should feel motivated & inspired by the fact that they are
working & devoting their life to a noble cause if they chose to be a part of it.
Why is it needed?
• Without it civil servants would find it difficult to perform their duties in difficult situations.
• To make civil servants emphatic to their job and empathetic to the people.
• To realize the ideals mentioned in the Constitution, like justice, equality etc.
How to develop it?
Since dedication to public service is more of a kind of aptitude, it has an internal motivational
component, which, if not present, is hard to develop. However, the following can still be done
to ensure dedicated civil services:
• The examination pattern should be so devised to gauge the aptitude of the prospective
recruits.
• By giving them sensitivity training, like the problems faced by different sections of the
population and how they can make a difference.
7.4.8. Empathy, Tolerance and Compassion towards the weaker section
7.4.8.1. Empathy
It is about understanding another person’s condition from their perspective. Simply put,
empathy means putting yourself in their shoes & feel what they must be feeling in the
circumstances they are faced with.
How empathy is different from sympathy?
Sympathy is instinctive and largely involves cognitive aspect. For example, seeing a poor man
at raining winter night you would think to do something for him, but may not necessarily do
anything. You can sympathize with his situation by expressing pity or remorse.
But empathy is deeper than this as it includes emotions as well, apart from that cognitive
aspect. Empathizing with the person means that you are able to view the situation from his
perspective and have knowledge of his circumstances
and hardships. You cannot empathize with an
abstract or detached feeling- you need to have
knowledge about who he is, what is he trying to do
and why. Therefore, it would make you feel to
understand his problems, causes and consequences
thereof. Thus, it is a step ahead of sympathy.
Empathy is stronger attitude than sympathy, hence
it’s better indicator of behavior.
For example, consider climate change. Developed countries may well sympathize with
developing countries which face immediate threats because of rising sea levels and
temperatures. However, their lack of ability to put themselves in the shoes of developing
countries struggling to meet multiple conflicting objectives (poverty alleviation v/s
environmental protection) has led to slow progress in implementing measures needed to
counter climate change and providing adequate climate finance. Thus, they are sympathizing
but lacks empathetic attitude.
Why is it needed?
• To break prejudices and enhance effectiveness of bureaucracy: The bureaucratic system
has become so complex that adhering to rules has become an end in itself rather than

DELHI JAIPUR PUNE HYDERABAD AHMEDABAD LUCKNOW CHANDIGARH


30 www.visionias.in # 8468022022 ©Vision IAS
Google it:- https://upscpdf.com
https://t.me/UPSC_PDF Download From > https://upscpdf.com https://t.me/UPSC_PDF

pursuit of the end which was to be achieved by those rules. More time is spent in Student Notes:
transactional business rather than working for welfare of people.
• To improve Emotional intelligence and Decision making: Empathy helps us understand
others’ emotions, therefore empathy is required to increase one’s emotional intelligence.
• To improve social cohesion and inclusivity: Lack of empathy may lead to social unrest and
intolerance towards women/minorities/ children/disabled as we can’t understand their
actual problems and specific needs. An empathetic civil service is the need of the hour to
usher in inclusive growth.
How to inculcate it?
• Sensitivity training: Role playing games, put yourself in the shoes of other people.
• Open conversation: When members of different sections interact with each in open
environment then the long held prejudiced are dismantled, and thereby empathy is
inculcated.
• Sarvadharma Sambhav: When members of different cultural groups participate in each
other’s cultural festivals then tolerance is developed for each other and empathy is
inculcated.
• Art and literature: They can also sensitize people about the conditions of some specific
group of people.
• Cultivation of interest in other cultures: It means extending our knowledge of ways of life
which are unfamiliar to us and appreciating the underlined ethical values.
7.4.8.2. Compassion
Compassion literally means “to suffer together or co-suffering.” It is defined as the feeling that
arises when you are confronted with another’s suffering and feel motivated to relieve that
suffering. Buddhism teaches the practice of compassion, called karuna.
Compassion and empathy are fundamentally
different but are closely related i.e.:
• Empathy is the feeling of awareness towards
other people's emotions and an attempt to
understand how they feel while compassion is
when those feelings and thoughts include the
desire to help. So, empathy precedes
compassion.
• Empathy is putting oneself in someone else’
shoes to understand what they are going
through, but compassion spurs one to action. For
example: empathetic people may cry with the
person experiencing a loss or if something unfortunate happens but a compassionate
person would rather do something to alleviate/reduce the other person’s suffering.
• While empathy is a very internal feeling, compassion moves outwards as it compels us to
dive into another person’s environment and offer help, without concern for ourselves or
the cost involved.
Importance of having compassion towards weaker section in administration:
• Civil servants are the agents of change. Compassion is ranked as one of the great virtues
for civil servants which has bearing upon the functioning of the administration and society.
Compassion drives the civil servant to help people and ensure the welfare of people.
• Compassionate people often have other positive traits like generosity, kindness, and
understanding. Compassion makes a civil servant lead with not only head but also heart. It
gives rise to active desire to alleviate the sufferings of others.

DELHI JAIPUR PUNE HYDERABAD AHMEDABAD LUCKNOW CHANDIGARH


31 www.visionias.in # 8468022022 ©Vision IAS
Google it:- https://upscpdf.com
https://t.me/UPSC_PDF Download From > https://upscpdf.com https://t.me/UPSC_PDF

• Compassion ensures a people-centric, humane, accommodative administration. It makes Student Notes:


administration sensitive and responsive to the needs of every citizen, thus contributes to
efficient service delivery.
• Compassion helps in getting a better understanding of ground realities and grass root
problems and hence helps in better planning. It also establishes mutual trust and
coordination between society and administration.
• A civil servant with virtues of compassion will make him help the needy and weaker
sections of society even if he may have to deal with external pressure. It helps civil servants
to earn the trust of the public, which enhances his confidence, gives a sense of
satisfaction, and energy to work harder.
How to develop it?
• Increasing awareness about the life and living condition of powerless. It can be done by
getting to know about the downtrodden sections like by visiting their neighborhood.
• Enhancing sensibility towards people by valuing others life and understanding their
perspective.
• Following universal values that are part of one's religion such as Buddhism’s 4 Noble Truths
(Existence of suffering, its cause, its end, Path that leads to its end), etc.
7.4.8.3. Tolerance
Tolerance comprises of allowing people to exercise their rights, religious or constitutional. It is
the midpoint on a spectrum ranging between prohibition/unacceptance at one end to
acceptance at the other. For example, a vegetarian might be convinced it is wrong to eat
animals. Such a person may never accept the practice as part of their own life. However, they
may not want to stop others eating meat.
It teaches us to exercise restraint in sensitive matters. Tolerance is defined as a fair and
objective outlook towards those whose lifestyle differs from a person. It refers to the skills a
person needs to live together peaceably. In Jainism tolerance is captured in the ideal of
Syaadavaada, which means that every view is correct from its own perspective, but no
particular view is absolutely correct.
It means showing respect for the race, gender, opinions, religion and ideologies of other people
or groups, and to admire the good qualities and good work of others. And to express one’s
point of view in a decent and respectful way while respecting the sentiments of others.
Why is it needed?
1. Without tolerance and harmony, the lasting peace of societies cannot be maintained.
2. Tolerance encourages freedom of expression, which is necessary for perusal of truth and
progress. Without it differing views can’t be expressed freely and thereby the society would
become status-quoist and decadent, where new inventions can’t take place.
3. It is necessary to uphold the moral worth of every individual, as according to J.S. Mill, all
individuals have equal moral worth, and therefore should be allowed to express their views
without any constraint.
4. Human development is possible only when we allow everyone to express their views and
pursue their interest.
5. In a diverse society, like ours, civil servants are required to serve all the sections equally
well, which is not possible if they are not tolerant.
6. In fact, values of justice, impartiality and objectivity are not possible without tolerance.
How to develop it?
• Sensitivity training.
• Promoting the notion of Sarvadharma-Sambhav, where we participate in cultural festivals of
different sections of society.

DELHI JAIPUR PUNE HYDERABAD AHMEDABAD LUCKNOW CHANDIGARH


32 www.visionias.in # 8468022022 ©Vision IAS
Google it:- https://upscpdf.com
https://t.me/UPSC_PDF Download From > https://upscpdf.com https://t.me/UPSC_PDF

• Developing empathy for others would lead to tolerance by itself. Student Notes:
7.4.9. Miscellaneous Values
7.4.9.1. Serving the public interest
Civil servants and public officials are expected to maintain and strengthen the public’s trust and
confidence in government, by demonstrating the highest standards of professional
competence, efficiency and effectiveness, upholding the Constitution and the laws, and seeking
to advance the public good at all times.
7.4.9.2 Legitimacy
Civil servants and public officials are required to administer the laws, and to exercise
administrative power on behalf of the government, or the Parliament, or other such authority.
That power and authority should he exercised legitimately, impartially and without fear or
favour, for its proper public
purpose as determined by the
Parliament or their employer.
7.4.9.3. Fairness
Civil servants and public officials
should make decisions and act in a
fair and equitable manner, without
bias or prejudice, taking into
account only the merits of the
matter, and respecting the right of affected citizens.
7.4.9.4. Courage and bravery
It is another feature of gutsy bureaucrats, because they can take transformational steps only if
they have courage to accept the responsibility of failure, if there is any. Moreover, to uphold
highest order of integrity, they have to be courageous, otherwise they would remain spineless,
which is of no use.
7.4.9.5. Responsiveness
As agents and employees of the elected government, civil servants and public officials are
required to serve the legitimate interests and needs of the government, other civil servants,
and all citizens, in a timely manner, with care, respect and courtesy.
They have to be responsive to whatever new opportunities and challenges arise every day. And
must be able to respond to the emerging needs of the people. For example, arranging shelter,
food, financial help for migrants during lockdown.
7.4.9.6. Efficiency and Effectiveness:
Civil servants and public officials are required to obtain best value for public assets deployed in
or through public management, and to avoid waste and extravagance in expenditure and the
use of public assets
7.4.9.7. Innovativeness and Creativity
In a rapidly changing environment with new ICT technologies coming every day, civil servants
have to be innovative and creative to utilize them to make their administrative work faster,
smoother and more efficient.
Moreover, administration should be ecology based, and when there are fast pace changes in
ecology the civil servants must be creative enough to match the changing environment so as to
fulfill their duties innovatively.

DELHI JAIPUR PUNE HYDERABAD AHMEDABAD LUCKNOW CHANDIGARH


33 www.visionias.in # 8468022022 ©Vision IAS
Google it:- https://upscpdf.com
https://t.me/UPSC_PDF Download From > https://upscpdf.com https://t.me/UPSC_PDF

For example, in the era of fast paced advancements in technology civil servants are left with Student Notes:
little to no time to first understand the impact of advancements in technology and then frame
policies on them. Rather they’ll need to adapt to the circumstances and engage in pre-emptive
policy making.
7.4.9.8. Fortitude
It means to have courage in pain or adversity, having courage over a long period. Civil servants
work in dynamic environment where they may be subjected to various external pulls and
pressures. They must demonstrate steadfastness and commitment to values that they adhere
to. For example, trying to help helpless people during lockdown even when there are little
means at disposal.
7.4.9.9. Perseverance and Tenacity
It means continued effort and determination. It means doing something despite difficulties or
delay in achieving success. Single-minded focus of researchers that keep on repeating
experiments for number of years is a good example of perseverance. For civil services,
perseverance is a key value. Changes that policies bring, for e.g. removing open defecation, or
improving sex-ratio in a district are goals that cannot be achieved overnight. There will be lot of
people opposing the schemes because it did not show results in one year or two years. Civil
servants have to persevere if they honestly believe that the current policy/scheme is the best
way to achieve desired goals, although there may not be an immediately visible impact.
Tenacity is very similar- it means the quality of being determined, persisting.
7.4.9.10. Resilience
Resilience is the quality to recover quickly and bounce back. It is the individual’s ability to adapt
in face of crisis and return back to pre-crisis status quickly, better prepared this time. Failure is
inevitable in a task that has uncertainties and decisions that are based on incomplete
information. The task of a civil servant is not to be bogged down by eventual failure of efforts
but to dissect the causes of failure and develop a mechanism to incorporate negative feedback
to improve up on future decisions. A positive attitude, ability to manage emotions and
optimism are key qualities of people who are resilient.

7.5. Consequences of Erosion of Values


• Corruption and absence of public service which is central to any administrative system.
• The administration becomes unresponsive to public needs as it tries to fulfill the ends of
the political party it is affiliated to.
• It results in the destruction of meritocracy, where merit is rewarded and all service
conditions are based on objective assessment of merit. Therefore, transfers, posting etc.
are guided solely by affiliations rather than the capability of the respective civil servant. This
results in inefficiency, ineffectiveness, complacency, and lack of morale.
• It reduces the popular trust in the public services, thereby hurting their credibility.
This erosion of values leads to various concerns, dilemmas and issues. We shall discuss these
concerns and the measures to strengthen ethical administration in the next document of Ethics
in Administration.

8. Previous Years UPSC GS Mains Questions


2013
1. What do you understand by the following terms in the context of public service? a) Integrity
b) Perseverance c) Spirit of service d) Commitment e) Courage of conviction f) Personal
opinion.

DELHI JAIPUR PUNE HYDERABAD AHMEDABAD LUCKNOW CHANDIGARH


34 www.visionias.in # 8468022022 ©Vision IAS
Google it:- https://upscpdf.com
https://t.me/UPSC_PDF Download From > https://upscpdf.com https://t.me/UPSC_PDF

2. Indicate two more attributes which you consider important for public service. Justify your Student Notes:
answer.
3. The good of an individual is contained in the good of all. What do you understand by this
statement? How can this principle be implemented in public life?
2014
1. There is a heavy ethical responsibility on the public servants because they occupy positions
of power, handle huge amounts of public funds, and their decisions have wide-ranging
impact on society and environment. What steps have you taken to improve your ethical
competence to handle such responsibility?
2. Integrity without knowledge is weak and useless, but knowledge without integrity is
dangerous and dreadful. What do you understand by this statement? Explain your stand
with illustrations from the modern context.
3. What does ethics seek to promote in human life? Why is it all the more important in public
administration?
2015
1. “A mere compliance with law is not enough, the public servant also have to have a well-
developed sensibility to ethical issues for effective discharge of duties” Do you agree?
Explain with the help of two examples where (i) an act is ethically right, but not legally and
(ii) an act is legally right, but not ethically.
2. Public servants are likely to confront with the issues of “Conflict of Interest”. What do you
understand by the term “Conflict of Interest” and how does it manifest in the decision
making by public servants? If faced with the conflict of interest situation how would you
resolve it? Explain with the help of examples.
2016
1. Why should impartiality and non-partisanship be considered as foundational values in
public services, especially in the present day socio-political context? Illustrate your answer
with examples.
2. What do you understand by the term ‘voice of conscience’? How do you prepare yourself to
heed to the voice of conscience?
3. What is meant by ‘crisis of conscience’? Narrate one incident in your life when you were
faced with such a crisis and how you resolved the same.
2017
1. Conflict of interest in the public sector arises when (a) official duties, (b) public interest, and
(c) personal interest are taking priority one above the other. How can this conflict in
administration be resolved? Describe with an example.
2. Examine the relevance of the following in the context of civil service:
(a) Transparency
(b) Accountability
(c) Fairness and justice
(d) Courage of conviction
(e) Spirit of service
3. One of the tests of integrity is complete refusal to be compromised. Explain with reference
to a real-life example.
2018
1. State the three basic values, universal in nature, in the context of civil services and bring out
their importance.
2. What is mean by public interest? What are the principles and procedures to be followed by
the civil servants in public interest?
3. “In looking for people to hire, you look for three qualities: integrity, intelligence and energy.
And if they do not have the first, the other two will kill you.” – Warren Buffett. What do you
understand by this statement in the present-day scenario? Explain.
4. Explain the process of resolving ethical dilemmas in Public Administration.

DELHI JAIPUR PUNE HYDERABAD AHMEDABAD LUCKNOW CHANDIGARH


35 www.visionias.in # 8468022022 ©Vision IAS
Google it:- https://upscpdf.com
https://t.me/UPSC_PDF Download From > https://upscpdf.com https://t.me/UPSC_PDF

2019 Student Notes:


1. What is meant by the term ‘constitutional morality’? How does one uphold constitutional
morality?
2. What is meant by ‘crisis of conscience’? How does it manifest itself in the public domain?

9. Previous Years UPSC Mains Questions: Case Studies


1. Suppose you are an officer in-charge of implementing a social service scheme to provide
support to old and destitute women. An old and illiterate woman comes to you to avail the
benefits of the scheme. However, she has no documents to show that she fulfils the
eligibility criteria. But after meeting her and listening to her you feel that she certainly
needs support. Your enquiries also show that she is really destitute and living in a pitiable
condition. You are in a dilemma as to what to do. Putting her under the scheme without
necessary documents would clearly be violation of rules. But denying her the support
would be cruel and inhuman.
(a) Can you think of a rational way to resolve this dilemma?
(b) Give your reasons for it.
2. Rameshwar successfully cleared the prestigious civil services examination and was excited
about the opportunity that he would get through the civil services to serve the country.
However, soon after joining the services, he realized that things are not as rosy as he had
imagined. He found a number of malpractices prevailing in the department assigned to him.
For example, funds under various schemes and grants were being misappropriated. The
official facilities were frequently being used for personal needs by the officers and staff.
After some time, he noticed that the process of recruiting the staff was also not up to the
mark. Prospective candidates were required to write an examination in which a lot of
cheating was going on. Some candidates were provided external help in the examination.
Rameshwar brought these incidents to the notice of his seniors. However, he was advised
to keep his eyes, ears and mouth shut and ignore all these things which were taking place
with the connivance of the higherups. Rameshwar felt highly disillusioned and
uncomfortable. He comes to you seeking your advice. Indicate various options that you
think are available in this situation. How would you help him to evaluate these options and
choose the most appropriate path to be adopted?

10. Previous Years Vision IAS GS Mains Questions


1. One has not only a legal but also a moral responsibility to obey just laws. Conversely,
one has a moral responsibility to disobey unjust laws.
Approach:
• Briefly highlight your understanding of the quote.
• Discuss its importance in the current context.
• Conclude accordingly.
Answer:
In a democracy, a law not only has the statutory sanction but also reflects the will of the
people. Obedience to the law of the land is also a moral obligation for maintaining
social and political order in the interest of common good. Socrates when condemned to
death, did not flee the city. According to him, it would have been violation of law
amounting to mistreatment or disrespect of his fellow citizens.
If one has agreed to constrain his freedom being bound by the law, under the premise
that others will do likewise, then it’s unfair if others choose to disobey the law to suit
their convenience. This approach to obeying laws on the basis of convenience will
create chaos.

DELHI JAIPUR PUNE HYDERABAD AHMEDABAD LUCKNOW CHANDIGARH


36 www.visionias.in # 8468022022 ©Vision IAS
Google it:- https://upscpdf.com
https://t.me/UPSC_PDF Download From > https://upscpdf.com https://t.me/UPSC_PDF

However, not all laws can have moral sanction. Even in a democracy, a majority can take Student Notes:
over the legislative process and frame a law that may not be just to every section of the
society or may undermine the dignity of some. A law that caters to most, at the cost of
a few is an unjust law. And according to St. Augustine an unjust law is no law at all.
Mahatma Gandhi also argued that an unjust law is itself a species of violence. In the
present context, following examples could be seen in this light:
• Criminalizing Homosexuality: Many countries, including India till recently,
criminalised LGBT community for their sexual orientation.
• Adultery and sexual harassment- Until recently, only a man in India could be
prosecuted for adultery under Section 497 of IPC. On the hand only women can file
cases against sexual harassment in India. Men do not have legal recourse, if faced
with sexual harassment.
Historically laws related to apartheid in South Africa and racial discrimination in US
were morally corrupt. Therefore, these laws were withdrawn after widespread
opposition and resentment by the population. Martin Luther King also provided a
template for opposing unjust law. He said that one who breaks an unjust law must do
so openly, lovingly, and with a willingness to accept the penalty. There should be
acceptance of penalty of imprisonment in order to arouse the conscience of the
community over its injustice. This in reality expresses the highest respect for law.

2. "Law should be so succinct that it can be carried in the pocket of the coat and it
should be so simple that it can be understood by a peasant." Discuss.
Approach:
• Introduce by stating the relevance of this quote.
• Discuss the meaning of the statement – emphasising key words.
• Conclude with examples.
Answer:
The given statement was quoted by Napoleon, which also finds its mention in the 2nd
ARC report on ethics in governance. The statement focuses on why law should be brief
and concise so that one can comprehend it easily. Also, law should be away from
complexities, so that even the person existing at the lowest rung of the pyramid (in this
case, peasant) can decipher it.
The above statement gains relevance in the context of:
• The lengthy legal texts and intricate language used in framing the constitution, and
a no. of civil & criminal laws, etc.
• Expensive legal system for the poor and marginalised and termed as lawyer’s
paradise.
• Complex jurisprudential interpretation of law and policy.
• The complexities in the law often end in secrecy as well as harassment of the
ordinary people.
• Existence of obsolete and archaic laws.
In the present context, to make laws succinct and simple a substantive exercise must be
carried out of identifying those statutes and regulations that no longer serve a useful
purpose, but may become a tool for delay, obstructionism and harassment. There
should be systematic use of drafting techniques like sunset and review clauses, and
simplification of the procedure for repeal.
As a civil servant and a law-abiding citizen of the country, it is the duty of every citizen
to aid and assist the poor by not only promoting the cause of simple procedures and

DELHI JAIPUR PUNE HYDERABAD AHMEDABAD LUCKNOW CHANDIGARH


37 www.visionias.in # 8468022022 ©Vision IAS
Google it:- https://upscpdf.com
https://t.me/UPSC_PDF Download From > https://upscpdf.com https://t.me/UPSC_PDF

legislations, but by also making available free legal aid for socially and economically Student Notes:
vulnerable people as envisaged under Article 39A and the Legal Services Authorities
Act.
Several such initiatives have been undertaken to simplify legislative procedures. For
instance:
• The government undertook repeal of about 2000 archaic laws in the past decade
such as through Repealing and Amending Act, 2017.
• Civil society organisations taking law to tribals and poor by making them
understand and representing their causes. For example, civil societies working for
the rights of Adivasis in Chattisgarh.Single window clearance mechanisms have
expedited last mile service delivery and minimised hassles of complex procedures.
Apart from reforming the laws, The National Legal Services Authority (NALSA) has been
constituted under the Legal Services Authorities Act, 1987 to provide free Legal Services
to the weaker sections of the society and to organize Lok Adalats for amicable
settlement of disputes. Mahila Adalats, e-courts for effective and timely justice have
also been setup.
To conclude, a nation should strive for succinct and simple law which is easily
understandable by the people. It must also be noted that laws are drafted in conformity
with existing laws and principles of jurisprudence. It is never meant to be used as tool
for rough and ready delivery of justice. Only a court or a duly appointed authority can
provide for expression of of views of contending parties, and that inevitably brings in
lawyers, who may interpret the law according to convenience if it is not well-drafted.

3. Why are certain rights considered universal in nature? Explain using examples.
Approach:
• Briefly introduce with a definition of rights.
• Discuss why certain rights are considered universal in nature.
• Give a brief conclusion based on the discussion in the answer.
Answer:
Rights mean claims & entitlements of individuals and groups that are considered to be
necessary for leading a life of respect and dignity and are essential for their well-being.
There are certain rights that are considered universal in nature because:
• They are rights inherent to all human beings since all human beings are born free
and equal. They have a right to have rights. For example, Freedom is an essential
condition for life and entitlement of the right to freedom is hence a universal claim.
• They are natural rights, that is, they are derived from the law of nature and not
dependent on laws or customs of any particular culture or government. For
example, Right to life and liberty.
• They impact all people without distinction of any kind, such as race, colour, sex,
language, religion, nationality etc. For example, Right against Exploitation & Right to
Justice affects all people equally & hence makes a case for their universal
availability.
• They are inalienable as exclusion from any of such rights would lead to
discrimination, unequal access to resources and opportunities, inability to realize
their potential. For example, No one shall be held in slavery or servitude or be
subjected to torturous or degrading treatment.

DELHI JAIPUR PUNE HYDERABAD AHMEDABAD LUCKNOW CHANDIGARH


38 www.visionias.in # 8468022022 ©Vision IAS
Google it:- https://upscpdf.com
https://t.me/UPSC_PDF Download From > https://upscpdf.com https://t.me/UPSC_PDF

• They are required to meet the basic needs, for example, the right to develop one’s Student Notes:
own talent and skills, right to opportunities etc. ensures that certain sections are
not denied the equal chance to have a dignified life in terms of food, clothing and
shelter.
Although mostly individualistic, the above discussed rights are required for the overall
development of the society because an empowered individual only would lead to a
sustainably empowered society. Entitlement to these universal rights would help build a
world in which every man, woman and child lives life with liberty and is protected from
oppression, violence and discrimination.

4. Explain the significance of the following in the context of civil service: (i) Public trust
(ii) Objectivity (iii) Strength of character (iv) Empathy (v) Selflessness
Approach:
• Succinctly define the terms.
• Outline the significance of each, by using examples or otherwise, in civil services.
Answer:
The given terms are the assets that a civil servant must possess in order to be true to
his/ her duty towards the society. Significance of these is discussed below:
(a) Public Trust: It is the firm belief in the reliability, truth, or ability of the people in
public offices, institutions and officials i.e. the measure of public confidence and
faith commanded by an officer or an institution or a system. High public trust
enables a civil servant to take bold decisions, whereas, low public trust raises
question on every activity. It can be built by consistent performance and efficient
delivery of expected services. For example, the Election Commission enjoys high
trust and this has helped it to implement ‘Model Code of Conduct’ even without
Legislature’s backing. One of the reasons that bribery is regarded as a notorious evil
is that it contributes to a culture of political corruption in which the public trust is
eroded.
(b) Objectivity: It is the quality of being truthful, unbiased, impartial and sticking to the
facts beyond the influence of one’s feelings and prejudices. Objectivity helps a civil
servant to rise above his own biases and perceptions and take actions in the larger
interest. For example, a judicial magistrate with objectivity would go by the merits
of a robbery case rather than being influenced by the public perception of the
culprit.
(c) Strength of character: It is a measure of how much a person can persevere in
adverse circumstances and stand against wrong actions/deeds.. It is determined by
how strongly or weakly does one believes and adheres to certain values. It stands
to test in the face of hardships. If a person holds high strength of character, he/ she
would hold on to his principles even in the face of crisis be it social, economic or
political. This strength helps a civil servant to firmly say ‘no’ to anyone’s ill demands
that may go against the law or the general public e.g. during riots, passing of
tenders, work during disasters and so on.
(d) Empathy: It is the ability to place oneself in another’s position and understand their
feelings and experience their emotions. In context of civil services, unless the public
officials empathize with the common man, they will not be able to understand the
problems faced by him and consequently, public services will not improve e.g. an
empathetic official will ensure ramps etc. in his/ her office premises to aid the
movement of the physically disabled.
(e) Selflessness: It means to put others before oneself to the extent of having little or
no concern for one’s life, money, position etc. The job of civil servant demands that
public concerns be the top priority. There might be situations where an official has

DELHI JAIPUR PUNE HYDERABAD AHMEDABAD LUCKNOW CHANDIGARH


39 www.visionias.in # 8468022022 ©Vision IAS
Google it:- https://upscpdf.com
https://t.me/UPSC_PDF Download From > https://upscpdf.com https://t.me/UPSC_PDF

to give up family time at a stretch in order to fulfil professional responsibilities. Student Notes:
Further, selflessness helps in building an organization of integrity and honesty.

5. Courage is the first of human qualities because it’s the quality that guarantees the
others.
Approach:
• Define courage & establish its importance as an important human virtue.
• Discuss, with examples, how courage is a prerequisite for other human qualities.
• Conclude in a balanced manner.
Answer:
Courage is the virtue that enables a person to restrain fear in the face of danger,
difficulty or doubt. As Nelson Mandela put it, “Courage is not the absence of fear, but
the triumph over it”.
Courage guarantees other qualities
• Courage enables people to face tough consequences for their acts. For instance
whistleblowers like Edward Snowden often pay heavy price for disclosures.
• Without courage it is difficult to display qualities like leadership which entails laying
out roadmaps for the future amidst uncertainty. For example it is “courage” that
enabled Mahatma Gandhi to display the virtue of nonviolence against the
oppressive colonial regime.
• It encourages people to take firm decisions and attempt things that they have not
tried before. For instance, it takes courage to invest in novel & seemingly
impractical/commercially unviable ideas like the SpaceX.
• Various personal, social and professional feats are unthinkable without courage. It
may be the fight for social reforms or fearless decision making by public officials to
bring about public accountability, impartiality and truthfulness in daily life.
However, it must be borne in mind that courage must stay within limits defined by
reason (like the courage displayed by law-enforcing personnel vis-à-vis criminals).
Moreover life provides daily opportunities and instances of courageous acts (for
instance standing up for injustice, actively supporting a just cause like cleanliness in
public places etc.) as distant from notions of courage as extreme or dramatic physical
heroism.

6. What are the factors that have influenced the contemporary attitude of the state and
the society towards homosexuality in India? Also, comment on the changing attitude
and the factors driving this change.
Approach:
• State the factors that are affecting currently held attitude towards homosexuality in
India.
• Mention the factors that result in changing attitude towards homosexuality and
what this change is all about.
• End with a way forward.
Answer:
While acceptance of homosexuality has not been fully entrenched in India, there is a
shift in stance of the society and state regarding it. The factors that have influenced the
contemporary attitude include religion, international discourse, individual rights, legal
pronouncements, advancement of medical understanding, etc.

DELHI JAIPUR PUNE HYDERABAD AHMEDABAD LUCKNOW CHANDIGARH


40 www.visionias.in # 8468022022 ©Vision IAS
Google it:- https://upscpdf.com
https://t.me/UPSC_PDF Download From > https://upscpdf.com https://t.me/UPSC_PDF

Reasons for the attitude opposing homosexuality: Student Notes:


• At the core lies the question that is it the natural expression of sexuality or merely a
product of closet desires? Those opposed to homosexuality generally consider it to
be the latter.
• Religion: Major religions such as Islam and Christianity consider homosexuality to
be a sin explicitly. Further, religious groups of all major religions support
criminalizing homosexuality.
• Traditional values: Indian culture espouses marriage, procreation and continuity of
traditional values. People suppress homosexual tendencies to prevent ostracization
by family members.
• Social stigma:For many Indians, homosexuality is still a taboo and a Westernized
concept.
• Legal provisions: Section 377 of IPC criminalizes homosexuality and thus strict legal
provisions have also shaped the attitude.
• Political reasons: The political class has shown apathy to LGBT rights in India mainly
due to prevailing attitude in the society which can affect vote bank.
• Homosexuality has been considered a medical problem and claimed to be treatable
with indigenous therapy.
However, off late, there has been considerable shift in the attitude of some sections of
the society which led to the filing of PIL to quash the law criminalising homosexuality.
Reasons for the changing social attitude towards homosexuality:
• Globalisation: With the free flow of information and ideas, the liberal attitude
towards homosexuality held found in the developed world is being adopted by
Indians as well.
• Social media: Social media networks are important platforms to accommodate
homosexuals who need a support system.
• Film industry: The film industry is also increasingly depicting favourable attitude
towards homosexuals.
• Support from organizations: Organisations like Humsafar Trust, Naz Foundation etc.
support decriminalising homosexuality. They are aided by the favourable
judgements by the higher judiciary, like in 2017; the SC stated that freedom to
express sexual orientation is guaranteed under Article 21.
• Open social support: There is increasing youth support through participation in
events like gay parade in Delhi, Bangalore etc.
Thus, both legal and social attitudes regarding homosexuality are witnessing change.
There is a need to support legal rights of the LGBT community and develop positive
social attitude towards them to prevent marginalization.

11. Previous Years Vision IAS GS Mains Questions: Case


Studies
1. You are posted as a District Collector in one of the districts in India. It has been
brought to your notice that a structure has been built by few members of a religious
community on the public land without getting due permissions. In keeping with the
guidelines of the Supreme Court of India, disallowing the construction of any
permanent religious structures on public land, you are contemplating its removal.
However, the leaders of the community in question have requested you to permit the
structure saying that it is for the period of month long religious festival only.
Moreover they say that there is no other religious place nearby where community
members can celebrate their festival. Your seniors and the political leader of the area

DELHI JAIPUR PUNE HYDERABAD AHMEDABAD LUCKNOW CHANDIGARH


41 www.visionias.in # 8468022022 ©Vision IAS
Google it:- https://upscpdf.com
https://t.me/UPSC_PDF Download From > https://upscpdf.com https://t.me/UPSC_PDF

also support their views. However you are skeptical that after the festival is over, it Student Notes:
may not be easy to remove the religious structure from the public land due to the
involvement of community members at large.
1. Perform an objective and subjective analysis of the case.
2. What will you do in such a situation?
Answer:
Objective Analysis:
• The constitution of India caters to treat everybody as equal before of the law
irrespective of his religion. Thus, religious community in the question should not be
dealt with any special treatment.
• As a civil servant an officer should follow the guidelines of SC as its interpretation is
final and binding in India.
• If he violates the law he has to answer to his superior for such a violation.
• The religious community too is part of Indian population. Therefore, their customs
and beliefs should be respected.
Subjective Analysis:
Religion plays an important part in lives of people in India. If the structure is removed,
the action can be given a communal color. Since our model of secularism respect all the
religion equally, their sentiment should also be respected. Leadership quality of civil
service also demands not to antagonize any community.
Course of Action:
1st Approach
Firstly, leaders of the community should be persuaded that though District
Administration fully respect the values and customs of their religion but due strict
guideline administrators have to suffer from penal action if structure is not removed.
Second, if leaders do not agree, then the matter should be discussed with the superiors
that are there any authority which can give such permission. If it is not possible, then
he should remove the structure.
If such solution is not available, then he should check whether it is possible to limit the
number of people attending the festival, based on law and order administration
available and with the help of leaders of the community? If it is possible, then he can
allow them to take up the festival because structure can be removed later if number of
people involved is not much. If it is not possible, then he should remove the structure.
2nd Approach
Community activities can be taken up on a government land but proper permission
needs to be taken so I will ask to that community to take requisite permission in written
manner.
As a district officer it is my responsibility to maintain law and order and peace in district
and also to ensure sentiments of community is not hurt. Therefore to maintain such
balance permission will be granted till festive month but after that demolition will be
carried out under municipal act, following proper procedures and discussions with
influential and important members of religious community such that peace would not
be disturbed.

DELHI JAIPUR PUNE HYDERABAD AHMEDABAD LUCKNOW CHANDIGARH


42 www.visionias.in # 8468022022 ©Vision IAS
Google it:- https://upscpdf.com
https://t.me/UPSC_PDF Download From > https://upscpdf.com https://t.me/UPSC_PDF

2. You are the director of a unit in a regulatory agency that is charged with monitoring Student Notes:
the use of potentially harmful commercial chemicals. Geeta, a junior project manager
under your supervision, is responsible for studying a broad-spectrum insecticide that
is used not only in agriculture by small food-grain farmers and cotton farmers, but
also in the livestock sector as an animal spray. She has been assigned to determine
whether this product should be removed from the market. At a social event, Geeta
met a man named Siddharth, who she later learned was the Mumbai representative
for the insecticide manufacturer. After meeting Siddharth several times, she became
rather fond of him and wanted to pursue the relationship further. However, Geeta
realized that their professional roles created a potential conflict of interest for her,
and she decided to tell you about the situation. She intended to continue seeing
Siddharth and said she considered herself mature enough to maintain a separation
between her professional and private lives. Geeta insisted that her feelings for
Siddharth would not influence her judgment in any way; in fact she and Siddharth had
never even discussed the chemical in question. What would you do in such a
situation? While you evaluate the alternatives available to you, what are the moral
codes and maxims that come to your mind as reference points for arriving at a
decision?
Answer:
1st Approach
Approach
In this case the ethical situation is not much clear. Has Geeta done anything that
represents a breach of professional ethics? Because of her relationship with Siddharth,
it might well be difficult for her to maintain objectivity in discharging her duties. But
perhaps it might not be. People differ in their ability to manage tensions of this kind.
And what is your responsibility? Is it more important to avoid even the appearance of
unethical conduct within your organization or to support an employee’s right to
freedom in her private life? Should Geeta be trusted until her behaviour demonstrates
otherwise? Examine your alternatives and arrive at a suitable conclusion.
Answer
You have to think about how to handle this highly sensitive situation. Some alternatives
may come immediately to your mind:
1. Order Geeta to stop seeing Siddharth
2. Transfer her to another task
3. Discuss the matter with your superior
4. Trust Geeta to do the job without being biased by the relationship
Then you consider the possible consequences:
• Geeta may resign
• Progress on investigating the chemical may be delayed
• The media may pick up the story
• A biased decision may be reached about the chemical, with serious consequences
for the public
• You may be blamed for irresponsible conduct if your superior discovers the
relationship without being informed by you
As you evaluate the alternatives and their possible consequences, various moral rules
and maxims come to mind as reference points for arriving at a decision:
• “You should be fair with subordinates under your supervision.” Would you handle
this situation differently if it involved a male member of your staff?

DELHI JAIPUR PUNE HYDERABAD AHMEDABAD LUCKNOW CHANDIGARH


43 www.visionias.in # 8468022022 ©Vision IAS
Google it:- https://upscpdf.com
https://t.me/UPSC_PDF Download From > https://upscpdf.com https://t.me/UPSC_PDF

• “Avoid even the appearance of evil.” Even if Geeta performs in an objective, Student Notes:
professional manner, will the credibility of your organization be eroded if this
situation is picked up by the press?
• “Honesty is the best policy.” If you take any action that Geeta perceives as
punishment or distrust, are you discouraging honest communication from your
staff? Should you tell your superior, or should you maintain Geeta’s confidence and
accept responsibility for dealing with the situation yourself?
As we review the facts of the case, the alternatives for action, and their likely
consequences on the one hand, and associate them with our stock of relevant moral
rules on the other, the field of alternatives begins to narrow and one or two rules
emerge as crucial. We can move towards a decision, with the practical consequences
and the moral justification related in some way that is acceptable to us.
After defining the situation as objectively as possible and defining the ethical issue, the
most difficult requirement is resisting the inclination to view the alternatives in
dichotomous terms, as meaning that you must do either this or that. Either you tell
Geeta to stop seeing Siddharth, or you trust her to handle the relationship in a
professional manner. This either-or view is the most common trap in the ethical
process. Rarely does such an ethical issue have only two or three possible solutions: If
you tell Geeta to stop seeing Siddharth, what is the likely outcome? What if you transfer
her to another position? Ask another member of the staff to work along with her?
Tighten your supervision of her work? What chain of events will likely unfold, and
toward what end?
There may also be regulations concerning conflict of interest that could apply to this
case. Moreover, the importance of individual dignity should be considered along with
respect for privacy: values that are highly important. The question is not only whether
you should observe the regulations or be responsive to Geeta’s wishes. Rather, it is a
matter of how the intent of the regulation can be upheld while still showing respect for
Geeta’s dignity and privacy. The tension between these two obligations is a healthy one
for any superior to address and reflect on. Simply ordering Geeta to stop seeing
Siddharth would be treating her in a mechanical and disrespectful manner, but simply
giving in to her would run the risk of accepting real or perceived conflicts of interests.
Thus one solution might be to talk it through with her, offering assurance that you
understand her dilemma and want to be helpful. You might then discuss the various
alternatives for protecting her dignity and privacy while fulfilling your duty to the law.
You should attempt searching her for a win-win solution that does not punish her for
having a private life outside work.
2nd Approach
Analysis
• Regulatory organization is very crucial as it can affect the lives of people, animal
and environment. Geeta’s relationship can have detrimental effects on the
stakeholders.
• If this issue comes into the public domain, people can lose the faith in regulatory
framework of the country.
• Though Siddharth have not asked her to help in this matter, but in future it can
happen.
• If she is not allowed to continue her relationship, it can affect her efficiency.

DELHI JAIPUR PUNE HYDERABAD AHMEDABAD LUCKNOW CHANDIGARH


44 www.visionias.in # 8468022022 ©Vision IAS
Google it:- https://upscpdf.com
My action Student Notes:
First, I will find out whether code of conduct or code of ethics of the organization
clearly debars such relationship. If it is so, then I will convey it to Geeta. I will also
explain how important it is to remain neutral in regulatory works. I will explore the
options of transferring her to some other departments. If no other solution work, I will
ask her to make a choice between the relationship and the office.
If no code anywhere mention about such relationship, even then I owe the
responsibility to carry out the regulatory functions with autonomy and unbiased. Thus,
I will transfer the case of Siddharth’s company to the other officials than Geeta.

Copyright © by Vision IAS


All rights are reserved. No part of this document may be reproduced, stored in a retrieval
system or transmitted in any form or by any means, electronic, mechanical, photocopying,
recording or otherwise, without prior permission of Vision IAS

DELHI JAIPUR PUNE HYDERABAD AHMEDABAD LUCKNOW CHANDIGARH


45 www.visionias.in # 8468022022 ©Vision IAS
EMOTIONAL INTELLIGENCE - CONCEPTS, AND Student Notes:

THEIR UTILITIES AND APPLICATION IN


ADMINISTRATION AND GOVERNANCE
Contents
1. Emotions: The Essential Constitute of Human Psyche ............................................................... 2
1.1. Structure of Emotions ......................................................................................................... 2
1.2. Classification of Emotions ................................................................................................... 2
1.2.1. Primary emotion and Secondary emotion ................................................................... 2
1.2.2 Positive and Negative Emotion ..................................................................................... 3
2. Concept of Intelligence .............................................................................................................. 3
2.1. The Social Aspect of Intelligence......................................................................................... 3
2.2. From Social Intelligence towards Emotional Intelligence ................................................... 4
3. Relation Between Emotions and Intelligence ............................................................................ 4
4. Emotional Intelligence: Integration of Emotions and Intelligence ............................................. 5
4.1. Model of Emotional Intelligence by Mayer and Salovey..................................................... 6
4.2. Model of Emotional Intelligence by Daniel Goleman ......................................................... 6
5. A Perspective on Intelligence Quotient (IQ) versus Emotional Quotient (EQ) ........................... 8
6. Importance of Emotional Intelligence ........................................................................................ 9
7. Qualities of people with Emotional Intelligence ...................................................................... 11
8. Can Emotional Intelligence be developed? .............................................................................. 12
9. Role of Emotional Intelligence in Administration and Governance ......................................... 13
9.1. Traits of an Emotionally Intelligent Administrator ............................................................ 13
10. Dark Side of Emotional Intelligence ....................................................................................... 15
11. Way Forward .......................................................................................................................... 15
12. Previous Years Questions of UPSC ......................................................................................... 16
13. Previous Years case studies of UPSC ...................................................................................... 16
14. Previous Year Questions from Vision IAS Test Series ............................................................. 17
15. Previous year case studies from Vision IAS Test Series: ......................................................... 20

Copyright © by Vision IAS


All rights are reserved. No part of this document may be reproduced, stored in a retrieval system or
transmitted in any form or by any means, electronic, mechanical, photocopying, recording or
otherwise, without prior permission of Vision IAS.

DELHI JAIPUR PUNE HYDERABAD AHMEDABAD LUCKNOW CHANDIGARH


1 www.visionias.in # 8468022022 ©Vision IAS
https://t.me/UPSC_PDF Download From > https://upscpdf.com https://t.me/UPSC_PDF

1. Emotions: The Essential Constitute of Human Psyche Student Notes:

Emotions are complex reactions that involve


• Intense subjective feelings such as joy, anger, sorrow etc.,
• Emotional expressions
• The ability (or abilities) to understand emotional information i.e. the ability to “read” the
emotional reactions of others.
• Discrete and consistent responses to internal or external events which have a particular
significance for the organism
In other words, emotions are generally understood as intense feelings, favorable or
unfavorable, that are directed at someone or something.
They are biologically given and a result of evolution because they provided good solutions to
ancient and recurring problems that faced our ancestors. Therefore, they are the essential
constitute of human mind. It is a well-established fact that it is almost impossible for humans to
live without emotions.

1.1. Structure of Emotions


Though, there is no unanimity, but it is generally believed that emotions, as complex reactions,
consist of three major components. These are:
• Physiological changes within our bodies- like shifts in heartbeat, blood pressure etc.
• Subjective cognitive states- the personal experiences we label as emotions
• Expressive behavior- outward signs of these internal reactions.

1.2. Classification of Emotions


1.2.1. Primary emotion and Secondary emotion
Primary emotion
Primary emotions are direct
emotional reactions to a
situation, and they are called
primary because they come
first. It is your very first
reaction to a situation or
event, and they alert you
about your needs.
A typical example is if you get
angry as a result of someone
being ruthless to you or
someone you love, and you
feel a need to protect or set
boundaries. Then the feeling
of anger is a primary emotion
and it helps you to protect
what is important to you by
making it more likely you
assert yourself.
Secondary emotion
A secondary emotion is an emotional response to a primary emotion, thus an emotion about
what you feel. For example, if someone who is important to you says something hurtful to you,

DELHI JAIPUR PUNE HYDERABAD AHMEDABAD LUCKNOW CHANDIGARH


2 www.visionias.in # 8468022022 ©Vision IAS
Google it:- https://upscpdf.com
https://t.me/UPSC_PDF Download From > https://upscpdf.com https://t.me/UPSC_PDF

you may become sad. That would be a primary sadness. If experiencing sadness for some Student Notes:
reason might be difficult to you, you may also notice that you get angry. The anger then is a
secondary reaction, since it is a reaction to you sadness.
1.2.2 Positive and Negative Emotion
Positive Emotion-
Certain emotions like joy, interest, contentment, love, and similar that are pleasant and
rewarding, are called positive emotions.
They open up new possibilities and build up our personal resources.
Negative Emotion
On the other hand, negative emotions are associated with actions that probably helped our
ancestors save their skins: escaping, attacking, expelling poison.
Negative emotions can also be valuable and constructive. For example, persistent distress may
motivate a person to seek help, mend a relationship, or find a new direction in life.
There is a natural tendency that people enjoy positive emotions while treating negative
emotions as misery.

2. Concept of Intelligence
Intelligence is defined as the capacity of an individual to think rationally, act purposefully and
deal effectively with his environment. In other words, it is the mental quality that consists of
the abilities to learn from experience, adapt to new situations, understand and handle abstract
concepts, and use knowledge to manipulate one’s environment.
“The true sign of intelligence is not knowledge but imagination” – Albert Einstein.
“I know that I am intelligent, because I know that I know nothing” – Socrates.
It can be more generally described as the ability to perceive or infer information, and to retain it
as knowledge to be applied towards adaptive behaviors within an environment or context.
Although, different investigators have emphasized different aspects of intelligence in their
definitions, they all emphasized upon some sort of cognitive energy as the ultimate ground of
intelligence. It is this cognitive ground which enables a person to effectively use his abilities.
However, their emphasis on the rational component as the most important constituent of
intelligence, was discarded by some later thinkers.

2.1. The Social Aspect of Intelligence


Social Intelligence (SI) is the ability to get along well with others, and to get them to cooperate
with you. These are sometimes simplistically also referred to as "people skills."
The original definition was given by Edward Thorndike in 1920 as "the ability to understand
and manage men and women and girls, to act wisely in human relations".
Social intelligence is a person’s competence to understand his or her environment optimally
and react appropriately for socially successful conduct. Thus, SI includes an awareness of
situations; the social dynamics that govern these situations, and knowledge of interaction styles
and strategies that can help a person achieve his or her objectives in dealing with others.
It also involves a certain amount of self-insight and a consciousness of one's own perceptions
and reaction patterns. Thus, SI is the ability to connect with people and influence them
effectively.
It is equivalent to interpersonal intelligence, one of the types of intelligence identified in
Howard Gardner's theory of multiple intelligences. Some authors have restricted the definition
of social intelligence primarily to deal only with the knowledge of social situations, perhaps

DELHI JAIPUR PUNE HYDERABAD AHMEDABAD LUCKNOW CHANDIGARH


3 www.visionias.in # 8468022022 ©Vision IAS
Google it:- https://upscpdf.com
https://t.me/UPSC_PDF Download From > https://upscpdf.com https://t.me/UPSC_PDF

more properly called social cognition or social marketing intelligence, as it pertains to trending Student Notes:
socio-psychological advertising and marketing strategies and tactics.
Since, Social Intelligence is a combination of skills expressed through learned behaviour, it can
be developed by assessing the impact of one's behaviour on others. This can be measured as
the degree to which one is successful in dealing with others.
One can experiment with new behaviours and new interaction strategies. In the simplest terms,
this is the ability to "get along with people," which - it is assumed - people learn as they grow
up, mature, and gain experience in dealing with others. Some examples of the people high on SI
include Mahatma Gandhi, Dalai Lama, and Nelson Mandela etc.

2.2. From Social Intelligence towards Emotional Intelligence


While some practitioners have included "people skills," or Social Intelligence in EI theory, but in
practical terms it makes more sense to think of EI and SI as two distinct dimensions of
competence.
Social intelligence (Gardner's "interpersonal intelligence") is separate from, but complimentary
to emotional intelligence (Gardner's "intrapersonal intelligence"). But we need both models in
order to understand ourselves and the way we interact with others. Some deficits in SI arise
from inadequate development of EI; conversely, some deficits in SI may lead to unsuccessful
social experiences which may undermine a person's sense of self-worth which is part of EI.

3. Relation Between Emotions and Intelligence


The traditional notion of intelligence as logical or mathematical ability invariably reduces it to
cognitive ability. Cognition refers to processes such as memory, attention, language, problem
solving, and planning. Many cognitive processes often involve so-called controlled processes,
such as when the pursuit of a goal (e.g., maintaining information in mind like retaining some
facts) needs to be protected from interference (e.g., a distracting stimulus like a nagging noise).
Traditionally, it was believed that emotion, being non-cognitive, can’t facilitate the cognitive
processes. In fact, it was believed that emotions, were counter to cognitive task, because they
are intense feelings.
Thus, the earlier notion was either of no relation between emotion and intelligence or negative
relation. For example, when we are experiencing negative emotions, like anger or depression,
then it becomes very difficult to perform a constructive task, like solving a puzzle, or making
good decisions.
However, Mayer and Salovey, in their concept of Emotional Intelligence (EI), discarded this
necessarily negative relation between Emotions & Intelligence. It was realized that emotions
aren’t necessarily bottlenecks in our thinking or decision making. This leads us to the topic of
Emotional Intelligence.

DELHI JAIPUR PUNE HYDERABAD AHMEDABAD LUCKNOW CHANDIGARH


4 www.visionias.in # 8468022022 ©Vision IAS
Google it:- https://upscpdf.com
https://t.me/UPSC_PDF Download From > https://upscpdf.com https://t.me/UPSC_PDF

Student Notes:

4. Emotional Intelligence: Integration of Emotions and


Intelligence
The term Emotional Intelligence was introduced in 1990 by Mayer and Salovey. It is described
as a set of skills that involve the ability to monitor ones’ own and other’s feelings/emotions, to
discriminate among them, so as to use that information to guide one’s thinking and action.
Thus, it integrates emotions and intelligence.
Simplistically speaking, it is the ability to channelize emotions for constructive purposes. It
must be known that emotional intelligence is not the opposite of intelligence. It is not the
triumph of heart over head, rather, the unique intersection of both.

DELHI JAIPUR PUNE HYDERABAD AHMEDABAD LUCKNOW CHANDIGARH


5 www.visionias.in # 8468022022 ©Vision IAS
Google it:- https://upscpdf.com
https://t.me/UPSC_PDF Download From > https://upscpdf.com https://t.me/UPSC_PDF

Mayer and Salovey introduced this concept as a challenge to the traditional notion of Student Notes:
intelligence as monolithic ability i.e. only focused on cognitive ability, and to the thinkers who
held emotions as obstructive to cognitive activity. EI includes the intra- and inter-personal
intelligence, i.e. the ability to know oneself and others, in terms of abilities, current emotional
state.

4.1. Model of Emotional Intelligence by Mayer and Salovey


Mayer and Salovey described EI more specifically in 1997 by outlining the competencies it
encompasses. They organized those competencies along the four branches:
Four Branch Model
• Perceiving emotions: It refers to the ability to perceive i.e. recognize emotions and also the
ability to specifically categorize the current emotional state, in oneself and in others. For
example, it enables the person to understand the kind of feeling he/she or other person is
going through.
• Understanding emotions: It is the ability of a person to comprehend the emotions in the
given messages better. It also enables one to understand the relationship between
emotions, thought, and behavior. For example, with this ability one can accurately
understand as to how a particular emotion can affect the thinking or action of oneself and
that of others.
• Using emotions to facilitate thought: This model is a logical corollary of the
aforementioned models, i.e. from the above two it can be deduced that emotions can act
as facilitators of thinking by channelizing emotions for constructive purpose, like making
appropriate decision or solving some problem.
• Managing emotion: It refers to the ability to take responsibility for one’s emotions, and
converting the negative emotions into positive ones. Its utility lies in generating the
desirable emotions in one or in others, which are conducive to the task at hand. It is
because emotions are energy for motions; therefore, if desired emotions are created then
efforts in the direction of task can be made easily. For example, motivating oneself and
others to perform some task, like focusing on studies when otherwise it is difficult to focus;
or managing the stressful situation without losing temper etc.

4.2. Model of Emotional Intelligence by Daniel Goleman


Goleman further refined the concept of emotional intelligence in 1999.
Five Component Model
• Self-awareness: It is the ability to recognize and understand personal moods and emotions
and drives, as well as their effect on others. Hallmarks of self-awareness include self-
confidence, realistic self-assessment, and a self-deprecating sense of humor. Self-awareness
depends upon one's ability to monitor one's own emotion state and to correctly identify
and name one's emotions.
• Self-regulation: It is the ability to control or redirect disruptive impulses and moods, and
the tendency to suspend judgment and to think before acting. Some of its hallmarks include
trustworthiness and integrity; comfort with ambiguity; and openness to change.
• Internal motivation: This refers to a passion to work for internal reasons that go beyond
external rewards like money and status. This can be driven by an inner vision of what is
important in life, a joy in doing something, curiosity in learning, a flow that comes with
being immersed in an activity. Its hallmarks include a strong drive to achieve, optimism
even in the face of failure, and organizational commitment.
• Empathy: It is the ability to understand the emotional makeup of other people. It involves
imaginatively stepping in someone’s shoes and emoting as per the context. It is a skill that

DELHI JAIPUR PUNE HYDERABAD AHMEDABAD LUCKNOW CHANDIGARH


6 www.visionias.in # 8468022022 ©Vision IAS
Google it:- https://upscpdf.com
https://t.me/UPSC_PDF Download From > https://upscpdf.com https://t.me/UPSC_PDF

helps one treat people according to their emotional reactions. Its hallmarks include Student Notes:
expertise in building and retaining talent, cross-cultural sensitivity
Three kinds of empathy are important to emotional intelligence:
1. Cognitive empathy – the ability to understand another person’s point of view;
2. Emotional empathy – the ability to feel what someone else feels;
3. Empathic concern – the ability to sense what another person needs from you.
Cultivating all three kinds of
empathy, which originate in
different parts of the brain, is
important for building social
relationships.
• Social skills: It includes
proficiency in managing
relationships and building
networks, and the ability to
find common ground and to
build rapport. Hallmarks of
social skills include
effectiveness in leading
change, persuasiveness, and
expertise building and
leading teams.
Thus, Goleman’s model is more comprehensive and is most suited for effective functioning in a
social system, especially in governance and administration.
An Example of Emotional Intelligence
• Some of the greatest moments in human history were fueled by emotional intelligence.
When Martin Luther King, Jr. presented his dream, he chose language that would stir the
hearts of his audience. He promised that a land “sweltering with the heat of oppression”
could be “transformed into an oasis of freedom and justice.”
• Delivering this electrifying message required emotional intelligence—the ability to
recognize, understand, and manage emotions. Martin Luther King demonstrated
remarkable skill in managing his own emotions and in sparking emotions that moved his
audience to action. King delivered “a perfectly balanced outcry of reason and emotion, of
anger and hope. His tone of pained indignation matched that note for note.”
Emotional Intelligence and the Bhagavad-Gita
1. Attachment is the root of all misery, says the ‘Gita’. Attachment leads to desire; desire when not
fulfilled leads to anger; anger to delusion; delusion to indiscriminate action which in turn leads a
man to his ruin. Attachment is the web that ensnares a person in this materialistic world
2. The way to emotional stability is to remain unattached to the fruits of action
3. One must follow and fulfill his ‘swadharma’, his obligatory duties. When he does that without
expecting anything in return, he becomes a ‘karma yogi’ which is the first step towards becoming a
‘sthithapragnya’ – the emotionally stable person.
4. It is the duty of a person says ‘Gita’, to act according to his dharma. Upholding this dharma of his
without getting attached to the fruits of his action is the best way to tackle any conflicting situation.
5. One should not claim the fruits of his actions because the moment he does that, he is bound. This
bondage will cloud his sense of right and wrong and thus lead him to misery.
6. one should focus on one’s duty alone. When a person firmly anchors himself to this principle he is
not swayed by any confusion or dilemma. His mind is clear; his judgment is not muddled; he takes
the right decisions; he succeeds in life.

DELHI JAIPUR PUNE HYDERABAD AHMEDABAD LUCKNOW CHANDIGARH


7 www.visionias.in # 8468022022 ©Vision IAS
Google it:- https://upscpdf.com
https://t.me/UPSC_PDF Download From > https://upscpdf.com https://t.me/UPSC_PDF

How does One choose between conflict Obligation Actions which result in societal well being – Student Notes:
Lokasangrahamevapi sampasyankartumarhasi – is the answer. That which denounces selfishness and
results in the well being of the society at large is the best choice.

5. A Perspective on Intelligence Quotient (IQ) versus


Emotional Quotient (EQ)
Which is Better-EQ or IQ?
EQ is believed to be a better indicator of success at the workplace. People with high EQ usually
make great leaders and team players because of their ability to understand, empathize, and
connect with the people around them.
According to Goleman, success at workplace is about 80% or more dependent on EQ and about
20% or less dependent upon IQ. As a result, many persons, high on IQ, may not be successful in
life, while contrary to this, most successful people are high on EQ.

DELHI JAIPUR PUNE HYDERABAD AHMEDABAD LUCKNOW CHANDIGARH


8 www.visionias.in # 8468022022 ©Vision IAS
Google it:- https://upscpdf.com
https://t.me/UPSC_PDF Download From > https://upscpdf.com https://t.me/UPSC_PDF

The success of most professions today depends on our ability to read other people’s signals and Student Notes:
react appropriately to them.
It’s not the smartest people that are the most successful or the most fulfilled in life. There are
people who are academically brilliant and yet are socially inept and unsuccessful at work or
in their personal relationships.
Intellectual intelligence (IQ) isn’t enough on its own to be successful in life. Ones IQ can get him
into college, but it’s the Emotional Intelligence that manages the stress and emotions when
facing final exams or during an interview.
EQ, on the other hand, is the ability to effectively use IQ and all other potentialities that an
individual possesses to the greatest advantage. Thus, in a way, IQ is like a vehicle, but it is EQ
which determines the destination. As a result, EQ is more important than IQ in reaching the
highest potential development.

6. Importance of Emotional Intelligence


The chances of succeeding are skewed towards people who are better able to manage
themselves and others emotionally; one’s who are likeable and trustworthy. Research shows
that more than 80% percent of success is due to skills in “human engineering,” personality and
ability to communicate, negotiates, and leads. Only 15 percent is due to technical knowledge.
Additionally, Nobel Prize winning psychologist, Daniel Kahneman, found that people would
rather do business with a person they like and trust rather than someone they don’t, even if the
likeable person is offering a lower quality product or service at a higher price.
Hence, instead of exclusively focusing on conventional intelligence quotient, one should make
an investment in strengthening his/her EQ (Emotional Intelligence). The concepts of EQ may be
difficult to measure, but their significance is far greater than IQ.
Simply put, emotional intelligence is that “something” within us that help us to sense how we
feel and enables us to truly connect with others and form a bond. It gives us the ability to be
present and listen to someone when they most need it.
It is that sense of internal balance within us that enables us to keep our composure, make good
decisions, communicate successfully, and maintain effective leadership even when under stress.
To be specific,
• Profession: Proficiency in EI is becoming a vital prerequisite in prolonged or intense areas of
‘emotional work’ such as nursing, social work, the service industry, and management roles.

DELHI JAIPUR PUNE HYDERABAD AHMEDABAD LUCKNOW CHANDIGARH


9 www.visionias.in # 8468022022 ©Vision IAS
Google it:- https://upscpdf.com
https://t.me/UPSC_PDF Download From > https://upscpdf.com https://t.me/UPSC_PDF

• Relationship: Emotional intelligence is an integral part of forming and developing Student Notes:
meaningful human relationships.
• Health: The ability to manage emotions and stress – and the ability to solve personal, as
well as interpersonal problems, are also significantly related to physical health. As
accumulated and persistent stress leads to the various cardiovascular diseases.
• Decision making - Whether we like it or not, our emotions do influence our decisions.
Positive and negative emotions can both lead to bad decision-making. People who are
emotionally intelligent don’t remove all emotions from their decision-making. They
remove emotions that have nothing to do with the decision.
• Social Harmony: Emotional intelligent people can understand other outrage or outburst
thus avoid violent conflict and preventing communal issue. Emotional intelligence can help
eradicate many social evils. For example-Impact of emotional intelligence on Racism-
Emotional Intelligence (E.I.) is vital to having all parties be heard and increases the chance
of a mutually beneficial understanding and resolution. The Black Lives Matter movement is
asking for change and action. It is diving into having a deeper conversation. It is requiring
society to listen to find the most
effective way to ensure what black
voices are saying is being received.
• Differentiate between Needs and
wants: The emotionally intelligent mind
is able to discern between things that
they need versus things that would be
“nice to have” that classify more aptly as
wants. A need, particularly in the
context of Abraham Maslow’s “Hierarchy
of Needs” is the basic level stuff like
safety, survival and sustenance. Once
those things are met, then we can
progress to other needs and of course,
wants. A “want” is a big house, nice car,
a Smartphone, etc. We do
not need those things to survive, but
rather we want them based on our own personal desires or what we
An Illustration
An accountant, Julie, working on a long-term and complex project needed to learn new skills that no-one
else in the organization had. For a time she was becoming disheartened and not getting the support she
needed because her manager did not sufficiently understand the complexity of the task.
After being left on her own for several weeks she was making slow, steady progress.
Unexpectedly, she got an email from her manager which was very positive about her progress and her
approach to the project. The email was a total surprise as it was atypical of that manager to send
anything like that. The result was a burst of enthusiasm from Julie and a greater willingness to battle on,
learn more and put "the slog" in to meet the unrealistic deadline.
The manager was showing his ability to manage the emotions of others which is the sixth dimension of
emotional intelligence.
From this example, it is illustrated that how generating appropriate emotions in the colleagues or
subordinates can do wonders in accomplishing difficult goals. The idea is emotions are the energiser of
our behaviour, therefore emotional intelligence enables us to manage emotions and sustaining them.

DELHI JAIPUR PUNE HYDERABAD AHMEDABAD LUCKNOW CHANDIGARH


10 www.visionias.in # 8468022022 ©Vision IAS
Google it:- https://upscpdf.com
https://t.me/UPSC_PDF Download From > https://upscpdf.com https://t.me/UPSC_PDF

7. Qualities of people with Emotional Intelligence Student Notes:

There are some general qualities which are found in the people with high EI.

• Acting with integrity. Because integrity means consistency in what we think and what we
do. Therefore, if one is emotionally intelligent then he would be aware of his inner self and
the surrounding environment. Therefore, there would be minimum chances of mismatch.
• EI leads to reduced stress levels. It is because the emotionally intelligent people are good at
managing and regulating their emotions. Authenticity and legitimacy In Action and
behavior
• Improved career prospects. It is because, every organization is a social system where
people form an inter-related and inter-dependent organic whole. People with higher EI are
better at social relationships.
• Improved communication with others is the very basic attribute of EI.
• Feeling confident and positive, because of self-awareness and self-regulation.
• Respect from others: It is because EI brings in tactfulness, sensitivity, cooperativeness, and
good listening qualities etc. which are necessary for favorable rapport.

DELHI JAIPUR PUNE HYDERABAD AHMEDABAD LUCKNOW CHANDIGARH


11 www.visionias.in # 8468022022 ©Vision IAS
Google it:- https://upscpdf.com
https://t.me/UPSC_PDF Download From > https://upscpdf.com https://t.me/UPSC_PDF

• People with high EI are more empathetic, because understanding others’ emotions and Student Notes:
their perspective is also essential for EI. Improved inter-personal relationships and therefore
better satisfaction of social and esteem needs.
• Learning from mistakes, because EI makes a person assertive, bold, and responsible for
their deeds. Therefore, EI reduces the tendency of individuals to repeat mistakes.
• Benefit from criticism: Nobody enjoys negative feedback. But you know that criticism is a
chance to learn, even if it's not delivered in the best way. And even when it's unfounded, it
gives you a window into how others think. When you receive negative feedback, you keep
your emotions in check and ask yourself: How can this make me better?
• Increased creativity, because it is believed that creativity is facilitated by the positive
emotions. On the other hand, EI enables one to manage their stress levels, and be
optimistic in the face of adversities.
• Managing change more confidently. It is because change is, generally, resisted by the
stakeholders and a person high on EI is able to bring the stakeholders on board through
persuasion and conviction.
• Fewer power games at work, because of increased cooperation and coordination induced
by emotionally intelligent people at work.
• Protecting oneself from emotional sabotage: Emotional intelligence also has a dark side--
such as when individuals attempt to manipulate others' emotions to promote a personal
agenda or for some other selfish cause. And that's why one should continue to sharpen
his/her own emotional intelligence--to protect oneself.

8. Can Emotional Intelligence be developed?


There are differing perspectives on the ability of people to develop EI. Some researchers
suggest that emotional intelligence can be learned and strengthened, while others claim it is an
inborn characteristic. In this context, some thinkers make distinction between emotional
intelligence (EI) and emotional quotient (EQ).
EI refers to innate potentiality, such that each individual is born with some innate potential for
emotional literacy and emotional learning ability, and this potential is realized only when he
gets favorable environment.
The core of this favourable environment constitutes emotional lessons. These emotional
lessons are given to us through socialization by our parents, teachers, peers etc., during our
childhood or adolescence. The result of which is what is called as EQ.
EQ is hence the relative measure of one’s healthy or unhealthy development of innate EI. It is
possible that two children with the same EI may have different EQ or vice-versa, depending
upon the socialization experiences. However, it must be clarified that either of them is not a
numerical entity, like IQ.
Emotional quotient is best inculcated from an early age by encouraging qualities like sharing,
thinking about others, putting oneself in another person's shoes, giving individual space and
the general principles of cooperation. There are tools like toys and games available to increase
emotional quotient, and children who do not do well in social settings are known to perform
significantly better after taking SEL (Social and Emotional Learning) classes. Adult EQ can also
be enhanced, although to a limited extent through effective coaching.
There are some medical conditions like high functioning autism (HFA) or Asperser’s where one
of the symptoms may be low-empathy. While some studies found that adults with Asperser’s
have low-empathy, there are have been studies with control groups that indicate EQ can be
changed in individuals with HFA or Aspergers.

DELHI JAIPUR PUNE HYDERABAD AHMEDABAD LUCKNOW CHANDIGARH


12 www.visionias.in # 8468022022 ©Vision IAS
Google it:- https://upscpdf.com
https://t.me/UPSC_PDF Download From > https://upscpdf.com https://t.me/UPSC_PDF

9. Role of Emotional Intelligence in Administration and Student Notes:

Governance
Many civil servants despite being extraordinarily talented, conceptually brilliant and having a
very high IQ., are not particularly likeable people. Many of them are aggressive and brutal in
their response to the outside world. They have little or no feeling for people around them.
They feel physiologically awkward in their relationships; have no social graces or even a social
personal life. Being uncomfortable with themselves and making people uncomfortable becomes
a routine response in their life.
Apart from this, it has also been noticed that risk taking behavior, and bold decisions are
needed in discharging the responsibility of public services, especially in a developing country
like India.
Civil servants need to be adept at handling people effectively since it forms a major part of their
responsibilities. Further, the civil servants are the trustees of public interest and are entrusted
to make policies. Therefore, they need to be high on EI, because without EI it would be
difficult to be empathetic to different sections of the society, to be firm in their approach, and
to be good change agents.

9.1. Traits of an Emotionally Intelligent Administrator


Emotionally intelligent leaders are centered and grounded. Such leaders display a stable mood,
aren’t erratic or extremely unpredictable in their behaviour, and they tend to possess these
traits:
• High self-regard: Good leaders have high self-regard. Leaders who claim to know it all tend
to be poor leaders. Good leaders know their strengths and capitalize on those strengths, as
well as know their weaknesses and fill the gaps with people who have strong skills in these
areas.
o During the seventeen years he was the Prime Minister, Nehru strode the Indian political
stage like a colossus. But he never imposed his political will and always had an ear for
what others had to say. Though not in favour of linguistic states, he adhered to popular
wishes. He did not choose chief ministers but allowed the party organisation at the
state-levels to elect their leaders. When courts challenged his land reform programmes,
instead of being critical of the judges, he chose to undertake constitutional
amendments.
• Maintain balance in life: Good leaders also seem to know how to balance their personal
and work lives. They tend to avoid burning out by managing their time well. If a person can
manage his own life well — including stress, home life, fitness, and diet — then he has a
better chance of managing the workplace well.
• Model the way: Successful leaders say what they want to accomplish and get it done. The
leader needs to walk the talk if he wants others to follow. In emotional intelligence terms,
this practice involves assertiveness and independence.
o People who are assertive and articulate have no difficulty expressing their thoughts,
feelings, and beliefs. Also, people, who are independent, listen are to and take in the
advice of others, but in the end, make their own informed decisions. Independence
implies acting in order to carry things out.
o For Example, Pankaj Jain, the district Collector of Katni district in Madhya Pradesh has
set an example for others to emulate. Pankaj Jain could afford to send her daughter to
a high profile and costly play school in the city. But he instead chose to send her
daughter to a rural childcare centre or Anganwadi for pre-schooling.

DELHI JAIPUR PUNE HYDERABAD AHMEDABAD LUCKNOW CHANDIGARH


13 www.visionias.in # 8468022022 ©Vision IAS
Google it:- https://upscpdf.com
https://t.me/UPSC_PDF Download From > https://upscpdf.com https://t.me/UPSC_PDF

Student Notes:

• Inspire a shared vision: As a leader, one must convince others that he/she understands
others needs and have their best interests at heart. Inspiring a shared vision requires a
good deal of empathy and optimism for it gives our vision a positive and desirable flavour
so that others want to share in it.
o Our empathy ensures that we hit the right chord in terms of what others want to see
and hear from us. One can connect the Various slogan Given by our Leaders for
example Mrs Indira Gandhi slogan of “Garibi Hatao ,Desh Bachao” and Recently Our
Prime minster Stated the slogan of “Sabka Sath,Sabka Vikas” is modeled on the line of
Inspiring shared vision.
• Challenge the process: An emotionally intelligent leader strives for change. He looks for
opportunities to improve and grow and also to experiment and take risks. One of the key
emotional intelligence skills that are needed in order to challenge the status quo is
flexibility.
o Flexible people are more likely to try new things, take risks, and face new challenges
without fear. For example, Abraham Lincoln challenged the existing system of slavery in
USA. He was instrumental in Bringing the 13 amendment which abolished slavery.
o Raja Rammohan Roy, a 19th century moderate leader from Bengal advocated against
the cruel practice of the burning of the widow as a way to guarantee that both the
widow and the deceased husband would reside in heaven. Thus created a Movement
which eventually abolished the cruel practice.
• Enable others to act: Success requires a team and leaders, by definition, require followers.
Leaders can empower others in a variety of ways. They enable others by fostering
collaboration and building trust.

DELHI JAIPUR PUNE HYDERABAD AHMEDABAD LUCKNOW CHANDIGARH


14 www.visionias.in # 8468022022 ©Vision IAS
Google it:- https://upscpdf.com
https://t.me/UPSC_PDF Download From > https://upscpdf.com https://t.me/UPSC_PDF

o Successful leaders share power, delegate well, and do what’s necessary to help others Student Notes:
perform. In terms of emotional intelligence, there is a need of self-regard and
interpersonal skills to enable others to act. In order to build successful relationships,
you need the skills to engage and relate to others in a meaningful way.
• Stay composed under pressure: Good leaders don’t flare up or lose control under difficult
circumstances.
• Encourage others: A key component of this practice involves recognizing the contributions
of others. Rewarding people for their participation goes a long way in motivating them to
be part of our team. Leaders who encourage others not only need to know how those
people feel but need to be capable of building relationships with them, as well.

10. Dark Side of Emotional Intelligence


• Manipulating others- Emotional intelligence is important, but the unbridled enthusiasm
has a dark side as well. New evidence shows that when people hone their emotional skills,
they become better at manipulating others.
o When you’re good at controlling your own emotions, you can disguise your true
feelings. When you know what others are feeling, you can tug at their heartstrings
and motivate them to act against their own best interests. Especially when they have
self-serving interests, EI becomes a weapon for manipulating others.
• Inciting Emotion – Robbing of our capacity to Reason- When a leader gave an inspiring
speech filled with emotion, the audience was less likely to scrutinize the message and
remembered less of the content. Ironically, audience members were so moved by the
speech that they claimed to recall more of it.
o Hitler’s persuasive impact came from his ability to strategically express emotions—he
would “tear open his heart”—and these emotions affected his followers to the point
that they would “stop thinking critically and just emote.” Leaders who master
emotions can rob us of our capacities to reason. If their values are out of step with our
own, the results can be devastating.
• Personal Gain- Emotional intelligence helps people disguise one set of emotions while
expressing another for personal gain. Emotionally intelligent people “intentionally shape
their emotions to fabricate favorable impressions of themselves,”
• Hidden Cost- In settings where emotions aren’t running high, emotional intelligence may
have hidden costs. In jobs that involved fewer emotional demands, the results reversed.
The more emotionally intelligent employees were, the lower their job performance.
For mechanics, scientists, and accountants, emotional intelligence was a liability rather than an
asset. Although more research is needed to unpack these results, one promising explanation is
that these employees were paying attention to emotions when they should have been focusing
on their tasks.

11. Way Forward


Succeeding in life largely depends on succeeding socially, and a large part of social success
depends on EQ. But as a growing body of research shows, EQ can be used to orchestrate ‘win-
lose’ as well as ‘win-win’ outcomes. Of course, people aren’t always using emotional
intelligence for nefarious ends. More often than not, emotional skills are simply instrumental
tools for goal accomplishment.
Appropriate level of EI demands capacity for appreciation of ‘interconnections’ of matters
which are seemingly unrelated but together influence the outcome of a public policy or project.
Emotional Intelligence, thus, has to be factored in administrative justice, and it may even be
called ‘constructive emotional intelligence’.

DELHI JAIPUR PUNE HYDERABAD AHMEDABAD LUCKNOW CHANDIGARH


15 www.visionias.in # 8468022022 ©Vision IAS
Google it:- https://upscpdf.com
https://t.me/UPSC_PDF Download From > https://upscpdf.com https://t.me/UPSC_PDF

12. Previous Years Questions of UPSC Student Notes:

• What is ‘emotional intelligence’ and how can it be developed in people? How does it help
an individual in taking ethical decisions? (2013)
• Anger is a harmful negative emotion. It is injurious to both personal life and work life. (a)
Discuss how it leads to negative emotions and undesirable behaviours. (b) How can it be
managed and controlled? (2016)
• How will you apply emotional intelligence in administrative practices? (2017)
• “Anger and intolerance are the enemies of correct understanding. “Mahatma
Gandhi (2018)
• Emotional Intelligence is the ability to make your emotions work for you instead of
against you”. Do you agree with this view? Discuss. (2019)

13. Previous Years case studies of UPSC


1. ABC Ltd. is a large transnational company having diversified business activities with a huge
shareholder base. The company is continuously expanding and generating employment.
The company, in its expansion and diversification programme, decides to establish a new
plant at Vikaspuri, an area which is underdeveloped. The new plant is designed to use
energy efficient technology that will help the company to save production cost by 20%. The
company's decision goes well with the Government policy of attracting investment to
develop such underdeveloped regions. The Government has also announced tax holiday for
five years for the companies that invest in underdeveloped areas. However, the new plant
may bring chaos for the inhabitants of Vikaspuri region, which is otherwise tranquil. The
new plant may result in increased cost of living, aliens migrating to the region, disturbing
the social and economic order. The company sensing the possible protest tried to educate
the people of Vikaspuri region and public in general that how its Corporate Social
Responsibility (CSR) policy would help overcome the likely difficulties of the residents of
Vikaspuri region. In spite of this the protests begin and some of the residents decided to
approach the judiciary as their plea before the Government did not yield any result.
(a) Identify the issues involved in the case.
(b) What can be suggested to satisfy the company's goal and to address the residents'
concerns?

2. A private company is known for its efficiency, transparency and employee welfare. The
company though owned by a private individual has a cooperative character where
employees feel a sense of ownership. The company employs nearly 700 personnel and they
have voluntarily decided not to form union. One day suddenly in the morning, about 40
men belonging to political party gate crashed into the factory demanding jobs in the
factory. They threatened the management and employees, and also used foul language.
The employees feel demoralized. It was clear that those people who gate crashed wanted
to be on the payroll of the company as well as continue as the volunteers/members of the
party. The company maintains high standards in integrity and does not extend favours to
civil administration that also includes law enforcement agency. Such incident occur in
public sector also. (20 Marks) (250 Words)
(a) Assume you are the CEO of the company. What would you do to diffuse the volatile
situation on the date of gate crashing with the violent mob sitting inside the company
premises?
(b) What can be the long term solution to the issue discussed in the case?
(c) Every solution/action that you suggest will have a negative and a positive impact on you
as (CEO), the employees and the performance of the employees. Analyse the consequences
of each of your suggested actions.

DELHI JAIPUR PUNE HYDERABAD AHMEDABAD LUCKNOW CHANDIGARH


16 www.visionias.in # 8468022022 ©Vision IAS
Google it:- https://upscpdf.com
https://t.me/UPSC_PDF Download From > https://upscpdf.com https://t.me/UPSC_PDF

14. Previous Year Questions from Vision IAS Test Series Student Notes:

1. “Emotion can lead to our worst decisions or our best ones: The difference is emotional
intelligence.” In light of the given statement, illustrate how emotional intelligence is
critical in decision making. How can it help a civil servant in taking prudent decisions?

Approach:
The basic theme of the question is "criticality of emotional intelligence in decision
making". Accordingly, the answer can be framed in following manner:
• In the introduction, discuss how emotions influence the decisions of an individual
and how emotions lead to good or bad decisions.
• Thereafter, introduce the concept of emotional intelligence. Demonstrate how EI
makes the difference in decision-making.
• Conclude by citing relevance of emotional intelligence in decision-making activity of
a civil servant.
Answer:
Almost from the beginning, the decision-making experts encouraged to think that
making decisions is a rational process involving facts and analysis. However, in reality
decisions are acts of judgment made by people. These judgments are shaped and
formed in our brain, through processes that are just now being understood. In fact, in
many instances, the real driver of our actions is our emotions.
Many experts and empirical studies warn decision-makers about the perils of making
decisions when one is emotionally aroused. The important thing here is not the
presence of emotions, but the way in which the individuals interpret and deal with
emotions. When an individual appropriately deals with emotions, he/she is able to
make better decisions. The more skilled a person is in dealing with his/her emotions,
the more likely that person is to make more correct decisions.
Emotional Intelligence is defined as individual abilities that enable an individual to
perceive, thoughtfully use, understand, and manage emotions to achieve a productive
end. When emotions are elicited in any decision making process, they are processed by
abilities included under emotional intelligence. Decisions involve the use of cognitions
and emotions in varying amounts.
For example, when faced with a situation eliciting intense anger, a person could make
an unethical decision prompted by that anger, rather than going through a rational,
multi-step process. Take example of law enforcement agencies. When faced with
hostile environment, they may commit revengeful acts.
In the civil services, therefore, emotional intelligence is beneficial to manage the
emotions in a manner that will be helpful to achieve productive outcomes. High-EI
individuals, therefore, could reduce the effects of negative emotions like anger or rage
on their decision-making. Rage and anger are not only removed as hindrances, but also
used to enhance the quality of decisions. Therefore, emotionally intelligent civil
services could result in better decisions especially when more negative emotions
accompany decisions.

DELHI JAIPUR PUNE HYDERABAD AHMEDABAD LUCKNOW CHANDIGARH


17 www.visionias.in # 8468022022 ©Vision IAS
Google it:- https://upscpdf.com
https://t.me/UPSC_PDF Download From > https://upscpdf.com https://t.me/UPSC_PDF

2. Is Emotional Intelligence influenced by cultural moorings? How would you Student Notes:
differentiate the applicability of Emotional Intelligence in Indian society from that of
Western society?
Approach:
• Discuss how EI gets affected by cultural values.
• Differentiate the application of Emotional Intelligence in Indian and western
context.
Answer:
Emotional intelligence is the ability to identify and manage our own emotions and the
emotions of others. Studies have shown that a part of emotional intelligence ability is
innate while another part can be changed through learning and experience; this later
part dominated by culture provides the structures, guidelines, expectations, and rules
to interpret behaviors.
Although emotions are known as universal phenomena, most researchers believe that
the way in which they are being experienced, expressed, perceived, and regulated can
be influenced by cultural norms.
Culture affects emotional functioning by identifying which emotions are negative or
positive, when emotions should be expressed and even how emotions should be
displayed.
Several ethnographic studies suggest there are cultural differences in social
consequences, particularly when it comes to evaluating emotions.
Explicit expression of emotions like anger, love, frustration etc. are considered as
uncouth and are restrained in public in collectivistic cultures, but the same is
considered essential in individualistic cultures. Thus in America, parents encourage
emotional expression while suppression of emotion is often disapproved.
Thus, the way people perceive, regulate and exhibit emotions varies according to their
cultural background.
Applicability of EI in India and the West
EI is embedded in Indian philosophic tradition, which stresses the powerful nature of
emotions. These emotions need to be harnessed for a harmonious life. For example,
the Bhagavad-Gita refers to the emotionally intelligent person as a ‘Sthithapragnya’ (the
emotionally stable person). A ‘Sthithapragnya’, according to Lord Krishna is one who
remains unperturbed in the face of calamity, and takes good or evil with equanimity. He
has the power to emotionally attach or detach from any situation, at his will.
The following differences may be listed:
• Western approach deals largely with how we conduct ourselves with people around
us in different settings. But Indian approach also has stereological purpose, i.e. to
achieve self-control for getting Moksha by performing our Dharma. It also teaches
us to maintain equanimity of mind by detaching ourselves from the end results.
• In India, individuals see themselves as interdependent with their groups (collectivist
culture), whereas in West, people are independent and give more importance to
personal goals and personal needs (individualistic culture).
• Indian employees usually prefer a more directive, task-oriented style of leadership,
compared to the participative style advocated by western managers.

DELHI JAIPUR PUNE HYDERABAD AHMEDABAD LUCKNOW CHANDIGARH


18 www.visionias.in # 8468022022 ©Vision IAS
Google it:- https://upscpdf.com
https://t.me/UPSC_PDF Download From > https://upscpdf.com https://t.me/UPSC_PDF

3. Emotions, earlier considered as an irrational factor in decision-making, is now Student Notes:


recognized as a critical factor of judgment. In this regard, answer the following
questions: (a) how can Emotional Intelligence help in coping with the intense pressure
and occupational stress faced by police officers and armed forces in discharge of their
duties? (b) What are the some of the concerns in incorporating and assessment of
emotional intelligence skills in public service?
Approach:
• Give a brief explanation of Emotional Intelligence (EI).
• Explain how it can help police and armed forces in dealing with occupational stress
and pressure.
• List some of the concerns that arise while incorporating and assessing EI skills in
public service.
Answer:

Emotional intelligence (EI) is the ability to identify, use, understand, and manage one’s
emotions as well as emotions of others in positive ways to relieve stress, communicate
effectively, empathize with others, overcome challenges and defuse conflict.
Emotions were earlier treated as an irrational factor that distort reasoning and thus,
affects decision-making. However, with emotional intelligence, emotions were
recognized as an important part of decision making. In the work of public officials,
certain situations involving issues related to vulnerable sections of population require
understanding of emotions for optimal decisions. Thus, emotional intelligence is
necessary. It can help segregate purely emotional decisions or decisions arising out of
anxiety from decisions based on rational factors.
(a) Police officers and members of the armed forces face occupational stress due to
factors like isolation from family and friends, hostile environment especially in areas
where people oppose their posting, adherence to strict rules and occupational
hierarchy, intense workload, role ambiguity, abysmal living conditions, danger/threat to
their lives especially in combative areas, among others.
In such situations, EI can help them cope with pressure and stress. The cornerstones of
EI include:
• Self-awareness: knowing one’s emotional and mental state in such physically taxing
jobs and taking actions to avoid depression as well as extreme steps, such as recent
news of officers committing suicide.
• Self-management: ability to control oneself even in difficult times such as
overstretched working hours, emergency situations etc.
• Social awareness: including empathy towards fellow officers especially
subordinates as well as citizens while dealing with their grievances.
• Relationship management: maintaining a balance between professional and
personal relationships and developing the ability to support one’s family
emotionally even when one can’t be there physically.
Moreover, in conflict situations where the public is wary of officers, EI can help in open
communication and respect for opposing viewpoints.
However, it should be noted that the duties of police personnel and members of armed
forces essentially constitute use of proportionate force to maintain law and order and
hierarchical leadership is essential for maintaining the chain of command in these
forces. Hence, understanding of EI in these forces should be modeled along these
parameters.

DELHI JAIPUR PUNE HYDERABAD AHMEDABAD LUCKNOW CHANDIGARH


19 www.visionias.in # 8468022022 ©Vision IAS
Google it:- https://upscpdf.com
https://t.me/UPSC_PDF Download From > https://upscpdf.com https://t.me/UPSC_PDF

(b) Some concerns in incorporating and assessment of EI in public services include: Student Notes:
• Measurement of EI: Although, there are tests for measuring EI such as Mayer-
Salovey-Caruso EI test (MSCEIT), Wong’s EI scale etc., their validity is questioned as
the results are not deemed objective and reliable. Hence, lack of validity dilutes
the importance of administering these tests among public servants.
• Appropriate stage: Even if incorporated, there is no indication on whether the
tests will be conducted during hiring stage or promotion stage. The weight age of
EI tests in successive stages has not been determined
• Inflexibility in services: Public services are characterized by rigid structure and
disinclination towards change, right from the hiring process to promotion and
retirement. Hence, there is less scope of introduction of new practices in such
inflexible organizations.
• Dynamic environment: Public servants work in dynamic environment where
various kinds of emotions need to be dealt with. Thus, ability to completely
incorporate and assess the emotional intelligence in this scenario will be difficult.
• Balancing with other parameter: Factors such as competitive exams, educational
qualifications, physical fitness etc. are given more importance while hiring
personnel in the public services, thus there is a need to strike a balance.
Hence, EI is not yet an important component of public services in India. However, it
should be considered as pertinent to public servants as they should be responsive and
sensitive to people’s needs, especially, in an environment of constant change.

15. Previous year case studies from Vision IAS Test Series:
1. You are officer-in-charge of a very important railway junction, which is an artery of
trade and commerce. A peasant disturbance has been brewing in your district for the
past few weeks. Their discussions with the political and district leadership has borne
no fruit and it has come to the stage that now they are protesting by organizing a sit-
in on the railway tracks near the station. They have thereby succeeded in blocking
movement of all trains. This disruption is causing significant harassment for the
passengers waiting at the platform as well.
(a) What will be your course of immediate action?
(b) How can emotional intelligence act as a tool in handling this issue?
(c) What steps will you take so that such incidents are not repeated in the future?
Approach:
• Mention what your immediate action will be given the situation.
• Explain the importance of emotional intelligence in handling the issue.
• Then highlight the steps that you would take so that such incidents are not
repeated in the future.
Answer:

a) Being an officer in charge of the railway station it is my duty to ensure that the
railway operations do not get affected by the ongoing protest. I will make sure that the
railway tracks are cleared by using persuasion, warning and all other legal means
available. I will also seek help from district administration to ensure that smooth
functioning of the critical railway junction is not hampered. Additionally, I will ensure
that the passengers face minimum inconvenience by providing timely communication
to the passengers about the current situation, ensuring basic amenities like water,
medical aid etc. at the station. Safety of the passengers at the station will also be taken
care of.
b) Emotional intelligence is very crucial to handle the above situation.

DELHI JAIPUR PUNE HYDERABAD AHMEDABAD LUCKNOW CHANDIGARH


20 www.visionias.in # 8468022022 ©Vision IAS
Google it:- https://upscpdf.com
https://t.me/UPSC_PDF Download From > https://upscpdf.com https://t.me/UPSC_PDF

• Emotional intelligence is helpful in keeping oneself calm and composed. Student Notes:
Coordination at many levels – district administration, angry protesters, passengers
and senior officials of railway – in such situation is likely to generate huge mental
pressure.
• The officer should empathize with the emotions of the crowd and win the trust of
angry peasant protesters who are full of doubt. The officer must base arguments
on huge economic loss, inconvenience to general public and the legal
consequences to convince them for dropping this method of protest and engage in
meaningful talks with the political leaders.
• Some passengers may have urgent reasons to travel and the current situation
requires that the emotions of passengers showing regret with the current service
needs to be understood. The officer must show emotional maturity in dealing with
such passengers.
(c) First, I would ensure that those who are involved in blocking the railway lines get
adequately punished. This will dissuade anyone in future to attempt the same.
Additionally, I will create public awareness about the legal aspects of such actions
which will educate the public and will prevent such incidences.
While protesting for certain issues is not wrong but the mechanism to be adopted
should be peaceful and within legal means. In any case, illegal means of protest like
blocking railway lines is not acceptable in a democracy where various avenues of
staging dissent are available.

2. You are the CEO of a film production company, which has not been doing well
financially for some time. Your company is now relying on its forthcoming movie,
which is about to be released shortly. The movie has a cast of reliable actors and even
before its release, trade pundits have predicted that the movie will be a hit. However,
you face a conundrum as one of the actors in the movie is a citizen of a neighbouring
country with which relations have been disturbed and a war like situation exists.
While the political atmosphere was different when the movie was being filmed, now
there is a widespread public demand, with a local political party at the forefront, for
replacing the actor from the movie or a ban on the release of the movie itself, if the
actor is not replaced. While you are aware of the mood of the nation and the public
repercussions of releasing the movie in its present form, you also know that it is not
feasible to replace him at this stage as he has a substantial role in the movie. There is
also a section in the film fraternity that does not want you to compromise in the wake
of threat by the local party as it compromises freedom of speech and expression as
well as artistic creativity.
(a) What are the options available to you?
(b) Evaluate each of the options and state their merits and demerits.
(c) What course of action would you take and why?

Approach:
• State the options available to you as a CEO of the company.
• Evaluate the merits and demerits of the available options.
• State your course of action, with reasons.
Answer:
In this case, the issues involved are
• Securing corporate interests as a CEO of the company.

DELHI JAIPUR PUNE HYDERABAD AHMEDABAD LUCKNOW CHANDIGARH


21 www.visionias.in # 8468022022 ©Vision IAS
Google it:- https://upscpdf.com
https://t.me/UPSC_PDF Download From > https://upscpdf.com https://t.me/UPSC_PDF

• Stifling of freedom of speech and expression guaranteed by the Constitution under Student Notes:
Article 19 (1) (a) and political coercion and intimidation.
• Questioning one’s nationality based on one’s professional decisions, interference
with artistic creativity and hiring practice.
• Widespread public sentiment guided by the changing political atmosphere.
The stakeholders involved are:
• CEO of the production company who is relying financially on the upcoming movie.
• The movie star who belongs to a neighbouring country with which a war like
situation exists.
• Rest of the cast and crew who worked hard to complete the film.
• Members of the local party who have threatened to ban the movie due to the
presence of the mentioned actor.
• Film fraternity as a whole with the broader issue of freedom of speech under
question.
• Audience in general who will be the ultimate judge of the movie and its casting.
Options available in this scenario and their merits and demerits are:
• Agree to the demands of the party and replace the actor.
o Merit: As a CEO, my ultimate interest is the company’s profit. The company is
relying financially on the forthcoming film and it is important that the film
releases.
o Demerit: However, doing so leaves avenues for political parties to repeat such
threats. Additionally, replacing the actor requires reshooting his scenes, which
is not feasible.
• Take a firm stand and release the film as it is.
o Merit: It is a courageous act of protecting the interests of company, its staff and
the whole film fraternity. It will be step towards building free and liberal society
by sending a wider message of artistic creativity and freedom of speech and
expression.
o Demerit: However, this can lead to chaos and unanticipated consequences if
the party does what it has claimed. Also, since there is a widespread public
opinion, I may suffer losses.
• Minimize the number of scenes the said actor is a part of with only the essential
scenes.
o Merit: This will help in neutralizing the widespread public opinion against the
movie which will help me financially since it is predicted to be a success.
o Demerit: However, this implies suppression of freedom of speech and
expression guaranteed by the Constitution and artistic creativity. Further, there
is no guarantee that such demands won’t be repeated in the future.
• Postpone the release of film for some days and wait for the public sentiments to
subside
o Merit: It will protect the film in its original form, provide more time for decision
making and protect the interests of the movie star. Since the movie is good, it
may get a good response, which may help in generating high revenues for the
company.
o Demerit: A delay in release has certain financial implications and it may
demoralise the crew members. Also it may only amount to delaying the
inevitable rather than resolving the issue.

DELHI JAIPUR PUNE HYDERABAD AHMEDABAD LUCKNOW CHANDIGARH


22 www.visionias.in # 8468022022 ©Vision IAS
Google it:- https://upscpdf.com
https://t.me/UPSC_PDF Download From > https://upscpdf.com https://t.me/UPSC_PDF

Course of action Student Notes:


• I will approach the party leader and reason it out with him/her. But, if he/she
refuses, I will release the film with the said actor. I will also approach the local
police and state leaders to ensure that the film releases without any obstacles.
• Additionally, I will request the support of the film fraternity and associations of
various producers, theatre owners etc. as a measure of solidarity. I will also
request the audience to give the film a fair chance and separate an actor’s work
from his/her nationality via social media and other mediums of interaction.
• It is important to take a stand as art transcends national boundaries and hiring
actors who belong to other countries is not a yardstick for measuring patriotism
and nationalism. Moreover such actions at home may also adversely affect Indian
artists working in the neighbouring country.

3. You and your friend are living together in a metropolitan city and preparing for civil
service examinations. While you manage to meet your daily targets, you sense that
your friend is unable to cope up with the pressure of the exam. Fear of not passing the
exam and meeting the expectations of his family is stressing him further. You realise
that with time your friend is losing interest in everything and often talks about
committing suicide if he is unable to meet his goal. He is also missing out on meals
and prefers to stay indoors when asked to venture out. When you sought professional
help, the doctor diagnosed your friend with clinical depression. Being aware that your
friend needs professional counselling and psychiatric care, you reach out to his
parents who reside in a rural area. They rebuff you for suggesting counselling and
instead reiterate that nothing is wrong with him. They sternly mention that your
bringing up the matter will only make people engage in loose talk. They also ignore
you when you politely inform them that it is not wise to ignore one’s mental health.
Your friend’s parents see his state as a sign of failure and decide to call him home
where he can continue with his preparation. You are aware of the gravity of the
situation if your friend goes back to his house. You also know that there is very little
awareness about mental health and that the solution is not to ignore it but to take
necessary measures to tackle it.
(a) As a concerned friend and an aspiring civil servant who can frame policies for the
public in the future and has a moral duty towards them, what are the options
available to you in such a situation?
(b) Evaluate each of these options and choose the option you would adopt, giving
reasons.2017-13-861
Approach:
• State the available options and their impacts in this situation
• List the pros and cons of each of the possible options and state the eventual course
of action.
Answer:
a) The options available to me in this scenario are:
1. Convincing my friends’ family to keep my friend in the city and taking
responsibility for his treatment.
2. Not pursuing the matter any further once his parent don’t listen to my advice.
3. Let him go back to the village as his parent desire but helping him in other
possible ways.

DELHI JAIPUR PUNE HYDERABAD AHMEDABAD LUCKNOW CHANDIGARH


23 www.visionias.in # 8468022022 ©Vision IAS
Google it:- https://upscpdf.com
https://t.me/UPSC_PDF Download From > https://upscpdf.com https://t.me/UPSC_PDF

b) Evaluating each option Student Notes:


1) Convincing my friends’ family to keep my friend in the city and taking
responsibility for his treatment.
Considering the fact that my friend is suffering from suicidal tendencies, his life
is at great risk and as a friend and well-wisher I must ensure that he gets
professional help from a psychiatrist. The fact that his family is not aware about
the gravity of the situation due to rural background necessitates my
intervention even further.
However, this will come at the cost of my precious time which I need to devote
to my studies. This may also enrage my friend’s family members who are
against any medical intervention to the problem due to social fears. Medical
cost may also be an issue which may arise.
2) Not pursuing the matter any further once his parents don’t listen to my advice.
As my friend’s parents do not consider my advice useful, I may focus back on
my studies leaving the matter solely in their hands.
This will be an escapist route and may lead to adverse consequences for my
friend, his family and even for me as any harm to my friend will also impact my
mental state and my preparation for the upcoming exams. This will amount to
selfish and mean behaviour.
3) Let him go back to the village as his parents desire but helping him in other
possible ways.
This may help him as he will be close to his family, which may provide the much
needed psychological support. As health is the utmost priority here, a bit of
sacrifice of regular studies is not an issue.
But professional advice and intervention is still sought which may not be readily
available in a rural area.
Possible Course of Action
First, I will try to educate my friends’ parents about Depression, its implications and will
apprise them of the professional advice which was given in this case. Asking the doctor
to convince my friends’ parents about the treatment may also be useful.
Regarding whether my friend should stay here or go back to his village for studies should
be decided on the basis of professional advice, availability of psychiatrist in the village
and his convenience.
Taking a break may help especially in reviving his interests which he is losing fast. I will
also try to help him in meeting his daily targets which are bothering him and are a cause
of stress for him. Additionally, I will also suggest him to break the targets into smaller
tasks.
Lastly, I will make my friends’ parents aware about the hard work my friend is putting
into this exam which should be praised.
As competitive exams are extremely stressful and mentally taxing, an aspirant may at
times face anxiety or depression which must be treated like any other health anomaly.
As a friend/partner/relative etc. of a person who suffers from depression, we should
ensure that we do our best to normalize the situation for the person diagnosed with it
and support him/her in every stage of treatment.
Copyright © by Vision IAS
All rights are reserved. No part of this document may be reproduced, stored in a retrieval system or
transmitted in any form or by any means, electronic, mechanical, photocopying, recording or
otherwise, without prior permission of Vision IAS.

DELHI JAIPUR PUNE HYDERABAD AHMEDABAD LUCKNOW CHANDIGARH


24 www.visionias.in # 8468022022 ©Vision IAS
Google it:- https://upscpdf.com
CONTRIBUTION OF MORAL THINKERS, Student Notes:

PHILOSOPHERS, REFORMERS AND


ADMINISTRATORS
Contents
1. Introduction ............................................................................................................................... 2
2. Contribution of Moral Thinkers and Philosophers ..................................................................... 2
2.1. Indian Thinkers .................................................................................................................... 2
2.1.1. Kautilya ......................................................................................................................... 2
2.1.2. Buddha ......................................................................................................................... 3
2.1.3. Thiruvallur .................................................................................................................... 5
2.1.4. Swami Vivekanand ....................................................................................................... 6
2.2. Western Thinkers ................................................................................................................ 8
2.2.1. Plato ............................................................................................................................. 8
2.2.2. Aristotle ........................................................................................................................ 9
2.2.3. Emanuel Kant ............................................................................................................. 10
2.2.4. Karl Marx .................................................................................................................... 10
2.2.5. Weber ......................................................................................................................... 11
2.2.6. Locke .......................................................................................................................... 11
2.2.7. Hobbes ....................................................................................................................... 12
2.2.8. John Rawls .................................................................................................................. 12
2.2.9. John Stuart Mill .......................................................................................................... 13
3. Lessons from the Lives and Teachings of Great Leaders .......................................................... 14
3.1. Mahatma Gandhi .............................................................................................................. 14
3.2. Amartya Sen ...................................................................................................................... 16
4. Lessons from the Lives and Teachings of Great Reformers ...................................................... 16
4.1. Raja Ram Mohan Roy ........................................................................................................ 16
4.2. Baba Amte ......................................................................................................................... 17
5. Lessons from the Lives and Teachings of Great Reformers ...................................................... 18
5.1. E. Shreedharan .................................................................................................................. 18
5.2. TN Sheshan........................................................................................................................ 19
6. Way Forward ............................................................................................................................ 19
7. Previous Years UPSC GS Mains Questions ................................................................................ 19
8. Previous Years Vision IAS GS Mains Questions ........................................................................ 20

Copyright © by Vision IAS


All rights are reserved. No part of this document may be reproduced, stored in a retrieval system or
transmitted in any form or by any means, electronic, mechanical, photocopying, recording or
otherwise, without prior permission of Vision IAS.

DELHI JAIPUR PUNE HYDERABAD AHMEDABAD LUCKNOW CHANDIGARH


1 www.visionias.in # 8468022022 ©Vision IAS
https://t.me/UPSC_PDF Download From > https://upscpdf.com https://t.me/UPSC_PDF

1. Introduction Student Notes:

Ethics is the philosophical study of morality. Ethics as branch of philosophy arises from the
curiosity about the values involved in the human behavior. This behaviour can be studied from
various dimensions in the lives of virtuous people. From the ancient past to the recent decades,
we have witnessed various philosophers, moral thinkers, leaders, reformers as well as
administrators which can help us develop new knowledge, values and principles to induce
moral progress in human society and make it better.

2. Contribution of Moral Thinkers and Philosophers


2.1. Indian Thinkers
2.1.1. Kautilya
The teachings of Kautilya have been primarily derived from his book on Statecraft i.e. the
Arthashastra. It was written primarily as a book of codes for efficient administration. However,
it also includes various learnings with regards to ethical administration. He propounds various
duties of the king, the ministers, the officials as well as for the common man.

Key lessons from philosophy of Kautilya-


• Concept of Welfare State (Yogakshema)- As per Kautilya, the king is responsible to his
subjects. He says, “In the happiness of the subjects lies the happiness of the king; in their

DELHI JAIPUR PUNE HYDERABAD AHMEDABAD LUCKNOW CHANDIGARH


2 www.visionias.in # 8468022022 ©Vision IAS
Google it:- https://upscpdf.com
https://t.me/UPSC_PDF Download From > https://upscpdf.com https://t.me/UPSC_PDF

welfare his welfare. The king shall not consider what pleases himself as good; whatever Student Notes:
pleases his subjects is only good for him”. Accordingly, he highlighted the crucial role of the
State/Government for ensuring material well-being of the nation and its people.
• Concept of Materialism- His gives the idea of ‘Artha’, which has a broader meaning beyond
the personal possessions. He regarded that the society should be above the individual
interest. He regarded money as valuable only to the extent it serves as a means to acquire
goods. Thus, he guides individuals to mould their lives for wealth generation in an ethical
manner. In modern times, corporate scandals like Enron, Tyco and Satyam have time and
again brought forth the importance of ethics in economic and corporate affairs.
• Ideals of ethical conduct- He lays down ideals of ethical conduct for each individual, to
whichever social class he may belong, the duties of ahimsa (refraining from injury), satya
(truthfulness), sauca (purity), anasuya (freedom from malice), anrsamsya (compassion) and
kshama (forgiveness).
• Views on corruption- He highlighted the endemic invisibility of corruption in
administration. He compared corrupt administrators with an analogy of like a fish in water
who cannot be caught when it drinks water. Hence, he proposed the need for strong penal
framework for tackling corruption and ensuring probity in civil administration.
• Maintenance of social order- Arthashastra lays down that the ruler is required to safe-
guard the social order based on the ancient varna and the asrama system. This order is
believed to have been prescribed in the Veda and, as such, divinely ordained. Therefore, it
is considered immutable. The state had no role in its creation. So, it does not have any role
to try to modify it. The duty of the state is only to preserve this order and not to allow it to
be disturbed in any way. Even though these ideas are not in consonance with modern
constitutional setup, still they hold relevance towards the ethical conduct of individuals as
well as the state for the maintenance of social order.
2.1.2. Buddha
Essentially, according to Buddhist teachings, the ethical and moral principles are governed by
examining whether a certain action, whether connected to body or speech is likely to be
harmful to one's self or to others and thereby avoiding any actions which are likely to be
harmful.
In Buddhism, there is much talk of a skilled mind. A mind that is skillful avoids actions that are
likely to cause suffering or remorse. Buddha focuses on moral discipline in which he focuses on
the five Precepts. These apply to all the common people and are similar to basic lists of
prohibitions in other great world religions. It seeks to make a commitment to refrain from
killing, stealing, sexual misconduct, lying, and drunkenness.
All his teachings were derived from his core ideas which he propounded in the form of four
noble truths, which are-
• Dukkha- There is suffering
• Samudaya- There is a cause of suffering
• Nirodha- There is a solution to every suffering
• Marga- There is a path to cessation of suffering.
Buddha teaches us that if one understands these noble truths, one can get freedom from all the
sufferings in this world. For this he propounds the eight-fold path, which can be seen as
follows-

DELHI JAIPUR PUNE HYDERABAD AHMEDABAD LUCKNOW CHANDIGARH


3 www.visionias.in # 8468022022 ©Vision IAS
Google it:- https://upscpdf.com
https://t.me/UPSC_PDF Download From > https://upscpdf.com https://t.me/UPSC_PDF

Student Notes:

Key lessons from the philosophy of Buddha-


• Mindfulness- Here, Buddha tries to notice present thoughts, feelings and sensations
without judgement. The aim is to create a state of “bare awareness”. What was once a tool
for spiritual exploration has been turned into a panacea for the modern age i.e. a cure-all
for common human problems, from stress, anxiety and depression.
• Momentariness- In this core aspect, Buddha states that everything that has originated is
bound to perish, and there is temporariness or momentariness of everything. Thus, world
is at every moment distinct from the world in the previous or next moment. By using this,
every person can practice non-attachment and selflessness as nothing is permanent in this
world. This can be of immense help in the modern day, as it can lead to conflict resolution
in the domains of social conflicts, administration as well as international relations.

DELHI JAIPUR PUNE HYDERABAD AHMEDABAD LUCKNOW CHANDIGARH


4 www.visionias.in # 8468022022 ©Vision IAS
Google it:- https://upscpdf.com
https://t.me/UPSC_PDF Download From > https://upscpdf.com https://t.me/UPSC_PDF

• Linkage between individual action and world- In his concept of Karma, Buddha Student Notes:
propounds that if a person has a right mindset, his actions will be beneficial not just to
himself but to the whole world, including the environment. As per him, our actions affect
the planet in a harmful way because we are selfish and we crave things. These actions will
only result in more suffering in the future. The effects of karma will continue to work in a
person's rebirth, so by being compassionate, we will improve our own future and that of
the Environment.
• Bhavana- It means the training of the mind. The word itself etymologically means
development - a further development of the mind. The Buddha believed that everything
emanates from the man’s mind. And that reflects the first line of the first verse of the
Dhammapada. A pure mind, a trained mind, a well-developed mind, a mind that can be
controlled at will, a mind that does not go on to subjects that are conducive to tension and
boredom, but keeps alert, keeps on developing itself, discovering itself and within itself the
secret of life, the problems of life and the reality of life, is man’s greatest treasure.
• Karuna- Compassion is more easily generated. When we see somebody in trouble, our
heart moves towards that person and we rush to help him. Last of all comes the fourth
aspect of loving kindness and that is total equanimity, Upekkha. We have no friends, no
enemies, no one higher, and no one lower. We have absolutely no distinctions between
one person and another, and are totally merged in a kind of unity with all beings, all things
and all situations. So once you are able to live a life in which all these four characteristics
govern your actions, there is no place for hatred, rivalry and competition.

2.1.3. Thiruvallur
Thiruvallur was a weaver by profession and belongs to Valluva caste whose profession it was in
the ancient days to announce the commands of kings by beating of drums. His book
"Thirukurral or Kural" is regarded as the first book on ethics. This book contains the seed of
modern ideas which was considered as important for living good life even in contemporary
times. To honor his work, Indian Government has decided to install his statue in the banks of
Ganges in Haridwar.

DELHI JAIPUR PUNE HYDERABAD AHMEDABAD LUCKNOW CHANDIGARH


5 www.visionias.in # 8468022022 ©Vision IAS
Google it:- https://upscpdf.com
https://t.me/UPSC_PDF Download From > https://upscpdf.com https://t.me/UPSC_PDF

Key lessons from the philosophy of Thiruvallur- Student Notes:


• Universal Ethics- He focuses on universal values related to humanity, love, relations and
contracts.
• Codification of Ethics- Thiruvallavur work touches different aspects of human life and
associated moral dilemmas. He prescribed there resolution through pursuit of reasoning
and wisdom. He also laid down the principles to be followed in personal life as moral code
to avoid degradation of self. He discussed the various dimensions of life in three major
heads of Aram (Dharma/Virtue), Porul (Wealth/Artha) and Inbam (Love/Kama).
• Personal and Public life are intertwined- According to him, in order to life good personal
life, person in public life should have certain fundamentals of conduct and character. This
line of thought also underlines the importance of honesty, integrity and consistency in
individual’s life.

2.1.4. Swami Vivekanand


Vivekanand was a modern thinker and reformer who attempted to combine
Indian spirituality with Western material progress, maintaining that the two supplemented and
complemented one another.

DELHI JAIPUR PUNE HYDERABAD AHMEDABAD LUCKNOW CHANDIGARH


6 www.visionias.in # 8468022022 ©Vision IAS
Google it:- https://upscpdf.com
https://t.me/UPSC_PDF Download From > https://upscpdf.com https://t.me/UPSC_PDF

Student Notes:

Key lessons from the philosophy of Vivekanand-


• Intrinsic purity and oneness of the Atman- As per him, the prevalent morality is mostly
based on fear (both for individual and society). This fear could from the police, public
ridicule, god’s punishment, fear of karma and so on. However, these do not explain why a
person should be moral and be good to others. We should be pure because purity is our
real nature and also the true divine Self or Atman. So, we should love and serve our
neighbours because we are all one in the Supreme Spirit. This helps in formation of attitude
of non-individuality, which can form healthy team spirit and culture.
• Ethics more than Laws: He believed that laws which are enacted by parliament does not
make a nation good or great, it’s the people and their conduct. For him ethics or morality
has something more than laws of a country, he believed that laws act as means and ethics
as end. Morality for him means renouncement of all temptations.

DELHI JAIPUR PUNE HYDERABAD AHMEDABAD LUCKNOW CHANDIGARH


7 www.visionias.in # 8468022022 ©Vision IAS
Google it:- https://upscpdf.com
https://t.me/UPSC_PDF Download From > https://upscpdf.com https://t.me/UPSC_PDF

• Ethical secularism- He propounded humanistic approach to religion which holds relevance Student Notes:
to the rising communal tendencies in the present context. He declared that "if you are born
as a Muslim be a good Muslim if as Hindu be a good Hindu".
• Compassion for all beings: He said the basis for ethical conduct is unselfishness - a person
should be willing to put other's needs ahead of his own. Unlike other ethical thinkers, he
clarified that the basis for such ethical conduct is essential oneness of all humans, not just
the need to be ethical. He expanded the scope of ethical behaviour to all living creatures
and not just humans.
• Education as Character Building exercise: He propounded that education is by which
character is formed, strength of mind is increased, the intellect is expanded, thus the
manifestation of the perfection already in man.

2.2. Western Thinkers


2.2.1. Plato
Plato was a great Greek philosopher. He is known as a true disciple of Socrates. He believed in
two basic premises-
• Goodness consists of the natural and proper functioning of human nature. Besides, man is
social by nature; therefore, society is a normal background of moral life of human beings.
• Good life is the life of virtues. Like later Greek and mediaeval thinkers1, Plato assumes that
virtue is necessary for happiness
He also propounded the framework of four cardinal virtues in his theory of morality, i.e.
wisdom, courage, temperance and justice, which together constitute a morally good life.
Key lessons from Plato’s four cardinal virtues-
• Wisdom- It includes knowing and mastering the non-rational elements such as spirited
element and passions. Thus, a man who is wise, shall always be guided by rational rules
rather than his impulses.
• Courage- It includes excellence in the activity of will in a person. A courageous man is one
who can stay intact to the instructions of his intellect. It is of two types viz. physical courage
of a soldier and moral courage of a thinker or a reformer. Thus, one can be courageous in
war as well as in intellectual convictions.
• Temperance- It includes the self-control in keeping bodily satisfactions within limits. But it
does not mean complete abstinence, rather it focusses on controlling and ordering the
natural instincts, desires and sensuous pleasures.
• Justice- It includes proper integration of all rational and non-rational parts of one’s self.
Justice consists in doing one’s own job, being morally perfect and therefore is tantamount
to being wise, valiant, temperate and just.

1
Medieval philosophy is the philosophy in the era now known as medieval or the Middle Ages, the period
roughly extending from the fall of the Western Roman Empire in the 5th century C.E. to the Renaissance
in the 16th century.

DELHI JAIPUR PUNE HYDERABAD AHMEDABAD LUCKNOW CHANDIGARH


8 www.visionias.in # 8468022022 ©Vision IAS
Google it:- https://upscpdf.com
https://t.me/UPSC_PDF Download From > https://upscpdf.com https://t.me/UPSC_PDF

Student Notes:

2.2.2. Aristotle
Aristotle was the disciple of Plato. He was the first one to write the book on ethics. He claimed
that virtue can be learned only through constant practice implies that there are no set rules we
can learn and then obey. Instead, virtue consists of learning through experience.
Key lessons from the philosophy of Aristotle
• Teleology of Nature: Nature works toward a telos, or end goal. Every human activity aims at
a certain telos.This teleological view gives Aristotle’s Ethics a clear sense of direction. Our
goal in life is to achieve our true nature, and this true nature consists essentially of
rationality.
• The highest human good is happiness-All human activities aim at some end that we
consider good. Most activities are a means to a higher end. The highest human good, then,
is that activity that is an end in itself. That good is happiness. When we aim at happiness,
we do so for its own sake, not because happiness helps us realize some other end. The
goal of the Ethics is to determine how best to achieve happiness.
• Doctrine of mean - Happiness depends on living in accordance with appropriate virtues.
That is, a virtuous person is naturally disposed to behave in the right ways and for the right

DELHI JAIPUR PUNE HYDERABAD AHMEDABAD LUCKNOW CHANDIGARH


9 www.visionias.in # 8468022022 ©Vision IAS
Google it:- https://upscpdf.com
https://t.me/UPSC_PDF Download From > https://upscpdf.com https://t.me/UPSC_PDF

reasons, and to feel pleasure in behaving rightly. Virtue is a mean state between the Student Notes:
extremes of excess and deficiency. This mean varies from person to person, so there are no
hard and fast rules as to how best to avoid vice.
• Justice- He has classified it into general and particular justice. The former deals with virtue
expressed towards other people whereas the latter can distributive as well as rectificatory.
o Distributive justice deals with the distribution of wealth among the members of a
community.
o Rectificatory justice remedies unequal distributions of gain and loss between two
people.
• Life of Contemplation: Aristotle maintains that contemplation is the highest human activity.
In discussing the various intellectual virtues, Aristotle extols wisdom as the highest, since it
deals only with unchanging, universal truths and rests on a synthesis of scientific
investigation and the intuitive understanding of the first principles of nature. The activity of
wisdom is contemplation, so contemplation must be the highest activity of human life.
2.2.3. Emanuel Kant
Kant, the Germany philosopher is known for de-ontological approach. Under this, he argues for
duty-based ethics for modern life. His ideas are critical to understand the complex system of
ethics that affects our current lives.
Key lessons from the philosophy of Kant
• Duty based ethics: His work placed special emphasis on the relationship between
duty and the morality of human actions. Acts are inherently good or evil regardless of the
consequences of act. This theory is also called as Deontological theory. It gives importance
to the performance of duties irrespective of the consequences.
• Role of moral will in ethics: Kant believed that moral law arises only from the pure will
which is self-determined and self- governed. A person is free when he is bound by his own
will i.e. moral courage and not by the will of others.
• Authority behind Morals: Moral laws are autonomous and the authority behind ethics is
the individual’s own will. The moral laws do not operate through the influence of external
factors. Each person’s own reason is the authority, the legislator and the executor of the
moral law. Hence ethics is autonomous, universal and unconditional. For example: An
autonomous state is one in which the laws are made by the will of the people in that state.
The laws have no legitimate authority when they are imposed by another state as it
happened during colonization.
• Categorical Imperative as the moral law imposed by practical reason- Categorical
Imperative is the internal law imposed by conscience upon itself. Kant distinguishes
Categorical Imperatives from Hypothetical Imperatives.
2.2.4. Karl Marx
Karl Marx inspired the foundation of various communist regimes in the modern world. He
turned away from philosophy in his mid-twenties, towards economics and politics.
Marx’s theory of history is centered around the idea that forms of society rise and fall as they
further and then impede the development of human productive power. Marx sees the historical
process as proceeding through a necessary series of modes of production, characterized by
class struggle, culminating in communism.
Key lessons from Marxism
• The issue of morality in Marxism is a complex question. Marxists believe that end justifies
the means. Many thinkers consider Marxists as immoral because they recommend violence
and killing but many others believe that it Marxism possess different kind of morality.

DELHI JAIPUR PUNE HYDERABAD AHMEDABAD LUCKNOW CHANDIGARH


10 www.visionias.in # 8468022022 ©Vision IAS
Google it:- https://upscpdf.com
https://t.me/UPSC_PDF Download From > https://upscpdf.com https://t.me/UPSC_PDF

• Marxists believe that “old morality”—the morality of the reigning capitalist class—exploits Student Notes:
the working class. According to this view, old religious moral codes must be abandoned. For
Karl Marx and Frederick Engels “Thou shalt not steal” establishes a society in which some
have property and some do not; such an establishment is the root of the problem.
• When pursuing Marxist ethics, revolution is the most efficient means for creating a society
without class distinctions. According to Marxists, revolution is unavoidable and it is the only
way to overthrow the bourgeoisie and lift up the proletariat.
• The obligation to work toward the overthrow of the bourgeoisie may very well include the
duty to kill.
2.2.5. Weber
Weber was a German sociologist. He devised ideal bureaucratic model. some of its
characteristics were, bound by rules, sphere of competence, principle of hierarchy, need for
specialized training, impersonal detachment, documentation, career service and a non-
bureaucratic head.
Weber believed in political superiority over the bureaucracy. He asserted that a civil servant
should not take interest in politics. He should engage in only impartial administration of his
office.
Key lessons from the philosophy of Weber
• Legal Rational Authority- Weber was in favour that authority should be derived from a
system of rules & norms, which are rationally derived. He opposed charismatic and
traditional authority for the bureaucracy.
• Protestant ethic as the source of Capitalism: The protestants believed that on performing
actions with honesty god will bless them with riches. The protestant ascetism holds that the
individual needs hard, continuous bodily or mental labour. This served as the foundation of
the capitalism.
• Civil Servants ethics: Weber said that, If the official receives orders with which he
disagrees, he should make his views known to his superior; but if the superior insists, the
official must comply to the best of his capability. In other words, a sense of duty should
prevail over personal view. This should be an integral part of the civil servant’s ethic.
• Accountability through Principle of hierarchy: Each lower office is under the control and
supervision of a higher one. This ensured every subordinate in the administrative hierarchy
is accountable to his superior for his actions.
2.2.6. Locke
John Locke was an English philosopher and political theorist, who is also hailed as the father of
“Liberalism”. His conceptions of the government and natural rights of the human beings laid
the foundation of USA constitution.
Key lessons from the philosophy of Locke-
• Three Natural Rights- To locke a man has three natural rights: life, liberty and property.
Property is the product of a person’s labour. This conception of property became the
foundation of Karl Marx’s Socialism and Adam Smith’s capitalism.
• Accountability of government- He rejected the divine right of the king. According to him,
societies form governments by their mutual consent. Therefore, a society may remove the
king when it loses faith in him, i.e. consent of the governed.
• Importance of education- He advocated early education, as education shapes the mind and
provides an individual with necessary mental faculty to take decisions based on their
preferences in a rational and logical manner.
• Tolerance- He argued that governments should have non-interfering approach when it
comes to freedom of religion. The only exception that he cites was a threat to public order.
To him governments were simply not in a position to decide which belief is right.

DELHI JAIPUR PUNE HYDERABAD AHMEDABAD LUCKNOW CHANDIGARH


11 www.visionias.in # 8468022022 ©Vision IAS
Google it:- https://upscpdf.com
https://t.me/UPSC_PDF Download From > https://upscpdf.com https://t.me/UPSC_PDF

2.2.7. Hobbes Student Notes:


Thomas Hobbes was an advocate of materialism. As per him, the human beings strive to fulfill
their various desires such as food, shelter, clothing, social acceptance etc. As per him, two
situations prevail in the world i.e. the resources are scarce and all persons are equal in their
power.
Based on these basic situations, he propounded two theories-
• State of nature is state of war- He believed that human beings will engage in a fierce
struggle for scarce resources to fulfil their self-interest.
• Right of nature- Here, he says that every individual is justified in the use of force and fraud
to defend his/ her state of nature.
Key lessons from the philosophy of Hobbes-
• Social Contract- Being innately selfish in nature human beings require some external
coercivce power to follow the articles of peace. This agreement between individuals to
establish laws that make communal living possible and an agency to enforce those laws is
called social contract. In Hobbes social contract, tradeoff between liberty for safety is
essential, otherwise there will be ‘war of every man against every man”.
• Objective Morality- As per him, morality is objective such as there are objective reasons we
shouldn’t kill or lie.
• Emphasis on Civic Education: Hobbes believed that state should reform the curriculum and
focus on civic education. To him arguing on matters of mind and free will etc is useless
instead state should emphasize on instilling the values of obedience amongst the citizens
through educating them.
2.2.8. John Rawls
He was an American moral and political philosopher, who described a method for moral
evaluation of social and political institutions. He propounded his theory of justice using a
though experiment called the “original position” and “Veil of Ignorance”.
Key lessons from the philosophy of Rawls-
• Justice as Fairness: Rawls recommends equal basic rights, equality of opportunity, and
promoting the interests of the least advantaged members of society.
o Fairness demands ignorance-Rawls thought most people’s definition of justice reflected
what was good for them instead of anything shared or universal. To get to the core of
things, Rawls asks us to imagine a situation where society doesn’t exist yet and no one
knows where they will end up in life.
o If we each play along with Rawl’s hypothetical, we are likely to imagine fairness in a
particular way. Rawls thought we would only join a society where everyone, no matter
what circumstances we were born in, has their needs met.
• Principles of social Justice: Rawls thinks a just society will conform to rules that everyone
would agree to in the original position. He developed principles of justice for a liberal
society and challenged utilitarian political philosophy.
• Reflective Equilibrium: It is the consistency and coherence between various opinions and
beliefs. We often form opinions based on gut reaction, instinct, or by focussing on the
specifics of a situation. Each of these approaches can at times be unreasonable and
inconsistent. By forcing us to test how our different beliefs fit together, Rawls encourages us
to do make sure our set of beliefs make sense and are consistent.

DELHI JAIPUR PUNE HYDERABAD AHMEDABAD LUCKNOW CHANDIGARH


12 www.visionias.in # 8468022022 ©Vision IAS
Google it:- https://upscpdf.com
https://t.me/UPSC_PDF Download From > https://upscpdf.com https://t.me/UPSC_PDF

Student Notes:

2.2.9. John Stuart Mill


John Stuart Mill carried forward the philosophy of utilitarianism, which was initially
propounded by Jeremy Bentham. Mill fully accepted Bentham’s devotion to greatest happiness
principle as the basic statement of utilitarian value.
Key lessons from the philosophy of Mill-
• Quantitative utilitarianism/ Ethical hedonism- As per this, an action is right, if it produces
the greatest good for the greatest number. An action is right if it produces pleasure and
wrong if it produces pain. Also, every individual will seek pleasure and avoid pain. As per
Bentham, since all pleasures are equal, so a suitable arrangement of these pleasures and
pains should be achieved where greatest good of greatest number can be achieved.

DELHI JAIPUR PUNE HYDERABAD AHMEDABAD LUCKNOW CHANDIGARH


13 www.visionias.in # 8468022022 ©Vision IAS
Google it:- https://upscpdf.com
https://t.me/UPSC_PDF Download From > https://upscpdf.com https://t.me/UPSC_PDF

• Qualitative utilitarianism- Mill distinguished between quantitative and qualitative Student Notes:
pleasures (known as Rule-Utilitarianism) to highlight that Intellectual pleasures are better
than sensuous pleasures that we ought to seek satisfaction of higher capacities. He declares
that more valuable moral pleasures are those which employ “higher faculties.
• Idea on liberty- It highlights that harm principle which propounds that the people should
be free to act however they wish unless their actions cause harm to somebody else.
Individuals are rational enough to make decisions about their well-being. Government
should interfere when it is for the protection of society.

3. Lessons from the Lives and Teachings of Great Leaders


3.1. Mahatma Gandhi
‘My Experiments with Truth’, the biography of Gandhi is well known, however from ethics
point of view his ideas are very crucial. His ethical values were guided by the leanings in ancient
text of Bhagvad Gita and for him, dharma is everything. As the Dharma in the Indian tradition,
commands morality in the sense of righteous conduct. Mahatma Gandhi has raised dharma to a
higher pedestal, signifying a quality through which we know “our duty in human life and our
relation with other selves”.
In his life, Gandhi set an example of self-sacrifice, demonstrated the highest standards of
personal integrity, honesty, pristine purity, transparent public and private life, fairness and
justice which are the basic ingredients of Gandhi’s ethics.
Key Gandhian lessons at individual level-
• Concept of Truth and God- According to his ideas, Truth is divine hence it should be an
ideal of human conduct. As per him, truth is a relative truthfulness in word and deed and
the absolute truth i.e. absolute reality is god and morality. Devotion to Truth is the sole
justification for our existence. All our activities should be directed towards finding the
Truth. In his words, “Truth should be the very breath of our life”.
• Relationship between Means and Ends- Gandhi highlighted indivisibility of means and ends
i.e. means (method/techniques) and ends (goals/outcome) both should mutually justify
each other, and no one should give preference over another. This moral sentiment is
reflected when he said, "be the change you wish to see in the world". No wonder, he called
off non-cooperation movement in 1922 when it was on peak, considering the means (use of
violence) is not matching the ends.
• Non-violence- According to Gandhi, Ahimsa is infinite love/ infinite capacity of suffering. He
used non-violence to realise truth. He was of the opinion that Non-violence is strongest
force and attribute of very strong character.
• Seven deadly sins- He explained the seven unethical practices in his idea of Seven Deadly
Sins i.e.
o Wealth without work
o Pleasure without conscience
o Science without humanity
o Knowledge without character
o Politics without principle
o Commerce without morality
o Worship without sacrifice.
• Importance of sanitation- Gandhi's views on cleanliness. Mahatma Gandhi said "Sanitation
is more important than independence". He made cleanliness and sanitation an integral part
of the Gandhian way of living. His dream was total sanitation for all. Cleanliness is most
important for physical well-being and a healthy environment. It has bearing on public and
personal hygiene. Mahatma Gandhi said, “I will not let anyone walk through my mind with
their dirty feet.” He dwelt on cleanliness and good habits and pointed out its close
relationship to good health.

DELHI JAIPUR PUNE HYDERABAD AHMEDABAD LUCKNOW CHANDIGARH


14 www.visionias.in # 8468022022 ©Vision IAS
Google it:- https://upscpdf.com
https://t.me/UPSC_PDF Download From > https://upscpdf.com https://t.me/UPSC_PDF

Student Notes:

Key Gandhian lessons at community level-


• Sarvodaya- It is based on concept of unity of existence. It teaches universal love as the only
love of life. Sarvodaya means universal welfare of all living being. Gandhi advocates that
Sarvodaya is the true panacea for all types of social or political problems experienced by
Indian society.
• Concept of Trusteeship- According to the theory of trusteeship all material property of the
society should be seen as a common wealth belonging to the society. The property owners
are trustees of this wealth therefore was not required to take more than what was needed
for a moderately comfortable life. The other members of society who were associated with
the property were jointly responsible with the owner for its management and were to
provide welfare schemes for all for the ethical wellbeing of society.
• Idea of Sarva-dharma-sambhav- To prevent conflicts caused by religious bigotry, Gandhi
suggested "Sarva Dharma Sambhav". According to it all religions are true and man cannot
live without religion so he recommends attitude of respect and tolerance towards all
religions.

DELHI JAIPUR PUNE HYDERABAD AHMEDABAD LUCKNOW CHANDIGARH


15 www.visionias.in # 8468022022 ©Vision IAS
Google it:- https://upscpdf.com
https://t.me/UPSC_PDF Download From > https://upscpdf.com https://t.me/UPSC_PDF

• Satyagraha- It is characterized by adherence to truth, non-violence, and self-suffering, by Student Notes:


operating within a conflict situation, aims at a fundamental social and political change.
• Concept of Gram Swaraj- The fundamental concept of Gram swaraj or village self-rule is
that every village should be its own republic, independent of its neighbours for its own vital
wants and yet interdependent for many others in which dependence is necessary. Each
village should be basically self-reliant, making provision for all necessities of life - food,
clothing, clean water, sanitation, housing, education and so on, including government and
self-defence, and all socially useful amenities required by a community.
• Women empowerment- Gandhi ji was of the opinion that women were capable of
infinite strength, which they only needed to realize and channel. Women had a key role
to play in the family, in Gandhi’s opinion. The family was the crucible of society
where future citizens, leaders and lawgivers were nurtured. Gandhi strongly favored
the emancipation of women and opposed ills such as Purdah, child marriage, sati
among others.
• Untouchability- According to Gandhi, practice of untouchability is moral crime. All human
beings are equal and harijans to have right for social life. He was of the opinion to bring
basic change in caste structure by uplifting untouchability and not by abolishing caste.

3.2. Amartya Sen


Amartya Sen is an Indian Economics Nobel Laureate. He has contributed to the cause of human
development and poverty alleviation. Sen through his seminal work, Poverty and famine
argued that deprivation largely occurs due to the absence of entitlement to food, rather than
the absence of food itself. He recognises that the market can provide entitlement provided all
people can get work and a reasonable wage.
Key lessons from the life and works of Sen-
• Capability development approach- He has broadened the sphere of human development
by bringing people at the centre of development. He argues that the purpose of
development is to enlarge all human choices and not income. Thus, state should invest in
people and enable them to use those capacities appropriately.
• Idea of Justice- Sen argues that objective of justice should be more concerned with the
elimination of removable injustices rather than defining a perfectly just society. He links this
with capability approach and states that a state is more just, if its people enjoy more
freedoms to live a life they have a reason to value.
• Tolerance- He also tried to establish that societies where a healthy culture of debate and
dissent is cultivated and people exercise tolerance to each other’s point of view, they
flourish economically as well.

4. Lessons from the Lives and Teachings of Great


Reformers
4.1. Raja Ram Mohan Roy
He was famously termed as Father of Indian Renaissance. He had a broad social vision and a
striking modernity of his thought. He was a tireless social reformer and fought for various social
evils in the eighteenth century, the most prominent one being the practice of sati. He also
revived interest in ethical principles of Vedanta school as a counterpoise to increasing Western
influence on Indian culture.

DELHI JAIPUR PUNE HYDERABAD AHMEDABAD LUCKNOW CHANDIGARH


16 www.visionias.in # 8468022022 ©Vision IAS
Google it:- https://upscpdf.com
https://t.me/UPSC_PDF Download From > https://upscpdf.com https://t.me/UPSC_PDF

Student Notes:

Key lessons from his life and teachings-


• Humanism- He preached love for human beings, stop animal sacrifice and offerings. It came
forward for the promotion of charity, morality, piety, benevolence, virtue and the
strengthening of the bonds of union between men of all religions and creeds.
• Cosmopolitanism- He was a truly cosmopolitan figure, a global citizen, who argued for
political liberty, civic freedom and social justice for all humanity using the Enlightenment
tool of universal reason.
• Scientific temper- He always raised a view that every social practice must fulfil the
principles of science and rationality.
• Universal spirit of religion- He attacked the various ceremonies, practices and ills which
crept in Hinduism and alleged that they alienated Hinduism from the poor masses. He
reaffirmed that every religion’s spirit should be towards service all its believers, rather
than a few dominant ones.

4.2. Baba Amte


A noted social worker, activist and Gandhian, he was known particularly for his work for the
rehabilitation and empowerment of the marginalized and the ostracized. He devoted his life to
many other social causes, most notably the Quit India movement, raising public awareness on
fighting corruption and the importance of ecological balance, wildlife preservation. He also
used Gandhi's principles to fight against corruption, mismanagement, and poor, shortsighted
planning in the government.

DELHI JAIPUR PUNE HYDERABAD AHMEDABAD LUCKNOW CHANDIGARH


17 www.visionias.in # 8468022022 ©Vision IAS
Google it:- https://upscpdf.com
https://t.me/UPSC_PDF Download From > https://upscpdf.com https://t.me/UPSC_PDF

Key lesson from his life and teachings Student Notes:


• Compassion- He founded the Anandwan Ashram, which was a rehabilitation centre for
leprosy patients where they were taught to become self-sufficient through hard work.
• National Integration- He organized Bharat Jodo Abhiyan to re-infuse the spirit of national
integration, when there was cynicism due to communal strife in the country.
• Environmentalism- He worked in Narmada Bachao Andolan, which fought against unjust
displacement of local inhabitants as well as damage to the environment due to construction
of Sardar Sarovar Dam.

5. Lessons from the Lives and Teachings of Great


Reformers
5.1. E. Shreedharan
Commonly known as Metro man, who by the virtue of his hardwork and dedication brought
comfort to the lives of masses. His work and vision have redefined the public transport be it
Delhi Metro or Konkan Railways. He was known for his clear thinking and adherence to
deadlines.

Key lesson from his life and work-


• Diligence and Perseverance- Despite working with government organization, Shreedharan
had to face many challenges in the form of political interference and resource paucity in the
process of completion of project, he was about to quit the project due to disagreement
with Railway ministry over choice of guage. However, he persevered and deliver the project
not only on scheduled time but completed it with transparent integrity.

DELHI JAIPUR PUNE HYDERABAD AHMEDABAD LUCKNOW CHANDIGARH


18 www.visionias.in # 8468022022 ©Vision IAS
Google it:- https://upscpdf.com
https://t.me/UPSC_PDF Download From > https://upscpdf.com https://t.me/UPSC_PDF

• Equal access to public utility- As a firm believer in the democratic ethos of public utility, he Student Notes:
believed that every citizen should have equal access to it. These values is clearly seen in
operation of various project he had undertaken like Delhi Metro.
• Ethical work culture- He described punctuality, integrity, professional competence and
social accountability as the four cardinal pillars of work culture and attributed his success in
life to scrupulous adherence to these values. He believed that ethics must be a compulsion,
not an option.

5.2. TN Sheshan
An IAS officer of 1955 batch from Tamil Nadu cadre, managed to stamp his authority on the
country's electoral system during his term as the country's 10th Chief Election Commissioner
from 1990 to 1996. He was awarded the Ramon Magsaysay award in 1996 for "his resolute
actions to bring order, fairness, and integrity to elections in India, the world's largest
democracy.
Key lesson from his life and work
• Leadership- He changed the face of elections in the country. He identified over 100
common electoral malpractices, including the preparation of inaccurate election rolls,
mistakes in setting up polling stations, coercive electioneering, spending more than the
legal limit, using goons to snatch polling booths and general abuse of authority.
o He enforced the Model Code of Conduct, strictly monitored limits on poll expenses, and
cracked down on several malpractices like wall graffiti. The issuance of Voter IDs for all
eligible voters came into being under his strict watch.
• Disciplinarian- He was ruthless and unforgiving when it came to electoral rules and
practices. He enforced strictness on his electoral staff as well as on politicians for any kind
of electoral malpractice or indiscipline. In 1994, he advised then Prime Minister to remove
two sitting cabinet ministers from the cabinet for reportedly influencing voters.
• Resoluteness- He believed in two principles i.e. zero delay and zero deficiency. He faced
various challenges, but he responded to them in the following way- “I am like a ball. The
more you kick me, the more I will bounce back.”

6. Way Forward
The actual transformation of ethical values from an option to compulsion takes place when
these values are internalized and practiced with integrity. It is crucial that apart from
remembering ethics, one should ensure that ethics should become his/her way of life. Also,
there should not be any compromise in the domain of ethics.

7. Previous Years UPSC GS Mains Questions


1. Discuss Mahatma Gandhi’s concept of seven sins.
2. Analyse John Rawls’s concept of social justice in Indian Context.
3. “Great ambition is the passion of a great character. Those endowed with it may perform
very good or very bad acts. All depends on the principles which direct them.” – Napoleon
Bonaparte. Stating examples mention the rulers
a. who have harmed society and country,
b. who worked for the development of society and country.
4. “If a country is to be corruption free and become a nation of beautiful minds, I strongly feel
there are three key societal members who can make a difference. they are father, the
mother and the teacher.” – A. P. J. Abdul Kalam. Analyse.
5. What do each of the following quotations means to you in the present context?
(a) “The true rule, in determining to embrace, or reject any thing, is not whether it has any
evil in it; but whether it has more evil than good. There are few things wholly evil or
wholly good. Almost every thing, especially of government policy, is an inseparable

DELHI JAIPUR PUNE HYDERABAD AHMEDABAD LUCKNOW CHANDIGARH


19 www.visionias.in # 8468022022 ©Vision IAS
Google it:- https://upscpdf.com
https://t.me/UPSC_PDF Download From > https://upscpdf.com https://t.me/UPSC_PDF

compound of the two; so that our best judgment of the preponderance between them Student Notes:
is continually demanded.” – Abraham Lincoln.
(b) “Falsehood takes the place of truth when it results in unblemished common good.” –
Tirukkural
6. “An unexamined life is not worth living.” – Socrates
7. “A man is but the product of his thoughts. What he thinks, he becomes.” – M.K.Gandhi
8. “Where there is righteousness in the heart, there is beauty in the character. When there is
beauty in the character, there is harmony in the home. When there is harmony in the
home, there is order in the nation. When there is order in the nation, there is peace in the
world.” – A.P.J. Abdul Kalam
9. Given are two quotations of moral thinkers/philosophers. For each of these bring out what
it means to you in the present context.
(a) “The weak can never forgive; forgiveness is the attribute of strong.
(b) “We can easily forgive a child who is afraid of the dark; the real tragedy of life is when
men are afraid of the light”
10. Given below are three quotations of great moral thinkers/philosophers. For each of these
quotations, bring out what it means to you in the present context:
(a) “There is enough on this earth for every one’s need but for no one’s greed.” -Mahatma
Gandhi
(b) “Nearly all men can withstand adversity, but if you want to test a man’s character, give
him power.” - Abraham Lincoln
(c) “I count him braver who overcomes his desires than him who overcomes his enemies.”
– Aristotle

8. Previous Years Vision IAS GS Mains Questions


1. Bring out Gandhiji's philosophy of Means and Ends.
Approach:
• Briefly define Means and Ends.
• Explain Gandhian perspective on the relative importance of Means and Ends.
• Conclude with the relevance of his philosophy in the modern world.
Answer:
In simple terms, ends are the goals or results. Means are the methods used to achieve
goals. There are differing schools of thought with some justifying any means for
achieving the goals while others uphold the need for right means to achieve the
objectives in true sense.
For Gandhiji, there is some inviolable connection between means and ends similar to
the one between a seed and a tree. Gandhiji, stated that it is means, rather than ends,
that provide the standard of morality. As per him, the only thing that is completely
within control is the means to achieve the goal. For Gandhi ji, violence and non-
violence cannot be different means to serve the same end, since they are morally
different in quality and essence, they must necessarily achieve different results.
Gandhiji, in his moral and political thought, gave truth and non-violence the highest
importance and said that ‘ahimsa’ is the means to reach ‘satya’, which is the end. His
view was that impure means can never lead to a pure end. Gandhiji also said that
choosing the rightful means to achieve a goal may appear to be too long, but he was
convinced that it is the shortest route. According to Gandhiji, means are after all
everything. He said, ‘as the means, so the ends.’
Gandhiji’s views on ends and means hold ground in view of the fact that India, after
gaining independence through his methods of satyagraha, adopted his principle of

DELHI JAIPUR PUNE HYDERABAD AHMEDABAD LUCKNOW CHANDIGARH


20 www.visionias.in # 8468022022 ©Vision IAS
Google it:- https://upscpdf.com
https://t.me/UPSC_PDF Download From > https://upscpdf.com https://t.me/UPSC_PDF

Sarvodaya (progress of all) and became the largest democracy in the world. It is Student Notes:
constantly growing towards becoming a better nation in terms of economy as well
human values. Adoption of his view on ends and means will help morality and ethics
prevail in the society, which is largely seen missing in personal as well as public life.

2. Highlight the important teachings of Swami Vivekananda, relevant to the youth of


today.
Approach:
• Introduce the answer by highlighting the need to remember teachings of Swami
Vivekananda.
• Discuss the teachings of Swami Vivekananda relevant to the youth in the current
context.
• Conclude the answer on the basis of the above points.
Answer:
The philosophy of Swami Vivekananda and the ideals for which he lived and worked are
a great source of inspiration for the youth today. He wanted the countrymen including
the youth to have ‘muscles of iron’, ‘nerves of steel’ and ‘minds like thunderbolt’.
Owing to this, his birth anniversary i.e. January 12th is commemorated and celebrated
as National Youth Day.
Certain teachings relevant to the current youth are as under:
• Character building: He believed that the nation’s youth held the key for solving
various problems facing India. According to him, each human being with his/her
own potential could guide himself/herself (Atma-Nirmana) and at the same time
play an active role in the society. This emphasis on individual potential is relevant to
the youngsters to develop their skills and carve an identity for themselves.
• Education and society: Vivekananda had attached great significance to the role of
education in society building. He believed that the Indian society had its own
fundamental characteristics like guru-shishya parampara etc. since the beginning,
which should be maintained at any cost.
• Philanthropy & service to mankind: Vivekananda once said, “so as long as the
millions live in hunger and ignorance, I hold each man a traitor.” Such a call
inspired the youngsters to serve the suffering humanity. This is relevant to the
youth, as it will encourage the spirit of social service, philanthropy, and
humanitarianism.
• Spirit of national awakening: He wanted youth who would eventually be leaders,
contribute to national awakening, preach and teach the eternal spiritual truths of
our forefathers. With his messages of “Arise, Awake and Stop not till the goal is
achieved”, he inspired youngsters to invoke the spirit of nationalism.
• Universal tolerance: His famous speech at the World Parliament of Religions,
emphasized universal tolerance. This teaching is significant for the youth in a highly
polarised global world order.
• Meaningful life: He wanted the youth to live a meaningful life through different
quests like physical quest (for ensuring physical fitness), social quest (for
undertaking social activities aimed not only for the betterment of society but also
the individuals), intellectual quest (for building intellectual strength) and spiritual
quest (for achieving spiritual and psychological well-being).
Cultural capital and Vedanta: His interpretation of the Vedanta, not only brought him
closer to the masses but also established several milestones in the cultural

DELHI JAIPUR PUNE HYDERABAD AHMEDABAD LUCKNOW CHANDIGARH


21 www.visionias.in # 8468022022 ©Vision IAS
Google it:- https://upscpdf.com
https://t.me/UPSC_PDF Download From > https://upscpdf.com https://t.me/UPSC_PDF

development of India. His teachings centre around the themes of Vedas and Student Notes:
Upanishads, which are relevant for the young population. Swami Vivekananda believed
that the young generation, which he called as the modern generation, is very powerful.
It can do anything and everything and has the eternal power to even revive the whole
country and realize his dream of making Bharat a Vishwa Guru once again.

3. Courage is the most important of all the virtues because without courage, you can't
practice any other virtue consistently
Approach:
• Explain what you understand by courage.
• State its importance as a virtue that is important to practice other virtues.
• Conclude on the basis of above points.
Answer:
Courage means strength, whether moral or physical, to confront adversities such as
danger, dilemma or opposition. It also means conviction to do what one believes in and
take decisions based on those beliefs. Aristotle recognized that courage is the first of
human virtues because it is the one that guarantees other virtues. Without courage,
one cannot practice any other virtue consistently, such as:
• Assertiveness: It refers to achieving goals by setting appropriate boundaries.
Prime Minister Nehru was courageous when he decided that India would remain
non-aligned and maintain equi-distance from the US and the Soviet Union during
the Cold War era despite enormous pressure.
• Integrity: Integrity is the practice of being consistent, honest and moral. Sanjeev
Chaturvedi displayed integrity while working as Chief Vigilance Officer of All India
Institute of Medical Sciences (AIIMS), even though he had to wage a legal battle to
face the consequences.
• Perseverance: It means continued effort and determination in order to achieve
something despite difficulties or delays. Courage was displayed by citizens who
persistently mobilized for the rights of the community as a whole in the face of
odds. For e.g. the Chipko movement, Appiko movement, Narmada Bachao
Andolan, India Against Corruption movement etc.
• Fortitude: It means display of strength during times of adversity. There are various
examples in the form of Holocaust survivors who fought against the oppression of
Nazis.
• Selflessness: It refers to the quality of caring more about what other people need
and want than about one’s own needs and wants. Dr. Arunoday Mondal, who set
up Sujan Sundarban, travels to remote Sundarban villages and has treated
numerous poor people.
• Truthfulness: Courage of conviction and truthfulness go hand-in-hand as a person
should be courageous to tell the truth even in times of distress. This what drives
the admirable efforts of RTI and Human Rights activists.
Thus, courage as a virtue enables other virtues and empowers a person to be honest
and unbiased. Further, it gives a person the strength to rise above considerations of
caste, community and religious identities.

DELHI JAIPUR PUNE HYDERABAD AHMEDABAD LUCKNOW CHANDIGARH


22 www.visionias.in # 8468022022 ©Vision IAS
Google it:- https://upscpdf.com
https://t.me/UPSC_PDF Download From > https://upscpdf.com https://t.me/UPSC_PDF

4. There is a higher court than courts of justice and that is the court of conscience. It Student Notes:
supersedes all other courts.
Approach:
• Briefly explain conscience and how it works
• Discuss the constraints of court of justice and how conscience can fill in those gaps.
• Discuss conscience as a positive tool to make moral decisions.
• Conclude with some ways to strengthen the voice of conscience.
Answer:
Conscience is an inner sense of right and wrong based on an individuals’ moral values.
In case of any wrongdoing, conscience creates internal dissonance leading to a feeling
of guilt, loss of self-respect and peace of mind. Court of justice, on the other hand is
external, it is a post-mortem exercise. It is constrained in its functioning in several ways-
Immanuel Kant aptly observed that, “In law a man is guilty when he violates the rights
of others. In ethics he is guilty if he only thinks of doing so.” The courts of law have
limited scope of activities. Conscience, on the other hand, is one’s constant companion.
The meaning of the given quote can be further illustrated by the following:
• Courts of law are based on procedures - they require establishment of guilt beyond
doubt for a person to be punished. A powerful person can evade court of justice by
exploiting loopholes or for want of evidence. For example, perpetrators of
communal riots are seldom convicted in court of justice but before the court of
conscience, they are guilty of murdering people.
• Courts of law alone might not bring a transformation of criminal’s mind by imposing
punishment. However, conscience directs a person towards reformation by seeking
to change the behaviour at deeper levels i.e. emotional and psychological.
• Further, conscience acts as a positive tool in many cases. It acts as the source for a
sense of duty in a person to act morally. For e.g. health workers despite being at
risk of covid-19 disease themselves, are conscientiously doing their duty.
• Conscience also helps in solving issues like conflict of interest, corruption,
nepotism, crony-capitalism. For e.g. conscience may nudge a judge to recuse
himself from cases in which they have an interest.
In this context, conscience assumes importance as one can evade the courts of law but
the inner voice will hold the person to account. Thus though the court of conscience
has wider scope than the court of justice, still conscience needs to be educated to make
it work as an effective ethical decision making tool.
As, conscience may mean different things to different people, thus, it often fails in
preventing wrongdoing such as honour killing, dowry, cruelty towards animals,
extravagance, etc. Therefore, the court of conscience needs to be strengthened through
building one’s ethical competence. And court of law must ensure justice to maintain
public order.

5. Discuss Vivekananda’s views on nationalism. Do you think nationalism has become


more a divisive force in the world today than one that integrates?
Approach:
• Briefly, define Nationalism.
• Highlight Vivekananda’s views on nationalism.
• Provide arguments and examples to support the statement.
• Give an appropriate conclusion.

DELHI JAIPUR PUNE HYDERABAD AHMEDABAD LUCKNOW CHANDIGARH


23 www.visionias.in # 8468022022 ©Vision IAS
Google it:- https://upscpdf.com
https://t.me/UPSC_PDF Download From > https://upscpdf.com https://t.me/UPSC_PDF

Answer: Student Notes:


Nationalism is defined as loyalty to a nation or sense of national consciousness,
elevating one nation above all others. It primarily emphasizes on promotion of one’s
own national culture and interests as opposed to those of other nations or
supranational groups.
Vivekananda’s views on nationalism
Vivekanand’s views on nationalism were not based on geographical or political or
emotional unity or a feeling that ‘we are Indians’. His views on nationalism were deeply
spiritual. It was spiritual integration of the people, a spiritual awakening of the soul. He
recognised the prevailing diversity on so many basis and suggested that nationalism in
India cannot be sectarian like the west.
For him, Indian people were deeply religious and from it can be derived the unifying
strength. A unity in purpose and action can be achieved through a cultivation of
national ideals. He recognized compassion, service and renunciation as the national
ideals. Hence, nationalism for Vivekananda was based on universalism and humanism.
• He believed that there is one all dominating principle manifesting itself in the life of
each nation and India’s theme was religion. Swami Vivekananda’s nationalism is
deeply rooted in religion, Indian spirituality and morality, contrary to western
nationalism based on secularism.. Here, the spirituality is viewed as point of
convergence for all religious forces of India capable of unifying into a national
current.
• He also considered ideals of Humanism and Universalism as a basis of nationalism.
These ideals guided people to first get rid of self-inflicted bondages and resultant
miseries.
During the last two centuries, nationalism has passed through many phases and
emerged as one of the most compelling forces, which both united people and divided
them. In the nineteenth century, it led to the Europe’s unification and the breakup of
colonies in Asia and Africa.
However, contemporary world is witnessing significant rise of hardline nationalism. The
swaying away of USA from established conventions of international politics, BREXIT,
demand for second referendum for Scottish independence are some examples of the
same. As such, a parochial view of nationalism has taken stronghold in many groups,
who want to ascertain their rights and privileges on others. Nationalism in this form
divides nations, isolates them and leads to economies which heighten inequality, drive
away people who can contribute to the nation.
In contrast to the divisive forces of modern nationalism, Swami Vivekananda articulates
a vision that is universal in its reach and the focuses on oneness of spiritual identity. It is
time to imbibe his understanding of “enlightened nationalism” which stresses that
there could be no spiritual or ethical justification for one nation occupying another.

6. The golden rule of conduct is mutual toleration, seeing that we will never all think
alike and we shall always see Truth in fragment and from different points of vision.
Mahatma Gandhi.
Approach:
• Briefly discuss the meaning of the quotation.
• Bring out the relevance of given statement in the present time.
• Conclude the answer.

DELHI JAIPUR PUNE HYDERABAD AHMEDABAD LUCKNOW CHANDIGARH


24 www.visionias.in # 8468022022 ©Vision IAS
Google it:- https://upscpdf.com
https://t.me/UPSC_PDF Download From > https://upscpdf.com https://t.me/UPSC_PDF

Answer: Student Notes:


Our behavior is based on our own reasoning. Our reasoning is based on our values,
beliefs, experiences, etc. Every individual is subjected to a variety of values, beliefs and
experiences during his/her lifetime. As such, in their own way, people act correctly, but
differently. It is because people think that they act correctly, there is always a scope of
difference of opinion and action with others, who in their own stead also act correctly.
It is in this recognition of basic human behavior and the limitation of human mind to
know everything that Gandhiji made this statement- because we cannot always think
alike, the best way to conduct is by tolerating the divergent or different opinion.
In a society with diverse values, culture and social norms, it is natural to have differing
viewpoints on the same issue. For such a diverse society to be in peace, tolerance is
essential. An individual’s conduct should be guided by tolerance and empathy for
diverse opinions, ideologies and cultures while valuing the basic universal values of
love, non-violence and basic human rights. Rigid and negative attitude towards diversity
ignores the fact that truth is fragmented, and misses the larger vision.
Although conscience is a good guide to moral conduct, it is not the same for all, and
therefore, imposing one’s own conduct on others amounts to insufferable interference
with their freedom of conscience. This vision is also emphasized in Jain concept of
‘Syadavad’- relativity of Truth. It implies that there can be many truths rather than
adopting one rigid approach.
In contemporary times, the statement serves as a guiding light for the world witnessing
divisions across multiple lines- region, religion, wealth, color, and many more. An
attitude of intolerance leads to restriction of space for expression of ideas. Divergent
views enrich the discussion and are the source of innovation, progress and inclusive
growth.
The intellectual basis for tolerance comes from respect, curiosity and skepticism and
rationality. Also, it is necessary to differentiate between tolerance and acceptance.
Divergent views must be encouraged, but their acceptance should not depend merely
because they are divergent. Acceptance should be based on debates and discussions in
which everyone gets to know the ‘truth’ of the other and hence, become more aware
and rational. But for debate to take place, tolerance again becomes the necessary
aspect of conduct, and hence it is the golden rule.

7. The mind of the superior man is conversant with righteousness; the mind of the mean
man is conversant with gain. Confucius.
Approach:
• Briefly, elaborate the quote.
• Provide arguments /examples to highlight the present-day relevance of the quote.
• Conclude the answer.
Answer:
‘What is the right thing to do?’ may be different for different people. In the given
quote, Confucius is trying to argue that superior amongst people are those who are
concerned with what is right, and not merely what is right for them alone. As an
example, a person who takes a long term view or whose thoughts, speech and action
are aligned with the collective good of society, would be a superior man as compared to
a person who remains selfish, greedy and follows self-centred tendencies and unethical
means, who would be a mean man.

DELHI JAIPUR PUNE HYDERABAD AHMEDABAD LUCKNOW CHANDIGARH


25 www.visionias.in # 8468022022 ©Vision IAS
Google it:- https://upscpdf.com
https://t.me/UPSC_PDF Download From > https://upscpdf.com https://t.me/UPSC_PDF

Generally, it is easier and more profitable to pursue greedy self-interest when a person Student Notes:
has to choose between it and the collective societal good. Preferring immediate
gratification and short-term gains over long-term collective gains to the society is often
clearly visible, and hence becomes the chosen course. Such “mean man” tendencies
are often seen in daily acts like tax evasion, littering public places, paying no regard to
traffic rules, un-mindful use of water/electricity, displaying no compassion towards
animals or making electoral choices purely out of religion/caste considerations etc.
Worst form of “mean tendency” is seen when people indulge in abhorrent acts like
human trafficking, drug abuse, terrorism and adulteration which have long term
negative consequences for the society. On the other hand, a superior man, with simple
acts such as saving and segregating waste to be disposed-off properly undertakes more
effort with lesser immediate benefits to himself.
It also raises the important issue of the limited understanding of what is the right thing
to do. Broadly, the scope of righteousness constitutes: (i) Correct personal conduct, (ii)
Not indulging in personal aggrandizement, (iii) Doesn’t cause harm or violate other
citizens rights or cause social trouble. The goals that we desire and the principles that
we use in their pursuit are the primary parameters to judge what is righteous or not.
People like Swami Vivekananda, Mahatma Gandhi and Dr Abdul Kalam rose above self-
interest and devoted one’s life and energy towards humanity are role models for
modern society. However, it is imperative that everyone cannot be a mass leader and
sacrifice most self-interest for others. Still, everyone can be a leader in one’s own right
and have a longer vision for oneself and the society. This way, everyone can aspire to
not be a mean man and progress towards being a superior man. Even by doing simple
tasks such as following the taxation laws in letter and spirit makes one a superior man.
In this case, the task of deciding what is the righteous thing has been left to the State-
which decides the laws; the task of man is not to be the mean man by selfishly seeking
short-term gains.

8. "I learned that courage was not the absence of fear, but the triumph over it. The
brave man is not he who does not feel afraid, but he who conquers that fear"- Nelson
Mandela
Approach:
• Elaborate and explain the given statement.
• Mention the significance of courage in personal and public life.
• Give a suitable conclusion.
Answer:
Courage is not simply absence of fear. It is the virtue of taking righteous decision or
action in the face of fear. It is rightly said that courage is ‘being terrified but going ahead
and doing what must be done’.
Fear is something that we all have struggled with at some point in our lives - fear of
expressing your honest opinion on something, fear of pursuing your dreams etc. The
one who feels no fear is a fool and the one who lets fear rule him is a coward.
In the recent Me Too movement, ordinary citizens (victims) raised their voice against
high and mighty individuals. It takes immense courage to challenge persons occupying
powerful positions and seek justice.
The life of Nelson Mandela is testimony to such courageous acts. He stood for a
justified cause of ending apartheid. He remained undeterred even in the face of
discrimination and threat to his own life.

DELHI JAIPUR PUNE HYDERABAD AHMEDABAD LUCKNOW CHANDIGARH


26 www.visionias.in # 8468022022 ©Vision IAS
Google it:- https://upscpdf.com
Relevance in the present context, particularly in public service and public life Student Notes:
• Thriving in adversity: Courage is to stand for a justified cause, even in the face of
adversity. It is sine qua non for good governance and efficient public administration.
A civil servant should not yield to unreasonable demands by public officials, elected
members of legislatures, other representatives of people etc. They also need to
exercise courage while providing frank, independent and unbiased advice to the
executive.
• Self-belief and commitment to social cause or cause that the state stands for:
Only courageous mind can truly stay committed to the cause of oppressed and
deprived. Such state of mind can be attained when a person has eternal belief in his
cause and purpose for life. This instills sentiments of compassion and empathy for a
bureaucrat. But one of the most important elements of courage is to continue
doing what one believes in. To persevere for justified cause is important for
individual self, society and nation at large.
Mandela’s 27-year long struggle in prison and his tenure as the first-ever black
President symbolize the importance of self-belief and courage of conviction. This is a
lesson civil servants of today can take while working for vulnerable and oppressed
sections of the society.

9. "Not everybody can be famous, but everybody can be great because greatness is
determined by service"- Martin Luther King.
Approach:
• Briefly discuss the significance of service in this context.
• Mention your own understanding of the quote.
• Highlight the relevance of quote in the present context.
• Give a suitable conclusion.
Answer:
Service in this context means ‘Service to others or the mankind’. Serving other makes a
person look beyond self. Service to other leads to selflessness. It means doing
something for someone else without expecting any reward or gain. Making a difference
in their lives.
However, it is possible only when a person’s heart and soul is full of grace and love.
Otherwise, it will be merely acting for the purpose of gain. The life of Martin Luther
King was dedicated for the well-being and upliftment of others. He was dedicated to
serving others and end racial inequality in the USA.
However not everyone involved in the service of others are recognized by others. They
remain unsung hero. It is the spirit to serve humanity and alleviate the sufferings of
others without any expectation from society makes them great.
We have multiple examples of individuals who have tirelessly toiled for years without
yearning for acknowledgement and recognition.
• Mr. Karimul Haq, popularly known as ‘Bike-Ambulance Dada’, turned his two-
wheeler into an “ambulance” to hospitalize patients in remote areas of West
Bengal. He has saved more 3500 lives of road accident victims and emergency
patients.
• Saalumarada Thimmakka, a 106 years old environmentalist, has planted and
nurtured more than 8000 plants in last 80 years.

DELHI JAIPUR PUNE HYDERABAD AHMEDABAD LUCKNOW CHANDIGARH


27 www.visionias.in # 8468022022 ©Vision IAS
Recently both of them were awarded Padma Shri. They acted without desire for Student Notes:
recognition or publicity. They had an inherent urge to remove sufferings from others life
which encouraged them to work relentlessly. On the other hand, there are instances of
certain activists and NGOs, which became famous for reasons other than service to
others.
Keeping this in mind, a civil servant should always yearn to serve others in best of their
capabilities, without longing for fame. This also assumes importance given that
anonymity is a foundational value for civil servants. In case they are tempted to gain
individual publicity, they should remind themselves of Mahatma Gandhi who rightly
quoted that “the best way to find yourself is to lose yourself in the service of others.

10. A nation should not be judged by how it treats its highest citizens, but its lowest ones.
Nelson Mandela
Approach:
• Briefly explain what you understand by the given quote.
• State the issues faced by the disadvantaged groups in India that need to be
addressed.
• Conclude suitably.
Answer:
The quote captures the essence of inclusivity in a society. A nation may grow
economically but it truly progresses when the fruits of growth are not merely
appropriated by those who contribute the most to it, but equitably by those who need
them the most. As a corollary, it also means that rule of law is supreme and that it
cannot be different for the ‘highest’ and the ‘lowest’ citizens - it should be equal for all
citizens. This is essentially what the framers of the Constitution also envisioned for an
independent India i.e. a just and equitable society.
In the present context, it applies even more as increasing inequality has created large
gaps in the opportunities - those at the bottom are finding it more difficult to reach the
social, economical and political level that they aspire for. In order to achieve the above,
India needs to reduce the gap between the haves and the have-nots across these
domains. The concerns of the deprived and disadvantaged members of the society
need to be addressed. These include:
• Pulling out approximately 73 million people out of poverty and providing a basic
standard of living for all its citizens.
• Ensuring that no one is subjected to social and occupational discrimination based
on ascriptive factors like caste.
• Taking into consideration the relationship of people (such as farmers, tribals) with
the land for their rehabilitation in case of acquisition of land.
• The literacy rates of marginalized sections like SCs, STs, minorities remain below
the national average. The proportion of children suffering from under-nutrition and
malnutrition is high, access to health, education and jobs is poor amongst such
groups.
• Ensuring that rule of law is there not merely on paper but in practice as well. The
over-representation of marginalised communities in prisons reflects that those
with means are able to navigate their way out of the clutches of law more easily as
compared to others.
• There should be increased opportunities for employment for women and
conducive work environment in order to reduce gender gap in employment.
Further, they need adequate representation in politics.

DELHI JAIPUR PUNE HYDERABAD AHMEDABAD LUCKNOW CHANDIGARH


28 www.visionias.in # 8468022022 ©Vision IAS
These issues reveal entrenched discrimination in society. The measure of India’s Student Notes:
progress will not be judged merely by the number of Indian billionaires, strides it
makes in the field of space science, lavish urban communities it creates, its roads &
highways etc. But it will be measured by how much of that development percolates to
the bottom most sections of the society. The paradoxical state of India where affluence
co-exists with abject poverty and privilege coexists with lack of rights needs to be
resolved. The focus of the policy-makers should be to resolve them through
development schemes, affirmative action, policies based on inclusive growth, etc. After
all, equality is a pre-condition for meeting the challenge of reducing poverty,
promoting sustainable development and building good governance.

11. The highest education is that which does not merely give us information but makes
our life in harmony with all existence. Rabindranath Tagore
Approach:
• Start with the significance of education.
• Explain the quote and bring out its various dimensions.
• Discuss the present day relevance.
• Conclude suitably.
Answer:
Education is the basic foundation of human character. It gives information about the
world around us and helps build opinions and viewpoints. However, this should not be
the ultimate purpose of education. Tagore suggested that through information people
may become powerful, but they attain fullness of being through sympathy. The purpose
of true education should be to assist humans in gaining that sense of completeness.
Education should create healthy and wholesome personalities, which are capable of
rising above differences and have a broader outlook towards life and diversity. It
should inculcate the spirit of empathy, service and self-sacrifice, which can help in
improving the conditions of marginalized people. Further, it should also generate a
sense of the importance of things around us, which are important for human life.
Therefore, promoting effective and sustainable use of resources for the good of
mankind. Ultimately, the need is to create unity between people without imposing
uniformity that may lead to forcing the individual to blindly imitate the dominant
patterns in society. Therefore, it creates a peaceful and harmonious society.
Challenges such as climate change, refugee crisis etc. require collective action. It
requires understanding of the issue from different perspectives. Then only one would
be able to see the complete picture and act accordingly.
In this scenario, value-based education enables citizens to appreciate the differences
and live peacefully that would lead to the evolution of society rather than stagnation.
Knowledge about the significance of the things surrounding human life would lead to
sustainable and judicious use of resources and promote practices such as resource
conservation, adoption and mitigation. As such the curriculum should revolve
organically around nature. The Indian society that is divided along numerous fault lines
and faces a resource crunch stands to gain the most from adopting such changes to its
education system.
Creation of such an education system requires an active contribution from all the
stakeholders of society. It should focus on producing ethically grounded, global citizens
rather than profit-makers. Therefore, need is to transform the education system in ways
that bring a happy synthesis between the individual and society and help to realize the
essential unity of the individual with the rest of humanity.

DELHI JAIPUR PUNE HYDERABAD AHMEDABAD LUCKNOW CHANDIGARH


29 www.visionias.in # 8468022022 ©Vision IAS
12. Explain the main elements of integral humanism as propounded by Deen Dayal Student Notes:
Upadhyay and highlight its contemporary relevance.
Approach:
• Introduce the concept of integral humanism.
• Highlight its main elements.
• Discuss its contemporary relevance.
Answer:
Integral humanism was advocated by Deendayal Upadhyaya in 1965. Tracing its origins
to the non-dualistic philosophy of Advaita Vedanta, integral humanism propagated the
oneness of various souls, be it of human, animal or plant origin. Rejecting the intrinsic
diversity based on race, colour, caste or religion, it identified all human beings as part of
this one organic whole, sharing a common consciousness of national thought.
Main elements of doctrine of integral humanism
• Man at the centre of development: The philosophy considered that it was of
utmost important for India to develop an indigenous economic model with the
human being at center-stage. It did not reject western philosophies completely
rather it evaluates both capitalism and socialism on their respective merits, while
being critical of their excesses and alienness.
• Refutes individualism: It stresses on the need of an organic relation between an
individual and society. There has to be coordination between common goals and
individual goals whereby individual goals should be sacrificed for the sake of
broader social goals. It encourages the significance of family and humanity to build
a complete society.
• Cultural ethos: It talks about the integration of indigenous culture with social,
economic and political fabric of the nation. It maintains that the essence of Indian
culture and its uniqueness should form the background of any political philosophy
or development model to be adopted by India.
• Integrated view: While accepting various differences, it focuses on the inter-
dependency, association and unity rather than divergence, denial and
disagreement in various aspects of life. Hence, it works for the welfare of
everyone.
• Dharma Rajya: It represents the ideal duty-oriented state where every individual
has power as well as obligations towards its state.
• Antodaya: This concept ensures that decision is taken considering that the person
who is last in the queue is also taken care of.
Contemporary relevance
• It advocates the holistic idea of human welfare. The philosophy of integral
humanism opposes unbridled consumerism and rapid industrialisation without its
benefits trickling down to the poorest of the poor – which is relevant in today’s
context of inclusive development for all.
• The philosophy of integral humanism is also in sync with the modern notions of
democracy, social equality and human rights since respect and equality of all faiths
and castes is an important feature of the Dharma Rajya.
• Integral Humanism aims to provide a life of dignity to every human being, thus,
promoting policies and principles that can balance the use the labour, natural
resources and capital at the same time.
• Adopting this philosophy may change the approach towards politics as Pandit
Deen Dayal believed that the purpose of politics is to bring about social and

DELHI JAIPUR PUNE HYDERABAD AHMEDABAD LUCKNOW CHANDIGARH


30 www.visionias.in # 8468022022 ©Vision IAS
economic changes. This is required in today’s context where criminal elements, Student Notes:
money power etc. is beginning to dominate politics.
• It may lead to strengthening of family institution as this philosophy highlights the
role that families and society plays in building a nation
A world where large population lives in poverty this can be an alternative model for
development in which social, political and economic needs converge and which is
integral and sustainable in nature.

13. Elucidate Swami Vivekananda's ideas on nationalism.


Approach:
• Briefly introduce the life of Swami Vivekananda.
• Bring out the various foundations on which his idea of nationalism is based upon.
• Conclude with the relevance of his views in present-day India.
Answer:
Swami Vivekananda was a major force in the formulation of the concept of nationalism
in colonial India. Regarded as the patriot saint of modern India, he inspired India’s
dormant national consciousness. Though growth of Nationalism is attributed to the
Western influence but Swami Vivekananda’s nationalism is deeply rooted in Indian
spirituality and morality. He contributed immensely to the concept of nationalism in
colonial India and played a special role in steering India into the 20th Century. Basic
tenets of his ideas on nationalism include:
• Spiritualism: Vivekananda’s nationalism is associated with spiritualism. Humanism
and Universalism, the two cardinal features of Indian spiritual culture, are
prominent in his ideas on nationalism. He called for Indians to understand India’s
spiritual goal, its place amongst other nations and its role in contributing to the
harmony of races.
• Religious basis: His nationalism is based on religion which he considered to be the
life blood of the Indian people. He taught the essential oneness of all religions and
condemned any narrowness in religious matters. He condemned the caste system
and blind faith in rituals and superstitions, urging evaluation of religious texts based
on its conformity to logic, reasons and science. This helped Indians in acquiring a
modern, secular and national outlook. He established motherland as the only deity
to be worshiped in the mind and heart of countrymen.
• Roots in Indian Philosophy: Vivekananda felt that Indian nationalism had to be
built on the stable foundation of India’s past historical heritage. He revived and
advocated the Indian philosophical tradition, particularly Vedanta as being a fully
rational system. Hence Vivekananda worked for awakening the masses, the
development of their physical and moral strength and creating in them a
consciousness of the pride in the ancient glory and greatness of India. This helped
in fostering greater self-respect, self-confidence, and pride among Indians.
• Humanism: For Swami Vivekanada, humanitarian morality should favour human
progress. He laid emphasis not only on personal salvation but on social service as
well, that he maintained as the pre-requisite for national awakening. He drew
attention to the extent of poverty in India, and stressed upon elimination of
poverty.
• Nationalism vis-à-vis internationalism: He encouraged people to mingle with the
life of other individuals and nations. His love for mankind transcended the
geographical limitations. His interests were not confined to India alone but were

DELHI JAIPUR PUNE HYDERABAD AHMEDABAD LUCKNOW CHANDIGARH


31 www.visionias.in # 8468022022 ©Vision IAS
extended to international level. He called for the harmony and good relationship Student Notes:
with other nations.
• Ending Cultural Isolation: Address at the Parliament of Religions held in Chicago in
1893 started the process of ending India’s cultural isolation from all other nations
which was leading to her degeneration. He opposed blind imitation of the West and
was against colonisation of culture and thought, while welcoming positive elements
of modern culture.
Since the onset of 21st Century, the world is in turmoil and passing through transition
period of a kind. At this hour of human history the message of Swami Vivekananda
promoting spiritual integration of the nation and world on the basis of universal
brotherhood and goodwill becomes all the more relevant.

14. The best way to find yourself is to lose yourself in the service of others. Mahatma
Gandhi
Approach:
• Introduce the context of the statement given.
• Explain in brief your understanding of the quotation given in present context.
• Support your argument with suitable examples.
• Conclude on the basis of the above points.
Answer:
The quote captures the essence of the life of Mahatma Gandhi who dedicated his life
for the upliftment and betterment of others. It highlights that the true core of a person
is not inherently selfish, but rather selfless. It means that one can find the true innate
nature of self not just by focusing on oneself, but by devoting oneself in the service of
other people around them.
The quote is based on the premise that till the time we are self-centered and thinking
about personal gains, the true inner identity of the individual won't surface up . It is
only when we act out of selflessness and dedicate ourselves to a larger societal cause,
that we could realize altruistic qualities of our hidden ‘character’ – empathy,
compassion, compromise, charity, inner strength etc. In this regard, service is a fulfilling
activity that can help to see that “we” are not defined by an identity, but by ability,
sympathy, connection, and solidarity. Therefore, to lose oneself in the service of others
implies nothing but the sacrifice of egoism, self-centeredness, hatred and
discrimination based on caste, creed & religion.
Further, by serving others, we not only find more about ourselves but also help others
in the process. It helps in achieving the dual objective of self-service and service to the
mankind. When we become totally engrossed in the process of helping others, we tend
to lose sight of our own petty problems, the things that we think, define us and
consequently our true self reveals itself. People like Kailash Satyarthi have found
greatness within themselves, only by losing themselves in the service of others.
True fulfillment comes when we are ready to help others. Self-empowerment is when
we empower our fellow beings. The moment we dedicate ourself in the service of
others, it is that time when we discover our true self. These remarks holds true even in
the present age of information and technology, which is exemplified with the life of Bill
Gates and Warren Buffet who after achieving all the materialistic gains, have decided to
give away their wealth for service of others, thus reflecting that the most essential
purpose of human beings is to be virtuous.

DELHI JAIPUR PUNE HYDERABAD AHMEDABAD LUCKNOW CHANDIGARH


32 www.visionias.in # 8468022022 ©Vision IAS
15. So long as you do not achieve social liberty, whatever freedom is provided by the law Student Notes:
is of no avail to you. B.R. Ambedkar
Approach:
• Introduce the context of the statement given.
• Explain in brief your understanding of the quotation given in present context.
• Support your argument with suitable examples.
• Conclude on the basis of the above points.
Answer:
The Indian Constitution guarantees various Fundamental Rights to the citizens, one of
which is Right to Freedom. But the freedom guaranteed by the fundamental law of the
land does not mean that people enjoy freedom in true sense.
Various social evils existing in the society curtail the individual freedom guaranteed by
law. The hierarchy of caste system and the attached notions of purity and pollution
have curtailed the livelihood opportunities for certain sections of the society. For
instance, the practice of manual scavenging limits the mobility of the persons
associated with it and in turn hampers their growth potential.
Over a period of time, the society tends to develop its own mechanisms via norms and
cultures to ensure the oppressive system remain in place irrespective of the freedoms
provided by law. The recent agitation for temple entry movements by women across
the country is an example of how social acceptance of regressive norms (menstruation
as pollution) violates the basic fundamental right of women. Further, despite existence
of laws, the social liberties of women are curtailed due to prevalence and dominance of
patriarchy, thus subverting education and career options for women in a subtle manner.
Social liberty is a state of being free within society where the opportunities are
available to all individuals, irrespective of caste, gender, religion, etc. to attain a position
in society commensurate with their capabilities. The absence of social liberty creates
inequalities with regard to access to a variety of social ‘goods’, such as education,
healthcare, the labour market etc.
In the current context, the violence related to inter-caste marriages and the poor
acceptance of homosexuality in the society also reflects the dissonance between the
societal culture and norms and freedom ensured by law. Therefore, a right, which is
guaranteed by law but is opposed by the society, is of no use at all.
Therefore, it is important to ensure progressive change in the society in congruence
with the progressive legislations. Bringing social sensitization by awareness building
and imparting education can help ensure social liberty for all sections of the society.

16. Intolerance is itself a form of violence and an obstacle to the growth of a true
democratic spirit. Mahatma Gandhi
Approach:
• Briefly introduce the term ‘intolerance’ in the introduction.
• Talk about how intolerance breeds violence and threatens democratic spirit.
• One can also talk about how tolerance can help in deepening democracy.
• Conclude appropriately.
Answer:
Intolerance is a lack of respect for practices or beliefs other than one's own. It can
develop due to differences amongst people on the basis of religion, caste, race, gender,

DELHI JAIPUR PUNE HYDERABAD AHMEDABAD LUCKNOW CHANDIGARH


33 www.visionias.in # 8468022022 ©Vision IAS
opinions and affiliations, say to a political ideology/party. When this intolerance creeps Student Notes:
over a person, he ceases to grow as he closes himself to other ideas.
This further leads to discrimination, and culminates in violence & persecution. It has
the potential to create a polarising impact and a sharply divided polity, which could
spell danger, particularly in a multi-layered, multi-religious and multi-ethnic country like
ours.
Gandhiji identified ‘intolerance’ amongst the enemies of correct understanding. This is
vital for nurturing democratic spirit, which enables us to make correct decisions
ourselves, keeping in mind the overall interests of the society.
Tolerance, on the other hand, strengthens the democratic spirit in people and
communities. According to the Socrates, tolerance is about the pursuit of truth. It is
about being prepared to see the value of another person’s perspective or truth. This is
fundamental to democracy, which is about equality and the right of people to believe
what they believe. Agreement or consensus may be a desired end, if only to resolve
conflict, but it is not a democratic imperative. Tolerance is the tool that helps us
democratically manage the dynamics of any plural community or society.
In protecting and cultivating diversity, tolerance also creates the conditions for human
flourishing. Disagreement and dissent can be seen as processes of “mutual incitement”.
When people disagree, they are in fact inciting each other to think anew and differently
about established ideas or beliefs. It is through disagreement that we dislodge
ourselves from rigid ways of thinking and being – and refine our ideas and positions.
Former Indian President P. Mukherjee once remarked that “Intolerance and violence is
a betrayal of the letter and spirit of democracy." India is a nation that prides itself on
adhering to values enshrined in its Constitution. As its ardent followers, our aim must
be to develop scientific temper and build a tolerant environment.

17. Try not to become a man of success but rather try to become a man of value. Albert
Einstein
Approach:
• In the introduction, give a brief idea of the meaning of the term success.
• Explain why pursuit of success is problematic.
• Conclude while writing in brief about the need to pursue a value-based approach.
Answer:
Humans are fascinated with what is possible in life and what they can do or become.
They have dreams, hopes and aspirations, passions and thoughts. It is when these
dreams, hopes, aspirations turn into reality that they think of themselves as successful.
In terms of pure semantics, success is merely the accomplishment of an aim or
purpose. There is no virtue signaling around the aim or purpose itself i.e. it could be
good, bad or indifferent to the society at large.
When Einstein suggests that we must strive to become men of value not of success, his
wisdom brings to light the underlying problems around the idea of success. In fact, the
fault doesn’t lie in the conception of success itself but understanding of the value
underlying it. It is what we hold important and how we intend to achieve it that must
be regarded.
As much as one might think of success being purely personal, everything around us
seems to suggest otherwise. For instance, there is an endless array of advertisements,
blogs etc. around ways to become successful and almost all of them measure success in

DELHI JAIPUR PUNE HYDERABAD AHMEDABAD LUCKNOW CHANDIGARH


34 www.visionias.in # 8468022022 ©Vision IAS
terms of quantifiable metrics such as wealth, fame, audience etc. This imparts a sense Student Notes:
of superficiality to the notion of success. As such, many ‘successful’ people can be
found regretting their life choices, while some even succumb to the pressure of leading
a successful life, as witnessed in the CCD case recently.
This often eclipses the need to think about character and morality. Obsession with
success alone can cause one to be blinded to morality of both the means and the end.
For instance, in pursuit of making windfall profits, corporates often bribe public
functionaries.
On the other hand, even as value is a relative concept (i.e. beliefs about right and
wrong may or may not be considered moral), chasing values is a harmless pursuit. This
is because ‘immoral value’ is an oxymoron. In a narrow sense, value is that which is
good, desirable, or worthwhile. They are the motives behind purposeful actions. While
pursuit of success may blind someone, values guide humans by imparting a sense of
introspection and assessment of their actions.
Success in itself is not a value. Only when it is qualified by the right set of values, can it
hold meaning to both the individual and society at large. More often than not, humans
driven by the sense of value also ended up being successful.

18. Discuss the contributions of Dr. B.R. Ambedkar to the idea of social justice in India.
Approach:
• Give a brief introduction of Dr. B.R. Ambedkar highlighting the influence of his
personal experiences on his beliefs and philosophy of social justice.
• Decode his views on social justice in India.
• Conclude on the basis of above points.
Answer:
Social justice involves the creation of a just and fair social order to one and all. It aims
to tackle the social inequalities and injustices existing in India owing to differences
based on caste, community, gender etc. Even after almost seven decades of
independence, social justice remains a distant dream for various sections of Indian
society.
Dr. B.R. Ambedkar’s contributions to idea of social justice in India
• His idea of social justice included all kinds of justice, namely, legal, economic,
political, divine, religious, natural, distributive, administrative as also welfare of
children and women.
• His concept of social justice did not merely mean the distribution of social wealth.
Instead, it was based on the concept of social democracy consisting of three
concepts of justice namely liberty, equality and fraternity of all human beings,
which draws inspiration from the French Revolution of 1789.
• He ensured that India’s Constitution acknowledges the longstanding social
injustices of the past and provides for measures of affirmative action to facilitate
access to opportunities for disadvantaged sections of Indian society. In this regard,
various provisions in Directive Principles and Fundamental Rights aim towards
achieving social justice.
• Being a victim of social inequalities and injustice himself, he had been relentless in
his pursuit of a social system, which is based on right relations between man and
man in all spheres of life.
o He argued for extensive economic and social rights for depressed classes,
women and labour.

DELHI JAIPUR PUNE HYDERABAD AHMEDABAD LUCKNOW CHANDIGARH


35 www.visionias.in # 8468022022 ©Vision IAS
o He called for annihilation of caste as it is highly discriminatory and promotes Student Notes:
social immobility, disruptive tendencies, endogamy and anti-social spirit.
Article 17 of the Indian Constitution declares untouchability as abolished.
• He put forward the idea of “Educate – Unite – Agitate” to access the path to social
justice. It means people who have been discriminated against for the past several
centuries due to their social origin have to first get ‘Educated’ then ‘United’ and
finally they have to ‘Agitate’ for their rights.
• As a rationalist and humanist, he did not approve of any type of hypocrisy,
injustice and exploitation of man by man in the name of religion. He stood for a
religion that is based on universal principles of morality and is applicable to all
times, to all countries and to all races. That is why he embraced Buddhism.
• His thoughts on social justice were progressive and he considered the press to be a
powerful tool for social changes for justice and freedom and published Mook
Nayak, Janata and Samata magazines.
Hence, Dr. B.R. Ambedkar’s ideas of social justice remain relevant in contemporary
Indian society in promoting constitutional and legal methods for upholding the rights
and dignity of the vulnerable sections in India.

19. If people scrutinize their own faults as they do the faults of others, mankind will be
freed of all evil.
Approach:
• Introduce the answer with a brief interpretation
• List the reasons behind people finding faults in others’ actions
• Discuss the positive implications of scrutinizing & owning up to our own faults.
• Conclude
Answer:
Thiruvalluvar suggested that if man could scrutinize his own faults instead of finding
faults in others, mankind will be freed of all evils. Obsessing over others’ faults, instead
of concentrating upon our own mistakes, makes us vulnerable to vices of conceit,
blaming & ill will.
The human tendency to find faults in others is commonplace. For example, people
blame the government for corruption, while partaking in bribery themselves or
avoiding registration of complaints. Similarly, women complain of gender bias while
some of them are active perpetrators of the same. It happens partly because of our ego
driven nature. Finding faults within ourselves challenges our cherished self-image of a
decent person. It may also happen because of complete unawareness of our mistakes
that makes owning up to it extremely difficult, while blaming others offers a convenient
escape.
In the present context, where information is available at the click of a button, people
tend to overlook their own mistakes and devote less time towards a rational thought.
The age of making quick decisions leaves little time for self-introspection. This is
especially dominant in inter-personal conflict today. Hiding one’s own ills or weaknesses
is considered a character of strength, however brushing them aside and being ignorant
of them is dangerous. And, since a person himself is the best evaluator of self, ignoring
this leads to an overall sub-optimal outcome for the society and leads to generation of
societal ills. Even though all ills of the society are not generated by lack of self-
evaluation, the conflicts certainly spiral out because of this.
Scrutinising our own faults has numerous advantages, both personal and for the
society. Personally, it helps us make better decisions since self-justifications distort

DELHI JAIPUR PUNE HYDERABAD AHMEDABAD LUCKNOW CHANDIGARH


36 www.visionias.in # 8468022022 ©Vision IAS
reality, create an alternate universe and decrease our ability to make good choices. It Student Notes:
prevents small mistakes from catapulting into bigger mistakes by addressing them in
the beginning itself. It engenders the respect of others and strengthens relationships
between people.
Such people will inculcate the right habits in their children, and can influence grown
ups as well. For instance, great leaders like Gautam Buddha, Ashoka, Mahatma Gandhi
could influence the world by their individual actions. Virtues like humility, truthfulness,
honesty etc. if practiced at the individual level, will eventually create a better society
and peaceful world free of evils. The similar philosophy was reaffirmed by Gandhiji in
his remarks-"be the change you want to see in the world."

20. "Nonviolence is not servile passivity but a powerful moral force which makes for
social transformation". Comment.
Approach:
• Mention the context of the statement and briefly explain its meaning.
• By citing examples, elaborate on how non violence can aid the process of social
transformation in a positive way.
Answer:
The statement was observed by Dr. Martin Luther King in his Nobel Prize acceptance
speech in 1964. It recognizes the power of non-violence in social transformation as it
provides answer to the crucial political and moral questions raised at times.
Non-violence is the personal practice of being harmless to self and others under every
condition. But in no sense, it means passivity or inaction. Rather it is a dynamic living
force, a willingness to suffer and sacrifice. It is based on the premise that non violence
is meant for the strong character, and not for the weak - that forgiveness is the virtue of
the strong, weak cannot forgive.
Non-violence works in a silent, subtle, unseen way and transforms the whole society. It
entails pitting one’s whole soul against the will of the tyrant. In modern times, it is the
greatest moral force at the disposal of mankind and is a powerful tool for revolutionary
socio-political change and justice. As Gandhiji held, non-violence works by striking at
the heart of the tyrant, rather than doing physical damage. The change brought by it is
rather permanent.
Non-violence ensures the requisite changes in the existing social order without
sacrificing the cherished values of humanity. This can be understood through various
examples such as:
• Martin Luther King's adoption of non violent methods to win civil rights for African
Americans.
• Gandhiji’s fight against age-old system of untouchability through non-violence by
working along with Harijans
• Gandhiji's non-violent struggle against the Rowlatt bill and his Salt Satyagraha
moulded the nationalist thinking of that time for betterment.
Non-violence knows no defeat. It has no limits and it is applicable to every aspect of
life; even no great expenditure of money is required for equipping an army of non-
violent resisters.
On the other hand, violent change does not guarantee success and long term peace
either. Violence can only destroy societies and not make or transform it. Several
examples ranging from French Revolution to the present-day problems of Vietnam,

DELHI JAIPUR PUNE HYDERABAD AHMEDABAD LUCKNOW CHANDIGARH


37 www.visionias.in # 8468022022 ©Vision IAS
Korea and Arab-Israel conflict testify the same fact. Violence never gains the moral Student Notes:
authority required to make appropriate social transformation.
A non-violent revolution does not simply seek the liberation of a class or a nation or a
race, but it seeks the liberation of mankind. Martin Luther King Jr. rightly remarked -
"Nonviolence is a powerful and just weapon. Indeed, it is a weapon unique in history,
which cuts without wounding and ennobles the man who wields it."

21. Income inequality is a matter of grave concern for the Indian society. In this context,
examine the relevance of Gandhiji's concepts of 'Sarvodaya' and 'Trusteeship'.
Approach:
• Briefly comment on the extent of income inequality in India.
• Mention the consequences of income inequality.
• Explaining the concepts of Sarvodaya and Trusteeship, examine their relevance in
phasing out income inequality from the Indian society.
Answer:
According to some estimates, Income inequality has been worsening in India in the past
two decades despite a historic progress on reducing poverty. According to the World
Inequality Report, 2018 the top 10% of the population accounted for 56% of the
national income in 2014. Additionally, the absolute income growth of top 1% since the
1980s exceeds that of the bottom 50% people taken together. Similarly, according to
the annual Oxfam survey, India’s richest 1% cornered as much as 73% of the wealth
generated in 2017.
Consequences of income inequality include:
• High level of economic inequality signifies that opportunities for earning are limited
and many people remain relatively poor, if not absolutely. This is associated with
increased crime and social upheavals.
• Wealthy citizens maintain disproportionate political power compared to poorer
citizens, which encourages inefficient tax structures ultimately favouring the
wealthy, thus perpetuating further inequality.
• High and sustained levels of inequality of opportunity can undermine an
individual’s educational and occupational choices.
• Higher inequality lowers growth by depriving the ability of lower-income
households to accumulate physical and human capital.
• Extreme inequality may damage trust on the government and can lead to conflict
and unrest. Eg, Naxalbari Movement.
In this context, Gandhiji’s concepts of ‘Sarvodaya’ and ‘Trusteeship’ assume relevance,
since they advocate equality and fairness.
Sarvodaya: It means 'progress of all'. In the Sarvodaya society, conceptualized by
Gandhiji, every member will be free from greed for limitless acquisition of material
wealth and will get equal opportunities to earn sufficiently through honest work. As the
Sarvodaya society is based on the basic tenets of socio-economic equality and
individual liberty, it rules out any form of unhealthy competition, economic exploitation
and class-hatred. Rather, it provides enough scope for unhindered development of
individual’s skill set, thus it helps in the process of equitable redistribution of income
across different sections of the Indian society.
Trusteeship: It is Gandhiji’s socio-political philosophy, which propounded that the
wealthy would hold wealth and property, not by virtue of being owners but by virtue of

DELHI JAIPUR PUNE HYDERABAD AHMEDABAD LUCKNOW CHANDIGARH


38 www.visionias.in # 8468022022 ©Vision IAS
being trustees who look after the welfare of the people in general. It would help in Student Notes:
phasing out income inequality, as the philosophy mandates the wealthy people to part
with their wealth to help the poor, thus, eliminating extremities of poverty and
affluence.
Though, difficult to implement by a particular country in the wake of competition, but
can be implemented if agreed to by all major countries. If implemented, these concepts
provide alternate dimensions to alleviate the problem of inequality and introduce new
tools to distribute wealth symmetrically to all classes. They will not only help in
reducing economic equality, but also foster social equality and political stability in the
long run.

22. Examine the contemporary relevance of Gandhiji's talisman as a means of resolving


ethical dilemmas in day to day life.
Approach:
• Highlight the key message of talisman (in brief) – ‘sarvodaya through antyodaya’.
• Explain how it can promote emotional intelligence in citizens and help them resolve
ethical dilemmas.
• Discuss its contemporary relevance for citizens, businesses and the Government.
Answer:
Gandhiji’s talisman– “whenever in doubt, think if the decision would empower or
marginalize the poorest?” provides an ethical test to judge everyday actions. It
champions the cause of ‘sarvodaya through antyodaya’ implying the welfare of all
through the weakest of the society which lies at the core of Indian Constitution.
It is a clarion call to the citizens to develop emotional intelligence so that they are in a
better position to assess the needs of fellow (vulnerable) citizens, and work towards
their upliftment through individual actions.
India of the 21st century is witnessing sectoral violence, socio-economic inequality, low
tolerance threshold and increasing consumerism. Global politics is witnessing
xenophobia, armed conflicts and multiple refugee crises. The talisman can hence be an
effective tool to evaluate the relevance of government policies, business priorities and
our daily actions on the society. All these entities can assess their actions in public as
well as private life against the talisman to ameliorate ethical dilemmas.
Applying the talisman to our daily lives invariably suggest the following:
• Government policies must necessarily focus on the vulnerable and marginalized
sections of the society even if it requires making special provision for such classes.
• At personal level, self-serving acts like tax evasion, littering in public places etc.
must be shunned. Derogatory practices in society like manual scavenging, caste-
based discrimination, exploitation of women and the poor, dowry, nepotism etc.
must be condemned in practice.
• At societal level, it encourages altruism, compassion and communal harmony
towards fellow human beings when encountered with negative vibes of hatred and
violence.
• Businesses and Industries should strive to reduce polluting activities, ensure
optimal working conditions and must honestly comply with social security
legislations. Further, it encourages business houses to go for Corporate Social
Responsibility for achieving greater inclusiveness in the society.

DELHI JAIPUR PUNE HYDERABAD AHMEDABAD LUCKNOW CHANDIGARH


39 www.visionias.in # 8468022022 ©Vision IAS
• Public servants should ensure transparency and integrity in implementation of Student Notes:
welfare provisions like PDS, MGNREGA, Old Age Pension Scheme etc. so that their
steps lead to swaraj in true sense.
Thus, Gandhiji’s talisman empowers us to judge our actions and is a timeless beacon of
inspiration for generations to come.

23. "Man by nature is a political animal". Explain with reference to Aristotle's idea of the
state in life of the society.
Approach:
• Bring out the different connotation of the statement given.
• Elaborate on why Aristotle regarded human beings as naturally political.
• Discuss the role of state in the society in the context of man as political animal.
• Conclude.
Answer:
Aristotle believes that humans are like the animals in many ways, but are above them in
one distinctive way - politics. In the given statement there are two components, one is
“by nature” which connotes something that is innate and the other one is “political
animal” which means the animals who live together in a setting. The entire statement
thereby implies that it is the inherent nature of humans to live together and form
deliberative and governing bodies that seek to betterment of all those involved- that
man is naturally sociable and that they are naturally drawn to various political
associations in order to satisfy their social needs. And this process is possible only
because humans have the power of speech and moral reasoning.
To elaborate, since we see that every city-state is a sort of community and that every
community is established for the sake of some good (for everyone does everything for
the sake of what they believe to be good), it is clear that every community aims at
some good, and the community which has the most authority of all and includes all the
others aims highest, that is, at the good with the most authority. This is what is called
the city-state or political community. Aristotle opines that state enjoys highest rank or
position in the society or social structure. State is characterized by natural growth. But,
during its different stages of progress, man-made laws and conventions have
intervened. But these have benefitted man and facilitated functioning of state. The fact
that government, tribes, monarchs, other groups, etc which are one or other form of
state do exist, are proof that humans operate politically. It has helped state to ensure
peace and prosperity for the people associated with it. Together what humans have
achieved today would have been impossible to achieve. Be it technological
advancements, prosperous and peaceful societies to the development of shared human
values of compassion, love, respect, sympathy, tolerance among others.
But the same political nature which has brought humans together has also divided
them. Rise of nation-states has led to fragmentation of society as well as forced
agglomeration of social groups and rendered many as unwanted population. It has
uprooted many from their culture and customs and forced them to accept diktat of a
state they don’t associate with. It has led to problems of war, secession, insurgency,
terrorism and refugees. Globally under the ruse of protecting one's own country, the
very groups formed on the premise of good for the society, propagate violence.
It is up to the governments and other groupings to bring the humans closer and make a
just society where everyone understands and accepts universal human values of peace,
love, compassion towards others. Also the society has to change because “if you
change the society and a culture, the politics will follow”.

DELHI JAIPUR PUNE HYDERABAD AHMEDABAD LUCKNOW CHANDIGARH


40 www.visionias.in # 8468022022 ©Vision IAS
24. The policy of Dhamma advocated by Ashoka through his edicts remains relevant in Student Notes:
the context of issues in public life even today. Elucidate with examples.
Approach:
• Explain the concept of Dhamma advocated by Ashoka.
• Discuss with examples why his concept of Dhamma is relevant even today.
Answer:
After the war in Kalinga, Ashoka advocated a code of social ethics based on pillars of
tolerance, liberalism and compassion, termed as ‘Ashoka’s Dhamma’. It was neither a
new religion nor a new political philosophy. It was a way of life, a code of conduct and
practically stood for absence of sin, good deeds, charity, donation, truthfulness and
purity. It is basically a moral code to guide individual behaviour in society according to
universal moral laws
This doctrine becomes essential, especially for public servants in a society mired with
challenges such as inequalities, intolerance and various ethical-political dilemmas.
Present day relevance in the context of public life
• Secularism and tolerance: Ashoka’s dhamma explicitly mentions that 'one
shouldn't extol one's own sect or disparage those of others, as doing so will harm
others'.
• Duties of Public servant- Special officials were appointed for distribution of alms to
all sects so that all thrive and co-exist. They also taught people about ethics and
read out messages for people who could not read.
• Foreign Policy and Soft Diplomacy: Internationally, his policy is also relevant in
today’s approaches to soft diplomacy, as he sent messages to spread ideas about
Dhamma to other countries.
• Sustainable Development Goals: Invoking these ideas in present day governance
and administration, will not only create social capital and harmony, but also enable
India to achieve targets in Sustainable Development Goals.
• Human face to administration: It advises public servant to be free from jealousy,
anger, cruelty, hastiness, laziness and fatigue.
• Environmental democracy and justice (Environmental Ethics): The idea planting of
trees along the roads to provide shade; building of guest-houses for travelers and
construction of several watering places lays the foundation of modern
environmental ethics by highlighting the importance of nature for a sustainable life.
Therefore, Ashoka’s policy of Dhamma becomes imperative in present times as it
always kept people above profits, focuses on moral duties and secular harmony among
all.

25. Given below are two quotations of moral thinkers/ philosophers. For each of these,
bring out what it means to you in the present context. (a) "Silence becomes cowardice
when occasion demands speaking out the whole truth and acting accordingly" -
Mahatma Gandhi. (b) “We must not only tolerate others, but positively embrace
them" - Swami Vivekananda.
Approach:
• In the introduction, bring about the intentions of the moral thinkers- both Gandhi
and Vivekananda.
• Discuss the values in association with the statements.
• Discuss relevant context in which these values can be applied.
• Conclude by emphasizing the importance of these statements.

DELHI JAIPUR PUNE HYDERABAD AHMEDABAD LUCKNOW CHANDIGARH


41 www.visionias.in # 8468022022 ©Vision IAS
Answer: Student Notes:
(a) Gandhiji has been the one of the biggest supporters of moral politics, upholding
high principles like truth, honesty and courage in public life. As a public figure he
believed in walking the talk.
He believed in strong force of the soul and courage to be truthful in all walks of life.
But to be able to speak the truth a person needs to have strength of the soul.
Gandhi ji believed that a seeker of truth has to be silent. Proneness to exaggerate,
to suppress or modify the truth, wittingly or unwittingly, is a natural weakness of
man and silence is necessary in order to surmount it.
However, Gandhiji also believed that silence becomes cowardice when silence only
helps the oppressor and not the oppressed. He held that one need to cultivate
courage,it is a virtue worth living for and dying for. A person needs to hold on to the
truth and defend it with the force of the purity of soul, also appealing to the soul
force of the opponent. He called it “Satyagraha”, which he had practiced on various
occasions to fight the mighty British Empire as well as social evils such as
untouchability.
The given quote is of great significance even in the present context which is evident
from the following:
• Courage to stand against the tide of corruption- Today, the public life has
become corrupt and opaque and when the system itself is corroded, it requires
great courage to stand against the wrong tide and to walk on the path of
honesty and truth. For example, the act of whistleblowing.
• Commitment to bring transparency to public system- Various provisions and
policy tools like RTI are being brought into force to ensure transparency, but it
cannot happen until the public servants do not commit to transparency
completely. Truth has to be vociferously defended if transparency needs to be
brought to the system.
• Raising awareness: For e.g. the cultural movement #MeToo, encouraged that
the whole truth needs to be spoken out by victims of sexual abuse which can
give people a sense of the magnitude of the problem
Silence or half-hearted effort is never an option if the principles and values of
freedom struggle have to be put into practice to defend the true spirit of
democracy. The only answer is speaking the truth and acting courageously.
(b) Above mentioned words of Swami Vivekananda were spoken at the world
parliament of religions where he addressed the American people as his brothers
and sisters. This shows his general idea of acceptance towards each and every
being irrespective of religion, nationality, race etc.
The values inherent in the statement:
1. Vasudev Kutumbakam: The concept of ‘whole world as a family’ is inherent in
Indian culture. We have to own the differences amongst people and accept
them as part of one humanity.
2. Multiculturalism- Owning and not tolerating: Multi culturalism means
primarily an interest in other cultures, respect for the " differences of all
segments of the population" and ' oneness with the world". Tolerance is a
negative word, the positive aspect of living in a diverse society is to assimilate,
respect and accept the differences. That is what makes the society truly
multicultural.

DELHI JAIPUR PUNE HYDERABAD AHMEDABAD LUCKNOW CHANDIGARH


42 www.visionias.in # 8468022022 ©Vision IAS
3. All religions are different paths to same goal- The need to hold inter faith Student Notes:
dialogue right from the school to societal level on regular basis to bring home
the truth about the inherent unity of all faiths that Sri Ramkrishna preached
and practiced. India is a ' civilisation state' and not just a ‘nation state’ in the
European sense of the term and founded on dynamic multi cultualism.
Vivekananda did not see the goals of different religions as essentially different.
The religions are in harmony with each other owing to their goals and
principles. The followers of the religion must understand the same and be
brethrens in the national and social habitation.
In a world divided on race, religion, ethnicity among other factors these
propositions expounded by Vivekanand are extremely relevant.
• Avoiding intermixing of Religion and Politics- somewhere in the misuse of the
religion for political gains, the true goals of the religion that is peace, harmony
and higher purposes of life are getting misplaced.
• Check hatred and Violence in the name of religion- there are many unsocial
elements who try to incite hatred in the name of religion. Many a times it leads
to riots and tension among various groups. As a result, weaker sections and the
society at large suffers.
• For the protection of the united and diverse nation- India as a nation is diverse
as well as united. This unity can only be preserved if the society as a whole
accepts each other and flourishes together.
In today’s context the teachings of Swami Vivekananda on religious acceptance is
more than relevant. Not only are his words that of a religious monk preaching true
secularism to the society but also his preaching is very revolutionary in providing us
an idea of a diverse yet united nation.

26. What do you understand by Amartya Sen's 'capability approach'? Analyse its
significance in understanding social realities and in making a pro poor development
strategy.
Approach:
• Explain Amartya Sen’s ‘capability approach’.
• State the various dimensions covered by it..
• Assess its significance in understanding social realities and in making a pro poor
development strategy.
Answer:
Giving bicycles to poor girls can a means to encourage them to go to school. But what
good is a bicycle for a person who cannot see? Development economists have debated
about how to judge the progress of a nation. Per capita income may be high, but
inequality may be pervasive. People may be wealthy but they may not be happy. In this
debate, Amartya Sen proposed the ‘capability approach’ to measure the progress of a
country. A capability is defined as the measure of people’s freedom to pursue and
achieve goals which they would like to. These are the goals that they deeply value, for
e.e. education is necessary to be employable, hence people would like to be educated.
So whether the society allows them to be educated or not will determine how much
progress the society is making. Capability approach was proposed as a practical theory
to remove injustices in particular instances against the prevailing theories of
universalising justice, such as utilitarianism or resourcism.
In his ‘capability’ theory, he defines capability as ‘the ability to fulfil one’s own will’ i.e.
the ‘freedom to choose’ must be associated with ‘capability to choose’. It particularly

DELHI JAIPUR PUNE HYDERABAD AHMEDABAD LUCKNOW CHANDIGARH


43 www.visionias.in # 8468022022 ©Vision IAS
identifies specific incapacities which hinder justice for all. For example, people in Student Notes:
remotest regions face incapacity of transportation; women face the incapacity because
of social norms etc. The approach is ' capability enhancement of the people " as the
main plank of the strategy to bring about inclusive development and hence to reduce
inequality.
In this way, the approach focuses on the quality of life of individuals in terms of
‘functionings’ and ‘capability’. ‘Functionings’ are states of ‘being and doing’, such as:
being well nourished and healthy, having adequate shelter etc. Capability refers to a set
of valuable functionings that a person has effective access to. Thus, a person’s
capability represents his/her effective freedom to choose a life that he/she has reason
to value.
This approach addresses various dimensions such as individual physiology, political and
economic freedom, social factors, local environment conditions, differences in access to
resources etc. therefore it helps us understand the existing social realities, especially in
a developing country like India where different sections of population face different
issues. Further its significance in understanding social realities and in making a pro poor
development strategy lies in:
1. It puts people at the focus of development and aims at expanding their capabilities
as per their social reality.
2. It supplements the GDP model of development and simultaneously points out its
shortcomings as a sole measure of economic progress.
3. It centres the growth and development debate around ‘development of the
individual’. .
4. Several indices have been created based on the approach such as HDI, Gender
Inequality Index, Gender-Related Development Index etc.
The approach is also significant in making a pro-poor development strategy as it
explains poverty as a result of capability deprivation and states that government should
emphasize on social sector expenditure such education, health etc. to improve the
capabilities of poor people.

Copyright © by Vision IAS


All rights are reserved. No part of this document may be reproduced, stored in a retrieval system or
transmitted in any form or by any means, electronic, mechanical, photocopying, recording or
otherwise, without prior permission of Vision IAS.

DELHI JAIPUR PUNE HYDERABAD AHMEDABAD LUCKNOW CHANDIGARH


44 www.visionias.in # 8468022022 ©Vision IAS
Student Notes:
ETHICS IN ADMINISTRATION- CONCERNS,
DILEMMAS AND SOLUTIONS
Table of Contents
1. Governance, Good Governance and Ethical Governance .........................................2
2. Status of Ethical Standards in Indian administration ...............................................3
2.1. Ethical issues in Indian administration ........................................................................ 3
2.2. Ethical dilemmas in Indian administration .................................................................. 4
3. Ethical Concerns and Dilemmas in Private/Business Institutions .............................5
3.1. Ethical Issues of Private Sector in General ................................................................... 5
3.2. Ethical Issues of Employers......................................................................................... 5
3.3. Ethical Issues of Employees ........................................................................................ 6
4. Existing Framework for Ethical Standards in India ..................................................6
4.1. Central Civil Services (Conduct) Rules 1964 ................................................................. 6
4.1.1. Critique of the Rules ................................................................................................................... 7
4.2. Draft Public Service Bill 2006 ...................................................................................... 7
4.3. Code of Conduct for Legislators .................................................................................. 8
4.4. Code of Conduct for Ministers .................................................................................... 8
4.5. Code of Conduct for Judges ........................................................................................ 9
5. Code of Ethics and Code of Conduct ........................................................................9
5.1. Purpose of Code of Ethics and Conduct ..................................................................... 10
5.2. Key Elements ........................................................................................................... 11
5.3. Importance of Code of Ethics.................................................................................... 11
5.4. Limitations of Code of Ethics .................................................................................... 12
5.5. First Initiative for Code of Ethics- May 1997 .............................................................. 12
5.6. Second ARC Recommendations with respect to Code of Conduct and Code of Ethics.. 13
6. Strengthening of Ethical and Moral Values in Governance .................................... 14
6.1. General Strategies to Strengthen ethical and Moral Values........................................ 14
6.2. Specific Strategies to be Considered ......................................................................... 14
7. Way Forward ....................................................................................................... 15
8. Previous Years UPSC GS Mains Questions ............................................................. 15
9. Previous Years UPSC Mains Questions: Case Studies ............................................. 15
10. Previous Years Vision IAS GS Mains Questions .................................................... 16
11. Previous Years Vision IAS GS Mains Questions: Case Studies ............................... 28

Copyright © by Vision IAS


All rights are reserved. No part of this document may be reproduced, stored in a retrieval
system or transmitted in any form or by any means, electronic, mechanical, photocopying,
recording or otherwise, without prior permission of Vision IAS

DELHI JAIPUR PUNE HYDERABAD AHMEDABAD LUCKNOW CHANDIGARH


1 www.visionias.in # 8468022022 ©Vision IAS
https://t.me/UPSC_PDF Download From > https://upscpdf.com https://t.me/UPSC_PDF

Note- Student Notes:


This document shall cover the following sections from the syllabus-
• Ethics in Public Administration: Status and Problems; Ethical Concerns and Dilemmas in
Government and Private Institutions
• Codes of Ethics, Codes of Conduct, Strengthening of Ethical and Moral Values in
Governance.

1. Governance, Good Governance and Ethical Governance


Governance is the exercise of economic, political and administrative authority to manage a
country’s affairs at all levels. Good governance is perhaps the single most important factor in
eradicating poverty and promoting development.
Without good governance, no number of developmental schemes can bring improvements in
the quality of life of the citizens. On the contrary, if the power of the state is abused, or
exercised in weak or improper ways, those with the least power in the society – the poor- are
most likely to suffer.
Though, good governance is a dynamic concept, but it is largely said to have the elements of
efficiency, economy, effectiveness, equity, transparency, accountability etc. which are essential
for promoting prosperity of all.

Ethical governance, on the other hand, is a step ahead of good governance in that it seeks to
realize certain universal desirable values, and not merely virtues of administrative efficiency. For
example, the Government of India Act 1858 was also called as “Act for Good Government of
India”, but it was intended for the benefit of the British at the cost of Indians. Therefore, what is
‘good’ keeps on changing with time and space. Hence, it is ethical governance that is needed.
Key elements of ethical governance
Ethical governance means governance based on certain value premise, which is also “good”. For
example, probity, integrity, compassion, empathy, responsibility, social justice etc. without
which ethical issues can’t be upheld.
Elimination of corruption is not only a moral imperative but an economic necessity for a nation
aspiring to catch up with the rest of the world. The most important elements of ethical

DELHI JAIPUR PUNE HYDERABAD AHMEDABAD LUCKNOW CHANDIGARH


2 www.visionias.in # 8468022022 ©Vision IAS
Google it:- https://upscpdf.com
https://t.me/UPSC_PDF Download From > https://upscpdf.com https://t.me/UPSC_PDF

governance to eliminate corruption and reduce bureaucratic delays should be rule of law, Student Notes:
probity, and responsibility.
• Probity would ensure that the sole purpose of administration is public interest, thereby
devoid of any wrongdoing.
• Responsibility, not merely accountability, ensures the inculcation of internal accountability
for every act of omission or commission in the form of judgement based on one’s
conscience. If this is attained then there would be no question of corruption.
• Rule of law checks arbitrariness in governance, thereby reducing chances of misusing
discretion.

2. Status of Ethical Standards in Indian administration


The Indian civil services suffer from strange paradoxes-
• Poor self-actualization: Rigid adherence to procedure combines with a ready susceptibility
to personal pressure and intervention. While a bureaucrat may give the appearance of
being preoccupied with correctness and propriety, in practice he may be committing
endless irregularities and improprieties.
• One size fits all approach: An apparent pursuit of the uniform application of absolute
justice may contain glaring anomalies. It is a curious reflection on their attitudes and
thinking that Indian bureaucrats are willing to tolerate such contradictions between theory
and practice.

2.1. Ethical issues in Indian administration


• Excess of personal authority or rank position: Intentionally, officials make actions that are
out of their position responsibilities and rights that, finally makes damage to the interests
of state or certain citizens.
• Negligence: A public official either does not perform his professional responsibilities or
performs them in a delinquent manner, causing damage to the state or community. This is
mostly because of the lack of interest that one has in one’s duties and responsibilities
• Bribery: Bribery and corruption have come to become an acceptable part of the society, as
a necessary evil greasing the wheels of the economy
• Complacency replaces the hard work- Although there is a core of exceptionally
hardworking, dedicated and conscientious officers, they stand today overwhelmingly
outnumbered by the complacent, who are obsessed with status, rank and emoluments and
addicted to habits of personal luxury and indolence
• Psychology of evasion- When confronted with a difficult decision, the Indian bureaucrat
seldom makes any attempt to tackle the problem with initiative and imagination. Instead,
he will refer the matter to another department or make a series of unnecessary references
to subordinates to gain time.
• Patronization: The post-retirement assignment of senior officers to Regulatory bodies and
other important posts is largely done on patronage with no set guidelines.
• Administrative Secrecy- Secrecy is the hallmark of bureaucracy. In the name of public
interest private interests are served while maintaining secrecy. Transparency therefore is
one of the most vital virtue of Ethical Governance
• Nepotism- The practice of nepotism (the appointment of relations and/or friends to public
positions, thereby ignoring the merit principle), may lead to the downgrading of the quality
of the public service.
• Lack of compassion: Indifference towards the feelings or the convenience of individuals and
by an obsession with the binding and inflexible authority of departmental decisions,
precedents, arrangements or forms, regardless of how badly or with what injustice they
work in individual cases.

DELHI JAIPUR PUNE HYDERABAD AHMEDABAD LUCKNOW CHANDIGARH


3 www.visionias.in # 8468022022 ©Vision IAS
Google it:- https://upscpdf.com
https://t.me/UPSC_PDF Download From > https://upscpdf.com https://t.me/UPSC_PDF

• Abuse of personal authority or rank position for private benefit: Intentionally, public Student Notes:
official may use the authority and facilities exercisable by virtue of holding a position (and
provided to make that exercise of power more effective), in the guise of public service for
personal interest. It appears when:
o Leaking official information- It is usually of sensitive nature and disclosure of such
information can lead to chaos, corrupt practices or, for some individuals, improper
monetary gains. Leaking official information at a date prior to the public announcement
thereof is a violation of procedural prescriptions
o Performing administrative actions while ignoring the facts and without a just motive;
o Use of public technical means, cars, communications, premises in a public official’s
private interests;
o Inactivity causing damages for community

2.2. Ethical dilemmas in Indian administration


Public servants find themselves in dilemmas that include conflict between:
• Between different values of public administration – such as efficiency v/s accountability
• Aspects of the code of conduct- accepting rewards or gifts for performance of duty
• Personal values v/s those of senior or a governmental directive
• Professional ethics v/s following an unjustified order by a supervisor/authority
• Blurred or competing accountabilities- such as towards department or society

The set of fundamental principles or criteria that integrate the process of dealing with ethical
dilemmas in public administration are:
• Democratic accountability of administration,
• The rule of law and the principle of legality,
• Professional integrity; and
• Responsiveness to civil society.
This can be described as the ALIR (Accountability, Legality, Integrity, Responsiveness) model of
imperatives of ethical reasoning in public administration.

DELHI JAIPUR PUNE HYDERABAD AHMEDABAD LUCKNOW CHANDIGARH


4 www.visionias.in # 8468022022 ©Vision IAS
Google it:- https://upscpdf.com
https://t.me/UPSC_PDF Download From > https://upscpdf.com https://t.me/UPSC_PDF

3. Ethical Concerns and Dilemmas in Private/Business Student Notes:

Institutions
Business ethics is the predominant source of guidance in Private/business institutions. The
philosophy may vary from organization to organization; however, fundamentals remain the
same. Individuals often find themselves in a situation of conflicting desires. Sometimes they
have to choose amongst two rights or wrongs. For example, whether to leak information which
is harmful to public interest but serves the interest of the private corporation? Likewise, other
general Ethical issues in Private sector are discussed below:

3.1. Ethical Issues of Private Sector in General


• Favoritism, Nepotism and partisanship: Conflict of interest in appointments- especially in
family run companies- to positions such as board of directors.
• Integrity of the audit process- Companies have been found to fudge their balance sheets. It
is the job of auditors to conduct proper audit and flag any violations, which they have failed
to do in many instances. For example DHFL case. AT times even third party auditors failed to
highlight incongruence in the balance sheet.
• Insider trading and manipulation of share prices: Excessive competition in the market
often forces people for unethical practices. It is also when personal interests take
precedence over the organizational and the share-holders interest. Companies poach such
employee for insider information. This may have debilitating results.
• Cartelisation and manipulation of markets: Big conglomerates often to displace the new
entrants form cartels having monopoly over the market. Such cartels have been seen in
drug, communication, cement etc. These cartels manipulate the market due to their hefty
share. Price manipulation is one big impact of the same.
• Lobbying with the government for favourable policies: Lobbying in some countries is
ethical. In India there is no law which defines Lobbying. Defence contracts are often heated
topics for the lobbying angles involved in the same. Lobbying in a regulated manner is often
recommended by experts.
• Oligopoly or Monopoly: For example, Communication sector is suffering from monopoly of
few big conglomerates. The price wars have displaced many players from the market.
Burning money to replace competitors is an unethical practice and competition commission
in India regulates it.

3.2. Ethical Issues of Employers


• Favouritism: This means that the employer may favour a particular person with regard to
promotions and bonuses and evidently neglect other eligible employees. This conduct is
considered highly unethical on the part of the employer.
• Sexual harassment at work place: is not legal/ethical/moral whether in the workplace or
out of it. Harassing an employee sexually, or refraining from taking action against those who
are involved in such offences, is strictly forbidden. Despite having laws, most of the
companies do not have internal committees to address the complaints.
• Hire and Fire culture: Terminating an employee without any notice. In some cases, for
reasons like budget management, companies opt for mass retrenchment to reduce the
number of employees. Such steps should be undertaken after prior indications and notice
of at least a month or two, so that the person can find another job.
• Gender neutrality: To develop gender neutral architecture in organizations is another
ethical issue. Organizations are often found to be insensitive and partial. Some
organisations hire less women due to maternity leaves and other gender associated
prejudices. Various studies have found that despite having better leadership skills women
face glass ceilings within the organization. Women are considered to be less productive.

DELHI JAIPUR PUNE HYDERABAD AHMEDABAD LUCKNOW CHANDIGARH


5 www.visionias.in # 8468022022 ©Vision IAS
Google it:- https://upscpdf.com
https://t.me/UPSC_PDF Download From > https://upscpdf.com https://t.me/UPSC_PDF

• Accessibility: Organizational infrastructure should be accessible to all, including disabled. Student Notes:
Most of the organizations lack sensitivity and awareness about disabled.

3.3. Ethical Issues of Employees


The employees in an organization may fall prey to many unethical practices, knowingly and
unknowingly. Some of the important ethical issues that employee may face are as following:
Ethical Issues Explanation
Work Ethics • Any and every organization has some work culture and employees there follow
some work ethics.
• Some of the important values of which are; Punctuality, proactive communication,
Documentation, Upholding the dignity of fellow employee and regularity. Any
breach of these values is an ethical concern.
• E.g. taking excessive leave beyond the allowed number is a breach of ethical code. It
not only leads to losses for the company but also brings the employee a bad name.
Breach of • Accepting terms and conditions are usually done as a part of joining procedure.
rules and • Violating any of these rules may lead to undesirable issues between the company
regulations and the employee.
of the • E.g. Failing to maintain the privacy policy of the company is sort of breaking of rules
company
Misuse of • Utilisation of organizational resources to fulfill personal needs is unethical.
resources • E.g. Making unnecessary phone calls at the company’s cost.
• E.g. Some employees pocket tools and stationery such as staplers, pins, papers etc.
to use them at home
Using office • Because employees tend to spend so much of their weekday hours on the job, they
hours for often are tempted to conduct personal business on company time.
private work • E.g. This can include setting up doctor's appointments on company phone lines etc.
Working for • Companies usually prohibit employees to work in more than one organization,
multiple especially in a competing company where confidential information may be used.
organizations • This also questions the loyalty one has for employer.
Taking Credit • Taking the credit of success and disowning the blame for failure is a common
for Others' unethical practice that leaders do.
Work • E.g. Prof. Satish Dhawan (then chairman of ISRO) demonstrated this quality while
accepting failure for first Satellite Launch Vehicle mission by a team led by Dr. APJ
Abdul Kalam. However, such ethical traits have been found missing in many
organisations.
Harassing • Employees often don't know what to do if they see one of their co-workers
Behaviour harassing another employee either mentally, sexually or physically.
• Employees may worry for their jobs if they attempt to report a superior for
harassment. They may fret that they'll be labelled a troublemaker if they report co-
workers who display inappropriate behaviour toward other employees.

4. Existing Framework for Ethical Standards in India


India inherited the colonial bureaucratic structure and with time functional and structural
unethical behavior started to appear. Important ethical standards were therefore created to
improve the quality of administration. Some of the key Ethical standards in India are as follows:

4.1. Central Civil Services (Conduct) Rules 1964


Conduct rules governing the conduct of India’s bureaucracy largely ensure that officers do not
divulge confidential information or criticise government policy, and remain politically neutral.
• Officers from the administrative, police and forest services are governed by the All India
Service (Conduct) Rules, 1968.
• Officers from the Indian Revenue Service, Indian Information Service, Indian Customs &
Central Excise Service, Indian Postal Service, Indian Audit & Accounts Service, etc. are
governed by the Central Civil Service (Conduct) Rules, 1964.

DELHI JAIPUR PUNE HYDERABAD AHMEDABAD LUCKNOW CHANDIGARH


6 www.visionias.in # 8468022022 ©Vision IAS
Google it:- https://upscpdf.com
https://t.me/UPSC_PDF Download From > https://upscpdf.com https://t.me/UPSC_PDF

• Some services like the Indian Railway Service have their own set of conduct rules, though Student Notes:
effectively they are all similar.
Some important and widely used/misused rules are:
• Rule 3(1), “every member of the service shall at all times maintain absolute integrity and
devotion to duty and shall do nothing which is unbecoming of a member of the service”.
• The rule is elaborated in a sub-section to say that every officer must maintain the following
– high ethical standards, integrity and honesty, political neutrality, promoting of the
principles of merit, fairness and impartiality in the discharge of duties, accountability and
transparency, responsiveness to the public, particularly to the weaker sections, and
courtesy and good behaviour with the public.
• According to Rule 7 of the AIS Conduct Rules, no member of the service can criticize any
policy or action of the central or state government on a radio broadcast, communication
over any public media, in any document, in any communication to the press, or in any
public utterance.
• Further, they are prohibited from saying anything “which has the effect of an adverse
criticism of any current or recent policy or action of the central government or a state
government” or “which is capable of embarrassing the relations between the central
government and any state government”.
4.1.1. Critique of the Rules
• No mention of values or Code of ethics: While the Central Government has issued conduct
rules for government employees known as Central Civil Services (Conduct) Rules 1964, it
does not lay down the values, which civil servants should follow or a code of ethics.
• Nature of the Rules: The rules are more in the nature of “do’s” and “don’ts”. The Conduct
Rules cover matters such as property transactions, acceptance of gifts, joining non-political
organizations and a host of other issues covering almost every activity, which a normal
individual undertakes.
• Restrictive in Nature: The rules are highly restrictive, seriously curtailing freedom of
operation of a government employee, couched in vague language and sometimes
impractical to follow.
• Primitive service rules for government employees: Shah Faesal, who was suspended for
tweeting against rape culture in the subcontinent, was accused of not “maintaining
absolute honesty and integrity in discharge of official duty and thus acted in a manner
unbecoming of public servant”.
• Golam Mohiuddin vs State of West Bengal, AIT 1964: In this case Calcutta high highlighted
the ambiguity inherent in the rules, leading to subjective satisfaction of the government or
disciplinary authority.
Recent disciplinary actions against bureaucrats under the Central Services Conduct Rules:
• The Department of Telecommunication (DoT) suspended Ashish Joshi, a 1992-batch Indian
Post and Telecommunication Accounts and Finance Service officer, for reportedly misusing
his official letterhead to file a complaint against Delhi MLA Kapil Mishra for uploading an
incendiary video online.
• The central government asked the West Bengal government to take action against five
officers from the Indian Police Service (IPS) for sharing the stage with Chief Minister
Mamata Banerjee, during her protest against the CBI. They allegedly violated conduct rules,
which prohibit officers from taking part in political protests.

4.2. Draft Public Service Bill 2006


In 2006 the Department of Personnel drafted a Public Service Bill which enumerated
fundamental values of Public Services, a Code of Ethics, a Management Code etc. with the
object of developing public services as a professional, politically neutral, merit based and

DELHI JAIPUR PUNE HYDERABAD AHMEDABAD LUCKNOW CHANDIGARH


7 www.visionias.in # 8468022022 ©Vision IAS
Google it:- https://upscpdf.com
https://t.me/UPSC_PDF Download From > https://upscpdf.com https://t.me/UPSC_PDF

accountable civil service. The main values by which the Public Servants shall be guided are as Student Notes:
follows:
• Allegiance to the Constitution and the law, democracy, nationalism, sovereignty, integrity of
India and the security of the nation;
• Function in apolitical manner;
• Act objectively, impartially, honestly, equitably, and in a fair and just manner;
• Act with integrity and in a courteous and just manner;
• Establish high standards, and ensure quality service, effective working and prompt decision
making;
• Be accountable for the decisions;
• Establish merit as the fundamental principle in employment, promotion and placements;
• Discharge functions with due regard to diversity of the nation/community and religion but
without discrimination of caste, community, religion, gender or class and duly protecting
the interest of poor, underprivileged and weaker sections;
• Provide honest, impartial and frank advice to political executive;
• Ensure that public money is used with utmost economy and care;
One problem with the draft bill was that it intended to fulfil too many objectives. Apart from
values and ethics, the Bill envisaged laying down principles of management of public services,
principles which should govern appointment to public services, performance indicators for
public services etc. With such wide ranging and diverse coverage of matters relating to service
matters, it is difficult to reach consensus and secure legislative approval. As such, the bill lapsed
for want of both political will and societal will.

4.3. Code of Conduct for Legislators


The Committee on Ethics of the Rajya Sabha and the Lok Sabha have drafted some guidelines
for the legislators of both the houses of the Parliament. Some of the common principles
include-
• Members must not do anything that brings disrepute to the Parliament and affects their
credibility.
• Members must utilize their position as Members of Parliament to advance general well-
being of the people.
• If Members are in possession of confidential information owing to their being Members of
Parliament or Members of Parliamentary Committees, they should not disclose such
information for advancing their personal interests.
• Members must keep uppermost in their mind the fundamental duties listed in Part-IV of
the Constitution.
• Members should maintain high standards of morality, dignity, decency and values in
public life.

4.4. Code of Conduct for Ministers


The Ministers of both the Union and State Governments have to follow ethical conduct based
on the following-
Before taking office-
According to the Representation of the People Act, 1951 a person before taking office as a
Minister, shall:
• disclose to the Prime Minister, or the Chief Minister, as the case may be, details of the
assets and liabilities, and of business interests, of himself and of members of his family.
• sever all connections, short of divesting himself of the ownership, with the conduct and
management of any business in which he was interested before his appointment as
Minister.

DELHI JAIPUR PUNE HYDERABAD AHMEDABAD LUCKNOW CHANDIGARH


8 www.visionias.in # 8468022022 ©Vision IAS
Google it:- https://upscpdf.com
https://t.me/UPSC_PDF Download From > https://upscpdf.com https://t.me/UPSC_PDF

After taking office- Student Notes:


So long as he remains in office, the Minister shall:
• furnish annually by the 31st March to the Prime Minister, or the Chief Minister, as the case
may be, a declaration regarding his assets and liabilities;
• refrain from buying from or selling to, the Government any immovable property except
where such property is compulsorily acquired by the Government in usual course;
• refrain from starting, or joining, any business;
• report the matter to the Prime Minister, or the Chief Minister as the case may be, if any
member of his family sets up, or joins in the conduct and management of, any other
business.
Apart from this, a minister should not accept valuable gifts except from close relatives, and he
or members of his family should not accept any gifts at all from any person with whom he may
have official dealings.

4.5. Code of Conduct for Judges


The Supreme Court of India in its Full Court Meeting held on May 7, 1997 unanimously adopted
a charter called the ‘Restatement of Values of Judicial Life’, generally known as the Code of
Conduct for judges. It includes the following-
• No member of his family, who is a member of the Bar, shall be permitted to use the
residence in which the Judge actually resides or other facilities for professional work.
• A Judge should practice a degree of aloofness consistent with the dignity of his office.
• A Judge shall not hear and decide a matter in which a member of his family, a close
relation or a friend is concerned.
• A Judge shall not enter into public debate or express his views in public on political
matters or on matters that are pending or are likely to arise for judicial determination.
• A Judge is expected to let his judgments speak for themselves. He shall not give interviews
to the media.
• A Judge shall not accept gifts or hospitality except from his family, close relations and
friends.
• A Judge shall not hear and decide a matter in which a company in which he holds shares is
concerned unless he has disclosed his interest and no objection to his hearing and deciding
the matter is raised.
• A Judge shall not speculate in shares, stocks or the like.

5. Code of Ethics and Code of Conduct


A code of ethics is an assembly of institutional guidelines used to reduce ethical vagueness.
Within an organization and serve as a means of reinforcing ethical conduct.

DELHI JAIPUR PUNE HYDERABAD AHMEDABAD LUCKNOW CHANDIGARH


9 www.visionias.in # 8468022022 ©Vision IAS
Google it:- https://upscpdf.com
https://t.me/UPSC_PDF Download From > https://upscpdf.com https://t.me/UPSC_PDF

Student Notes:

5.1. Purpose of Code of Ethics and Conduct


The codes are developed based not only on past organizational or individual experience, but
also based on actions that the organization wishes to prevent from ever occurring
Three main purposes that the codes of ethics serve, which are as following:
(a) Codes assure people outside the profession or organization that they can expect a degree
of uniformity as relates to expectation of performance and moral conduct from employees
of the profession or organization.
(b) Codes assure individuals within the organization or profession that they can rely upon
colleagues within the organization and profession to maintain a level of standards in
exchange for that individual conducting himself in adherence to the same principles upon
which the others are held.
(c) Codes serve as a notice that people outside of the organization or profession are not bound
by the code and, perhaps may be seen as adhering to lower standards pertaining to ethics.

DELHI JAIPUR PUNE HYDERABAD AHMEDABAD LUCKNOW CHANDIGARH


10 www.visionias.in # 8468022022 ©Vision IAS
Google it:- https://upscpdf.com
https://t.me/UPSC_PDF Download From > https://upscpdf.com https://t.me/UPSC_PDF

Student Notes:

5.2. Key Elements


The key elements of such a
Code are:
• Personal Responsibility
• Compliance with the Law
• Relations with the Public
• Limitations on the
Acceptance of Gifts,
Rewards, Hospitality and
Discounts
• Avoiding Conflicts of
Interest
• Limitations on Political
Activities
• Conduct in Money Matters
• Confidentiality and use of Official Information
• Use of Official Property and Services
• Private Purchases of Government Property by Employees
• Work Environment
While writing the code of ethics following things can be kept in mind;
(a) It should be written to a general audience
(b) It should be attainable
(c) It should be written in clear, yet specific language
(d) Follow a logical order

5.3. Importance of Code of Ethics


(a) Organization displays that it is willing to take responsibility for organizational ethics
(b) Serves as decision making guide and waiver of liability
(c) Reduces cognitive dissonance and stress associated with some decision-making processes.

DELHI JAIPUR PUNE HYDERABAD AHMEDABAD LUCKNOW CHANDIGARH


11 www.visionias.in # 8468022022 ©Vision IAS
Google it:- https://upscpdf.com
https://t.me/UPSC_PDF Download From > https://upscpdf.com https://t.me/UPSC_PDF

Student Notes:

5.4. Limitations of Code of Ethics


Code of Ethics, no matter how well intentioned may generate several limitations, some of which
are as following;
(a) A code of ethics cannot provide guidance for every individual in every situation.
(b) Ethical code will through guidelines helps ensures public trust but cannot be enforced
through criminal or civil code. It serves little to establish code of ethics that cannot be
defended in courts.
(c) Limited to the imagination of individuals. Behaviour cannot be guided by set of rules, it
must come from within the individuals and the organisations.
(d) Public service is a profession that must be governed by an inherent set of rules that do not
come from reading a specific code.

5.5. First Initiative for Code of Ethics- May 1997


The Department of Administrative Reforms of the Government of India had prepared a Code
of Ethics for public services, as part of an Action Plan for an Effective and Responsive
Government, which was presented in a conference of Chief Ministers presided by the Prime
Minister held in May 1997. The objective of the Code was to prescribe standards of integrity
and conduct that are to apply to public services. The salient features of the code are as follows:
• Assist the government: The public services should assist the government in formulating
and implementing policies and administering public services in the most effective way.
• To act with probity and integrity and in Public interest: Employees in public services should
uphold the rule of law and respect for human rights and act solely in public interest. They
must maintain the highest standards of probity and integrity.

DELHI JAIPUR PUNE HYDERABAD AHMEDABAD LUCKNOW CHANDIGARH


12 www.visionias.in # 8468022022 ©Vision IAS
Google it:- https://upscpdf.com
https://t.me/UPSC_PDF Download From > https://upscpdf.com https://t.me/UPSC_PDF

• Building Public Trust: They should conduct themselves in such manner that the public feels Student Notes:
that the decisions taken, or recommendations made by them are objective and transparent
and are not calculated to promote improper gains for the political party in power, for
themselves, or for any third party.
• Cooperation with the government: They should not seek to frustrate or undermine the
policies, decisions and action taken in public interest by Government by declining or
abstaining from action.
• When to approach the higher authority: Where an employee in public service has
reasonable grounds to believe that he or she is being required by superior authority to act
in a manner, which is illegal or against prescribed rules and regulations, he should decline to
implement the instructions. He will have the right to bring the fact to the notice of superior
authority.
• Conflict of Interest: Employees in public service should refrain from decisions; i) which are
calculated to benefit any particular person or party at the expense of the public interest; ii)
shall disclose any clash of interest when there is conflict between public interest and private
interest.
• Independence, Dignity and Impartiality: They should maintain their independence and
dignity and impartiality by not approaching politicians and outsiders in respect of service
matters or private benefits, and exercise peer pressure to dissuade those within their own
cadre who do so and to set in motion disciplinary proceedings against such persons.
• Accountability to Citizens:
o Employees in public services should be accessible to the people and practice
accountability to them in terms of quality of service, timeliness, courtesy, people
orientation, and readiness to encourage participation and form partnership with
citizen groups for responsive government.
o They should be consistent, equitable and honest in their treatment of the members of
the public.
o They should accept obligation to recognize and enforce citizen’s right for speedy
redressal of their grievance.
• Financial prudence: They should have concern for public assets and funds, avoid wastage
and extravagance and ensure effective and efficient use of public money within their
control.
• Non- abuse of official position: Employees in public services have a responsibility to take
decisions on merits, as they are in a position of trust, they must not use their official
position to influence any person to enter into financial or other arrangements with them or
anyone else.
The code also deals with issues such as: public comment, release of official information,
integrative role of public services and continuous improvement through professionalism and
teamwork.

5.6. Second ARC Recommendations with respect to Code of


Conduct and Code of Ethics
• Civil Service Values which all public servants should aspire, should be defined and made
applicable to all tiers of government and parastatal organizations.
• Any transgression of these values should be treated as misconduct, inviting punishment”
• In order to create a regime under which quick disciplinary action can be taken against
delinquent Government servants, the ARC has recommended deletion of Article 311 of the
Constitution, with a proviso that legislation under article 309 be made to protect public
servants against arbitrary action.
• The Commission has also suggested certain measures to protect honest Civil Servants
against malicious complaints.

DELHI JAIPUR PUNE HYDERABAD AHMEDABAD LUCKNOW CHANDIGARH


13 www.visionias.in # 8468022022 ©Vision IAS
Google it:- https://upscpdf.com
https://t.me/UPSC_PDF Download From > https://upscpdf.com https://t.me/UPSC_PDF

• The ARC in its 10th Report on Personnel Administration has re-emphasized the need for Student Notes:
prescribing Civil Service Values and laying down a Code of Ethics.
• The Code of Ethics should include: integrity, impartiality, commitment to public service,
open accountability, devotion to duty and exemplary behaviour.

6. Strengthening of Ethical and Moral Values in


Governance
The discontent with respect to bureaucracy has shown a sharp rise in recent past. There have
been protests to oppose the glaring fall in the ethical conduct of the public servants. Not
surprisingly, there is a great deal of skepticism concerning the morality of governmental action
and the ethics of its officials, and unless public trust and confidence are restored, moral
government will remain a distant dream.
That restoration will depend upon how public officials are able to demonstrate the highest
standards of ethics and responsibility in the use of the power entrusted to them. Towards this,
'they (the elected as well as appointed public servants) will need to reinforce their sense of
mission to serve the public, by acknowledging the place of administrative theology, and by
acquiring the necessary education and training in public service ethics and responsibility

6.1. General Strategies to Strengthen ethical and Moral Values


• There is a need to resurrect the concept of service and vocation so that a public
administrator can rise above individualistic leanings and become a true 'public servant'.
Exemplary action against improprieties and malfeasance committed by either the elected
or appointed officials would have a salutary effect on the morale of all concerned, including
the public.
• The exercise of discretion should serve the public interest. A public official should push
back bounds on rationality so that deliberation may take place, provide truthfulness in the
discharge of official responsibilities, demonstrate procedural respect, exercise restraints
on the means chosen to accomplish organizational ends.
• Education and training in virtue and morality are prerequisites to holding a public office
• By demonstrating the highest standards of personal integrity, honesty, fairness and
justice, which are the key ingredients of a moral government, public officials can inspire
public confidence and trust.
• Public servants should be aware of the conduct expected of them, not only by the state
which employs them, but also by the public. A set of principles and guidelines with the
force of law should be developed, and an office be established for enforcement and
resolution of conflicts.

6.2. Specific Strategies to be Considered


• Effective laws: Which require civil servants to give reasons for their official decisions.
• New Management approaches: To encourage all public officials and civil servants to deal
positively with corruption and unethical practice when encountered.
• Strengthening the Whistle Blower Protection Regime: Whistle-blower’ protection law to
protect appropriate 'public interest disclosures' of wrongdoing by officials.
• Ethics audits: To identify risks to the integrity of the most important processes.
• New Human Resource Management strategies (which link, for example, ethical
performance with entry and advancement, and ethical ‘under-performance’ with
disciplinary processes), merit-based promotion and recruitment, anti-discrimination
protections.
• Training and development in the content and rationale of Ethics Codes, the application of
ethical management principles, the proper use of official power, and the requirements of

DELHI JAIPUR PUNE HYDERABAD AHMEDABAD LUCKNOW CHANDIGARH


14 www.visionias.in # 8468022022 ©Vision IAS
Google it:- https://upscpdf.com
https://t.me/UPSC_PDF Download From > https://upscpdf.com https://t.me/UPSC_PDF

professional responsibility. Effective external and internal complaint and redress Student Notes:
procedures.
• Developing administrative practices and processes which promote ethical values and
integrity: The system should promote role models and incorporate the process of
disseminating best practices and exemplary actions of civil servants to the entire
bureaucracy. Also, performance assessment systems have to be renewed in line with
growing scope of innovation in civil services.
• New techniques need to be undertaken to institutionalize ethically competent decision
making, disinterested advice to Government, and, ultimately, an ‘ethical culture’ which
supports professional responsibility, self-discipline, and support for the rule of law;
• Second ARC recommendation: In Its wide-ranging recommendations, it has suggested
partial state funding of elections; tightening of anti-defection law and code of ethics for
ministers, legislatures, judiciary and civil servants.
• To check corruption: Second ARC proposed tightening the provision of Prevention of
Corruption Act, making corrupt public servants liable for paying damages, confiscation of
property illegally acquired and speedy trials.

7. Way Forward
Moral administration does not mean that its officials exhibit only the negative obligations
such as to do no harm, to avoid injury or to keep out of trouble. On the contrary, the notion of
public sector ethics suggests that administrators actively undertake actions that are socially just
and moral. Only by actively pursuing the goals of social justice, equity and human dignity can
the officials and the state be moral and just.

8. Previous Years UPSC GS Mains Questions


1. What do you understand by the terms ‘governance’, ‘good governance’ and ‘ethical
governance’? (2016)
2. Discuss the Public Services Code as recommended by the 2nd Administrative Reforms
Commission. (2016)
3. Distinguish between “Code of ethics” and “Code of conduct” with suitable examples. (2018)
4. Explain the process of resolving ethical dilemmas in Public Administration. (2018)
5. Suppose the Government of India is thinking of constructing a dam in a mountain valley
bond by forests and inhabited by ethnic communities. What rational policy should it resort
to in dealing with unforeseen contingencies. (2018)

9. Previous Years UPSC Mains Questions: Case Studies


1. Suppose one of your close friends, who is also aspiring for civil services, comes to you for
discussing some of the issues related to ethical conduct in public service. He raises the
following points:
(i) In the present times, when unethical environment is quite prevalent, individual
attempts to stick to ethical principles may cause a lot of problems in one’s career. It
may also cause hardship to the family members as well as risk to one’s life. Why should
we not be pragmatic and follow the path of least resistance, and be happy with doing
whatever good we can?
(ii) When so many people are adopting wrong means and are grossly harming the system,
what difference would it make if only a small minority tries to be ethical? They are
going to be rather ineffective and are bound to get frustrated.
(iii) If we become fussy about ethical considerations, will it not hamper the economic
progress of our country? After all, in the present age of high competition, we cannot
afford to be left behind in the race of development.
(iv) It is understandable that we should not get involved in grossly unethical practices, but
giving and accepting small gratifications and doing small favours increases everybody’s

DELHI JAIPUR PUNE HYDERABAD AHMEDABAD LUCKNOW CHANDIGARH


15 www.visionias.in # 8468022022 ©Vision IAS
Google it:- https://upscpdf.com
https://t.me/UPSC_PDF Download From > https://upscpdf.com https://t.me/UPSC_PDF

motivation. It also makes the system more efficient. What is wrong in adopting such Student Notes:
practices?
Critically analyze the above viewpoints. On the basis of this analysis, what will be your
advice to your friend?

10. Previous Years Vision IAS GS Mains Questions


1. The universal adoption of common good approach poses the ethical dilemma of
putting collective interests over and above the individual interests. Discuss with
examples.
Approach:
• Start with defining the concept of common good with examples.
• Discuss the ethical dilemma before the individual in dealing with challenges of
common good. Illustrate with some examples.
• Conclude.
Answer:
An individual entity- a person, a group, a community, a nation- may choose to act in a
manner which brings it maximum good. This good usually comes at a cost to another
individual entity. There are two ways to minimize the cost to other entities. Either all
act in a selfish manner and compete with each other to maximise their own benefits,
with no consideration of the cost to other; or, everyone may act in a harmonious
manner so that benefits are maximized for all together rather than one entity. This way,
the cost to all is borne by all, and hence everyone strives to minimize it while
maximizing benefits. However, the ethical dilemma here is what if the other entity
works to maximise its own benefits, rather than everyone’s? Hence, it becomes clear
that universal adoption of common good approach poses the dilemma- Am I/Are we
being left behind? Are others committed to common good just like me/us? Will
pursuing my self-interest bring more and quicker happiness to me, and so on.
According to John Rawls , the common good refers to "certain general conditions that
are equally to everyone's advantage." It is either what is shared and beneficial for all or
most members of a given community. Examples- Universal public health care system,
an effective system of public safety, an unpolluted natural environment etc.
Because such systems, institutions, and environments have a powerful impact on the
well-being of members of a society, virtually every social problem is linked to how well
these systems and institutions function.
Examples:
• Keeping the neighbourhood clean would require efforts of all households and if one
of them does not properly dispose it’s waste, others will be demotivated to keep it
clean.
• One set of people- forest dwellers- being made to bear the entire burden of
relocation for mining projects without getting any benefit in return.
• Richer people disproportionately contributing to pollution through their lifestyle
lessens the motivation of others to cut down on their emissions.
• Developed countries pursuing their interests in WTO at the cost of developing and
poor countries lessens the incentive to respect and abide by global rules, treaties.
Challenges of ensuring Common Good
No one can be easily excluded from the common good. This leads to following
challenges:

DELHI JAIPUR PUNE HYDERABAD AHMEDABAD LUCKNOW CHANDIGARH


16 www.visionias.in # 8468022022 ©Vision IAS
Google it:- https://upscpdf.com
https://t.me/UPSC_PDF Download From > https://upscpdf.com https://t.me/UPSC_PDF

• As Accountability can’t be fixed on a particular individual, everyone expects others Student Notes:
to contribute while he himself focuses on his individual interest.
• In the face of such pluralism, efforts to bring about the common good may lead to
adopting or promoting the views of some, while excluding others.
• Individuals can become "free riders" by taking the benefits the common good
provides while refusing to do their part.
• In the individualistic culture it is difficult to convince people that they should
sacrifice some of their freedom, personal goals, and self-interest, for the "common
good".
• For the sake of common good, particular individuals or groups may have to bear
costs higher than others.
Despite these issues, appeals to the common good ought not to be dismissed. For they
urge us to reflect on broad questions concerning the kind of society we want to become
and how we achieve that society. Through dialogue and discussion, others can be
persuaded to act in a manner which maximises common good.

2. What do you understand by ethical dilemma? Bring out some examples of ethical
dilemma often faced by civil servants.
Approach:
• Define ethical dilemma and briefly explain it.
• With examples, explain ethical dilemma often faced by civil servants.
• Conclude by suggesting ways to deal with ethical dilemmas.
Answer:
An ethical dilemma is a complex situation, that often involves conflict between moral
imperatives. Civil servants hold public positions and utilize resources to deliver public
services. In performing duties, they come across various situations when they have to
deal with equally important set of principles. They may face ethical dilemmas due to
lack of legal/policy clarity on the issues faced by them. Some examples of ethical
dilemmas faced by civil servants are as follows:
• Allocation of limited resources amidst demands by various beneficiaries, especially
in case of unclear guidelines i.e. matters which potentially influence a civil servant’s
ability to work in the public interest and represent all constituents equally and
fairly.
• Economic development versus its cost to the environment, for example,
construction of a dam in a tribal area, which would protect them from floods and
provide electricity but lead to displacement.
• Conflicts between personal life and professional life. For example, maintaining
balance between catering to family needs and working beyond office hours.
• Maintaining balance between secrecy and transparency. While protecting sensitive
information is necessary for security reasons, but at the same time, transparency is
essential to uphold the public servants accountable.
• Dilemmas involving the faithful execution of official duties. For example, the
decision to participate/be a part of say an appointment committee, interview board
etc, when a civil servant has a conflict of interest which prohibits his/her
participation.
• Dilemmas involving acting with integrity, for example, in cases where a civil servant
is offered a legally acceptable gift for doing his/her duty diligently, where there may
be a quid pro quo involved.

DELHI JAIPUR PUNE HYDERABAD AHMEDABAD LUCKNOW CHANDIGARH


17 www.visionias.in # 8468022022 ©Vision IAS
Google it:- https://upscpdf.com
https://t.me/UPSC_PDF Download From > https://upscpdf.com https://t.me/UPSC_PDF

In these situations, public servants should follow certain fundamental principles such as Student Notes:
democratic accountability, respect for the rule of law and the principle of legality,
professional integrity and responsiveness to civil society. Adopting these principles can
integrate the process of dealing with ethical dilemmas in public administration.

3. What is the difference between an acceptable business gift and a bribe? What kind of
norms and structures should an organization have in place to help its employees
avoid ethical dilemmas on such issues and make the right decision?
Answer:
Gift is something of value given without the expectation of return while bribe is
something of value given with the hope of a future influence or benefit. In the context
of contracting with the government, a ‘bribe’ is the offering, giving, receiving or
soliciting something of value for the purpose of influencing the actions of an official in
the discharge of his or her public or legal duties. In general, a ‘gift’ is something given
as a sign of friendship or appreciation. While it is generally accepted that gift giving
may enhance the prospect and image of a company, businesses must tread carefully as
corporate gift giving has several legal, ethical and practical questions attached to it.
Standard management tools are used to control bribery - internal monitoring,
monitoring suppliers, reports to Boards of Directors, use of compliance manuals,
whistle-blowing facilities, signatures of directors, training, periodic compliance, reviews
by managers, employee signatures, internal auditing, disciplinary action and active
communication. But there is a very fine line between business gift giving and bribing,
and it is seldom clear when the line is crossed. This fine line between a gift and a bribe
could be determined by using a 'test of disclosure with comfort'. If the business
manager or the public official can unhesitatingly acknowledge and justify the giving and
receiving of a gift and its size and nature in a public forum without any adverse impact
on the company's work and reputation then and only then it is a gift.
It is recognised that the giving and receiving of business gifts is an integral part of the
way in which some businesses operate. To steer clear of bribery but to maintain a
healthy gift-giving culture which is beneficial to business, companies must have clear
policies related to gift giving and receiving to steer clear of potential problems arising
out of gift-giving.
There are some suggestions for policy-making in this context:
• Workplace gift giving should ideally be prohibited or restricted.
• A clear guideline should be set for type and value of gifts the employees can accept
from outside entities and a policy to return should be there for any gift that falls
outside the set parameters.
• For giving gifts to suppliers and other outside entities, it would be important to
define who can receive a gift, along with ‘how these gifts are selected and
presented’ and also to cognizance of the gift policies of any company that is being
included in the list.
• The receipt or giving of modest gifts may be expensed in the normal way if paid for
by a group company.
• The giving or receipt of more lavish gifts must be approved by the person's
manager.
• The manager should ensure that an appropriate record is maintained.
• In cultures where the refusal of an expensive gift would give offence, such gifts
may be accepted on the basis that they will become the local company's property,
unless the managing director of the local company otherwise determines.
These kinds of norms and structures can help employees to avoid ethical dilemmas on
such issues.

DELHI JAIPUR PUNE HYDERABAD AHMEDABAD LUCKNOW CHANDIGARH


18 www.visionias.in # 8468022022 ©Vision IAS
Google it:- https://upscpdf.com
https://t.me/UPSC_PDF Download From > https://upscpdf.com https://t.me/UPSC_PDF

4. Explaining what an ethical dilemma is, discuss how it reflects not merely a choice Student Notes:
between competing interests and values but also a test of strength of one's character.
Approach:
• Explain briefly the meaning of ethical dilemma.
• Discuss how ethical dilemma tests one’s character.
• Conclude by suggesting ways to resolve an ethical conflict.
Answer:
It is easy to make a choice amongst actions where consequences are unambiguously
right or wrong. However, an ethical dilemma arises when there is ambiguity about
goodness or badness of an act. A situation where one has to take a decision between
moral imperatives, none of which is unambiguously acceptable or preferable is called
an ethical dilemma. It requires a choice between competing sets of principles in a
given, usually undesirable or perplexing situation. A person may find herself/himself in
ethical dilemmas in personal and professional life due to conflict of interest or conflict
between personal values and professional ethics; blurred or competing
accountabilities etc.
Ethical dilemma reflects not merely a choice between competing interests and values
but also a test of strength of one's character as:
• Resolving ethical dilemmas requires a robust framework of integrity. Often people
are required to act in public interest at the expense of personal interest, which
requires dedication and honesty and an ability to resist temptation for personal
gains.
• Similarly courage, humility, prudence, optimism and determination are necessary
character traits as regards resolving ethical dilemmas.
• A strong personal character provides for the ability to identify important issues,
determine priorities and sort out competing values.
• Ethical standards do not by themselves ensure ethical behavior. Personal integrity is
required to ensure ethically compliant behavior. For instance corruption is a direct
manifestation of failure of one’s ethics.
• Laws, rules and ethical standards, while guiding an individual in resolving ethical
dilemmas, are inherently incomplete to enable one to resolve the myriad conflicts
that arise in real life situations.
Ethical dilemmas can be resolved by adopting a justice approach, which holds that
decisions must be based on standards of equity, fairness, and impartiality. A personal
moral compass, fortitude and ability to take responsibility for actions are therefore
necessary to help one when faced with ethical dilemmas.

5. Income inequality, resource mismanagement and health hazards are some of the
negative consequences of globalization. In this context, what are some of the ethical
dilemmas that civil servants face today? How has the approach to handling these
undergone a change?
Approach:
• Explain the key ethical dilemmas that civil servants face due to the changes brought
by globalization.
• Then, talk about the difference in approach to handle the challenges and resolve
the ethical dilemmas.

DELHI JAIPUR PUNE HYDERABAD AHMEDABAD LUCKNOW CHANDIGARH


19 www.visionias.in # 8468022022 ©Vision IAS
Google it:- https://upscpdf.com
https://t.me/UPSC_PDF Download From > https://upscpdf.com https://t.me/UPSC_PDF

Answer: Student Notes:


The process of Globalization is associated with both benefits with increasing pace of
poverty reduction and high growth rates as well as drawbacks such as increasing
inequality and resource mismanagement. Globalization has impacted not just
economies but also politics, societies and cultures across the world. This has brought
new challenges for the civil services in the form of following ethical dilemmas.
Ethical dilemmas in era of Globalization
• In the era of globalisation, the role of the State has changed. Economic
liberalisation has led to – on one hand, the diminishing role of state and on the
other, increasing role of private sector. The basic objective of private enterprises—
maximizing profits—does not always coincide with broader social concerns. Therein
lies the ethical dilemma of civil servants of balancing these two priorities, profits –
for sustaining economic activities and addressing social concerns - as part of the
welfare state.
• Globalization has resulted in the marginalisation of a section of society who have
not been able to take advantage of the economic opportunities provided by
globalization. The ethical dilemma here is the need for showing empathy and
compassion in helping them while at the same time following the rule of law.
• Fast pace of globalisation is mainly driven by the rapid advancement of
communication technology. Civil servants too have had to adopt technology in
public administration even if they personally do not like it. The ethical dilemma
here is between the personal choice of status quo and professional demands for
infusing technology in day-to-day operations.
• Globalization era is also associated with increasing demands for transparency and
accountability. The ethical dilemma which civil servants face is judicious use of
discretionary power while at the same time upholding the principles of
transparency and accountability.
Handling these challenges and ethical dilemmas require a multi-pronged approach for
upholding the foundational values of civil services. Therefore, there has been change in
the approach in the following manner:
• Civil servants are shifting their orientation from being controllers to facilitators and
from being providers to enablers. This way the State can focus more on the social
sector while economic activities are left to the private sector.
• Values such as non-partisanship, empathy and compassion are now considered as
foundational values along with the traditional values of integrity, objectivity and
public service dedication.
• Civil servants are being equipped with the necessary skills and capabilities to
master new technologies and new styles of functioning. This will bring about an
attitudinal change required to adopt technology.
• Along with the code of conduct which may not be able to cover every possible
scenario, code of ethics is also being focused upon so as to guide the actions of civil
servants while acting under discretion.

6. Examine how the moral order of any society is based on the norms like honesty,
truthfulness, discipline, fairness, tolerance and justice, and therefore the code of
ethics has to be based on them.
Approach:
• Firstly, examine how the foundation of morality in a society is based on certain
norms

DELHI JAIPUR PUNE HYDERABAD AHMEDABAD LUCKNOW CHANDIGARH


20 www.visionias.in # 8468022022 ©Vision IAS
Google it:- https://upscpdf.com
https://t.me/UPSC_PDF Download From > https://upscpdf.com https://t.me/UPSC_PDF

• Then discuss why the code of conduct and ethics has to be rooted in moral order of Student Notes:
the society and thus ultimately based on the norms established.
Answer:
• Norms are largely social in their origin, sanctions and functions and are an
instrument of society as a whole for the guidance of individuals and groups.
• While the origin of social norms may lie in small, close knit groups, they often
spread well beyond the narrow boundaries of the original group to large
populations.
• Further, norms are not just a discovery or invention of the individual for his own
guidance. They are a social enterprise and seen as constraining behaviour.
• Though they make demands on the individuals, usually individuals themselves
become spokesmen of these demands, through what is called “internalization’.
• Once they are internalized, norms come to be seen gradually as central to the
production of social order and social coordination.
• Though they cannot be explained solely on the basis of functions that they
perform, it is indeed a fact that they fulfill important social functions, such as
welfare maximization.
• The common values and moral order of societies have come to be based on such
norms and they have guaranteed the orderly functioning and reproduction of the
social system of societies across the world.
• They have helped ensure that citizens are aware of not only their roles in society,
but also customs, common practices, power relations, economic exchanges, rules
for interpersonal interaction etc. This has helped people to function successfully in
society and maintain the moral order by making people learn and accept the
shared values of society, thus learning how to act in society.
• While certain norms like honesty, truthfulness, discipline, fairness, tolerance and
justice are universal to the extent that they shape the moral order of societies; the
interpretation of these can differ across societies, depending upon the type of
socio-cultural-economic-political system that the society has. This difference in
interpretation according to the desired goals of societies is one of the reasons for
difference in moral codes across societies. For instance, while equality and justice
are acceptable social norms across most societies, there may have different notions
of what they mean. In communist societies, achieving equality of outcome may be
desired, while in societies with different socio-political systems ensuring equality of
opportunity may be the desired goal.
• Social norms have also been the basis of the code of ethics for various professions.
• Code of ethics represents values collectively imposed on individuals by
organizations, professional associations, or political jurisdictions.
• Though discretions exercised in organizations, like the bureaucracy, are necessarily
broad, but it must be generally consistent with the wishes of the citizenry. The
range of possible decision options should be constrained by the will of the people
as expressed through the moral order of the society and various acceptable norms.
• Code of ethics and ethics legislation provides these broad constraints within which
the ethical conflicts and dilemmas that one faces are to be resolved. Such codes
and laws are however, only the formal statements of the moral minimum for a
political community.
• They also provide sanctions for professionals, including public servants, caught
stepping beyond the limits established by the citizenry through the moral order and
norms of the society. Fines, prison sentences, and administrative penalties for
misconduct are ways of establishing the right of the people to require that their
norms be respected and enforced.

DELHI JAIPUR PUNE HYDERABAD AHMEDABAD LUCKNOW CHANDIGARH


21 www.visionias.in # 8468022022 ©Vision IAS
Google it:- https://upscpdf.com
https://t.me/UPSC_PDF Download From > https://upscpdf.com https://t.me/UPSC_PDF

• Legal sanctions confront the whims, greed and self assertion of an irresponsible Student Notes:
professional with a reminder that he is employment is in a social context and that
the limits imposed by the society may not be transgressed.
• Hence a code of ethics should be based upon the norms, like the ones mentioned
above and confirm to the moral order of the society as these core values support
good governance and ensure the achievement of the highest possible standards.

7. Examine the contemporary relevance of Gandhiji's talisman as a means of resolving


ethical dilemmas in day to day life.
Approach:
• Highlight the key message of talisman (in brief) – ‘sarvodaya through antyodaya’.
• Explain how it can promote emotional intelligence in citizens and help them resolve
ethical dilemmas.
• Discuss its contemporary relevance for citizens, businesses and the Government.
Answer:
Gandhiji’s talisman– “whenever in doubt, think if the decision would empower or
marginalize the poorest?” provides an ethical test to judge everyday actions. It
champions the cause of ‘sarvodaya through antyodaya’ implying the welfare of all
through the weakest of the society which lies at the core of Indian Constitution.
It is a clarion call to the citizens to develop emotional intelligence so that they are in a
better position to assess the needs of fellow (vulnerable) citizens, and work towards
their upliftment through individual actions.
India of the 21st century is witnessing sectoral violence, socio-economic inequality, low
tolerance threshold and increasing consumerism. Global politics is witnessing
xenophobia, armed conflicts and multiple refugee crises. The talisman can hence be an
effective tool to evaluate the relevance of government policies, business priorities and
our daily actions on the society. All these entities can assess their actions in public as
well as private life against the talisman to ameliorate ethical dilemmas.
Applying the talisman to our daily lives invariably suggest the following:
• Government policies must necessarily focus on the vulnerable and marginalized
sections of the society even if it requires making special provision for such classes.
• At personal level, self-serving acts like tax evasion, littering in public places etc.
must be shunned. Derogatory practices in society like manual scavenging, caste-
based discrimination, exploitation of women and the poor, dowry, nepotism etc.
must be condemned in practice.
• At societal level, it encourages altruism, compassion and communal harmony
towards fellow human beings when encountered with negative vibes of hatred and
violence.
• Businesses and Industries should strive to reduce polluting activities, ensure
optimal working conditions and must honestly comply with social security
legislations. Further, it encourages business houses to go for Corporate Social
Responsibility for achieving greater inclusiveness in the society.
• Public servants should ensure transparency and integrity in implementation of
welfare provisions like PDS, MGNREGA, Old Age Pension Scheme etc. so that their
steps lead to swaraj in true sense.
Thus, Gandhiji’s talisman empowers us to judge our actions and is a timeless beacon of
inspiration for generations to come.

DELHI JAIPUR PUNE HYDERABAD AHMEDABAD LUCKNOW CHANDIGARH


22 www.visionias.in # 8468022022 ©Vision IAS
Google it:- https://upscpdf.com
https://t.me/UPSC_PDF Download From > https://upscpdf.com https://t.me/UPSC_PDF

8. Consider the case of a religiously orthodox couple in Wonderland. Their son is dying of Student Notes:
cancer and needs a bone marrow transplant. While the doctor is devoted to reducing
pain and preserving life, the parents refuse permission to let the doctor give the
necessary bone-marrow transplant. In their view faith alone does the healing and
moreover, permitting the treatment would endanger the child’s life because it would
issue from lack of faith. The doctor is dismissal of these ideas and puts off all
treatment until the parents agree for the transplant. While the clashes between these
two worldviews continue, the child dies. Should the parents now be tried by the law
for criminal neglect? Was the doctor also being stubborn and betraying weakness, or
is he just being a doctor? Has he violated his code of ethics too?
Answer:
1st Approach
The principle of autonomy is one of the four guiding principles of medical ethics, the
others being beneficence, non-malfeasance, and justice. It means that patients have the
right to decide what is done to their own bodies. For minors, it is the parents decide for
them. What happens when parents refuse a treatment that their child’s doctor
recommends?
While the faith of the parents should be respected, any possibility of a miraculous cure
based on their religious faith should be entirely left out of account. In the given case, it
was a near medical certainty that the child would die without treatment; and the only
effective treatment was probably a bone marrow transplant.
If the doctor believed that the parents were not acting in the child’s best interest, he
should have gone to the court and tried to convince the judge that the court should take
temporary custody of the child and allow the treatment. Another way for the doctor was
to try persuading the parents to accept medical care. He could have even discussed
passages from the holy books of various religions (and probably even theirs), which talk
about healing and even prayed with them, as a sign that he also respected their faith. He
could have made them believe that he was not opposed to their beliefs per se, and was
not negating the power of their God, and in fact was a part of it. In the process he could
have tried to balance their view on faith and science and told them to keep praying for
their child, while he gives him the requisite treatment. It was important that he tried to
appreciate their point of view, rather than dismissing it altogether.
In such a situation it was important to balance the practical obligations of providing
medical care while adhering to doctors’ ethical obligations to report such cases to the
police and the courts. To this limited extent the doctor violated his code of ethics and
abdicated his responsibilities. He was also being stubborn and betraying weakness.
The doctor’s decision on bone marrow transplant decision was made on the basis of
medical knowledge and experience, evidence and reason. Though by definition, a faith
based miracle defies medical science and all known experience and reason, it was
important that the parents be made to realize that they were simply engaging in child
neglect. Assuming that there are adequate laws on child neglect, the parents should be
tried and penalized in some way under the same.
However, it must also be kept in mind that even parents, who are blinded by religious
faith and miracles, love their children. They would have loved to have their child survive,
but they believed in healing with prayer. It is unlikely that they were harming the
interests of their child intentionally; it’s just that they had a strong faith that others don’t
share.
The doctor should not have made a rash decision when faced with a family refusing
medical care, despite how he felt about the family’s religious beliefs. If he would have

DELHI JAIPUR PUNE HYDERABAD AHMEDABAD LUCKNOW CHANDIGARH


23 www.visionias.in # 8468022022 ©Vision IAS
Google it:- https://upscpdf.com
https://t.me/UPSC_PDF Download From > https://upscpdf.com https://t.me/UPSC_PDF

approached them in a more humanistic way and with compassion, compromise was Student Notes:
much more likely. Finding common ground with parents about religious views would
have helped in developing a compatible care plan and maybe saved the child. It was
important for the doctor to guard against his frustrations, which held him from working
with the family in saving the child.
Thus both the doctor as well as the parents has to share the blame for the child’s death.
While the doctor violated his code of ethics and was being irresponsible, the parents are
guilty of child neglect. Both could be tried and probably penalized in different ways.
2nd Approach
In this case we need to see the rights of child and parents to apply the principle of
autonomy because these rights vary with different cultural settings. So we need to
analyze above situation under different aspect of trusteeship and ownership. Under
ownership situation, it is observed that parents are best well-wisher of child and own
every decision on behalf of child. But trusteeship does not grants ownership to parents.
Here child is a responsibility of state and parents are care taker. Thus whether to initiate
any criminal proceedings against parents or not depends on law of that country.
But in case of doctor he surely has violated ethics of medical profession by putting of all
the treatment. He as a doctor needed to continue with the appropriate treatment of
child. To do nothing is a disservice both to the profession and to patient.

9. In spite of the existence of a Code of Conduct for public servants, its implementation
has been tardy and of limited success. Discuss. Further, suggest steps which could be
taken to improve the efficacy of the Code of Conduct. What role can Code of Ethics
play in reinforcing the Code of Conduct in public life?
Approach:
• Describe briefly code of conduct.
• Bring out the reasons for the limited impact the code of conduct.
• Suggest steps which could be taken to improve the efficacy of the Code of Conduct.
• Explain how code of ethics will help in improving efficiency of Code of conduct.
Answer:
Code of Conduct is a set of rules, standards, principles and values outlining the
expected behavior for the members of an organization. The purpose is to regulate the
conduct of members on various decisions and processes. Most of the public
organizations follow some code of conduct. These have been designed to
comprehensively cover almost all the situations in which public servants find
themselves. Yet, widespread corruption occurring is indicative that it has proved to be
ineffective. Following reasons can be attributed towards the minimal impact of code of
conduct in dealing with corruption in public life:
• Legally not binding: Not all the provisions of code of conduct are legally binding.
For example, only provisions of Central Civil Services (Conduct) Rules which are
violated under the Prevention of Corruption Act invite punishment. Else, their
enforcement boils down to departmental discretion. Similarly, Model Code of
Conduct by ECI is not legally enforceable and only its provisions violated under the
RPA attract punishment..
• Code of conduct has to be supported by strong accountability and transparency
mechanisms. This is still to be achieved.
• Poor awareness about the code of conduct among the public officials and public.
• Lack of proper training about dealing with situations amounting to conflict of
interest. As a result officials often fail to follow code of conduct.

DELHI JAIPUR PUNE HYDERABAD AHMEDABAD LUCKNOW CHANDIGARH


24 www.visionias.in # 8468022022 ©Vision IAS
Google it:- https://upscpdf.com
https://t.me/UPSC_PDF Download From > https://upscpdf.com https://t.me/UPSC_PDF

• Legal rules alone cannot ensure transparent and fair governance. What is needed is Student Notes:
self-regulation but there has been rapid erosion of values in public servants.
Following steps can be taken to enhance the effectiveness of Code of Conduct:
• The Code should be legally binding. Arbitrariness is against the Rule of Law and
hence, it should be eliminated. Every provision should unambiguously describe the
course of action that has to be taken in case of violation of the code.
• A comparative evaluation of effectiveness of code of conduct in public and business
organizations indicates that there has been considerable impact of the Code of
Conduct in business organizations. Hence, the methodology for implementation of
the code of conduct can be adopted and modified to suit public organizations.
• Certain practices in business organizations have resulted in employees identifying
themselves with the organization and its objectives. These best practices should be
adopted by public sector.
• Inculcation and rejuvenation of public service values at an early age and throughout
the career of employees should be done through activities and training.
• Code of conduct should be supported by service delivery standards and strong
accountability and transparency mechanisms to reign in corruption.
Code of ethics provides for the value system that public servants can look up to in times
of crisis and dilemma. A Code of Ethics creates a framework upon which all decisions
are made. This helps create a cohesive understanding of the boundaries within an
organization and the standards set for interacting with stakeholders. The ARC II
fervently pitched for Code of ethics and related reporting systems. Thus, it will be
complementary to Code of Conduct. However, values need institutional support to be
sustained. Hence, it’s important that Code of Conduct be made legally binding, with
penalty provisions. If both the codes are followed in letter and spirit, corruption and
misuse of public office will be curbed to great extent.

10. It is at the interface of public action and private interest that the need arises for
establishing not just a code of ethics but a code of conduct. In this context, highlight
the need for drafting a code of ethics as well as a code of conduct.
Approach:
• Discuss the need for regulating the conduct/behaviour of public servants.
• Explain the rationale behind having a separate code of conduct & code of ethics.
• Highlight the important differences between the two.
• Conclude by mention how both can be supplemented owing to similarity in their
objectives.
Answer:
It is imperative for a public service organization to conduct itself in ways that preserve
and enhance public trust and confidence in the integrity of government and its
institutions. In the case of public servants, the standards of behaviour must be more
stringent, because any person/organization that is privileged to guide the destiny of the
people must not only be ethical but must be seen to practice these ethical values.
However, any elaborate system of laws and rules cannot exhaustively account for all
situations especially when the ambit of discretion at the disposal of a public servant
widens. It is at this interface of public action and private interest that the need arises
for establishing not just a code of ethics but also a code of conduct.
A code of ethics covers broad guiding principles of good behavior and governance. They
act as a reminder for the members of an organization regarding broad moral standards

DELHI JAIPUR PUNE HYDERABAD AHMEDABAD LUCKNOW CHANDIGARH


25 www.visionias.in # 8468022022 ©Vision IAS
Google it:- https://upscpdf.com
https://t.me/UPSC_PDF Download From > https://upscpdf.com https://t.me/UPSC_PDF

e.g. maintaining integrity, ensuring transparency in functioning etc. They provide ethical Student Notes:
guidance in difficult or unclear situations.
However, it is not sufficient to have only a Code of ethics. Code of conduct is also
required , as code of conduct provides a list of acceptable and unacceptable behaviour
and action in a precise and unambiguous manner. The specific do's and don’ts such as
not accepting bribes, not giving out details on internal working of the organization to
the media etc. promote adherence. It makes public servants well aware of acts that are
wrong, recognize wrongdoing and actively prevent it. Hence, it introduces a mental
barrier into the minds of corrupt or potential miscreants against wrongdoing.
The need for having clear cut codes arises from the fact that civil servants as individuals
have their own principles to guide them. This may lead to discrepancies in actions from
individual to individual. Also, having a clearly formulated code leaves less scope for
interpretation and thus act as an objective tool to guide behaviour of civil servants.
Both the code of ethics and of conduct are interrelated. While CoE provides higher
principles to follow in performance of duty, a CoC provides specific set of guidelines,
derived from the higher principles. Codes of ethics can be used to revise the code of
conduct to cover behaviour or situation that is not already covered while code of
conduct helps in indirectly adhering to the code of ethics by setting standards against
which public servants can test their actions and eventually inculcate the habit of doing
the right thing for the right reasons. Thus, both serve to meet the same objectives and
supplement each other.

11. Ensuring that civil service values are recognised during the recruitment process and
ensured through a code of ethics after appointment is a necessary condition of
making the civil services an effective instrument of citizen centric governance.
Comment.
Approach:
• Briefly define code of ethics and civil service values.
• State the importance of recognizing civil services values during the recruitment
process of civil servants and its continuance thereafter through code of ethics.
• Assess whether it is an effective instrument of citizen-centric governance.
Answer:
Civil servants play a key role in nation building as they advise on and implement the
policies and programmes that have far reaching consequences. Since they are the
decision makers, it is pertinent that the power they exercise is fair and just. This has to
be ensured from within and externally. The internal value system of a person is largely
formed by the mid-twenties. As such, it is important that this be tested and recognised
during the recruitment process. Efforts must be made to ensure that not only the most
intelligent and hard-working but also the most ethical aspirants join civil services.
However, in order to take decisions that have far reaching consequences, merely
internal system of ethics is not sufficient. The institution of bureaucracy requires that
there also be an externally enforced code of conduct as well as general code of ethics
to ensure good governance.
Code of ethics is a set of guidelines issued by an organization to its workers and
management to help them conduct their actions in accordance with the primary values
and ethical standards of the organization. The fundamental tenets of a code of ethics
and values in civil services include integrity, empathy, objectivity, transparency,
professional competency, among others.

DELHI JAIPUR PUNE HYDERABAD AHMEDABAD LUCKNOW CHANDIGARH


26 www.visionias.in # 8468022022 ©Vision IAS
Google it:- https://upscpdf.com
Further issues such as red tapism, corruption, rigid attitude, pulls and pressures of Student Notes:
decision making involving discretion etc plague the efficiency of a Civil Servant during
his employment and hence the role of an ethical code of conduct becomes significant.
Attaining Citizen Centric Governance which includes grievance redressal mechanisms,
active citizens’ participation, accountability and maximization of citizen’s welfare, is an
important goal of civil services. Additionally, services should be provided effectively,
efficiently and equitably to all citizens. Certainly, this cannot be achieved without
adherence to civil service values.
In this context, it should be noted that there is no single code of ethics for Civil Servants
in India. However, there are several conduct rules such as the Central Civil Services
(Conduct) Rules, All India Services (Conduct) Rules etc., which prescribe allegiance to
the Constitution, apolitical functioning and objective decision-making. Several
nd
committees like the Santhanam Committee, Hota Committee, 2 ARC etc. have
recommended a code of ethics for Civil Services in India.

12. It is at the interface of public action and private interest that the need arises for
establishing not just a code of ethics but a code of conduct. In this context, highlight
the need for drafting a code of ethics as well as a code of conduct.
Approach:
• Discuss the need for regulating the conduct/behaviour of public servants.
• Explain the rationale behind having a separate code of conduct & code of ethics.
• Highlight the important differences between the two.
• Conclude by mention how both can be supplemented owing to similarity in their
objectives.
Answer:
It is imperative for a public service organization to conduct itself in ways that preserve
and enhance public trust and confidence in the integrity of government and its
institutions. In the case of public servants, the standards of behaviour must be more
stringent, because any person/organization that is privileged to guide the destiny of the
people must not only be ethical but must be seen to practice these ethical values.
However, any elaborate system of laws and rules cannot exhaustively account for all
situations especially when the ambit of discretion at the disposal of a public servant
widens. It is at this interface of public action and private interest that the need arises
for establishing not just a code of ethics but also a code of conduct.
A code of ethics covers broad guiding principles of good behavior and governance. They
act as a reminder for the members of an organization regarding broad moral standards
e.g. maintaining integrity, ensuring transparency in functioning etc. They provide ethical
guidance in difficult or unclear situations.
However, it is not sufficient to have only a Code of ethics. Code of conduct is also
required , as code of conduct provides a list of acceptable and unacceptable behaviour
and action in a precise and unambiguous manner. The specific do's and don’ts such as
not accepting bribes, not giving out details on internal working of the organization to
the media etc. promote adherence. It makes public servants well aware of acts that are
wrong, recognize wrongdoing and actively prevent it. Hence, it introduces a mental
barrier into the minds of corrupt or potential miscreants against wrongdoing.
The need for having clear cut codes arises from the fact that civil servants as individuals
have their own principles to guide them. This may lead to discrepancies in actions from

DELHI JAIPUR PUNE HYDERABAD AHMEDABAD LUCKNOW CHANDIGARH


27 www.visionias.in # 8468022022 ©Vision IAS
individual to individual. Also, having a clearly formulated code leaves less scope for Student Notes:
interpretation and thus act as an objective tool to guide behaviour of civil servants.
Both the code of ethics and of conduct are interrelated. While CoE provides higher
principles to follow in performance of duty, a CoC provides specific set of guidelines,
derived from the higher principles. Codes of ethics can be used to revise the code of
conduct to cover behaviour or situation that is not already covered while code of
conduct helps in indirectly adhering to the code of ethics by setting standards against
which public servants can test their actions and eventually inculcate the habit of doing
the right thing for the right reasons. Thus, both serve to meet the same objectives and
supplement each other.

11. Previous Years Vision IAS GS Mains Questions: Case


Studies
1. A junior member of staff has just returned to work after taking special leave to care
for her elderly mother. For financial reasons she needs to work full-time. She has been
having difficulties arranging proper care for her mother, which has led her to miss
important team meetings (usually taking place at the beginning of each day) and to
leave the office early. She is very competent in her work but her absence is putting
pressure on her as well as her overworked colleagues. You being her manager are
aware that the flow of work is coming under pressure due to this. One of her male
colleagues is beginning to make comments such as “a woman’s place is in the home”,
and is undermining her at every opportunity, putting her under even greater stress.
How will you deal with the situation?
Answer:
Fundamental Principles
1. Integrity: You need to be fair to all those involved and act in a straightforward
manner.
2. Confidentiality: You have a duty of maintaining confidentiality to the staff involved.
3. Professional behaviour: How should you proceed so as not to discredit yourself,
your profession or the practice for which you work?
Identify Relevant Facts
Consider the firm’s policies and, with legal assistance if required, applicable laws and
regulations.
Identify affected parties
1. The Junior staff
2. You
3. The member staffs
4. The Male colleague and the HR
Who should be involved in the resolution?
Consider not just who you should involve but also why and when. Do you have access
to appropriate staff in HR? Can you consult someone in the office in whom you can
confide?
Possible Action
1. Check the relevant facts: Clarify staff procedures with the senior HR manager. Take
legal advice if required.

DELHI JAIPUR PUNE HYDERABAD AHMEDABAD LUCKNOW CHANDIGARH


28 www.visionias.in # 8468022022 ©Vision IAS
2. Discuss the matter with the staff member: Suggest a more flexible approach to Student Notes:
team meetings – do these always have to be in the morning? Working from home
may be an option for the junior staff member, if possible.
3. Remind the male member of staff about proper conduct and how such behaviour
may amount to harassment and he might have to face legal problems along with
the chance of spoiling company’s reputation.
4. Throughout, you must be seen to be acting fairly: both towards the junior member
of staff, who is responsible for her mother’s care, and towards other members of
staff.

2. "The challenge of climate change presents the world with several fundamental ethical
dilemmas. From a global perspective, it presents the world with a collective action
problem: all countries have a collective interest in controlling global carbon emissions.
But each individual country also has incentives to over-consume in response to
societal demands for economic growth and prosperity. Developing nations faced with
these costs may encounter further challenges as the impact of climate change will
most likely fall disproportionally on the poor, thus also raising issues of fairness and
inequality. As an intergenerational problem, the consequences of actions taken by the
current generation will have the greatest impact on future generations. While it is
intuitive that the current generation has some ethical responsibility to leave an
inhabitable world to future generations, the extent of this obligation is less clear.
Answer the following questions in this regard:
(a) To what extent do humans have a moral responsibility to future generations that
are yet to be born?
(b) Do developed countries have a greater responsibility to take action and bear more
costs of controlling climate change than developing countries?"
Approach:
• Briefly introduce about the moral problem of climate change.
• With reasoning explain the extent of moral obligations of current generation to
future generation.
• Discuss about CBDR and explain the reasons why CBDR should be the way forward
in climate negotiations.
• Write a conclusion highlighting the future course of action.
Answer:
The fight against climate change is political, economic, socio-cultural and technological
but most importantly ethical/moral where humans as individuals and societies need to
limit their greed and selfishness and think about the future generations and their
survival.
a) We don't inherit the earth from our ancestors; we borrow it from our children.
The extent of moral responsibility of the present generation towards future
generations can be understood from following:
• We have to understand that once emitted a substantial proportion of
climate emissions typically remain in the atmosphere for hundreds of
years. This means that while the current generation benefits, the costs of
its actions will be borne by the future generations.
• Also, the benefits are comparatively modest for e.g., powerful vehicles,
but many of the projected costs are severe for e.g., severe flooding and
famine.

DELHI JAIPUR PUNE HYDERABAD AHMEDABAD LUCKNOW CHANDIGARH


29 www.visionias.in # 8468022022 ©Vision IAS
• If we don’t tackle the issue of climate change as an issue of Student Notes:
intergenerational ethics, future generations will follow suit. They may feel
entitled to emit more, which can make matters even worse for their
successors.
• Happiness of future generations can be maximized by a reasonable
investment in the present.
Therefore, the Paris Summit, 2015 and SDGs highlighted the moral
responsibility of the present generation towards the future generations. Each
country came up with its INDCs (Intended Nationally Determined
Contributions) to owe such responsibility and leave the world a better place
than they inherited.
b) It is by and large accepted that developed countries ought to be held
responsible more for the current predicament of our world and therefore
need to do more to improve it. The United Nations recognises the CBDR
(Common but Differentiated Responsibilities) principle in the climate change
negotiations. It forms a key component of the Paris agreement as well.
Arguments supporting CBDR are as follows:
• Increase in CO2 and other GHGs in the Earth’s atmosphere are a result of fossil
fuel based industrial activity in the industrialized countries of the world. Those
least responsible for past emissions are likely to suffer the most serious impacts
of climate change.
• While developing countries like India and China are major GHG emitters, the
per capita consumption in these countries is very low compared to developed
countries.
• If developed countries do not make reductions in their emissions, there will be
a progressively smaller carbon space available to accommodate the
development needs of developing countries.
• Developing countries currently cannot sustain themselves without relying
heavily on fossil fuels. These countries face the challenge of food security,
housing, and employment along with global warming.
• Developing countries lack the finances or the technology to develop alternative
green technologies to sustain their economies. So, it is the moral responsibility
of the developed countries to provide financial and technological support to
developing nations.
The only way forward is a path of shared responsibilities where all stakeholders are
ready to fulfill their obligations, grow sustainably and provide a healthy and
habitable world to our future generations.
3. "You are representing India in an international bidding for oil exploration in a country.
Other, richer countries are also bidding for the project. You are sure that your bid of
exploration is better as well as cheaper than that of others, and that you will
definitely win the bid. A day before the auction, you come to know that other
countries are employing every means, including bribing the authorities for being
successful. Some of the officials of the home country have also contacted you and
made some demands in exchange for assurance of India winning the bid. You are
aware of the criticality of this bid in terms of domestic economic and strategic
implications. Based on above information, answer the following questions. (a) Specify
the ethical dilemma(s) that you face in this situation. (b) Do ethical concerns really
matter in international transactions or are they secondary to domestic interests? (c)
What will be your course of action in the above situation? Justify with merits and
demerits."

DELHI JAIPUR PUNE HYDERABAD AHMEDABAD LUCKNOW CHANDIGARH


30 www.visionias.in # 8468022022 ©Vision IAS
Approach: Student Notes:
• Identify the ethical dilemmas that you face.
• Highlight the importance of ethical concerns in international transactions vis a vis
domestic interests.
• Then mention the course of action that you would follow. Justify it by taking into
account the merits and demerits of the decision.
Answer:
a) The situation in this case study presents the following ethical dilemma:
The dilemma is whether to pay the bribe vs being upright and avoid the temptation to
pay.
The former action may help India win the bid, but it will be an unethical course of
action and may spoil the image of the country in the long run when the truth comes
out in the public. It will also have adverse consequences for India’s relations with the
countries involved in bidding process. Moreover, this is inimical to a healthy
competition, level playing field and innovation. This action will also set a wrong
example to others. The action is not only unethical but also illegal as regards to Indian
laws. It may not bring in me a sense of accomplishment or content.
The latter course of action may lead to a possible defeat in the bidding process, setback
for my career and economic and strategic implications for the country. But it is the right
path to follow.
b) Advocates of national interest in international relations argue that national interests
are paramount. As Henry Kissinger has said- “there are no permanent ally or
permanent enemies, only interests are permanent”. These arguments are based on the
fact that the government of a county primarily works on the behalf of its citizens and
thus it is bound to uphold their interests. The political party in power has to face
general elections regularly and its report card of performance evaluates not only
domestic but international actions as well. Hence, national interest alone should be
paramount in international relations.
However, these arguments suffer from certain inconsistencies. If the national interest
alone is taken into account then wrong doings like colonization, regime change, arm
twisting of weaker nations etc. will be justified. Further, there exists a wide inequality
internationally and if strong nations justify their actions solely based on the national
interests than this gap will further widen. Moreover, the global commons will not
survive and sustainable development will remain a distant dream.
Thus, fairness, justice, apathy, sustainable development of whole world, equity etc. are
ethical principles which are as important as national interests and really matter in
international relations.
c) In such situation, I will pursue the following course of action:
a. Verifying, at my own level, the correctness of information related to bribery
activities in the auctioning process.
b. Informing my seniors, seeking their advice as they might have faced similar
situation earlier.
c. Approach the head of the authorities handling the whole process of auction
and inform them about inconsistencies which has come to notice and demand
a fair and transparent bidding process.
d. If grievances are not addressed at that level then, after taking my seniors into
confidence, we can approach other higher authorities of home country like
judiciary for intervention.

DELHI JAIPUR PUNE HYDERABAD AHMEDABAD LUCKNOW CHANDIGARH


31 www.visionias.in # 8468022022 ©Vision IAS
I will also demand that the officials involved in bribe-seeking activities must be Student Notes:
punished which will deter such malpractices in future. Those nations who are involved
in unfair practices must also be punished (by way of fines, blacklisting them or
cancelling their bids).
Justification of such course of action
In international transactions, sometimes unetical actions like bribery are also sought to
be justified in the name of national interest. However, on a closer look, such actions are
clearly against the national interests in reality. The revelation about involvement in
bribery would jeopardize the international relations of future generations of our
country.
Further, a single contract cannot be so important to our country that we sacrifice our
moral standards and higher values maintained for so long. Moreover, corruption can
never be the true foundation of prosperity. The gains obtained from it corrupt the
whole society.
By following the stated course of action, I will display faith in the governance of home
country, uphold our moral values and there will be higher chances of fair bidding
process. As India’s bid is better and cheaper, it will ensure India’s success. It will
generate the good will for our nation among the people of that country; set a right
example against corruption in international transaction. Overall, it will be a right step
towards the righteousness which we expect in international relations.

4. The steady decline in sex ratio suggests that marked improvement in the economy
and literacy rates do not seem to have had any impact on this index. In fact, the
availability of new technology and its easy access for the urban, wealthy and
educated have worsened the trend and harmed the status of women in Indian society.
(a) Explain why the phenomenon should not be simply viewed as a medical or legal
issue and more attention should be given to the ethical issues involved. (b) Give some
suggestions to tackle the problem of declining sex ratio. (c) Discuss the ethical
dilemma involved in Right to abortion vs. Prevention of female foeticide. How can this
be resolved?
Approach:
• Give a brief introduction of the case study
• Provide moral values and stakeholders involved in the given case study.
• Explain why the phenomenon must not be seen as medical or legal issue.
• Explain how declining sex ratio involves ethical issues.
• Provide some suggestions to tackle the problem.
• Discuss the ethical dilemma involved in right to abortion vs prevention of female
foeticide.
Answer:
In a large sample of population, biological probability suggests that distribution of male
and female children should be fairly equal. However, a continuously declining sex ration
suggests human interference to keep the sex-ratio adverse towards female. Use of
technology has only aided in this effort. Female feticide is indeed a blot on the society
who prays goddesses. The given case study highlights how the wealthy, educated,
urban people are misusing technology for selective sex abortion and have contributed
to declining sex ratio and have harmed status of women in the society.
The situation reflects patriarchal mindset and involves the unborn girl child, parents,
the government and the society at large, as stakeholders.

DELHI JAIPUR PUNE HYDERABAD AHMEDABAD LUCKNOW CHANDIGARH


32 www.visionias.in # 8468022022 ©Vision IAS
(a) The phenomenon should not be simply viewed as medical or legal issue as it is not Student Notes:
only about the falling Maternal Mortality Rate (that increases due to such procedures
carried out by quacks) or about the lapses in the government to implement the laws
such as Pre-Natal Diagnostic Technique Act effectively. It is an issue of ethical concern
given the following reasons:
• Majority of the affected women are actually forced to go for such abortions
hampering their right to life and right to choose.
• Mother of a female child is blamed for her birth and miseries of the family. She is
cursed and ill-treated. Even prosperity of the family does not bring attitudinal
change towards the girl child reflecting gender bias in society
• The skewed sex ratio in the society gives birth to anti social behaviour and violence
such as rapes in case of but not limited to lack of marriageability and family
prospects. Thereby, decreasing values of respect towards women.
• The demand for sex workers may increase which may further give rise to human
trafficking as well as objectifying or commodifying women in terms of sexual
desires of men
• Technology which is for the betterment of humanity is being used to deprive the
right to life of an unborn being.
• It is also symbolic of the non-recognition of women’s contribution to society mostly
because of the non-monistic nature of their work.
(b) Suggestions to tackle the problem of declining sex ratio:
• Strict implementation of laws banning female foeticide and dowry, including fast
track courts to deliver on these cases
• Sensitisation of the public by mass awareness campaigns as well as Free and
compulsory education for girls
• National helpline number to rescue the would-be mothers
• Reservation for women in specific occupations as a temporary means to achieve an
attitudinal change.
• Moral education in schools and colleges to ensure inculcation of a sense of gender
equality among people.
Female foeticide should become a political, economic and reformist issue instead of
being just a social issue. The schemes such as Beti Bachao, Beti Padhao is a good start in
this direction
(c) In India, legally, abortion of a feotus not older than twelve weeks is allowed. This can
be extended up to 20 weeks if two medical practitioners authenticate that either
mother’s life is in danger or the foetus is seriously handicapped. Ethical dilemma
revolving around this are:
• If mother’s life is in danger, there should be no time limit on termination of
pregnancy (Supreme Court judgement). But then, it widens the scope for sex
determination and consequent sex selective abortions.
• In the garb of finding out disabilities in the foetus, sex of the child can be examined.
• The right to safe and legal abortion is an essential right of self-determination. Not
providing that right is to turn women into birth-machines, but how can it be
determined that the woman's choice is not forced.
• Right to abortion is 'Pro-Choice', while prevention of female foeticide is 'Pro-Life'.
• Right to abortion comes within realm of Right to Privacy but looking at the sexist
and patriarchal set up of Indian society, decisions about pregnancy are sometimes
out of the control of women.

DELHI JAIPUR PUNE HYDERABAD AHMEDABAD LUCKNOW CHANDIGARH


33 www.visionias.in # 8468022022 ©Vision IAS
Since high number of women die due to unsafe abortions, so it must be legal, but the Student Notes:
Pre-Conception Pre-Natal Diagnostic Techniques Act should be implemented strictly.
There should be mandatory registration of pregnancies and all abortions should be
monitored. Reasonable criteria for abortion should be set and more stress should be
given upon contraception. Complicated abortions should be handled on case-to-case to
basis, monitored by the Medical Boards in government medical centres.
There is no ethical dilemma as far as choice between female feticide and right to
abortion is concerned. Former is criminal and unethical, while the latter is still a matter
of debate. The only dilemma arises whether one considers killing an embryo
(irrespective of sex) as an exercise of right to choose or an act of murder. Going
forward, this debate has to be resolved by societal discussion and evolving ethical
standards. But a right to choose cannot become a means for selective sex abortion.

Copyright © by Vision IAS


All rights are reserved. No part of this document may be reproduced, stored in a retrieval
system or transmitted in any form or by any means, electronic, mechanical, photocopying,
recording or otherwise, without prior permission of Vision IAS

DELHI JAIPUR PUNE HYDERABAD AHMEDABAD LUCKNOW CHANDIGARH


34 www.visionias.in # 8468022022 ©Vision IAS
ETHICS IN INTERNATIONAL RELATIONS Student Notes:

Contents
1. Introduction ............................................................................................................................... 2
1.1. Significance of Ethics in International Relations ................................................................. 2
1.2. Theories in International Relations ..................................................................................... 2
2. Ethics in Foreign Policy ............................................................................................................... 2
2.1. Elements of Ethical Foreign Policy ...................................................................................... 2
3. Ethical Concerns in International Relations ............................................................................... 3
3.1. Climate Change and Ethics: ................................................................................................. 3
3.1.1. Resolution to ethical challenges of climate change: .................................................... 4
3.2. Global Poverty and Inequality ............................................................................................. 4
3.2.1. Global Poverty and Ethics ............................................................................................ 4
3.2.2. Inequality and Ethics .................................................................................................... 5
3.3. Terrorism ............................................................................................................................. 6
3.3.1 Ethical Issues associated with terrorism ....................................................................... 6
3.3.2 Measures which can be taken ....................................................................................... 7
3.4. Genocide ............................................................................................................................. 7
3.4.1. Ethical concerns associated with Genocide ................................................................. 7
3.5. Organised Crime: Drug and Human trafficking ................................................................... 9
3.5.1. Factors behind organized crime ................................................................................... 9
3.5.2. Ethical concerns associated with organised crime ....................................................... 9
3.5.3. Way Forward .............................................................................................................. 10
3.6. International Aid ............................................................................................................... 10
3.6.1. Types and Objective of Foreign Aid............................................................................ 10
3.6.2. Ethical Arguments for Foreign Aid ............................................................................. 10
3.6.3. Issues and Criticism .................................................................................................... 10
3.6.4. Foreign Aid and Neo-Colonialism ............................................................................... 11
3.6.5. Way forward ............................................................................................................... 12
3.7. NGO Colonialism and Foreign Aid ..................................................................................... 12
3.7.1. Ethical Issues .............................................................................................................. 12
3.7.2. Way Forward .............................................................................................................. 13
3.8. Pandemic and Ethics ......................................................................................................... 13
3.8.1. Vaccine development and distribution ...................................................................... 14
3.8.2. Way forward ............................................................................................................... 16
3.9. Ethics of Humanitarian Intervention ................................................................................. 17
3.9.1. Responsibility to Protect ............................................................................................ 17
3.10. Just War Concept ............................................................................................................ 18
3.11. Global Commons ............................................................................................................. 18
4. Doctrine of International Community ...................................................................................... 20
4.1. Five rules of Intervention of International Community .................................................... 20
4.2. Global Communitarianism ................................................................................................ 20
4.3. Collective action in the times of COVID ............................................................................ 20
5. Moving forward towards Global Constitutionalism ................................................................. 21
6. Previous Year UPSC Mains Questions ...................................................................................... 22
7. Previous Year Vision IAS Mains Test Series Questions ............................................................. 22
8. Previous Year Vision IAS Mains Test Series Case Studies ......................................................... 32

Copyright © by Vision IAS


All rights are reserved. No part of this document may be reproduced, stored in a retrieval system or
transmitted in any form or by any means, electronic, mechanical, photocopying, recording or
otherwise, without prior permission of Vision IAS.

DELHI JAIPUR PUNE HYDERABAD AHMEDABAD LUCKNOW CHANDIGARH


1 www.visionias.in # 8468022022 ©Vision IAS
https://t.me/UPSC_PDF Download From > https://upscpdf.com https://t.me/UPSC_PDF

1. Introduction Student Notes:

International ethics is a set of universal values that governs the actions and behaviours of
nation states. For example, Human rights are to be protected, aggression by a nation is
prohibited, the targeting of the civilians and or hospitals during war is prohibited, Genocide is
prohibited.

1.1. Significance of Ethics in International Relations


In a globalised world where geographical boundaries don’t exist international, Ethics in
International Relations helps in the following ways-
• It brings fairness in the world economic order. Through global institutions such as UN and
ICJ, it establishes justice.
• It strives to remove poverty, destitution, inequality and prevents genocides and crimes
against humanity.
• It follows humanitarian ethics and sets rules of exchange between nations. It ensures that
relationship between individual and nations be founded upon the principles of democracy
and justice.
• It includes areas and debates such as social justice, human rights, caring for environment
across national borders, social responsibility and accountability, and interdependence
gained through encompassing world view.

1.2. Theories in International Relations


Theoretical approaches are necessary to understand the International relations and how ethics
evolved as a part of the same. Some of the predominant theories are as following;
• Realism: Realist believe that the Universal moral principles do not govern the state
behaviour, rather interest govern them. Politics is an autonomous sphere, independent of
economics and personal morality.
• Liberalism: It has four major constituents-
o Individualism, as it gives moral primacy to person against any claims of the society,
o Egalitarian, as it confers on all humans the same moral status,
o Universalist, in that it affirms the moral unity of human species and
o Melorist for its affirmation of the corrigibility and improvability of all institutions and
political arrangements.
• Cosmopolitanism: It can be defined as the belief that all human beings have equal moral
standing within and belong to a single world community. It asks international politics to
focus on the interest, rights or welfare of persons wherever they may reside rather than on
the interest of the states.
These theories serve as the guiding light behind foreign policies of the nations.

2. Ethics in Foreign Policy


An ethical foreign policy requires governments to take a more enlightened view of their own
self Interest. It means that the foreign policy shows commitment to alleviate human suffering
and promotes mutual understanding, cooperation and peaceful coexistence in the
international system.

2.1. Elements of Ethical Foreign Policy


An ethical foreign policy is an ideology driven project which has following key elements:
• Human rights at the heart of the foreign Policy: Support to democratic rights of the people
anywhere in the world is fundamental constituent of an Ethical Foreign policy or in short
ethical foreign policies have cosmopolitan virtues.

DELHI JAIPUR PUNE HYDERABAD AHMEDABAD LUCKNOW CHANDIGARH


2 www.visionias.in # 8468022022 ©Vision IAS
Google it:- https://upscpdf.com
https://t.me/UPSC_PDF Download From > https://upscpdf.com https://t.me/UPSC_PDF

• Commitment to multilateralism, prosperity and environmentalism: It strives to achieve Student Notes:


global consensus around issues of universal importance such as Environment, Poverty and
Inequality etc.
• Global Justice and Responsibility to protect: To act decisively against any act of injustice
around the world under the doctrine of responsibility to protect is another constituent of
ethical foreign policy.
• Principle of sacrifice: According to this principle, if it is in power of any nation to prevent
something very bad from happening, for example death due to starvation or genocide,
without thereby sacrificing anything of comparable moral significance, one ought to do it.
• An ethical and decisive leadership which strives to make this world a better place is often
seen as an important tenet of ethical foreign policy.
One of the important objectives of the ethical foreign policy is to build consensus around the
ethical concerns in International relations.

3. Ethical Concerns in International Relations


Normative and ethical concerns now govern the International relations. Nations are now
acknowledging that there are concerns beyond the subject of national boundaries. Some of the
important ethical concerns in International Relations are as following;

3.1. Climate Change and Ethics:


Climate change is one of the most challenging issues facing the world today. it is important to
understand the ethical issues associated with it.

Need for understanding the ethical dimensions of Climate change:


• International Equity Concerns: Countries having the lowest socio-economic capacity, least
responsible for climate change are the most affected ones. For example, Marshall Islands.

DELHI JAIPUR PUNE HYDERABAD AHMEDABAD LUCKNOW CHANDIGARH


3 www.visionias.in # 8468022022 ©Vision IAS
Google it:- https://upscpdf.com
https://t.me/UPSC_PDF Download From > https://upscpdf.com https://t.me/UPSC_PDF

• Resource scarcity may create Conflict: Climate change has the potential to bring about Student Notes:
conflict mobilized by the quest for scarce resources. (Syria is an example of the same)
• Other ethical issues include: How to define and differentiate responsibilities between
present and future generations, developed and developing countries, etc.
Different actions are required by different agents in different contexts to respond appropriately,
humanely and ethically to the challenges of climate change.
3.1.1. Resolution to ethical challenges of climate change:
The resolution of ethical challenge of climate change lies in creating an opportunity to
establish a productive dialogue between States and other relevant agents from which a new
consensus may emerge about the issues.
The link between foreknowledge and the duty to act on it:
• In ethics generally, the worth of actions and policies depends not only upon the values and
principles they realize, but also upon their effects. An agent should thus foresee the
possible effects of his/her actions.
• National governments and the international scientific community have a responsibility to
train future generations of climate scientists
• Scientific uncertainty has vast implications for policymaking at both the regional and
national level. In this context, adoption of precautionary principle in decision-making
assumes importance.
• Governing bodies should put into place structures and procedures that are, resilient and
sensitive to vulnerabilities of people and environment.
• Removing obstacles to sharing and differentiating responsibilities. The principal
acknowledges that the actual ability to tackle climate change varies from country to
country.
Those who have the ability to prevent or alleviate harm suffered by others, and are in a
position to do so without sacrificing a greater value than what is rescued, have a clear duty to
act.

3.2. Global Poverty and Inequality


3.2.1. Global Poverty and Ethics
Global Poverty is defined as the number of people who live on less than $1.90 a day. A recent
UN report cites that; 71 million people are expected to be pushed back into extreme poverty in
2020, the first rise in global poverty since 1998.
This forces us to look at the ethical concerns that poverty at international level creates, such as-
• Affecting the vulnerable population disproportionately: More than a billion slum dwellers
worldwide are at risk from the effects of covid-19, suffering from lack of adequate housing,
no running water at home, and limited access to formal health care facilities.
• Rise in insensitivity: Global poverty as Kaushik Basu argues largely remains out of sight for
those who are not living it. This enhances insensitivity amongst the well off.
• Right to dignified life: Poverty and deprivation lead to low self-esteem and self-worth. It
also affects the dignity of the individual.
• Whom to prioritise? The states being a stake holder in the global fight against poverty, face
an inherent dilemma, that whether they should prioritise citizens or non-citizens for the
allocation of the resources.
• Accessibility and affordability issues: Poverty is an obstacle in access to resources. It
therefore has compound impact. It deprives a human being from developing capabilities for
living happy and meaningful life.

DELHI JAIPUR PUNE HYDERABAD AHMEDABAD LUCKNOW CHANDIGARH


4 www.visionias.in # 8468022022 ©Vision IAS
Google it:- https://upscpdf.com
https://t.me/UPSC_PDF Download From > https://upscpdf.com https://t.me/UPSC_PDF

The nations should contribute resources to protect the vulnerable. Principle of sacrifice and Student Notes:
principle of sympathy can be the guiding force behind international action against poverty.
Rwanda is an example of the same.
3.2.2. Inequality and Ethics
Inequality is a situation where certain people have access to things - places, goods, services -
which others do not. Justifying inequality therefore entails arguing why some deserve more
wealth and hence greater access to places, to goods and services, to social opportunities, than
others.
3.2.2.1. Defenders of inequality typically rely on one of 3 ethical arguments
• Desert Ethics: According to the Desert ethics, it is only fair to treat a person, as the person
DESERVES. So, if somebody has created something, say wealth, he is entitled to it, and he
must have all the rights to exclude others from using it.
• Voluntarist Ethics: According to it, the exchanges which led to an unequal distribution were
voluntary - people CONSENTED to the transactions, therefore they consented to the
resultant outcome. So, inequality is just an outcome of what people have already
consented.
• The Ethics of 'Growing the Pie': Granting superior legal status to few, will result in more
wealth and opportunities for everyone. However, in reality rich becomes richer and poor
becomes poorer. Higher inequality results in lower growth of incomes of poor.
3.2.2.2. Ethical issues due to high inequality
Inequality creates a less virtuous society because,
• The virtue of trust among members of society is lost e.g. acute deprivation may force a
person to steal or lie.
• The virtue of individual self-determination is lost as inequality removes the level playing
field to achieve success in life e.g. a rich child has better access to education and nutrition
than a poor child.
• The virtue of respect and tolerance in the society is lost e.g. the wasteful expenditure of
rich undermines their rationality and intellect in the eyes of poor.
• The virtue of obedience is also lost e.g. people start protesting, breaking rules, creating
riots
• Inequality violates distributive justice e.g. wages are not distributed according to the
labour or efforts one puts in the work.
• Affects fundamental rights: The right to have a dignified Life, the right to equality, the right
to have an equal opportunity gets jeopardised in case of extreme Inequality leading to lack
of food, education, healthcare, etc. Inequality creates compulsion and takes away the right
to choose.
• Inequality creates sufferings: Which may lead to a society where rich force the poor into
positions of servitude. Morality lies in reducing such sufferings.
• Undermines democracy: Inequality also undermines democracy as it creates unequal
access to the political system and to the positions of power. (John Rawls)
Hence, development in solidarity is necessary for peace and harmony in this world. Ensuring
justice is not merely to satisfy some legal requirements, or to avoid wars and conflicts. It comes
out of the conviction that all people on earth basically form one single human community, that
everyone is related to everyone else and everyone is responsible for the well-being of all. The
concept of justice is fundamentally rooted in love and solidarity with all people on earth.

DELHI JAIPUR PUNE HYDERABAD AHMEDABAD LUCKNOW CHANDIGARH


5 www.visionias.in # 8468022022 ©Vision IAS
Google it:- https://upscpdf.com
https://t.me/UPSC_PDF Download From > https://upscpdf.com https://t.me/UPSC_PDF

3.3. Terrorism Student Notes:

Terrorism is commonly understood to refer to acts of violence that target civilians in the
pursuit of political or ideological aims. Violence and intimidation constitute the core of
terrorism. Some of the recent examples of Terrorist attacks are as following;
• Kabul Gurudwara Attack: 25 victims died and many wounded. ISIS-Khorasan took the
responsibility of this deadly attack on the Gurudwara.
• Hanau Shootings, Germany: In total 9 people died. The attacker had far-right leaning and
hate for Non-German people in a letter of confusion and a video.
• Kulgam Massacre: Labourers were killed by Kashmiri Militants.

3.3.1 Ethical Issues associated with terrorism


India along with other major countries has been suffering from Terrorism. Global terrorism
report ranked India as the seventh most affected state from terrorism. Some ethical issues
emanating due to terrorism are as following:
• Preserving human life and rights: The right to life of the innocent civilians itself is under
threat due to terrorist actions.

DELHI JAIPUR PUNE HYDERABAD AHMEDABAD LUCKNOW CHANDIGARH


6 www.visionias.in # 8468022022 ©Vision IAS
Google it:- https://upscpdf.com
https://t.me/UPSC_PDF Download From > https://upscpdf.com https://t.me/UPSC_PDF

• Recruiting children and women for terrorist activities: Small children and women are Student Notes:
brainwashed and recruited for terror activities. The action which state ought to take against
them is an ethical concern.
• Good Terrorist vs Bad terrorist: States oppose terrorists’ group at one place and support at
the other. This reveals selective and self-serving nature.
• Use of violence to suppress terrorism: Whether a non-violent and tolerant society should
use violence to suppress the acts of terror against it.
• Disbelief and distrust in the present global order and democratic values: Terrorist often by
blowing them up try to show that, they don’t believe in the existing global social practices
and oppose them.
• Denial of civil liberties: Civil societies around the world start to demand tougher laws
leading to mass surveillance and restriction on civil liberties thereby emboldening
dictatorial tendencies
• Conduct during the war: The treatment of illegal combatants, the use of torture in the
interrogation of the prisoners and the killing of civilians in the conduct of military
operations and the wholesale destruction of private civilian property, all strengthen the
ethical case of terrorists and its allied groups to be conducting a just war.
• Human rights Violation and Migration: Terrorist action for example in Syria and Iraq lead
to mass migration of the people to Europe. The ethical challenge that nations faced was to
either let the migrants die (for example deaths of Rohingya and Syrian migrants due to
drowning) or to compromise safety security and resources of their own citizens and that of
migrants.
3.3.2 Measures which can be taken
World bank report on rethinking Governance, talks of providing safety and security to citizenry
as the foundation of modern states. Being aware of this fact, many states are taking steps in the
same direction, such as-
• Countering the fundamentalist ideologies and propaganda machineries with a more
humanistic ideology. For example, Using Sufism to counter Islamist fundamentalism.
• Providing aid and working on developing capability building: Many organisations are
working in Nigeria, Somalia, Syria and middle east to help impoverished and needy people
develop skills to earn livelihood. E.g, Bill and Milinda Gates foundation and UN HCR in Africa
and Somalia.
• Governments around the world should not pass anti-immigrant laws and infact, as
supported by data, allow intermingling of people so as to improve social cohesion. Regular
sensitisation in the schools, community centres etc shall be done for the same.
• De-radicalisation of youth is important. Ethical decision-making skills if imbibed from the
childhood will help curtail criminal tendencies.

3.4. Genocide
Genocide is the most heinous crime against humanity. The term was coined in 1943 by a
Jewish-Polish lawyer Raphael Lemkin. His efforts culminated into UN Convention on Genocide
in December 1948, which came into effect in January 1951
3.4.1. Ethical concerns associated with Genocide
• Narrow definition: The definition excludes targeted political and social groups. It also
excludes indirect acts against environment that sustains people and their cultural
distinctiveness.
• Self Interest predominates foreign policy: Many states are not friendly with minority
groups. So, the consensus making on action against nations perpetrating genocide is hard
to achieve.

DELHI JAIPUR PUNE HYDERABAD AHMEDABAD LUCKNOW CHANDIGARH


7 www.visionias.in # 8468022022 ©Vision IAS
Google it:- https://upscpdf.com
https://t.me/UPSC_PDF Download From > https://upscpdf.com https://t.me/UPSC_PDF

• Do states which are not directly affected by the genocide and resultant migration crisis, Student Notes:
ought to intervene? Right to protect is vague and states find themselves in a dilemma. This
is what happened in Rwanda Case and Rohingya case.
• How many deaths will be sufficient to declare the incidence as genocide? There is no
consensus on it.
• Whether the states should directly intervene or arm the groups so that they may protect
themselves? Use of violence for just ends puts state into means-end dilemma.

Genocide is a crime against humanity and therefore global community has a responsibility to
protect those who can’t protect themselves. Following measures can be considered-
• Broaden the definition of Genocide thereby including the extermination of political groups
and social groups as Genocide.
• Environment and culture are intrinsic to human survival. UN convention on genocide must
include any systematic erosion of these as genocide.

DELHI JAIPUR PUNE HYDERABAD AHMEDABAD LUCKNOW CHANDIGARH


8 www.visionias.in # 8468022022 ©Vision IAS
Google it:- https://upscpdf.com
https://t.me/UPSC_PDF Download From > https://upscpdf.com https://t.me/UPSC_PDF

• States need to intervene under the Responsibility to protect. Since injustice anywhere is Student Notes:
threat to justice everywhere.
• Neighboring states shall unite and build pressure on the state perpetrating Genocide.
Migrants should be protected and humanitarian aid should be provided to them by the
global community.
Above all nations should strive to preserve the values of global constitutionalism and global
communitarianism and do not allow such heinous crime to go unnoticed and unpunished.

3.5. Organised Crime: Drug and Human trafficking


Organized crime is a continuing criminal enterprise that rationally works to profit from illicit
activities that are often in great public demand. Its continuing existence is maintained through
corruption of public officials and the use of intimidation, threats or force to protect its
operations.
Drug trafficking- Overall, drug consumption has increased by 30% between 2009 and 2017 -
from 210 million to 271 million, in part as a result of global population growth.
Trafficking in person and smuggling of migrants- These two offences are often confused, as
they can occur along the same routes and affect the same people. For instance, what may begin
as a case of migrant smuggling may then transform into one of trafficking in persons, since
smuggled migrants are particularly vulnerable to becoming victims of trafficking.
3.5.1. Factors behind organized crime
The ethical view sees crime as placing one's own self-interest above the interests of others.
• Moral failure in decision making: Crime is a result of a moral failure in making decisions.
Crime takes place when a person fails to appreciate the wrongfulness of an act or its
impact on the victim.
• Role of external factors: The ethical explanation of crime appreciates that external factors
have a role to play in influencing some people toward criminal conduct, but these factors
do not cause the conduct by themselves.
• The ethical perspective also agrees that a free-will decision underlies criminal behaviour.
• Lack of moral and ethical education and conditioning is the reason why people often do
not make decisions in ethical terms but follow self-interest. This is the reason they fail to
understand the legitimate interest of other communities
3.5.2. Ethical concerns associated with organised crime
Ethical Concerns associated with organised crime at the International arena:
• Cultivation of greed: A crime-oriented society perpetuates selfish and greedy behaviour.
Organised crime feeds upon this greed or lust for money.
• Respect for autonomy: Organised crime disrobes a victim of her/his autonomy to take
decisions. For example, forced prostitution, or drug trafficking etc. do the same with the
victim. Bodily Integrity of the victim is compromised.
• Commodification of human organs: Organised trade in human parts feed upon vulnerable
population. It exploits the needs and situation of the victim.
• Exacerbates injustice: Organised crime exacerbates injustice in the society and is threat to
social cohesion and ethical functioning of the society. It breaks the moral fabric of the
society.
• For state: The dilemma is that to what extent it should strengthen the rule of law without
compromising the civil liberties of its citizens.
Organised crime is a challenge to all the nations and merely strengthening the rule of law or
systemic machineries will not bring the necessary change.

DELHI JAIPUR PUNE HYDERABAD AHMEDABAD LUCKNOW CHANDIGARH


9 www.visionias.in # 8468022022 ©Vision IAS
Google it:- https://upscpdf.com
https://t.me/UPSC_PDF Download From > https://upscpdf.com https://t.me/UPSC_PDF

3.5.3. Way Forward Student Notes:


Situational crime prevention requires that crime prevention techniques be directed at five
areas:
• Increasing the effort for offenders (e.g., target hardening, controlling crime facilitators).
• Increasing the risks (e.g., surveillance of offenders and victims, screening entrances and
exits).
• Reducing the rewards (e.g., removing targets, controlling markets).
• Reducing provocations (e.g., reducing temptations, avoiding disputes); and
• Removing excuses (e.g., setting clear rules, alerting conscience).
Ethical decision-making and reinforcement from an early age would help inculcate the notion
of personal and social responsibility for one's own behaviour, and a greater appreciation of the
harm caused.

3.6. International Aid


Foreign aid means transfer of money, goods or technical knowledge, from the developed to the
under-developed countries
3.6.1. Types and Objective of Foreign Aid
Foreign aid gained importance after the Second World War. It was required for the nation
building by cultivating political stability and development. States also misused it to garner
allies which lead to neo-colonialism. Some important forms of foreign aid are-
• Military aid: It helps in gaining allies. The only objective of this kind of aid is to strengthen
the military capability of their respective allies.
• Technical assistance: It is the least expensive among the aid programmes with big
benefits. It aims at providing technical know-how instead of equipment and funds.
• Economic aid: Loans are given to be repaid over a long period of time for the development
schemes to be run in the developing countries. But these loans also involve exploitation
• Humanitarian assistance: Humanitarian aids are the actions designed to save lives,
alleviate suffering and maintain and protect human dignity during and in the aftermath of
emergency situations. Humanitarian principles are rooted in international humanitarian
law of humanity, neutrality, impartiality and independence.
3.6.2. Ethical Arguments for Foreign Aid
Indeed, the idea behind foreign aid is always national interest of the donor. It has still been
justified on the following grounds-
• Global Justice: Foreign aid is justified on the basis of upholding values such as global
justice. It helps address poverty, destitution and reduces consequent conflicts.
• Principle of Sacrifice: It is the duty of well off to sacrifice some of their wealth to protect
those who can’t protect themselves. Providing assistance therefore becomes a
responsibility.
• To compensate nations affected by the repercussions of conflict in the neighboring nation,
for example middle eastern nations from Syrian war or South Asian nations from genocide
of Rohingya muslims. It reduces the pressure on the limited resources that a nation can
have.
Foreign aid has played a vital role in improving conditions of poor across the world.
3.6.3. Issues and Criticism
While on the one hand foreign aid is in demand by the developing nations, on the other, they
are also suffering from several ethical issues generated by it, some of which are as following-

DELHI JAIPUR PUNE HYDERABAD AHMEDABAD LUCKNOW CHANDIGARH


10 www.visionias.in # 8468022022 ©Vision IAS
Google it:- https://upscpdf.com
https://t.me/UPSC_PDF Download From > https://upscpdf.com https://t.me/UPSC_PDF

• Proliferation of monoculturalism- These programmes are often aimed at inculcating certain Student Notes:
form of culture and have low regards to indigenous culture in the targeted nations.
• Self-interest driven nature of foreign aid: Programmes were run throughout the world to
serve self-interests, for example, the eradication of malaria programmes, was run to spend
the huge stocks of DDT which were lying useless in the stores of developed nations
• Modern technologies are preserved for profit motives: Profit motive guides developed
nation behaviour. Obsolete technologies are transferred instead of advance, to the
developing nations.
• Blackmailing countries into alliance: The supplier country can direct the course of a local
conflict by simply replenishing or withholding supply and spare parts, or worse still by
threatening to switch her affections to the other side.
In this way, the recipient countries have to dance to the tune of the advanced supplier
countries. This means a serious encroachment upon their autonomy.

3.6.4. Foreign Aid and Neo-Colonialism


Neocolonialism is the practice of using economics, globalisation, cultural imperialism and
conditional aid to influence a country. It results in a relationship of dependence and
subservience. Foreign aid is essentially a part of neo-Colonialism exercised by the developed
nations.

DELHI JAIPUR PUNE HYDERABAD AHMEDABAD LUCKNOW CHANDIGARH


11 www.visionias.in # 8468022022 ©Vision IAS
Google it:- https://upscpdf.com
https://t.me/UPSC_PDF Download From > https://upscpdf.com https://t.me/UPSC_PDF

It basically aims at creation of political stability in the recipient State. Also, it seeks to change Student Notes:
domestic and foreign policy of recipient country. It is also distributed as a reward for becoming
an alliance partner and the recipient to achieve its objectives. For example, suppression of
rebellion in a nation.
3.6.5. Way forward
The greatest challenges for humanitarian aid and development in the form of financial
assistance are efficiency, effectiveness and the extremely complex political, economic, and
social side effects associated with them. The funds can be channelized in the following ways:
• By determining accurate assessment of need.
• By doing away with disparities in assistance within particular contexts, owing to political or
security constraints imposed on humanitarian organizations. Such disparities in the level of
assistance can be the reason for large population movements.
• There should be coherence between humanitarian law of humanity, neutrality, impartiality
and independence.
• Involvement of local administration and civil society in the humanitarian work.
Sustainable long-term development obviously depends essentially on the political and
economic framework and on the institutional and physical infrastructure a State is able to
provide.

3.7. NGO Colonialism and Foreign Aid


Proliferation of NGO’s has generated several ethical concerns. NGO’s are often seen as actors of
particular nation and its ideology. Some of the ethical concerns are discussed below:
3.7.1. Ethical Issues
• NGO colonialism acts as safety valve: When bombs were poured in Vietnam,
simultaneously USA sent its NGO’s such as CARE, American Red Cross etc. People were
uprooted from their life by bombings in Iraq and Afghanistan and NGO’s were providing
healing touch.
• Corruption: Only one percent of humanitarian fund reach the affected population.eg West
Africa during Ebola Crisis.
• Support for regime changes, lack of accountability and ongoing racism in the humanitarian
system has helped destabilize countries in the grip of natural disasters such as Haiti and
Nepal.
• Removes the onus of ethical governance from the state: The state feels less accountable
and if the regulatory mechanism is not strong, a weak state may cause siphoning off of the
funds.
• Extension of foreign policy: USA uses NGO’s as an extension of its foreign policy and EU as
a substitute. Humanitarian intervention is thus an intervention to change social cultural,
economic, political and environmental architecture of countries without their consent.
• Dependency on the foreign aid: The state starts to lose its independence and relies on
foreign aid for socio-economic polices. State also withdraws from social security field
leaving the citizens vulnerable for unethical practices. As happened in the post neo-liberal
reforms.
• NGO’s in order to harness the Foreign Aid at times try and inflate the data: For example,
many NGO’s inflate the data of malnutrition, infant deaths (under five), number of
dependent in shelter homes etc. It affects the development policy of affected nation.
However, NGO’s also have played vital role in enforcing accountability for the acts done by the
states and have provided necessary help in the war torn and conflict-stricken nations.

DELHI JAIPUR PUNE HYDERABAD AHMEDABAD LUCKNOW CHANDIGARH


12 www.visionias.in # 8468022022 ©Vision IAS
Google it:- https://upscpdf.com
https://t.me/UPSC_PDF Download From > https://upscpdf.com https://t.me/UPSC_PDF

3.7.2. Way Forward Student Notes:


• Humanitarian donors, policymakers and practitioners should change their current attitudes
and values, examine the ways to change the system itself and genuinely collaborate with
the affected people as equal partners.
• Regulatory authorities in the developing world need to enforce the accountability
mechanisms on the NGO’s in letter and spirit.
• Third party independent audits should be done so as to build trust in NGOs affairs.
• Sensitisation drives of the NGO workers shall be done at regular intervals so as address
insensitivity and cognitive dissonances.
NGO’s are required to supplement the work of developing nations. They help channelize the
necessary resources in time to the affected people and thereby contribute to strengthening the
administrative capacity. A balance is what that needs to be created for the betterment of the
civilians.

3.8. Pandemic and Ethics


COVID-19 brought to stand still lives around the world. All the material advancement that
humanity achieved since industrial revolution felt helpless in front of this unforeseen pandemic.
Apart from raising challenges for civilians and governments around the world, It also raised
some fundamental moral and philosophical issues, some of which are as following;
• Disproportionate burden on poor: How to respect commitments to social justice in the
face of the overwhelming and entrenched inequalities.
• Equitable access to healthcare: A major issues is how the vaccines, hospital beds etc.
should be awarded to patients.
• Ethics of public health action taken in response of breakout of pandemic. E.g. separation
measures such as quarantine, isolation, and social distancing; and control of international
travel and borders,
• Obligation among countries and the obligation of intergovernmental organisations: How
should governments balance their duties to their own populations versus duties to other
countries and population, and what role should international organisations such as WHO
play in addressing the cross-border risks and obligations.
• Pandemic also raised ethical issues of both procedural and well as distributive justice. E.g.
Should all lives be valued equally, or should there be a preference for saving those most
productive for society, which would generally imply young to middle-aged workers? Or
should there be a preference for the youngest based on an argument sometimes referred
to as “fair innings,” namely that older persons—say 65 years and older—have already had
their fair share of opportunities and that persons of 15 or 25 deserve a chance to have
theirs.
• The fine balance between reducing disease spread through isolation and travel measures
while protecting the right of individual to freedom of movement.
• Infodemic is another name given to the pandemic due to mass spread of false rumours and
news during the period. Truthfulness and objectivity was missing from social media.
COVID and Social Stigma
Social stigma in the context of health is the negative association between a person or group of people
who share certain characteristics and a specific disease. From the early days of the HIV epidemic to the
current coronavirus crisis, a major challenge around infectious diseases has been dealing with the deep-
seated stigma around affected populations. For example, LGBT population was stigmatized in the past in
connection with HIV outbreak.
In the context of COVID-19, broadly three sets of people are being stigmatized:
• People in quarantine (whether tested positive or not) are being socially discriminated.
• Health and sanitary workers: From doctors down to the frontline workers are facing stigma and
discrimination including violence at work.

DELHI JAIPUR PUNE HYDERABAD AHMEDABAD LUCKNOW CHANDIGARH


13 www.visionias.in # 8468022022 ©Vision IAS
Google it:- https://upscpdf.com
https://t.me/UPSC_PDF Download From > https://upscpdf.com https://t.me/UPSC_PDF

• Already vulnerable sections: Those who traditionally face discrimination like migrant workers, Student Notes:
people from the northeast, victims of religious or other persecution are being associated with the
disease and are facing undue stigmatization.
Why such stigmatization happens?
• Fear and anxiety: It is a disease that’s new and for which there are still many unknowns. Lack of
knowledge about the disease tends to create fear and anxiety around it. It is easy for the mind to
associate that fear with ‘others’, resulting in stigmatization of ‘others’.
• Fake news and false information: Stigma can also be the result of people’s poor knowledge about
COVID-19 and how it spreads. This is compounded by the large amount of fake news and false
information being generated.
o For example, certain communities and areas are being labelled as infected purely based on false
reports floating in social media and elsewhere.
o The proliferation of fake news can indirectly reinforce the stereotypes and prejudices prevalent
in our mind. For example, only people from certain community are contracting the disease.
• Labelling and association: Stigma arises when the virus and the person with the virus are conflated;
when we change the question from what’s to blame to who’s to blame.
o Calling COVID-19 "the Chinese virus" associates COVID-19 to Chinese people even though not
everyone of that descent or from that region is specifically at risk for the disease.
o Stigma can also occur after a person has been released from COVID-19 quarantine even though
they are not considered a risk for spreading the virus to others. This happens because s/he has
been labelled as infected.

3.8.1. Vaccine development and distribution


There is tremendous interest in the development of a COVID-19 vaccine, with more than a
hundred initiatives under way around the world. Even if one or more vaccines emerge, the
public-health problem will not be eliminated. But policymakers can avert some foreseeable
problems by starting to address key questions about financing and distribution now.
But why vaccine distribution requires ethical decision-making?
Governments, international agencies and health systems have an obligation to ensure, to the
best of their ability, adequate provision of health care for all. However, this may not be possible
during a pandemic, when health resources are likely to be limited.
• Setting priorities and rationing resources in this context means making tragic choices, but it
is essential that these tragic choices be ethically justified.
• Ethical justifications can be provided in various ways, it is important to understand which
systems suits the context best. For example, the allocation of different resources may find
ethical justifications in different principles or values.
• Most importantly, a standardized ethical framework that can be used as a guide for
decision-making process from hospitals to administrators.
What values could be the basis for deciding who should get priority?
• Equality: Each person’s interest should count equally unless there are good reasons that
justify the differential prioritization of resources.
o Characteristics of individuals, such as race, ethnicity, creed, ability or gender, should not
serve arbitrarily as the basis for the differential allocation of resources.
o This value could be the most appropriate guide the allocation of scarce resources
among individuals or populations who can be expected to derive the same benefit from
the resource, for example, vaccines among high-risk populations.
• Best outcomes (utility): This principle can be used to justify the allocation of resources
according to receiver’s capacity to do the most good or minimize the most harm, for
example, using available resources to save the most lives possible.
o This value could be the most appropriate guide for allocation of scarce resources that
confer substantially different benefits to different individuals, for example, ventilators
to those expected to derive the most benefit.

DELHI JAIPUR PUNE HYDERABAD AHMEDABAD LUCKNOW CHANDIGARH


14 www.visionias.in # 8468022022 ©Vision IAS
Google it:- https://upscpdf.com
https://t.me/UPSC_PDF Download From > https://upscpdf.com https://t.me/UPSC_PDF

• Prioritize the worst off: This principle can be used to justify the allocation of resources to Student Notes:
those in greatest medical need or those most at risk.
o This value could be the most appropriate to guide the allocation of resources that are
designed or intended to protect those at risk, for example, PPE for health care workers,
vaccines for those most at risk of infection and severe illness, or those most in need.
• Prioritize those tasked with helping others: This principle can be used to justify the
allocation of resources to those who have certain skills or talents that can save many other
people, or because something is owed to them on account of their participation in helping
others.
o This value could be most appropriate to guide the allocation of resources to health care
workers, first responders, etc.
What can be done to ensure that these principles are applied fairly?
• Ensuring Transparency: The decisions and their justifications should be made public. This
implies that the population should be informed about the criteria guiding the decisions.
• Encouraging Inclusiveness: Those affected by allocation decisions – including individuals,
communities or countries – should be able to exert at least some influence over the
decision-making process as well as the decision itself.
o This also implies that decisions should be open to challenge and potentially revisable,
perhaps through an appeal process.
• Ensuring Consistency: Decisions should be consistent so that all persons in the same
categories are treated in the same way. This implies that favoritism towards one’s own
family, religious or political compatriots, or otherwise, should not be exercised.
• Maintaining Accountability: Those making decisions about allocation must be accountable
for those decisions – that is, they should justify their decisions and be held responsible for
them. Taking into account these values, what could be the best possible course of action?

Conclusion
Ethical considerations are vital to decision-making about the deployment of vaccines in acute
humanitarian emergencies. A fair system engenders solidarity and trust, which are vital to the
successful and sustained collective response necessary for dealing effectively with any
outbreak. This allocation of a limited supply of vaccine calls for a fine balance between utility
and equality and fairness. Following could be done to ensure this:
• Vaccine Delivery Platform: which enables coordination between health management
authorities, local government and state government. Further, the platform could be
integrated with the information repository created for the vaccination distribution.
• Maintaining Information Repository for identification and accordingly optimization of the
vaccine delivery process. Also, it could create a list of people who need vaccination on
immediate basis in a given area.

DELHI JAIPUR PUNE HYDERABAD AHMEDABAD LUCKNOW CHANDIGARH


15 www.visionias.in # 8468022022 ©Vision IAS
Google it:- https://upscpdf.com
https://t.me/UPSC_PDF Download From > https://upscpdf.com https://t.me/UPSC_PDF

Student Notes:

3.8.2. Way forward


• Governments of wealthier nations should support poor, early affected countries out of both
ethical responsibility and self-interest. Governments bear a moral responsibility to identify
where social injustices are likely to occur as the result of a pandemic and to take
reasonable steps to prevent or reduce the worst among them.
• Beneficence, non-maleficence, respect for persons, and justice are the four core principles
that should guide international actions.

DELHI JAIPUR PUNE HYDERABAD AHMEDABAD LUCKNOW CHANDIGARH


16 www.visionias.in # 8468022022 ©Vision IAS
Google it:- https://upscpdf.com
https://t.me/UPSC_PDF Download From > https://upscpdf.com https://t.me/UPSC_PDF

• Principle of transparency and Principle of Participation: Information about the processes Student Notes:
and bases of decisions should be made available to the affected population. The
stakeholders should be involved, through appropriate institutions and means, in the
processes of formulating the objectives and adopting the policies.
International community should be working with utility orientation with efficiency to preserve
the life and liberty of the community. Principles of review, revisability and effectiveness should
be incorporated at the global level.

3.9. Ethics of Humanitarian Intervention


International humanitarian intervention to stop gross violation of human rights is supported as
the just cause criteria of the just war theory. Responsibility to protect is a fundamental tenet of
it.
3.9.1. Responsibility to Protect
The idea came in the background of Holocaust perpetrated by Nazis. The idea is to identify and
define crimes that have, to borrow a phrase from Michael Walzer (1977), ‘shocked the
conscience of mankind’ and protect populations from them.
What is Responsibility to Protect?
• Protection from Four Crimes: Each individual state has the responsibility to protect its
populations from genocide, war crimes, ethnic cleansing and crimes against humanity.
• Democratic and Responsible Use of Power: To use collective actions against the crime
against humanity through Security Council in accordance with the UN charter.
• It attempts to reconcile principles of human rights and international order by working
through states, and existing international mechanisms, to implement legal obligations.
Ethical Concerns
The concept is criticized for its narrowness- it relates only to the four crimes identified in the
2005 World Summit Outcome Document. The concept does not relate to threats to human life
stemming from natural disasters, diseases, armed conflict in general, or repressive measures
adopted by non-democratic forms of government. Other major concerns are as following;
• An interference into sovereign matters: Weaker and smaller states fear that in the garb of
Responsibility to protect developed nations will undermine their sovereignty.
• Blurring of domestic and international responsibilities: It blurs the line between domestic
and international responsibility. International affairs are thought to affect domestic concern
which is not true always. .
• The prevention of conflict and the protection of victims of human rights abuses became an
end in itself rather than part of a broader political or ideological project. It results in
concomitant loss of life and human suffering. For example, Drone Raids in Afghanistan and
middle east lead to collateral damage. (civilian lives)
• Invasion of other nations in the garb of humanitarian intervention is used to reap the
rewards at home; For example, Global War on terror and the Invasion of IRAQ.
There is a general consensus that western government policy-makers have, in the last decade,
explicitly taken on board normative and ethical concerns, shifting away from a ‘realist’
approach in which a more narrowly conceived national interest was the basis of policymaking.
This policy shift has meant that the declarations of ‘ethical foreign policy’ emanating from the
governments of leading world powers are often uncritically taken at face value and assumed to
be ‘simply the right thing to do’.

DELHI JAIPUR PUNE HYDERABAD AHMEDABAD LUCKNOW CHANDIGARH


17 www.visionias.in # 8468022022 ©Vision IAS
Google it:- https://upscpdf.com
https://t.me/UPSC_PDF Download From > https://upscpdf.com https://t.me/UPSC_PDF

3.10. Just War Concept Student Notes:

The aim of Just War Theory is to provide a guide to the right way for states to act in potential
conflict. It provides a useful framework for individuals and political groups to use for their
discussions of possible wars. The theory is not intended to justify wars but to prevent them, by
showing that going to war except in certain limited circumstances is wrong, and thus motivate
states to find other ways of resolving conflicts.

3.11. Global Commons


Global commons have been traditionally defined as those parts of the planet that fall outside
national jurisdictions and to which all nations have access. International law identifies four
global commons, namely the High Seas, the Atmosphere, the Antarctica and the Outer Space.

DELHI JAIPUR PUNE HYDERABAD AHMEDABAD LUCKNOW CHANDIGARH


18 www.visionias.in # 8468022022 ©Vision IAS
Google it:- https://upscpdf.com
https://t.me/UPSC_PDF Download From > https://upscpdf.com https://t.me/UPSC_PDF

• Guiding principle for these resource domains is Principle of common heritage. Student Notes:
• Some of the problems within the scope of global commons are for example; spread of
zoonotic diseases like Covid-19, greenhouse gas emission, biodiversity reduction,
overfishing and the accumulation of plastic waste.
Challenges in managing Global Commons
• The key challenge of the global commons is the design of governance structures and
management systems capable of addressing the complexity of multiple public and private
interests, subject to often unpredictable changes, ranging from the local to the global level.
• There are differences in the shared culture and expectations of resource users; more
localized commons users tend to be more homogeneous and global users more
heterogeneous. This contributes to differences in the possibility and time it takes for new
learning about resource usage to occur at the different levels.
• Moreover, global resource pools are less likely to be relatively stable and the dynamics are
less easily understood. Many of the global commons are non-renewable on human time
scales. Thus, resource degradation is more likely to be the result of unintended
consequences that are unforeseen, not immediately observable, or not easily understood.

Measures to overcome challenges


To manage our global commons, we may take following steps;
• Facilitate and accommodate the self-governance of local commons. Provide safeguards to
them to avoid exploitation and manage risks.
• At local level, initiatives and solutions can be developed that fit the local context.
• When expertise are not available, higher level organisations can faciliate learnings from
peers.
• Higher level authorities need provide insurance that the outcomes from the successes of
local level experimentations will not be grabbed by the outsiders.
To achieve coherence in global governance, all three dimensions of sustainable development –
sustainable economic growth, social inclusion and protection of the environment and the
global commons – need to be integrated at the global level.

DELHI JAIPUR PUNE HYDERABAD AHMEDABAD LUCKNOW CHANDIGARH


19 www.visionias.in # 8468022022 ©Vision IAS
Google it:- https://upscpdf.com
https://t.me/UPSC_PDF Download From > https://upscpdf.com https://t.me/UPSC_PDF

4. Doctrine of International Community Student Notes:

Former British Prime Minister Tony Blair propounded doctrine of international community Its
important principles are as following;
Six principles for a doctrine of international community and its institutions:
• reform of the system of international financial regulation;
• a new push on free trade in the WTO;
• a reconsideration of the workings of the UN;
• a critical examination of NATO;
• greater cooperation on meeting the targets of Kyoto;
• scrutiny into the issue of third world debt.
Examples:
• NATO’s attack on Serbia over Kosovo in 1999 established the rule of a humanitarian
intervention. It followed the worldwide guilt felt at the failure by the UN, or anyone else, to
intervene in Rwanda.
• Killing of Saddam Hussain.

4.1. Five rules of Intervention of International Community


Five rules for intervention of international community outlined by the former British PM were:
• be sure of your case,
• exhaust all other options first,
• ask if military operations can be "sensibly" undertaken,
• prepare for the long-term
• identify if your interests are involved.
The Global war on terror leading to invasion of Afghanistan, Iraq were justified by this doctrine.
It later on was heavily criticized when No chemical weapons were found in Ira

4.2. Global Communitarianism


Global communitarianism envisages a supranational community having a common definition of
good. Normally communities have their own definition of good. For example, caste system is
good from the point of view of traditional Indian society. However, its not tenable to authorise
communities to decide good. Therefore, a global community is envisaged under Global
communitarianism having transnational norms (such as protecting environment), global second
language etc.

4.3. Collective action in the times of COVID


The pandemic of COVID has started a global debate on the need for collective action to face
such global emergencies. The former UK prime minister Gordon Brown has called for a new
‘global government’ to deal with the crisis.
Importance of collective action in such emergencies
• To develop consensus - as the countries should work tandem in their response to this
pandemic.
o In such situations, countries should listen to international experts and take scientific
inputs, rather than work in silos based on their homegrown demands.
• Common but differentiated responsibility- as all the countries may not have the kind of
resources required to deal with his menace, such as-
o Capital resources- In low income countries, assistance is required from international
community to provide for safety nets, cash transfers and food & logistic costs. E.g. the

DELHI JAIPUR PUNE HYDERABAD AHMEDABAD LUCKNOW CHANDIGARH


20 www.visionias.in # 8468022022 ©Vision IAS
Google it:- https://upscpdf.com
https://t.me/UPSC_PDF Download From > https://upscpdf.com https://t.me/UPSC_PDF

Central African Republic had just three ventilators to treat patients in this crisis. Student Notes:
o International supply chains need to be put on a wartime footing to produce the
supplies needed, from masks to ventilators, to treat rapidly growing numbers of
patients who need intensive care.
▪ Collective action is also needed to clear barriers to the development, manufacture
and equitable distribution of a vaccine.
• Sharing of information- in the form of scientific research, findings and data in different
parts of the world.
o Crowdsourcing-e.g. In the recent crisis, more than 45,000 research papers have been
collected in an open source database, which could prove useful for the entire humanity.
o Learning from past experiences e.g. Liberia’s experience during 2014 Ebola outbreak,
shows that national leadership, community driven programs and local solutions helped
slow the spread of disease.
o Success stories-Any kind of innovation, technology, idea which can be replicated in
other countries should be shared among the global community.
• Stop reactionary forces from misusing this situation- for which the countries need to come
together, such as-
o Private Sector-The big pharmaceutical companies should not be allowed to make hefty
profits from drugs and medical devices needed during the pandemic, rather work for
the public good.
o Terrorism-There are reports that the current crisis could be exploited by some terrorist
groups to further their agendas.
• Prevent erosion of public trust- If such emergencies are not tackled effectively by the
governments.
o India- There was a mass exodus of migrants from urban centres with many travelling
foot with limited help from government.
o Philippines- There was a largescale protest and riots against the government for lack of
food and relief supplies during the COVID-19 lockdown.
o Spain and Italy- Given their situation, the analysts believe the government’s response
might create anarchy and put a question on the entire notion of social contract.
Globalism vs. Nationalism debate in the context of vaccine distribution
• Several countries like Britain, France, Germany and the US have entered into pre-purchase
agreements with Covid-19 vaccine manufacturers, a development that has come to be known as
“vaccine nationalism”.
• There are fears that such advance agreements will make the initial few vaccines unaffordable and
inaccessible to everyone apart from the rich countries, in particular to countries with fewer
resources and bargaining power.
• In this context, WHO has warned that hoarding possible COVID-19 vaccines while excluding others
would further deepen the pandemic.
• The possible counter to vaccine nationalism is global collaboration, which is being done through
the WHO-backed COVAX Facility mechanism.
• The facility aims to procure at least two billion doses of COVID-19 vaccines by the end of next year
for deployment and distribution mainly in the low- and middle-income countries
This crisis presented an opportunity to correct the injustices present in global order. The
international community should focus on creating systems, structures and policies that can
always protect the marginalised in need and assure everyone to live with dignity.

5. Moving forward towards Global Constitutionalism


Global constitutionalism believes in the existence of principles of constitutionalism which are
followed around the world and by the global governance through its institutions and norms.
Some of important principles are as following;

DELHI JAIPUR PUNE HYDERABAD AHMEDABAD LUCKNOW CHANDIGARH


21 www.visionias.in # 8468022022 ©Vision IAS
Google it:- https://upscpdf.com
https://t.me/UPSC_PDF Download From > https://upscpdf.com https://t.me/UPSC_PDF

• Rule of law Student Notes:


• Separation of power/checks and balances
• Protection of human rights,
• Democracy and
• Solidarity.
Ethical Significance of Global Constitutionalism:
• Provides objective ethical standards through which domestic as well as international
endeavors of the nations can be judged. e.g. Any curtailment of individual freedom and
press freedom is inimical to human growth and therefore against basic human rights.
• Helps civilians understand where they stand globally interms of having and exercising basic
freedoms and rights.
• Provides moral principles that can be a guide for international institutions, like
international courts of justice.
Nation states do find global aspirations like such inimical to their growth and power. So
therefore, there is conflict in desires. It’s a gradual process and hopeful in the future we may
have strong global institutions founded on the principles of Global constitutionalism.

6. Previous Year UPSC Mains Questions


1. At the international level, bilateral relations between most nations are governed on the
policy of promoting one’s own national interest without any regard for the interest of other
nations. This led to conflicts and tension between the nations. How can ethical
consideration help resolve such tensions? Discuss with specific examples. (2015)
2. Strength, peace and security are considered to be the pillars of international relations.
Elucidate. (2017)

7. Previous Year Vision IAS Mains Test Series Questions


1. Income inequality, resource mismanagement and health hazards are some of the
negative consequences of globalization. In this context, what are some of the ethical
dilemmas that civil servants face today? How has the approach to handling these
undergone a change?
Approach:
• Explain the key ethical dilemmas that civil servants face due to the changes brought
by globalization.
• Then, talk about the difference in approach to handle the challenges and resolve
the ethical dilemmas.
Answer:
The process of Globalization is associated with both benefits with increasing pace of
poverty reduction and high growth rates as well as drawbacks such as increasing
inequality and resource mismanagement. Globalization has impacted not just
economies but also politics, societies and cultures across the world. This has brought
new challenges for the civil services in the form of following ethical dilemmas.
Ethical dilemmas in era of Globalization
• In the era of globalisation, the role of the State has changed. Economic
liberalisation has led to – on one hand, the diminishing role of state and on the
other, increasing role of private sector. The basic objective of private enterprises—
maximizing profits—does not always coincide with broader social concerns. Therein
lies the ethical dilemma of civil servants of balancing these two priorities, profits –

DELHI JAIPUR PUNE HYDERABAD AHMEDABAD LUCKNOW CHANDIGARH


22 www.visionias.in # 8468022022 ©Vision IAS
Google it:- https://upscpdf.com
https://t.me/UPSC_PDF Download From > https://upscpdf.com https://t.me/UPSC_PDF

for sustaining economic activities and addressing social concerns - as part of the Student Notes:
welfare state.
• Globalization has resulted in the marginalisation of a section of society who have
not been able to take advantage of the economic opportunities provided by
globalization. The ethical dilemma here is the need for showing empathy and
compassion in helping them while at the same time following the rule of law.
• Fast pace of globalisation is mainly driven by the rapid advancement of
communication technology. Civil servants too have had to adopt technology in
public administration even if they personally do not like it. The ethical dilemma
here is between the personal choice of status quo and professional demands for
infusing technology in day-to-day operations.
• Globalization era is also associated with increasing demands for transparency and
accountability. The ethical dilemma which civil servants face is judicious use of
discretionary power while at the same time upholding the principles of
transparency and accountability.
Handling these challenges and ethical dilemmas require a multi-pronged approach for
upholding the foundational values of civil services. Therefore, there has been change in
the approach in the following manner:
• Civil servants are shifting their orientation from being controllers to facilitators and
from being providers to enablers. This way the State can focus more on the social
sector while economic activities are left to the private sector.
• Values such as non-partisanship, empathy and compassion are now considered as
foundational values along with the traditional values of integrity, objectivity and
public service dedication.
• Civil servants are being equipped with the necessary skills and capabilities to
master new technologies and new styles of functioning. This will bring about an
attitudinal change required to adopt technology.
• Along with the code of conduct which may not be able to cover every possible
scenario, code of ethics is also being focused upon so as to guide the actions of civil
servants while acting under discretion.

2. “A world without nuclear weapons would be less stable and more dangerous for all of
us.” Critically examine the ethical dimensions of the given statement from the
perspective of international relations.
Approach:
First part of the answer should bring out how the issue of nuclear weapons has its own
moral dimensions. Then one should examine the different ethical dimensions of the
statement. They may be:
• Stability in world affairs v/s always living under fear.
• If Nuclear weapons in themselves are moral or immoral
• From the point of view of Utilitarianism and deontology.
• Money for weapons v/s money for poor and other developmental objectives
Suitable examples provided to support the arguments. Preferably the conclusion
should favor the disarmament.
Answer:
Nuclear weapons have the potential to destroy the entire ecosystem of the planet.
However, a handful of states insist that these weapons provide unique security
benefits, but reserve the sole right to possess them. Hence, the possession of nuclear
weapons leads to numerous moral/ethical dilemmas.

DELHI JAIPUR PUNE HYDERABAD AHMEDABAD LUCKNOW CHANDIGARH


23 www.visionias.in # 8468022022 ©Vision IAS
Google it:- https://upscpdf.com
https://t.me/UPSC_PDF Download From > https://upscpdf.com https://t.me/UPSC_PDF

The first question is whether the nuclear weapons are moral or immoral in themselves. Student Notes:
According to ethical theories, since morality cannot be attributed to non-human things,
hence nuclear weapons in themselves are neither evil nor good.
According to proponents of nuclear weapons, these weapons create deterrence and
stabilize the world order. Proponents of deterrence claim that nuclear weapons are not
so much an instrument for the waging of war but political instruments "intended to
prevent war by depriving it of any possible rationale.” They argue that nuclear weapons
deterred the full-scale war between USSR and US during the period of cold war.
But it can also be argued that nuclear weapons create an environment of constant fear
and jeopardise the life of millions of innocent people. Living constantly under fear
subdues the free will; it is no way to maintain a world order. It erodes the dignity of
human life.
From the utilitarian perspective, while nuclear weapons give a sense of security to the
nations, which possess them, but it instills fear of complete destruction in the mind of
billions. Even the citizens of nuclear-armed states cannot be sure of their safety. Hence,
on the touchstone of ‘maximum good to maximum people’ nuclear weapons falter.
Similarly from deontological perspective, it is the duty of the governments to make
their citizens empowered and free from fear. But nuclear weapons do exactly the
opposite. Although they lead to some stability in world relations but leave the people
incapable of defending themselves in the case of nuclear war. Nuclear weapons also
use human life and emotions as means; hence they also fail the deontological test.
Another dimension could be whether the money used for production of nuclear
weapons can be put to better use. If money is saved from refraining from arms race, it
can be used to strengthen the social welfare mechanism of governments. Definitely
spending on social upliftment is more moral than spending on weapons, which are
never supposed to be used.

3. The increasing speed and scope of globalization has raised several ethical concerns.
Analyze with examples.
Approach:
It can be seen that even though the world has become a better place to live in terms of
economic development due to the fruits of globalization, in terms of peace,
environment, human rights etc a number of ethical concerns have been raised. The
answer should focus on each of these issues such as the climate change, disarmament,
IPR etc. Students should elaborate on a few of them. List have been provided with
elaboration on a few of them.
Answer:
As interaction, competition and collaboration among the nations are continuously
increasing we find that it has raised many ethical issues in the global arena. It is
observed that though the world is becoming a better place to live in terms of economic
prosperity, but in terms of equality, peace, environment and human rights, there have
been number of issues involving some ethical concerns also. Some of them being:
Climate Change: Countries are divided on the issue of common but differentiated
responsibility and on technology transfer and funding arrangements. It is the
responsibility of both the big and the smaller powers to ensure that we leave behind a
world that is safe for future generations to live in.

DELHI JAIPUR PUNE HYDERABAD AHMEDABAD LUCKNOW CHANDIGARH


24 www.visionias.in # 8468022022 ©Vision IAS
Google it:- https://upscpdf.com
https://t.me/UPSC_PDF Download From > https://upscpdf.com https://t.me/UPSC_PDF

Disarmament: World today has become a treasury of all the kinds of mass weapons of Student Notes:
destruction. Ironically the cause of disarmament at the international stage is being
promoted by those states, which have massive reserves of nuclear armaments, missiles
and nuclear powered submarines. Also, countries like the USA impose economic and
other sanctions on countries like Iran to prevent them from developing nuclear
weapons. There are unanswered questions like how it is ethical for a country to impose
sanctions on the others without discarding their own weapons of mass destruction.
IPRs: The developed countries are depriving the poor countries from accessing the new
technologies by the restrictive clauses of IPRs. Ironically, the same restrictive provisions
are also applied on things like the life saving drugs. In this debate, it is essential to
determine whether it is justifiable for a country to defend its IPRs on the ground of
commercial benefits, or they should ethically share the technology for the greater
cause of the humanity.
Trade Negotiations: The Doha Round of world Trade Organization is pending to reach a
consensus. In this context, the basic question involved is whether the developing
countries are ethically and morally correct when they demand higher concession in
international trade.
Outer space
Humanitarian interventions
Management of Common goods
In such issues it has been observed that countries have gone on to give more
importance to national issues than to ethical responsibility as per humanitarian values.
This, in the long term, would be detrimental not only for the other counties but for the
country in question also. It is, therefore, important to consider the ethical grounds and
take a holistic approach to all issues in order to ensure equitable development.

4. The Vienna Convention on Diplomatic Relations provides blanket cover to the


activities of diplomats and their family members? What are ethical issues arising out
of such a wholesale cover? What would you suggest to address those problems?
Approach:
• Discuss the concept of diplomatic immunity and provisions in Vienna Convention.
• Bring out the ethical issues with help of examples of abuses of diplomatic immunity
by diplomats and their family members.
• Suggest measures to address the problems w.r.t. changes to be made in Vienna
convention and also efforts to be taken by countries.
• Conclude with a balanced opinion.
Answer:
Vienna Convention on Diplomatic Relation 1961 defines, characterizes and lists the
powers and immunities provided to the serving diplomats of the various countries.
Diplomatic immunity is based on the principle that diplomats should be able to
function without fear in a foreign country. Under Vienna Convention the diplomatic
agents are completely immune from civil as well as criminal jurisdiction of receiving
state for any act done either in official or personal capacity. The family of diplomat
enjoys same immunity status as that of agent.
Various cases of abuses of diplomatic immunity bring out the issues such as:
• Immunity has accorded impunity to diplomats and their family members even in
the cases of grave crimes such as rape, murder, imposition of slavery etc., for e.g.

DELHI JAIPUR PUNE HYDERABAD AHMEDABAD LUCKNOW CHANDIGARH


25 www.visionias.in # 8468022022 ©Vision IAS
Google it:- https://upscpdf.com
https://t.me/UPSC_PDF Download From > https://upscpdf.com https://t.me/UPSC_PDF

immunity claimed by diplomat from Saudi Arabia against allegations of rape of two Student Notes:
Nepali women, diplomat from Georgia claimed immunity in a case of drunk driving
in which he killed a girl.
• Since diplomatic immunity shields accredited persons against domestic jurisdiction,
it practically interferes with intended operation of a legal order.
• Diplomatic immunity often comes in collision with principles of natural justice and
human rights. The aggrieved individual is often left with no course of action to
achieve justice and has no remedy for compensation.
• The diplomats are representatives of their country; while the diplomatic immunity
does insulate them from harm; it brings bad reputation to their country and a blow
to the bilateral ties.
Following actions could be taken to address the problems:
• A permanent international diplomatic criminal court can be set up with mandatory
jurisdiction over diplomats accused of criminal acts.
• A bilateral claims fund can be maintained by countries to compensate those injured
by diplomats.
• Consular staffs are not indemnified from serious criminal offences. This provision
should be extended to diplomatic immunity.
• Diplomatic immunity can be limited to the serving diplomat rather than extending
it to their family which are involves in heinous criminal offences.
• The sending nations should adopt a practice of trying diplomats once they return.
• The Vienna Convention needs to be revisited and reformed to ensure that it does
not function as a shield for delinquent diplomats.
Immunity is essential for diplomats to work with freedom, however effective
mechanisms should be put in place to prevent and penalize the abuses of such
immunity. The responsibilities lies with the diplomats to pay respect to local laws while
pursuing their duties.

5. A broad ethical framework as a guiding light for international relations will not only
ensure harmonious relations between nations but will also lead to progress of the
human race. Discuss with examples.
Approach:
• Highlight, with examples, the ethical dimensions in international relations.
• Discuss how an ethical framework can act as a guiding light for harmonious
relations.
• Conclude with overall impact on people of such ethics based relations between
nations. Examples from climate change or nuclear arms or trade pacts negotiation
can be given.
Answer:
Relations between sovereign states are largely historical in nature. However, economic
and strategic considerations have led to formation of new and often opportunistic
alliances in recent past. In international forums, countries mostly negotiate in groups
based on their own perception of what is good – economically or strategically, for
them. This approach ignores the larger ethical framework to take decisions which may
be good in the long term for all and rather relies on short term good for a few.
A broad ethical framework for international relations can be framed keeping the
following principals at hand:

DELHI JAIPUR PUNE HYDERABAD AHMEDABAD LUCKNOW CHANDIGARH


26 www.visionias.in # 8468022022 ©Vision IAS
Google it:- https://upscpdf.com
https://t.me/UPSC_PDF Download From > https://upscpdf.com https://t.me/UPSC_PDF

• Equity, Justice and Human Dignity should form the bedrock of international Student Notes:
negotiations, conventions, agreements, treaties and protocols.
• Upholding these values in face of adversities like economic turmoil, climate change
or terror must inform the decisions taken. Issues must be resolved morally and not
merely politically.
• Equally important is transparency, which makes the whole decision making process
more acceptable.
• Cooperation between nations is crucial as number of human lives depend up on
success/failure of decision. As such, early and time-bound conclusion of
negotiations should be adhered to.
• The international community has a responsibility to assist the state to fulfill its
primary responsibility of protecting its citizens (As envisaged in Responsibility to
Protect (R2P) in UN resolution).
• For e.g., failure to reach consensus on Syrian conflict has led to loss of number of
lives; Climate change negotiations have produced too little too late, the result of
which is imminent threat of drowning faced by Pacific Island Nations. Similarly,
national anti-corruption measures need reinforcement at the international level
with mutual assistance and cooperation.
Examples:

Challenge Ethical Solutions

1. Terrorism: • Promote inter-faith dialogue;


• Adopt a multicultural approach;
• Emphasize secular values like peace, truth,
tolerance, honesty, non-violence,

2. Climate Change: Exploitation • Equitable distribution of responsibilities.


of resources; luxury v/s • Sustainable development through Recycling,
survival; Reuse, Optimisation and conservation.

3. Poverty and Financial • Issues must be discussed at international forums


Instability: Socio-economic like WTO, UN.
inequalities, insensitivity. • Unilateral/Multilateral actions should be
discouraged.

4. Disarmament : Morality of • Encouragement of principle of peace, harmony,


nuclear weapons, non- love and compassion
violence • Cooperation should give precedence over
competition

5. Organized Crime, Drugs, • Dignity of human life should be upheld under all
Human Trafficking costs with zero tolerance for crime.
• Mutual assistance

By overcoming these challenges through the principles of ethics, number of lives can be
positively changed. Decisions based on ethical principles are more logical and easy to
accept. However, political considerations at home usually mar the ability of negotiators
to arrive at them. Taking the opposition parties, private entities, NGOs, etc, in
confidence through transparency can remove these hurdles.

DELHI JAIPUR PUNE HYDERABAD AHMEDABAD LUCKNOW CHANDIGARH


27 www.visionias.in # 8468022022 ©Vision IAS

Google it:- https://upscpdf.com


https://t.me/UPSC_PDF Download From > https://upscpdf.com https://t.me/UPSC_PDF

6. Around the world, more than 125 million people need humanitarian aid. With special Student Notes:
reference to financial assistance, discuss the various ethical issues involved in
addressing the needs of those in dire circumstances. What measures can be
undertaken to ensure that donations and aids for such purposes are thoroughly
channelled to meet their targets.
Approach:
• Introduce your answer with humanitarian aid and the types of assistance or aids
available.
• Write the ethical issue involved with the humanitarian aid and assistance to the
areas of disturbance and calamities.
• Write measures to be taken for better channelization of donations and aids.

Answer:
Humanitarian aids are the actions designed to save lives, alleviate suffering and
maintain and protect human dignity during and in the aftermath of emergency
situations. Humanitarian principles are rooted in international humanitarian law of
humanity, neutrality, impartiality and independence.
In the areas of disasters or natural calamities the primary aim is preventing human
casualties and ensuring access to the basics for survival: water, sanitation, food, shelter,
and health care. Whereas, in the areas of conflict/war torn regions, the priority is to
assist people who have been displaced, prevent the spread of conflict, support relief
work, financial assistance and prepare for rehabilitation.
Ethical issues related with humanitarian assistance
• Assistance with certain conditionality- World organisations like World Bank, IMF or
some humanitarian organisations provide assistance to the war torn countries on
the basis of fulfilment of certain conditions like “peace conditionality”. This can
indeed force adherence to the peace process or impose reconciliation. But, we
have witnessed that the cooperation from local authorities was often pure lip
service in order to get rewards from reconstruction funds.
• Ill targeted and Misuse- It is not necessary that the assistance shall be used in the
relief work only. Ethical dilemma persists for whether one should assist the conflict
region even after knowing that the finances may not be used in the pre-decided
work.
• Funding to terrorism- Humanitarian organisations have to collaborate with the
rebel groups to intervene for assistance which can be used for their terror activities
in the region.
• Fuel to war- Humanitarian aid has been accused of fuelling war economies and
prolonging conflict by providing assistance, directly or indirectly, to combatants and
their military operations. The conflicts in Somalia, Liberia and Angola are usually
mentioned as prime examples.
• Implicit recognition to rebel groups- The negotiation with the groups in charge of a
certain area means giving recognition to their authority and legitimacy.
• Benefits from assistance- The groups might derive considerable financial benefits
from humanitarian operations by imposing charges on transports, levying taxes on
imports and employees’ salaries, and collecting rent for warehouses, offices and
residences
• Dilemma between neutrality and political activism

DELHI JAIPUR PUNE HYDERABAD AHMEDABAD LUCKNOW CHANDIGARH


28 www.visionias.in # 8468022022 ©Vision IAS

Google it:- https://upscpdf.com


https://t.me/UPSC_PDF Download From > https://upscpdf.com https://t.me/UPSC_PDF

Measures to be taken Student Notes:


The greatest challenges for humanitarian aid and development in the form of financial
assistance are efficiency, effectiveness and the extremely complex political, economic,
and social side effects associated with them. We can channelize funds by following
ways:
• By determining accurate assessment of need.
• By doing away with disparities in assistance within particular contexts, owing to
political or security constraints imposed on humanitarian organizations. Such
disparities in the level of assistance can be the reason for large population
movements.
• There should be coherence between humanitarian law of humanity, neutrality,
impartiality and independence.
• Coordination of activities among humanitarian organisations, the military and
political intervention forces.
• Involvement of local administration and civil society in the humanitarian work.
• Conditions should include requests for active participation in specific aspects of the
peace process.
Sustainable long-term development obviously depends essentially on the political and
economic framework and on the institutional and physical infrastructure a State is able
to provide.

7. Analyse the ethical dimensions of using nuclear deterrence as a self-defense strategy.


Approach:
• Explain the meaning of nuclear deterrence.
• Discuss the pros and cons of nuclear deterrence from ethical perspective.
• Conclude by giving your own suggestions.
Answer:
Nuclear deterrence is a psychological phenomenon which gained prominence during
the Cold War era. The strategy involves deterring an aggressive power from taking
harmful action or preventing the other state from resorting to the threat of use of
military force in pursuit of its own foreign policy goals. The success of deterrence lies
not only on the retaliators capability, but on the persuasive appeal of the impending
threat. The opponent must perceive the retaliatory threat as legitimate and serious.
Thus, it acts as a potent self-defense strategy of modern times with many countries
vying for nuclear weapons to protect themselves. For example, North Korean Missile
programme.
However, nuclear deterrence comes at a huge ethical, economic, welfare and
environmental cost. It has several ethical dimensions associated to it that demonise it
as well as give it credibility.
Ethical arguments in favour:
• From the utilitarian perspective, it aims to bring long term peace.
• It strives to maintain international peace and stability.
• The fact that there has not been a war between nuclear-armed states due to fear of
mutually assured destruction implies that deterrence has prevented aggravation of
conflicts.
• It has indirectly saved millions of lives as states otherwise would have resorted to
conventional warfare.

DELHI JAIPUR PUNE HYDERABAD AHMEDABAD LUCKNOW CHANDIGARH


29 www.visionias.in # 8468022022 ©Vision IAS

Google it:- https://upscpdf.com


https://t.me/UPSC_PDF Download From > https://upscpdf.com https://t.me/UPSC_PDF

Ethical arguments against: Student Notes:


• It is intrinsically wrong to put lives of other human beings at risk on the back of
nuclear deterrence.
• It is immoral to detonate an atomic weapon due to both short and long-term
catastrophic effects.
• Even with best intentions amongst nuclear-armed states, there are always risks of
accidents and inadvertent escalations that may trigger nuclear exchanges.
• Possibility that nuclear-armed states may go rogue, collapse, or fail to prevent their
arsenal from falling into hands of terrorists, cannot be ignored.
• Development and acquisition of deterrence requires deployment of huge human,
technological and financial resources. These have an opportunity cost in terms of
education, health, infrastructure and quality of life.
Though there may be apparently few benefits of nuclear deterrence, but certainly they
come at a price. Disarmament can be seen as the moral alternative to deterrence
because the worst possible outcome is less catastrophic. Recently concluded Treaty on
the Prohibition of Nuclear weapons is a right step in the direction and international
disarmament seems a distant but possible pursuit now.

8. Many argue that there are times, when war is morally permissible, and even
obligatory. Critically discuss.
Approach:
• Introduce the answer by explaining ethical dilemmas regarding a war.
• Then mention how a war can be just and morally permissible.
• Conclude by critically evaluating the premise that a war be obligatory and moral.
Answer:
In its raw form, war is nothing but a duel at an extensive scale. It is an act of violence to
compel our opponent to fulfill our will. Clausewitz defined war as a mere continuation
of policy by other means. Thus, war becomes a political instrument to achieve one’s
goals. Use of this instrument is many times conditioned on exhaustion of all other
instruments to achieve the same political goal. In view of subjective nature of
‘exhaustion of all other instruments’, war is many a times resorted to without active
effort to avoid it. Thus, there are three moral positions that can be taken in context of a
war. The pacifist perspective completely rejects the morality of war, from the
perspective of a realist, war is not a moral enterprise and thirdly there is the middle
approach of a just war. It states that under certain conditions war may be morally
permissible or even necessary. The ‘Just War Theory’ of St. Augustine (5th Century)
helps in ethical assessment of a war.
• As per the theory, a war is just if the reasons to go to war, the conduct of war and
post war resolution is right. War is ethical and just, if
o Waged by legitimate authority (e.g. after UN sanction)
o Has a just cause (e.g. against a State that indulges in human rights abuse)
o Waged with right intention
o Is the last resort
o Be proportional (e.g. no use of nuclear weapons against conventional weapons)
o only unjust combatants are legitimate targets of attack
• Similarly, Mahabharata outlines the principles and contours in conduct of a just war.
Some rules propounded were –armies were allowed to collect bodies, personnel
could meet for negotiations etc.

DELHI JAIPUR PUNE HYDERABAD AHMEDABAD LUCKNOW CHANDIGARH


30 www.visionias.in # 8468022022 ©Vision IAS

Google it:- https://upscpdf.com


• In Statecraft, war is obligatory and moral when a State is defender, not an Student Notes:
aggressor. For – example India has fought 4 wars since independence. And it has
never been an aggressor. Its actions were aimed at defending its citizens, territory,
resources and sovereignty.
• A war might be ethical but the means to wage the war is seldom principled
especially in modern warfare which indiscriminately uses landmines, tortures,
chemical and biological weapons and non-targeted bombings through drones. That
the war is “just” is also questionable. For instance, though the professed aim of
2003 Iraq war was to liberate Iraq, usher democracy and eliminate biological
weapons, none of the these aim was achieved and sufferings of Iraqis continue till
date. In fact, there was not even a UN sanction of the war before starting it.
• Even grounds of humanitarian interventions are not equal and proportional. Ex-
NATO intervened in 1999 justifying the campaign in Kosovo as a "humanitarian war"
but no one took cognizance of 1994 Rwanda genocide.
Hence, war in modern times is one of convenience to achieve geo-political goals,
sometimes garbed under the facade of ethics and human rights. Justifying interventions
has become increasingly difficult and a general trend has been towards avoiding war at
all costs because of the catastrophic after-effects it has. Therefore, the character of war
is changing fast and the ethics needs to keep pace with this change. Else a war remains
inherently unethical.

9. While in principle most nations claim commitment to universal values, in practice


these values are honoured more in breach than in the observance. In context of this
statement, comment on the relevance of values in foreign policy.
Approach:
• Introduce the answer by explaining universal values and discuss how countries
adhere to them.
• Explain the role of values in foreign policy.
• Conclude the answer by mentioning how India abides by these values while dealing
in international relations.
Answer:
A value is a universal value if it is recognized and considered significant uniformly by all.
In the arena of international relations, nation states recognize peaceful resolution of
international conflict, quest for justice, respect for the dignity of human life, sovereign
equality, and humanitarian cooperation and assistance, amongst others as universal
values. While most of these values existed as customs since ancient times, their
codification became significant in the aftermath of World War II.
More breach than observance
Though, most nations claim their commitment to the universal values, but on many
occasions, they find it difficult to conduct their foreign policy based on these values. It
is because the foreign policy in contemporary times is governed by realpolitik which
places national interest on top. International relations are conducted with self-interest
and in the background of war or a threat of war. This is evident from the way the power
is exercised or projected by instruments such as joint military exercises or imposition of
" sanctions' or higher tariff on imports. However universal values are critical factors in
dealing with global issues like the climate change or Human rights crisis in some states
or communicable disease and medical research.

DELHI JAIPUR PUNE HYDERABAD AHMEDABAD LUCKNOW CHANDIGARH


31 www.visionias.in # 8468022022 ©Vision IAS
For example, while most countries pledge their commitment to non-interference and Student Notes:
peaceful conduct of foreign policy, in reality, they are involved in power maximization
which pits them against their co-aspirants. Similarly, while most countries pledge
commitment to environmental protection, in reality their focus is solely on economic
growth, even at the cost of environmental destruction.
In certain cases, some states are compelled to put aside these values in their quest for
survival and dignity. For example, India’s pursuit for Nuclear power in spite of its
commitment to non-violence and peace.
Relevance of values in foreign policy
Since countries are sovereign entities in themselves, there is no external body that can
dictate their actions. Still, despite few aberrations, the world order has shown
remarkable tenacity to stick to certain core universal values because they give
continuity and predictability in negotiations with a foreign power. In fact, trust is the
key building block of international relations. Pursuing short-sighted interests may bring
immediate benefits to countries but it undermines the trust of the world order in them.
If a country undermines a treaty or unilaterally backs-off from it, it disincentives other
as well. After witnessing two world wars in 3 decades, any conflict of such scale for
next 70 years has been avoided, in part due to this adherence. Democracy is one such
universal value to which many erstwhile Asian and African colonies have shown
allegiance to.
India’s track record in observance of universal values is second to none. Despite having
a hostile neighborhood, India has never been an aggressor. In addition to this
humanitarianism has been core of India’s foreign policy. It has been home to highest
number of refugees coming from all around the world.

8. Previous Year Vision IAS Mains Test Series Case Studies


1. You are representing India in an international bidding for oil exploration in a country.
Other, richer countries are also bidding for the project. You are sure that your bid of
exploration is better as well as cheaper than that of others, and that you will
definitely win the bid. A day before the auction, you come to know that other
countries are employing every means, including bribing the authorities for being
successful. Some of the officials of the home country have also contacted you and
made some demands in exchange for assurance of India winning the bid. You are
aware of the criticality of this bid in terms of domestic economic and strategic
implications. Based on above information, answer the following questions.
(a) Specify the ethical dilemma(s) that you face in this situation.
(b) Do ethical concerns really matter in international transactions or are they
secondary to domestic interests?
(c) What will be your course of action in the above situation? Justify with merits and
demerits.
Approach:
• Identify the ethical dilemmas that you face.
• Highlight the importance of ethical concerns in international transactions vis a vis
domestic interest.
• Then mention the course of action that you would follow. Justify it by taking into
account the merits and demerits of the decision.
Answer:
a) The situation in this case study presents the following ethical dilemma:
The dilemma is whether to pay the bribe vs being upright and avoid the temptation to
pay.

DELHI JAIPUR PUNE HYDERABAD AHMEDABAD LUCKNOW CHANDIGARH


32 www.visionias.in # 8468022022 ©Vision IAS
The former action may help India win the bid, but it will be an unethical course of Student Notes:
action and may spoil the image of the country in the long run when the truth comes
out in the public. It will also have adverse consequences for India’s relations with the
countries involved in bidding process. Moreover, this is inimical to a healthy
competition, level playing field and innovation. This action will also set a wrong
example to others. The action is not only unethical but also illegal as regards to Indian
laws. It may not bring in me a sense of accomplishment or content.
The latter course of action may lead to a possible defeat in the bidding process, setback
for my career and economic and strategic implications for the country. But it is the right
path to follow.
b) Advocates of national interest in international relations argue that national interests
are paramount. As Henry Kissinger has said- “there are no permanent ally or
permanent enemies, only interests are permanent”. These arguments are based on the
fact that the government of a county primarily works on the behalf of its citizens and
thus it is bound to uphold their interests. The political party in power has to face
general elections regularly and its report card of performance evaluates not only
domestic but international actions as well. Hence, national interest alone should be
paramount in international relations.
However, these arguments suffer from certain inconsistencies. If the national interest
alone is taken into account then wrong doings like colonization, regime change, arm
twisting of weaker nations etc. will be justified. Further, there exists a wide inequality
internationally and if strong nations justify their actions solely based on the national
interests than this gap will further widen. Moreover, the global commons will not
survive and sustainable development will remain a distant dream.
Thus, fairness, justice, apathy, sustainable development of whole world, equity etc. are
ethical principles which are as important as national interests and really matter in
international relations.
c) In such situation, I will pursue the following course of action:
a. Verifying, at my own level, the correctness of information related to bribery
activities in the auctioning process.
b. Informing my seniors, seeking their advice as they might have faced similar
situation earlier.
c. Approach the head of the authorities handling the whole process of auction
and inform them about inconsistencies which has come to notice and demand
a fair and transparent bidding process.
d. If grievances are not addressed at that level then, after taking my seniors into
confidence, we can approach other higher authorities of home country like
judiciary for intervention.
I will also demand that the officials involved in bribe-seeking activities must be
punished which will deter such malpractices in future. Those nations who are involved
in unfair practices must also be punished (by way of fines, blacklisting them or
cancelling their bids).
Justification of such course of action
In international transactions, sometimes unetical actions like bribery are also sought to
be justified in the name of national interest. However, on a closer look, such actions are
clearly against the national interests in reality. The revelation about involvement in
bribery would jeopardize the international relations of future generations of our
country.

DELHI JAIPUR PUNE HYDERABAD AHMEDABAD LUCKNOW CHANDIGARH


33 www.visionias.in # 8468022022 ©Vision IAS
Further, a single contract cannot be so important to our country that we sacrifice our Student Notes:
moral standards and higher values maintained for so long. Moreover, corruption can
never be the true foundation of prosperity. The gains obtained from it corrupt the
whole society.
By following the stated course of action, I will display faith in the governance of home
country, uphold our moral values and there will be higher chances of fair bidding
process. As India’s bid is better and cheaper, it will ensure India’s success. It will
generate the good will for our nation among the people of that country; set a right
example against corruption in international transaction. Overall, it will be a right step
towards the righteousness which we expect in international relations.

Copyright © by Vision IAS


All rights are reserved. No part of this document may be reproduced, stored in a retrieval system or
transmitted in any form or by any means, electronic, mechanical, photocopying, recording or
otherwise, without prior permission of Vision IAS.

DELHI JAIPUR PUNE HYDERABAD AHMEDABAD LUCKNOW CHANDIGARH


34 www.visionias.in # 8468022022 ©Vision IAS
CORPORATE GOVERNANCE Student Notes:

Contents
1. Introduction ............................................................................................................................... 2

2. Evolution of Corporate Governance in India .............................................................................. 2

3. Models of Corporate Governance .............................................................................................. 2

4. Principles and Functions of Corporate Governance in India ...................................................... 3

5. Importance of Good Corporate Governance ............................................................................. 3

6. Issues of corporate governance in India .................................................................................... 4

7. Independent Directors ............................................................................................................... 5

7.1. Issues and challenges that Independent Directors face ..................................................... 6

8. Corporate Social Responsibility (CSR) ........................................................................................ 6

8.1. Government mandate for CSR ............................................................................................ 7

8.2. Need of CSR......................................................................................................................... 7

8.3. Issues with CSR.................................................................................................................... 7

8.4. Way Forward ....................................................................................................................... 8

9. Corporate sustainability ............................................................................................................. 8

9.1. Key Drivers to ensure sustainability .................................................................................... 8

9.2. Recent steps taken to improve Corporate Governance: ..................................................... 9

10. Moving towards Good Corporate Governance ...................................................................... 10

11. Previous year UPSC Mains Questions .................................................................................... 10

12. Previous Year UPSC Mains Case Studies ................................................................................ 10

13. Previous Year Vision IAS Mains Test Series Questions ........................................................... 11

14. Previous year Vision IAS Mains Test Series Case Studies ....................................................... 23

Copyright © by Vision IAS


All rights are reserved. No part of this document may be reproduced, stored in a retrieval system or
transmitted in any form or by any means, electronic, mechanical, photocopying, recording or
otherwise, without prior permission of Vision IAS.

DELHI JAIPUR PUNE HYDERABAD AHMEDABAD LUCKNOW CHANDIGARH


1 www.visionias.in # 8468022022 ©Vision IAS
https://t.me/UPSC_PDF Download From > https://upscpdf.com https://t.me/UPSC_PDF

1. Introduction Student Notes:

India adopted Liberalisation, Privatisation and Globalisation reforms in 1991 and successfully
moved towards an open economy. Since then stock markets have seen an increase in size.
Corporate governance has played an important role in the present economic condition of
India.
Corporate Governance is defined as a set of systems, principles and processes which ensure
that a company is governed in the best interest of all stakeholders. It is a system that
comprises of employees, customers, management and shareholders.
Corporate governance has to ensure that companies stick to their vision and mission and
uphold and sustain their core values.

2. Evolution of Corporate Governance in India


The present regime of corporate governance in India has undergone many changes. Some of
the important phases in the evolution of corporate governance in India are as follows:
• After the Cadbury Committee Report in UK, some committees were constituted by SEBI,
ASSOCHAM and Indian government to recommend framework for Good Corporate
Governance, such as-
• Desirable Corporate Governance Code (1998): It laid down four ideals, which should be
the guiding force of a company’s philosophy on Corporate Governance: Transparency;
Accountability; Disclosure and Value creation.
• Kumar Mangalam Birla Committee and Narayana Murthy Committee Report: Both were
set up by SEBI. They recommended a framework for corporate governance in India, to
strengthen the responsibilities of audit committees and improve the quality of financial
disclosures.
• Corporate Governance Voluntary Guidelines, 2009: It emphasized on the- Role of board in
shareholders and related party transactions; Separation of the offices of the Chairman and
the Chief Executive Officer; Whistle Blowing Policy and Risk Management Framework.

Finally, the Companies Act 2013 made Corporate Social Responsibility (CSR), a mandatory legal
requirement and provided with a legal framework for good corporate governance. The
cultivation of good corporate governance is an ongoing process in India. All these phases
contributed to this process.

3. Models of Corporate Governance


Corporate governance systems vary around the world. Some of the important models are-
• Anglo American model: This model is focused on separation of ownership and control. All
important decisions are taken after getting approval of shareholders.
• German Model: Also called two-tier model. Majority of shareholders are banks and
financial institutions.
• Indian Model: It is a mix of Anglo-American Model and German model. There are three
types of Corporations viz. private companies, public companies and public sectors
undertakings (which includes statutory companies, government companies, banks and
other kinds of financial institutions). Each of these corporations has a distinct pattern of
shareholding. For e.g. in case of Private Companies, the promoter and his family have
almost complete control over the company. They depend less on outside equity capital.
However, no model of corporate governance is successful unless the principles and functions
of corporate governance are followed in letter and spirit.

DELHI JAIPUR PUNE HYDERABAD AHMEDABAD LUCKNOW CHANDIGARH


2 www.visionias.in # 8468022022 ©Vision IAS
Google it:- https://upscpdf.com
https://t.me/UPSC_PDF Download From > https://upscpdf.com https://t.me/UPSC_PDF

4. Principles and Functions of Corporate Governance in Student Notes:

India
Corporate governance includes following principles and functions:
• Independence of board of directors to take decisions in best interest of Profits of the
company, People of the society and the Planet (3Ps).
• Fairness in actions in market that ensures trust of the shareholders and investors.
Excessive profit seeking behaviour through fraud practices is against this principle.
• Social responsibility: Good corporate governance requires that the business be done in a
socially responsible manner. Companies should invest in building social and human capital.
Following these principles, good corporate governance becomes a means to support economic
efficiency, usher in sustainable growth and achieve financial stability i.e. the triple bottom-
line. To follow the principles diligently and to function responsibly a good framework is
required.

5. Importance of Good Corporate Governance


Good corporate governance is vital for the prosperity of an economy. Some of the key aspects
of its importance are as following;
• To enhance corporate sustainability:
The board members act in the best
interest of the shareholders. It helps
build trust in the organisation and
enhances its sustainability in the long
run. It also emboldens risk
management system.
• Lawful and ethical conduct of the
company: Helps develop a culture of
commitment to integrity, fairness,
honesty, transparency and ethical
conduct
• Shareholders activism is valued and
encouraged: Shareholders enjoy the
right to participate in the governance
and receive fair treatment from the
management and board. Stakeholders rights are delineated and communicated.
Shareholders are regularly apprised and sensitized about the company affairs. Aware and
proactive shareholders bring necessary and timely reforms in the governance.

DELHI JAIPUR PUNE HYDERABAD AHMEDABAD LUCKNOW CHANDIGARH


3 www.visionias.in # 8468022022 ©Vision IAS
Google it:- https://upscpdf.com
https://t.me/UPSC_PDF Download From > https://upscpdf.com https://t.me/UPSC_PDF

• Improves information flow between management and boards for an efficient functioning. Student Notes:
• Improve gender equity and diversity: Good corporate governance is also required for
regular and proper gender sensitisation of the employee so as to develop a healthy and
participatory work culture. It helps break glass ceiling and diversifies board membership.
• To improve functioning of the boards: Good corporate governance curtails nepotism and
favouritism and thereby helps fill the capability gap in the organisations.
Good corporate governance is in increasing demand in the current scenario as the inequalities
are rising globally. While many organisations are playing important and responsible role in the
societal development, few are also engaged in unjust behaviours which needs to be curtailed in
a timely manner.

6. Issues of corporate governance in India


In the recent past there have been several instances of corporate governance failures, including
IL&FS, DHFL and Jet Airways. Corporate frauds for profit maximisation have eroded the public
trust. All the ethical codes are being subverted to maximise profits. This forces us to identify the
issues and challenges that corporate governance in India is facing, which includes-
• Accountability and Risk Management: Many companies have poor risk management, poor
internal audit, statutory audit and weak whistle blowing policies. Top management, in many
cases overrode the internal control mechanisms. This fosters a culture of impunity. For
example, Reebok India case where governance and operations in the company were
mismanaged.
• Poor transparency and lack of fairness in affairs: Many companies suffered disclosure
lapses, as witnessed with Franklin Templeton India and Sun Pharma Ltd. Some companies
also suffered insider trading such as Aptech India ltd.
• Inadequate monitoring and response failure by regulatory authorities: The speed with
which regulatory changes have been brought in, the enforcement machinery has not been
strengthened with that speed. As a result, the oversight by regulators still remains weak.
• Independent directors associated challenges: In most companies, the nomination
committee, nominates independent directors only with the approval of the promoter or
controlling shareholder or the incumbent management. Therefore, even today, the
question ‘how independent are independent directors’ remains valid.
• Family ownership: There are two key aspects which are as following;
o Family ownership is the most practiced model in India. There is a close-knit group of
corporate leaders. A highly concentrated shareholding could increase governance and
key-man risks and therefore is detrimental to good corporate governance.
o Promoter’s emotional attachment to the company acts as a blinder and he/she cannot
take the right decisions at the right time in the interest of the company. For example;
Naresh Goyal’s reluctance to relinquish control of Jet Airways. Similarly, turbulence in
Infosys due to Narayana Murthy’s emotional attachment to the company even after
creating an independent board and distancing himself from the management of the
company.
• Lack of succession planning and capital allocation mechanism: Companies have often
been found clueless about the successor of the founding directors. Most of the companies
lack good succession planning. For example, big corporations like Infosys have failed to find
a replacement of Mr. Narayan Murthy.
• Lack of Diversity: Corporate boards lack diversity. The representation of other genders in
the boards is nominal. France and Norway have mandatory 40% board representation. The
regulatory requirement in Austria, Belgium, the Netherlands, Italy, and Germany ranges
between 30% and 40 %. In India, women only accounted for 17% of the board seats for
Nifty 500.

DELHI JAIPUR PUNE HYDERABAD AHMEDABAD LUCKNOW CHANDIGARH


4 www.visionias.in # 8468022022 ©Vision IAS
Google it:- https://upscpdf.com
https://t.me/UPSC_PDF Download From > https://upscpdf.com https://t.me/UPSC_PDF

The recent scam in the Punjab National Bank was also a result of poor corporate governance, Student Notes:
under which fraudulent letter of undertaking worth US$1.4 billion were issued by the PNB.

It has been found that many of the recent corporate governance failures were due to the issues
associated with Independent directors and therefore it requires special attention.

7. Independent Directors
An independent director is defined as a non-executive director of a company who does not
have any fiduciary relationship with the company, and/or has not been an executive with the
company in the three preceding financial years. Independent directors are the “conscience” of
the board.
The number of independent directors is set to a minimum of one third of board strength and
they are required to hold at least one separate meeting in a year without the participation of
non-independent directors. The key role and functions of Independent directors are as
following:

DELHI JAIPUR PUNE HYDERABAD AHMEDABAD LUCKNOW CHANDIGARH


5 www.visionias.in # 8468022022 ©Vision IAS
Google it:- https://upscpdf.com
https://t.me/UPSC_PDF Download From > https://upscpdf.com https://t.me/UPSC_PDF

Student Notes:

Hence, they are expected to act as a strong instrument to check intended corporate scandals.
However, in the recent past experiences like ILF&S and DHFL disclosed that the independent
directors are not fulfilling their roles and responsibilities.

7.1. Issues and challenges that Independent Directors face


Some of the issues and challenges faced by Independent directors are as following;
• Appointments related issues:
o Political partisanship: Companies Act is not followed in letter and spirit and therefore
most of the independent directors are not independent. e.g. The appointment of
leaders of ruling party as independent directors in PSUs.
o Favoritism and Nepotism: Induction of Independent directors who are either known to
the promoter of top management of the company ignores the capability gap in the
board. Consequently, most independent directors do not have a complete
understanding of a complex business model, organisation structure and contexts.eg
relatives in private corporate organisations do not augur well for their objectivity.
• Reliability issues: Independent directors exit whenever they smell trouble. Strict
enforcement of independent director’s accountability for the omission and commission by
the company drives away ‘good’ independent directors.
• Lack of board evaluations: 2016 report by InGovern cited that, under its 5-star rating
system, only five of the top 100 companies merited three stars for providing effective
board evaluations.
• The average take home remuneration of independent directors increased by 21% in fiscal
year 2015-16. This reveals that provisions regarding their non-remuneration for their
objectivity and transparency may have been compromised (Prime Database)
There needs to be further clarity on the role of independent directors so that there is no
conflict of interest in the company and legal provisions have to ensure that they have no
financial stake in the company. Finally, independent directors need to operate on their own
moral compass and self-confidence. Their independence comes from the fact that they can
interact with anybody and everybody who is willing to give them inputs.

8. Corporate Social Responsibility (CSR)


CSR is a process with the aim to embrace responsibility for the company's actions and
encourage a positive impact through its activities on the environment, consumers, employees,
communities and all other members of the public sphere who may also be considered as
stakeholders.

DELHI JAIPUR PUNE HYDERABAD AHMEDABAD LUCKNOW CHANDIGARH


6 www.visionias.in # 8468022022 ©Vision IAS
Google it:- https://upscpdf.com
https://t.me/UPSC_PDF Download From > https://upscpdf.com https://t.me/UPSC_PDF

Corporate Social Responsibility can be explained as: Student Notes:


• Corporate - means organized business
• Social - means everything dealing with the people
• Responsibility - means accountability between the two
CSR has become an established part of the global corporate landscape. It means when a
corporation goes beyond making profit and engages in actions that results in social good.

8.1. Government mandate for CSR


Companies with a net profit of RS 5 crore should spend 2% of their average profit in the last
three years on social development-related activities such as sanitation, education, health care
and poverty alleviation, among others, which are listed in Schedule 7 of the rules.

8.2. Need of CSR


The resources and abilities of governments in the developing world are limited. Organisations
(Public or Private), therefore through CSR can play vital role in supporting government in the
development process. Apart from this, it also helps as-
• Creates a favourable public image: The reputation or brand equity of the products of a
company which understands and demonstrates its social responsibilities is very high. For
example, TATA is known for Trust, Honesty and Integrity.
• Encourage social involvement of employee: Employees like to contribute to the cause of
creating a better society. Employees become champions of a company for which they are
proud to work.
• Coalescing societal and organisational goals: Society gains through better neighbourhoods
and employment opportunities, while the organisation benefits from a better community,
which is the main source of its workforce and the consumer of its products.
• Gives greater freedom and flexibility in decision-making: The company's social
involvement discourages excessive regulation or intervention from the Government or
statutory bodies, and hence gives greater freedom and flexibility in decision-making.
• Encourages co-operative attitude and healthy competition: The good public image secured
by one organization by their social responsiveness encourages other organizations in the
neighborhood or in the professional group to adapt themselves to achieve their social
responsiveness.
More companies are complying with CSR funding norms (According to India CSR Survey Report
2019) over a period of time in India. Some companies are even going beyond the mandatory
norm of 2% annual funding. However, it has also been observed that CSR is facing some
peculiar issues and challenges which are discussed below.

8.3. Issues with CSR


CSR is failing to deliver for both companies and society. It is often too peripheral to the core
business model, too far from providing real shared value for the society. Role of private sector
in meeting its social responsibility is increasingly questioned due to various reasons such as:
• Eyeing profit and displaying greed: CSR is also seen by companies from profit motive as it
enhances their image in society.
• Lack of specialists: Most corporate either don’t hire specialists for CSR or don’t have that
capacity. This impacts the efficiency of CSR.
• Transparency Issues: NGOs or local agencies do not disclose the information about their
programs, address concerns, assess Impacts and utilize funds. This lack of transparency
creates an indelible impact on the relationship and trust between the companies and local
communities which is the key to the success rate of any CSR initiative.
• Imbalance in CSR spending: Most of the CSR funds are spent in a handful of activities like
education and health, where companies either donate to specialized organisations or their

DELHI JAIPUR PUNE HYDERABAD AHMEDABAD LUCKNOW CHANDIGARH


7 www.visionias.in # 8468022022 ©Vision IAS
Google it:- https://upscpdf.com
https://t.me/UPSC_PDF Download From > https://upscpdf.com https://t.me/UPSC_PDF

own enterprises in these sectors. Others like sanitation, infrastructure, etc. get very limited Student Notes:
share.
• Geographic equity: Five states: Maharashtra, Gujarat, Andhra Pradesh, Rajasthan and
Tamil Nadu account for well over one quarter of all CSR funding. Towards the bottom of list
are Nagaland, Mizoram, Tripura, Sikkim and Meghalaya-all from North-east. It reflects the
inclinations, interest and priorities of the business sector.
• Contradictory nature of CSR law: CSR is fundamentally an inspirational exercise, and it is
very difficult to legislate aspirations. For example, it would be difficult to build “excellent
schools” the legal requirement can be met merely by spending money on education.
Thus, there are demands for greater corporate accountability.

8.4. Way Forward


Following measures can be taken by organisations to improve their CSR funding efficiency and
effectiveness;
• Encourage two-way communication: To build and sustain successful partnerships,
companies and NGO should communicate regularly. Substantial donors may get a board or
steering committee position in the NGO, as in the case of the Village Transformation
mission (VSTF) in Maharashtra, where Tata Trusts, Deutsche Bank and HUL are all part of
the steering group.
• Collaboration and co-creation: Involving proactively with partner NGO’ as true partner and
attend joint meetings. Companies should make valuable suggestions to bring in more
professional approach.
• Putting in an accountability regime: Companies should set clear objectives and
expectations for aligning different stakeholders. Due diligence and stringent monitoring of
implementing agencies is necessary.
• Companies should go beyond CSR, to CSV i.e. creating shared value.
• New proposed changes to laws such as punitive action and stringent disclosure norms
should be passed and implemented to embolden the CSR.
• Companies should diversify their spending areas so as to bring in equity.
Organisations need to build in a culture of socially responsible behaviour and should not
restrict themselves just to the allocations of the funds. A willingness on the part of
organisations is necessary to align intentions with action.

9. Corporate sustainability
Corporate sustainability encompasses strategies and practices that aim to meet the needs of
stakeholders today while seeking to protect, support and enhance the human and natural
resources that will be needed in the future.

9.1. Key Drivers to ensure sustainability


• Internal capacity building strength – in order
to convert various risks into competitive
advantage.
• Social impact assessment – in order to
become sensitive to various social factors, like
changes in culture, living habits etc.
• Repositioning capability through
development and innovation.
• Crystallisation of all activities to ensure
consistent growth.

DELHI JAIPUR PUNE HYDERABAD AHMEDABAD LUCKNOW CHANDIGARH


8 www.visionias.in # 8468022022 ©Vision IAS
Google it:- https://upscpdf.com
https://t.me/UPSC_PDF Download From > https://upscpdf.com https://t.me/UPSC_PDF

Student Notes:
These may be derived by converting risks arising out of economic, environmental and social
activities of a corporate into business opportunities keeping in mind the principles of
sustainable development.

9.2. Recent steps taken to improve Corporate Governance:


Government constituted Uday Kotak Committee to look into the affairs of corporate
governance and formulate a report consisting of the recommendations to improve corporate
governance standards in India. SEBI accepted 40 recommendations out of 81 made by Kotak
panel, 18 were rejected and 8 have been referred to other agencies Some of the important
recommendations are as following:

Apart from above, following steps have been taken-


• Punitive action and setting the accountability: SEBI is planning punitive action in several
big cases including alleged corporate governance lapses by Raymond Ltd and InterGlobe
Aviation Ltd, fund diversion at CG Power and Industrial Solutions Ltd and NSE’s algorithmic
trading and co-location case.
• To build trust: Government amended the Companies Act 2013 for the re-categorisation of
16 ‘compoundable offences’ to ‘civil defaults’, and also set up the Company Law Committee,
which has recommended further reclassifying 46 offences.
• An Independent director data base has been set up to register experts. SEBI is in process of
strengthening norms of related party transactions.

DELHI JAIPUR PUNE HYDERABAD AHMEDABAD LUCKNOW CHANDIGARH


9 www.visionias.in # 8468022022 ©Vision IAS
Google it:- https://upscpdf.com
https://t.me/UPSC_PDF Download From > https://upscpdf.com https://t.me/UPSC_PDF

10. Moving towards Good Corporate Governance Student Notes:

A combination of global practices, existing legal provisions, good to have principles and
forward-looking concepts comprise the next level of corporate governance. Following
recommendations can be considered as what more can be done to achieve Good corporate
governance
• Right board leadership is essential to develop the culture of commitment to integrity,
fairness, honesty transparency and ethical conduct. This culture must be disseminated to
all the employees through training programmes.
• Companies need to put in place clear policies and practices for zero tolerance for bribery,
corruption, anti-corruption practices, to prevent market manipulation, as also precautions
to prevent money laundering.
• The integrity of financial statements is foundation of stakeholder’s trust. The functioning
audit committees is critical to building internal controls. All information should be made
available to auditors, including through direct engagement between audit committees and
auditors.
• The process of de-criminalisation of business laws should be continued.
• Concepts and principles of governance for a company need to embrace all stakeholders,
including the government, lenders, creditors, employees, customers, vendors and the
community.
• OECD Document on Corporate governance reforms: It cites certain immensely valuable
recommendations such as
o Providing training to independent directors on the business of the company
o Improving investor education for better participation at General Meetings
o Improving selection mechanism for independent directors
Good corporate governance, going beyond the letter of law promotes ease of doing business
and thereby adds to growth.

11. Previous year UPSC Mains Questions


1. Corporate Social responsibility makes companies more profitable and sustainable. Analyse.

12. Previous Year UPSC Mains Case Studies


1. ABC ltd is a large transnational company having diversified business activities with ahuge
shareholder base. The company is continuously expanding and generating employment.
The company in its expansion and diversification programme decides to establish a new
plant at Vikaspuri, an area which is underdeveloped. The new plant is designed to use
energy efficient technology that will help the company to save production cost by 20%. The
company’s decision goes well with the government policy of attracting investment to
develop such underdeveloped regions. The government has also announced tax holidays for
five years for the companies that invest in underdeveloped areas. However, the new plant
may bring chaos for the inhabitants of Vikaspuri region, which is otherwise tranquil. The
new plant may result in increased cost of living, aliens migrating to the region, disturbing
the social and economic order. The company sensing the possible protest tried to educate
the people of Vikaspuri region and public in general that how it’s corporate social
responsibility (CSR) policy would help overcome the likely difficulties of the residents of
Vikaspuri region. In spite of this the protests begin and some of the residents decided to
approach the judiciary as their plea before the Government did not yield result.
(a) Identify the issues involved in the case.
(b) What can be suggested to satisfy the company’s goal and to address the residents
concerns?

DELHI JAIPUR PUNE HYDERABAD AHMEDABAD LUCKNOW CHANDIGARH


10 www.visionias.in # 8468022022 ©Vision IAS
Google it:- https://upscpdf.com
https://t.me/UPSC_PDF Download From > https://upscpdf.com https://t.me/UPSC_PDF

2. A fresh engineering graduate gets a job in a prestigious chemical industry. She likes the Student Notes:
work. The salary is also good. However, after a few months she accidentally discovers that
a highly toxic waste is being secretly discharged into a river nearby. This is causing health
problems to the villagers downstream who depend on the river for their water needs. She is
perturbed and mentions her concern to her colleagues who have been with the company
for longer periods. They advise her to keep quite as anyone who mentions the topic is
summarily dismissed. She cannot risk losing her job as she is the sole bread-winner for her
family and has to support her ailing parents and siblings. At first, she thinks that if her
seniors are keeping quiet, why she should stick out her neck. But her conscience pricks her
to do something to save the river and the people who depend upon it. At heart she feels
that the advice of silence given by her friends is not correct though she cannot give reasons
for it. She thinks you are a wise person and seek your advice.
a) What arguments can you advance to show her that keeping quiet is not morally right?
b) What course of action would you advise her to adopt and why? (250 words) 20
3. A private company is known for its efficiency, transparency and employee welfare. The
company though owned by a private individual has a cooperative character where
employees feel a sense of ownership. The company employs nearly 700 personnel and they
have voluntarily decided not to form union.
One day suddenly in the morning, about 40 men belonging to political party gate crashed
into the factory demanding jobs in the factory. They threatened the management and
employees, and also used foul language. The employees feel demoralized. It was clear that
those people who gate crashed wanted to be on the payroll of the company as well as
continue as the volunteers/members of the party.
The company maintains high standards in integrity and does not extend favours to civil
administration that also includes law enforcement agency. Such incident occur in public
sector also. (20 Marks) (250 Words)
(a) Assume you are the CEO of the company. What would you do to diffuse the volatile
situation on the date of gate crashing with the violent mob sitting inside the company
premises?
(b) What can be the long term solution to the issue discussed in the case?
(c) Every solution/action that you suggest will have a negative and a positive impact on you
as (CEO), the employees and the performance of the employees. Analyse the consequences
of each of your suggested actions.
4. One of the scientists working in the R&D laboratory of a major pharmaceutical company
discovers that one of the company’s bestselling veterinary drugs has the potential to cure a
currently incurable liver disease which is prevalent in tribal areas. However, developing a
variant of the drug suitable for human being entailed a lot of research and development
having a huge expenditure to the extent of Rs. 50 crores. It was unlikely that company
would recover the cost as the disease was rampant only in poverty stricken areas having
very little market otherwise.
If you were the CEO, then (20 Marks) (250 Words)
(a) Identify the various actions that you could take
(b) Evaluate the pros and cons of each of your actions

13. Previous Year Vision IAS Mains Test Series Questions


1. Corporate governance is about ethical conduct in business. Comment.
Answer
• Corporate governance is the system of internal controls and procedures by which
individual companies are managed. It provides a framework that defines the rights,
roles and responsibilities of different groups – management, board, and controlling
as well as non-controlling shareowners.

DELHI JAIPUR PUNE HYDERABAD AHMEDABAD LUCKNOW CHANDIGARH


11 www.visionias.in # 8468022022 ©Vision IAS
Google it:- https://upscpdf.com
https://t.me/UPSC_PDF Download From > https://upscpdf.com https://t.me/UPSC_PDF

• At its core, corporate governance is the arrangement of checks, balances, and Student Notes:
incentives a company needs to minimize the conflicting interests between insiders
and external shareowners.
• Increasing revelations of deterioration in quality and transparency in companies,
have called for the adoption of internationally accepted ‘Best Practices’ or ‘Business
Ethics’. It is the acceptance of these that has given rise to the concept of ‘Corporate
Governance’.
• Thus the term ‘Corporate Governance’ encompasses commitment to values and
ethical business conduct to maximize shareholder’s interests on a sustainable basis,
while ensuring fairness to all stakeholders including customers, employees, and
investors, vendors, government and society at large.
• Corporate Governance influences how the objectives of the company are set and
achieved, how risk is monitored and assessed and how performance is optimized.
Sound Corporate Governance is therefore critical to enhance and retain investors’
trust.
• When ethical dilemmas arise due to conflicting interests of the parties involved,
managers usually make decisions based on a set of principles influenced by the
values, context and culture of the organization.
• Ethical leadership is also good for business as the organization is seen to conduct its
business in line with the expectations of all stakeholders. On the other hand,
unethical behaviour is inimical to the interests of stakeholders of a business in the
long run. It is this, which gives weight to adoption of Corporate Governance.
• Hence, Corporate Governance is nothing but the moral or ethical or value
framework under which corporate decisions are taken.

2. Discuss the recommendations of Narayana Murthy Committee on Corporate


Governance?
Answer:
The Securities and Exchange Board of India (SEBI) had constituted a Committee on
Corporate Governance in 2002, in order to evaluate the adequacy of existing corporate
governance practices and further improve these practices. It was set up to review
Clause 49, and suggest measures to improve corporate governance standards.
The terms of reference of the committee were to:
• Review the performance of corporate governance; and
• Determine the role of companies in responding to rumour and other price sensitive
information circulating in the market, in order to enhance the transparency and
integrity of the market.
The issues discussed by the committee primarily related to audit committees, audit
reports, independent directors, related parties, risk management, directorships and
director compensation, codes of conduct and financial disclosures.
The committee's recommendations in the final report were selected based on
parameters including their relative importance, fairness, accountability, transparency,
ease of implementation, verifiability and enforceability.
The key mandatory recommendations focused on:
• Strengthening the responsibilities of audit committees;
• Improving the quality of financial disclosures, including those related to related
party transactions and proceeds from initial public offerings;
• Requiring corporate executive boards to assess and disclose business risks in the
annual reports of companies;

DELHI JAIPUR PUNE HYDERABAD AHMEDABAD LUCKNOW CHANDIGARH


12 www.visionias.in # 8468022022 ©Vision IAS
Google it:- https://upscpdf.com
https://t.me/UPSC_PDF Download From > https://upscpdf.com https://t.me/UPSC_PDF

• Introducing responsibilities on boards to adopt formal codes of conduct; the Student Notes:
position of nominee directors; and
• Stock holder approval and improved disclosures relating to compensation paid to
non-executive directors.
Non-mandatory recommendations included:
• moving to a regime where corporate financial statements are not qualified;
• instituting a system of training of board members; and
• evaluation of performance of board members.
Analysis
• This recommendation, would be instrumental in breeding indiscipline as most likely
the audit committee would be flooded with frivolous complaints and minor issues.
• Many complainants might go by their personal likes and dislikes and thus the
possibility of the right of access to the audit committee being misused would
always be there.
• The committee had not said anything on providing evidence in support of a
complaint, disclosure of the identity of the complainant and the maximum number
of complaints that an employee could make in a year.
• Elimination of unethical or improper practices is the responsibility of respective
corporate promoters and management.
• The Narayana Murthy panel is for restricting the tenure of non-executive directors
to three terms of three years each, running continuously.
• Representatives of a promoter remain on the board of a company as non-
independent directors. The recommendation now made rules out continuation of
promoter-directors on the board beyond nine years at a stretch.
• It needs to be clarified whether a partner of an audit firm or a solicitor's firm can be
treated as an independent director of a company if his firm is the auditor or legal
advisor of another company in the same group.

3. What is corporate governance? How is it important in the Indian corporate sector?


How will the mandatory provision of corporate social responsibility change the
perception of private sector in the eyes of common man?
Approach:
The answer should deal with the basics of corporate governance; its basic features and
the importance it plays in management and regulation of private sector. The second
part deals with the issue of CSR; the answer should deal with the role played by CSR in
initiating welfare activities for the people and how it helps in overall upliftment of the
society.
Answer:
Corporate governance can be defined as a set of system, process and principles, which
ensure that a company is governed in the best interest of all the shareholders. It
ensures:
• Adequate disclosures and effective decision making to achieve corporate
objectives.
• Transparency in business transactions
• Protection of stakeholder’s interest
• Statutory and legal compliances
• Commitment to values and ethical conduct of business

DELHI JAIPUR PUNE HYDERABAD AHMEDABAD LUCKNOW CHANDIGARH


13 www.visionias.in # 8468022022 ©Vision IAS
Google it:- https://upscpdf.com
https://t.me/UPSC_PDF Download From > https://upscpdf.com https://t.me/UPSC_PDF

• Credible and confident investors, which lead to more patient long-term capital at Student Notes:
low cost.
This helps in ensuring that the corporate sector is accountable to all stakeholders
including the society at large. For effective corporate governance, its policies need to be
such that the directors of the company should not abuse their power and instead
should understand their duties and responsibilities towards the company and should
act in the best interests of the company in the broadest sense. The concept of
‘corporate governance’ is not an end; it is just a beginning towards growth of the
company for long term prosperity.
The concept of Corporate Governance emerged in India after the second half of 1990’s
due to economic liberalization and deregulation of industry and business. With the
changing times, there was also a need for greater accountability of companies to their
shareholders and customers. It is important that corporate India must commit itself as
reliable, innovative and prompt service provider to their customers and should also
become reliable business partners in order to prosper and to have all round growth.
Indian Corporate Bodies having adopted good corporate governance will reach
themselves to a benchmark for rest of the world; it brings laurels as a way of
appreciation. The importance of corporate governance lies in promoting and maintains
integrity, transparency and accountability throughout the organization. Several Indian
Companies like PepsiCo, Infuses, Tata, Wipro, TCS, and Reliance are some of the global
giants, which have their flag of success flying high in the sky due to good corporate
governance.
Corporate Social Responsibility (CSR) is a process, which aims to embrace responsibility
for the companies’ actions and encourage positive impact through its activities on the
environment, consumer, employee, community and all other members of public sphere
who may also be considered as stake holders. By making CSR provisions mandatory
under the Companies’ bill, it will function as a built in self-regulating mechanism
whereby a business monitors and ensures its active compliance within the spirit of the
law, ethical standards and international law. The CSR programs could range from overall
development of a community to supporting specific causes like education, healthcare,
environment etc. These will help in boosting the image of private sector in the eyes of
the common man.

4. What do you understand by corporate ethics? Giving examples, highlight the ethical
issues faced by CEOs in the current competitive environment, with special reference to
government owned enterprises.
Approach:
• Explain what you understand by corporate ethics.
• Mention the ethical issues faced by CEOs in a highly competitive environment.
• Expand on the ethical issues faced by CEOs of state owned enterprises.
• Give examples to substantiate your answer, if any.
Answer:
• Corporate ethics includes proper business policies and practices in corporate
governance regarding issues such as insider trading, bribery, interests of
stakeholders, corporate social responsibilities and fiduciary responsibilities.
• Ethical issues faced by CEOs in a highly competitive environment include adaptation
to new technology and innovation while ensuring that manpower is retained,
transparency, compliance with government rules, ensuring gender equality and
equality of opportunity in the workplace, among others.

DELHI JAIPUR PUNE HYDERABAD AHMEDABAD LUCKNOW CHANDIGARH


14 www.visionias.in # 8468022022 ©Vision IAS
Google it:- https://upscpdf.com
https://t.me/UPSC_PDF Download From > https://upscpdf.com https://t.me/UPSC_PDF

• CEOs of state-owned enterprises have to fulfill the twin objectives of commercial Student Notes:
efficiency, resulting in profits for stakeholders and fulfilling of social responsibility,
as one of the arms of the welfare state.
• They also have to ensure that there is no undue interference of the state in the
functioning of the enterprise as excess interference can leave the enterprise
vulnerable to being used to achieve short-term political goals to the detriment of its
efficiency.
• Additionally, they have to set a level playing field with the private sector as they
often receive preferential treatment through access to subsidies, bank credit and
procurement contracts.
• Further, they should function with transparency, adhere to compliance procedures,
update accounting and auditing practices and disclose the financial statements of
the enterprises on time.
• They also have to meet the demands of the workers, who look out for their
personal interests in an environment characterized by increased contractual work
and increased instances of downsizing by the enterprises.
• CEOs of state owned enterprises have had to manage instances of protests by
workers against their decisions when the industrial policies were rapidly changing
post liberalization.
• A nation wide scale occurred in 1991 against the government's announced plan to
close unprofitable state-owned industries to move the country from socialism
toward free-market capitalism.
• Similarly, in September 2016, majority of the trade unions carried out an all India
strike to protest against labour reforms, disinvestment in profit-making public
sector undertakings and contractualization of workforce.
• In such situations, globally accepted codes like the OECD Code (2005) and
guidelines issued by the Department of Public Enterprise (DPE) could be taken as
benchmarks for the CEOs to follow.

5. Has excessive profit seeking by corporates undermined the trust of public in the
private sector? Giving examples, examine the reasons for failure of corporate
governance in India.
Approach:
• In the first part, discuss whether want for excess profits has eroded the trust of
public in the private sector.
• In the second part, mention the reasons behind the failure of corporate governance
using examples wherever possible.
• In the conclusion, briefly talk about the importance of corporate governance.
Answer:
Private sector is largely driven by ‘profits’, which are important for sustainability of
business operations. However, with increasing instances of corporate frauds and crony
capitalism, questions have been raised on the mode of operations of the private sector.
It has also been argued that such illegal and unethical practices are traceable to the
want of excess profits.
Manifestations of such practices include the Satyam scam, Enron Scandal, Financial
Crises of 2008 whereby the public was misled for generating excess profits.
Many see the private sector being driven by self-interest without adequately
acknowledging the interest all stakeholders, especially consumers. India’s telecom
sector is an example. While penetration has increased significantly over the years with

DELHI JAIPUR PUNE HYDERABAD AHMEDABAD LUCKNOW CHANDIGARH


15 www.visionias.in # 8468022022 ©Vision IAS
Google it:- https://upscpdf.com
https://t.me/UPSC_PDF Download From > https://upscpdf.com https://t.me/UPSC_PDF

extremely competitive tariffs, the quality of service has left much to be desired with Student Notes:
rising instances of call drops.
This has meant that it is the consumers or the larger public which are at the receiving
end. Therefore, while profits are an important factor, it is the race for excessive profits
and the corresponding practices to earn those that has undermined the trust of the
public in the private sector.
Reasons for the failure of corporate governance in India
• Most corporate governance abuses in India arise due to conflict between the
majority and minority shareholders. In India, boards are not as empowered as in
several western economies and since the board is subordinate to the shareholders,
the will of the majority shareholders prevails.
• There is still a lack of awareness about various issues pertaining to corporate
governance, like, quality and frequency of financial and managerial disclosure,
compliance with the code of best practice, roles and responsibilities of Board of
Directors, shareholders’ rights, etc.
• Collusion between companies and their accounting firms, non-compliance with
standards, etc. for example, the Enron scandal is also one of the reasons.
• In India, family businesses constitute an important portion of corporate setup.
Family control also brings governance problems – which includes lack of checks and
balances over executive decision-making and behavior, and a lack of transparent
reporting to the outside world.
• The issues of enforcement of corporate governance norms also leave much to be
desired. There have been instances of substantial delay in the delivery of justice by
Indian legal system.
Corporate governance is perhaps one of the most important differentiators of a
business that has impact on the profitability, growth and even sustainability of
business. Sound corporate governance is, therefore, crucial for sustained and robust
growth which enhances the confidence of not just the investors but the larger public as
well. Therefore, there is a need for establishing clear principles of corporate governance
keeping in mind the welfare of all the stakeholders. This becomes important as there is
greater involvement of private sector in delivering essential services.

6. Corporate Governance provides a framework that defines the rights, roles and
responsibilities of various groups within an organization. (a) Elaborate the need to
incorporate the principles of Corporate Governance to enhance the effectiveness of
the public sector enterprises. (b) Identify the challenges specific to the public sector
when it comes to the application of good practices of corporate governance.
Approach:
• Give in brief, the definition and framework of corporate governance.
• Throw light upon the need for Corporate Governance in Public Sector Enterprises.
• Highlight the challenges in corporate governance of Public Sector Enterprises.
Answer:
Corporate Governance is defined as a set of systems, principles and processes which
ensure that a company is governed in the best interest of all stakeholders. It is a
system that comprises of employees, customers, management and shareholders.
As per World Bank, ‘Corporate Governance is concerned with holding a balance
between economic and social goals and between individual and community goals.’

DELHI JAIPUR PUNE HYDERABAD AHMEDABAD LUCKNOW CHANDIGARH


16 www.visionias.in # 8468022022 ©Vision IAS
Google it:- https://upscpdf.com
https://t.me/UPSC_PDF Download From > https://upscpdf.com https://t.me/UPSC_PDF

(a) With 24/7 social media alerts, as well as scrutinized government budgets, past Student Notes:
scandals, and legislation around the world targeting corruption, there is an increased
demand for accountability. Public skepticism, combined with unprecedented
transparency, is placing everyone under scrutiny. The boundaries of accountability and
responsibility are rapidly expanding, challenging individuals in authority to consider the
public and its agenda.
Consequently, a well-functioning public sector, delivering quality public services
consistent with citizen preferences that fosters market growth while managing fiscal
resources prudently, is considered critical.
Due to high concentration of wealth within the public sector, state-owned enterprises,
play a key role in society. As such, effective governance within the public sector would
encourage efficient use of resources, strengthen accountability for the stewardship of
those resources, and improve management and service delivery, thus contributing to
enhanced quality of living. Effective governance is also essential for building confidence
in public sector entities, which is in itself necessary if public sector entities are to be
effective in meeting their objectives.

(b) The organizational challenges peculiar to Public Sector Enterprises in


institutionalizing corporate governance comprise of:
• The focus of governance in companies is on the board of directors. However, in the
context of public sector, boards are sometimes difficult to identify and define, as
they operate in different statutory and managerial frameworks.
• High performance expectations while taking into account diverse socio-cultural
environment. The public sector entities have to satisfy a complex range of political,
economic and social objectives which subject them to a different set of external
constraints.
• Increasing competitive pressure from both public as well as private enterprises.
• Regulatory interference by multiple authorities with different enforcement
environments.
• Various stakeholders including ministers, other government officials, the electorate
(Parliament), customers and clients, the general public, shareholders, customers
etc. may have a legitimate interest in the public sector entities, but not necessarily
any ‘ownership rights.’
• Entry into new sectors with different types of ethical concerns.
• Balancing employee turnover with market needs of cost cutting.

DELHI JAIPUR PUNE HYDERABAD AHMEDABAD LUCKNOW CHANDIGARH


17 www.visionias.in # 8468022022 ©Vision IAS
Google it:- https://upscpdf.com
https://t.me/UPSC_PDF Download From > https://upscpdf.com https://t.me/UPSC_PDF

To conclude, effective corporate governance is essential for building confidence in Student Notes:
public sector enterprises – which is in itself is necessary if public sector entities are to
be effective in meeting their objectives.

7. Critically discuss the impediments to corporate governance in India. Also evaluate


how the Kotak panel recommendations can help in overcoming them.
Approach:
• Briefly explain corporate governance and need to strengthen it.
• Discuss the impediments to corporate governance in India.
• Discuss the recommendation by the Kotak panel.
• Highlighting the role of board, auditors and regulator to improve corporate
governance discuss how recommendations will help in overcoming impediments.
• Discuss other reform measures.
Answer:
Corporate governance is the system of rules, practices and processes by which a
company is directed and controlled. It essentially involves balancing the interests of all
stakeholders, and is important for attracting investors
Impediments to Corporate Governance in India:
• Though Board of directors is appointed by shareholders, majority shareholders of
large corporations in India are individuals or family. Thereby they are ineffective in
upholding the interests of small shareholders.
• Compromise of Audit committee; questionable independence of Independent
directors from management; complete absence of employment relationships.
• Lack of adherence to regulatory framework and weak enforcement and monitoring
systems.
• Lack of transparency and disclosure.
• Business relationships of investors with firms in which they invest.
The Kotak committee has extensively examined the current state of affairs and
recommended sweeping changes that will help improve governance and enhance
investor confidence. Emphasis has been laid on strengthening the three gatekeepers—
the board, the auditors and the regulator. It focused on the following:
Independent Directors
• Their strength should be increased from 33% to 50% of the board to protect small
investors.
• No board meeting can be conducted without the presence of an independent
director.
• Suggested measures so that independent directors inducted in the board are truly
independent.
Regarding Audit:
• An audit committee to look into utilization of funds infused by a listed entity into
unlisted subsidiaries, including foreign subsidiaries.
• It covers issues in accounting and auditing practises by listed companies and seeks
to improve the effectiveness of board evaluation practises.
Strengthening the regulator:
• Strengthening the role of the market regulator to improve the governance practises
at listed entities.
• SEBI should have the power to act against auditors if the need arises.

DELHI JAIPUR PUNE HYDERABAD AHMEDABAD LUCKNOW CHANDIGARH


18 www.visionias.in # 8468022022 ©Vision IAS
Google it:- https://upscpdf.com
https://t.me/UPSC_PDF Download From > https://upscpdf.com https://t.me/UPSC_PDF

• SEBI to develop capabilities to be able to regulate listed companies more effectively Student Notes:
and protect the interests of small shareholders.
Other Reforms:
• For government companies, it recommended that the board have final say on the
appointment of independent directors and not the nodal ministry.
• It made some radical suggestions to improve the safeguards and disclosures
pertaining to related-party transactions.
• It seeks to address the issues faced by minority investors on voting and
participation in annual general meetings.
Although most of the recommendations in the committee’s report are incremental in
nature, there a few far-reaching as well, they aim to strengthen corporate governance
further particularly in the realm of implementation. These would help in realizing the
objectives of Companies Act 2013 and SEBI’s subsequent reforms for improving
Corporate Governance.

8. Good corporate governance is not an end in itself. It is a means to support economic


efficiency, sustainable growth and financial stability. Discuss.
Approach:
• Define good corporate governance.
• Mention as to how it supports economic efficiency, sustainable growth and
financial stability thereby acting as a means rather than an end in itself.
Answer:
Corporate governance is the system of rules, practices and processes by which a firm is
directed and controlled. Corporate governance essentially involves balancing the
interests of a company's many stakeholders, such as shareholders, management,
customers, suppliers, financiers, government and the community.
Various pillars and principles of good corporate governance helps companies operate
more efficiently, improve access to capital, mitigate risk, and safeguard against
mismanagement. Corporate governance includes adhering to certain principles - such
as:
• Discipline in financial actions and dealing with stakeholders,
• Transparency, accountability and responsibility of the decision makers towards all
stakeholders.
• Independence of board of directors to take decisions in best interest of Profits of
the company, People of the society and the Planet (3Ps).
• Fairness in actions in market.
• Social responsibility.

Following these principles, good corporate governance becomes a means to support


economic efficiency, usher in sustainable growth and achieve financial stability i.e. the
triple bottomline.
• Following good corporate governance norms helps to build an environment of
trust, transparency and accountability so that the financial sector can serve the
needs of the economy in terms of access to long term capital and investment. Thus,
promoting financial stability.
• Practicing good corporate governance involves probity, integrity and fairness in the
actions of leaders. This ensures leadership is conscious of its obligations and is
ready to account for the commitments it has made towards stakeholders. It also

DELHI JAIPUR PUNE HYDERABAD AHMEDABAD LUCKNOW CHANDIGARH


19 www.visionias.in # 8468022022 ©Vision IAS
Google it:- https://upscpdf.com
https://t.me/UPSC_PDF Download From > https://upscpdf.com https://t.me/UPSC_PDF

enhances goodwill of the company amongst public. Thus, ensuring sustainable Student Notes:
growth.
• It also emphasizes on timely disclosure of information as well as results rather
than processes. The quality of corporate governance affects the cost for
corporations to access capital for growth and the confidence with which those who
provide capital can participate and share in their value creation on fair and
equitable terms. Thus, making economic efficiency a natural consequence of good
corporate governance.
Contrastingly, poor corporate governance weakens a company’s potential, can lead to
financial difficulties and in some cases can cause long-term damage to a company’s
reputation as can be seen in cases financial scams like Satyam, Sahara etc.

9. It takes more than a corporate governance policy to inspire ethical behavior and
sustain a truly ethical workplace. Discuss.
Approach:
• Briefly, describe corporate governance policy.
• Provide arguments to bring out the inadequacy of merely the corporate governance
policy to promote ethical behavior and suggest how to inspire and sustain ethical
workplace.
• Conclude the answer.
Answer:
Corporate governance is the set of mechanisms, rules and practices by which a
corporate entity is run. These usually consist of set standards defining code of conduct
of the employees, the management and the organization- the ways in which they are
expected to act. Important pillars of corporate governance are transparency,
accountability, fairness and protection of stakeholders’ interest. It is expected that it
will ensure the adherence to the legal and ethical practices by all through a compliance
policy and statement of values. A corporate governance policy highlights the
commitment towards the relevant laws, rules and regulations and constitutes a
statement of values, informing the stakeholders about the company’s priorities and
core belief, thus setting behavioral expectations.
Despite good intentions and sound ethical frameworks under corporate governance
policy, the organizations and its employees often do not behave in the manner that the
policy advocates. The test of CG Policy is not merely whether a company explicitly has it
or not. It must be effective and seen to be working. If employees and the company are
not ethical in their day to day behaviour, it denotes a failure of the policy. At the same
time, in critical matters where the company/shareholders indulge in unethical practices
such as insider trading, secretive corruption, etc., then it reflects that those responsible
for enforcement of policy themselves do not believe and follow it. This resort to
unethical practices can be due to the following reasons:
• Inability and psychological fear among employees to raise ethical concerns.
• Excessive pressure to achieve unrealistic performance targets.
• The setting of conflicting goals by the organization provokes a sense of injustice and
unfairness.
• Lack of conscious efforts on the part of leaders to set a positive example. For e.g.
allegations of misallocation of resources by CEO of a reputed bank sets a bad
precedent.
These instances show that the mere creation of an ethical framework is not enough. It
must be backed up by practices such as appointment of auditors who actually maintain

DELHI JAIPUR PUNE HYDERABAD AHMEDABAD LUCKNOW CHANDIGARH


20 www.visionias.in # 8468022022 ©Vision IAS
Google it:- https://upscpdf.com
https://t.me/UPSC_PDF Download From > https://upscpdf.com https://t.me/UPSC_PDF

an arms-length relationship with the company. Ethical practices must be visible in Student Notes:
decisions and actions of the organization and its employees in the following ways:
• The organization should adopt an open-door policy in letter and spirit
• Leaders must set an example for others to emulate
• Set realistic goals in participation with employees
• Strict adherence to ethical considerations in routine activities
• Recognize the ethical choices and discourage the unethical behaviour
• Develop a system of values within the organization which are strictly adhered to.
• Decisions must be reviewed before finalization.
In an age of corporate mistrust, creating ethical workplaces requires conscious effort
that necessitates not only adopting the highest ethical standards, but also their whole-
hearted implementation.

10. Companies adhering to the norms of corporate governance emerge as winners in the
long run. Evaluate with the help of suitable examples.
Approach:
• Briefly discuss the concept of corporate governance.
• Give reasons with examples why such companies emerge as winners in the long
run.
• Justify with examples how companies ignoring corporate governance do not sustain
in the long run.
• Conclude appropriately based on the above points.
Answer:
Corporate governance is the system of rules, practices and processes by which a firm is
directed and controlled. Corporate governance essentially involves balancing the
interests of a company's stakeholders, such as shareholders, management, customers,
suppliers, financiers, government and the community. It also provides the framework
for attaining a company’s objectives and encompasses practically every sphere of
management.
Companies adhering to the norms of corporate governance emerge as winners in the
long run because of following factors:
• Sustainable growth of company: As it maintains transparency, accountability and
responsibility of the decision makers towards all stakeholders. It also involves
abiding by the laws and regulations in letter and spirit and ensuring that all
statutory obligations are discharged on time. For e.g. Tata Group of companies
continues to be one of the biggest conglomerates even after 150 years of its
existence.
• Brings economic efficiency: As companies ensure discipline in financial actions, it
attracts stakeholders’, especially foreign institutional investors and also has a
positive influence on the market value of the company.
• Improved risk management: It encompasses the manner in which board and
committee meetings are conducted and decisions are recorded. This mechanism of
internal control allows better fraud prevention and detection and conflict
resolution between management and shareholder’s interests. E.g. HDFC Bank has
been able to insulate itself from the NPA crisis, owing to its Corporate Governance
Policy.
• Enhances goodwill of the company: Adhering to disclosure norms, regular
updation of records, regulatory requirements like taking care of minority

DELHI JAIPUR PUNE HYDERABAD AHMEDABAD LUCKNOW CHANDIGARH


21 www.visionias.in # 8468022022 ©Vision IAS
Google it:- https://upscpdf.com
https://t.me/UPSC_PDF Download From > https://upscpdf.com https://t.me/UPSC_PDF

stakeholders etc. leaves a lasting impression in the minds of the people. E.g. Infosys Student Notes:
has maintained the value of 'Walk the Talk' and gained the respect of its
stakeholders.
• Continuous feedback loop: By incorporating customer’s feedback and market
research in improving service delivery quality. E.g. Hindustan Unilever Co., the most
valuable FMCG in India has kept pace with changing times and consumer
preferences.
Further, poor corporate governance may be driven by narrow self-interest and lead to
failure of meeting long-term commitments, thereby hurting the company’s reputation.
For example, Satyam and Sahara scandals, Volkswagen emission scandal etc. It can also
lead to financial difficulties such as in the case of Kingfisher Airlines.
Recent crises faced by financial giants like YES Bank, IL&FS point towards the
importance of strict adherence to the norms of corporate governance, as it is perhaps
one of the most important differentiators of a business that has an impact on the
profitability, growth and even sustainability of business.

11. It has been argued that traditional approaches to corporate social responsibility (CSR)
are inadequate. Discuss. Also, examine the role of Social License to Operate (SLO) in
this regard.
Approach:
• Brief introduction about Corporate Social Responsibility.
• Why traditional approach is being questioned?
• How SLO (Social License to Operate) can help overcome the problems with
traditional CSR?
Answer:
Corporate Social Responsibility is a management concept whereby companies integrate
social and environmental concerns in their business operations and interactions with
their stakeholders. CSR is generally understood as being the way through which a
company achieves a balance of economic, environmental and social imperatives. CSR is
a process with the aim to embrace responsibility for the company's actions and
encourage a positive impact through its activities on the environment, consumers,
employees, communities and all other members of the public sphere who may also be
considered as stakeholders. Corporate social responsibility (CSR) has become an
established part of the global corporate landscape. It means when a corporation goes
beyond making profit and engages in actions that results in social good.
However, traditional CSR is failing to deliver for both companies and society. It is often
too peripheral to the core business model, too far from providing real shared value for
the society. Role of private sector in meeting its social responsibility is increasingly
questioned due to various reasons such as:
• Disregard for societal implications: The company generates its profits and creates
value without much consideration for wider societal impact and benefits.
• Lack of intention to serve: It raises funds and makes donations but it has little to do
with how they function as a company. It is often too peripheral to the core business
model and too far from providing real ‘shared value’.
• Eyeing profit and displaying greed: There is also a view that CSR is also seen by
companies from profit motive as it enhances its image in society.
• Lack of specialists: Most corporate either don’t hire specialists for CSR or don’t
have that capacity. This impacts the efficiency of CSR.

DELHI JAIPUR PUNE HYDERABAD AHMEDABAD LUCKNOW CHANDIGARH


22 www.visionias.in # 8468022022 ©Vision IAS
Google it:- https://upscpdf.com
https://t.me/UPSC_PDF Download From > https://upscpdf.com https://t.me/UPSC_PDF

• Imbalance in CSR spending: Most of the CSR funds are spent in a handful of Student Notes:
activities like education and health, where companies either donate to specialized
organisations or their own enterprises in these sectors. Others like sanitation,
infrastructure, etc. get very limited share.
Thus, there are demands for greater corporate accountability. Private sector needs a
new approach for engaging with the external environment. In this context, Social
License to Operate (SLO) can play an important role. It refers to the ongoing acceptance
of a company’s standard business practices and operating procedures by its employees,
stakeholders and the general public.
In order to protect and build SLO, companies are encouraged to first do the right thing
and then be seen doing the right thing. This means critically evaluating and re-
evaluating supply chains, human resource management and all the other aspects of a
business. A company must be seen operating responsibly, taking care of its employees,
environment, and being a good corporate citizen.
Thus, ‘social license’ is a much more useful concept than CSR because it ensures that
activities of a business respect the rights of all of those in any community and it can
never be self-awarded, it requires that an activity enjoys sufficient trust and legitimacy
and has the consent of those affected. Also, it cannot be reduced to a number game as
seen in the limits of CSR spending. Thus, social good flows automatically from SLO
unlike the CSR where it is effortful.

14. Previous year Vision IAS Mains Test Series Case


Studies
1. A manufacturing company is located in a small town where it is not easy for people to
find jobs otherwise. The company has stayed in the town even though it could find
cheaper workers elsewhere, because workers are loyal to the company due to the jobs
it provides. Over the years, the company has developed a reputation in the town for
taking care of its employees and being a responsible corporate citizen, owing to its
CSR activities. The manufacturing process used by the company produces a by-
product that has been flowing into the town river for some time now. According to
some media reports people living near the river have started reporting illnesses off
late and there is also damage to aquatic life. People, however, have refused to come
forward as their livelihood depends on the company. You being the Sustainability
Officer of the company believe it to be your duty to report this to the higher
management. You are of the opinion that this matter is associated with the prestige
of the company and should be taken seriously. In the meanwhile the new CEO, who
has just joined the company, has called you for briefing and understanding the local
issues. On the basis of the above case, answer the following: (a) Identify the
stakeholders in the case above. (b) What arguments would you put forth to the CEO
to convince him to act on this matter?
Approach:
• Introduce the answer.
• Identify the different stakeholders in the case above.
• Enlist the arguments you’d present to the CEO of the Company to take action in this
matter.
Answer:
The present case highlights the trade-off between short term economic gains (both for
company and the local population) and long term health and environmental losses. The
manufacturing company, though, has maintained huge trust among the people but is

DELHI JAIPUR PUNE HYDERABAD AHMEDABAD LUCKNOW CHANDIGARH


23 www.visionias.in # 8468022022 ©Vision IAS
Google it:- https://upscpdf.com
https://t.me/UPSC_PDF Download From > https://upscpdf.com https://t.me/UPSC_PDF

also responsible for certain activities which may not be in good interest of the people Student Notes:
and society at large.
(a) The stakeholders identified in the above case are:
• Local population of the area and the people staying near the river- the reported
dumping of the harmful by-product in the river has direct health costs for the local
population. This may lead to severe issues in the long run.
• The employees of the manufacturing company- the functions of the company may
be altered by these reports thereby impacting directly the employees of the
company.
• Management of the company- the management is under direct scrutiny for the
actions it takes on this episode. It will also have implications for the company
operations at different places as well as for the environment.
• District and local administration- they are directly involved as it is the district
administration that needs to preserve the livelihood and wellbeing of the
population. Also, it is responsible for implementation of rules and regulations that
impact the functions of the company.
• Environment including the river and its aquatic life- the destruction caused to the
river and the aquatic life may be irreversible and therefore, the ecology of the area
becomes a legitimate stakeholder in the case.
(b) Since the CEO is new, I as the Sustainability Officer shall update and apprise him of
the issues surrounding this matter based on the internal reports and then convince
him to take actions giving the following arguments-
• Firstly, since the discharge from the factory seems to be related with the reported
incidence of illness and disease, it is a matter of grave concern. The wellness of the
employees is, therefore, one of the primary concerns for the company. A healthy
workforce is necessary to maintain the productivity and efficiency of the operations
of the company.
• Secondly, the brand value of the company is at stake. If the illness spreads due to
the actions of the company, it will directly impact the revenues and credibility of
the company. The company may lose a lot of its goodwill and thereby lose trust
among the people.
• Also, since the case has been already highlighted by the media, it may lead to PILs
and other forms of legal challenges for the company. This will be a drain on the
resources of the company, alongside the loss of public faith.
• There is an urgency to tackle the matter as it has been reported by the media and
there are chances that it gets blown out of proportion. Therefore, objectively
dealing with the matter becomes a priority to escape the trial by media, which may
also be motivated by self-interested parties.
• Also, there is a long term sustainability issue as degradation of the environment
and disturbing the ecological balance of the region may lead to catastrophic
damages in the future.
Considering that the company provides jobs to the locals and their overall well-being is
essential not only for them, but even for an uninterrupted operation of the company– I
shall propose to the CEO to constitute an enquiry to ascertain the details of the
phenomenon, in the meanwhile weekly/bi-weekly health camps should be started by
the company in the affected areas. A medical allowance should be considered for the
employees and also for the affected people on a case by case basis. This could be
undertaken via the CSR or Contingency Management funds. Also, I shall convince the
CEO to undertake sensitisation drives personally, effective communication programmes
on safe health, village to village town halls in the nearby areas so as to build trust
among people, about the company’s CSR medical drives and goodwill exercises.
Further, I would urge transparency and full co-operation with the local administration.

DELHI JAIPUR PUNE HYDERABAD AHMEDABAD LUCKNOW CHANDIGARH


24 www.visionias.in # 8468022022 ©Vision IAS
Google it:- https://upscpdf.com
https://t.me/UPSC_PDF Download From > https://upscpdf.com https://t.me/UPSC_PDF

2. You are the chief executive of ABC Ltd., an automotive component manufacturing Student Notes:
company. Until recently, all of your production plants were located in City X. However,
the cost of production has risen, causing profits to decline. A number of factors have
led to an increase in the production costs. First, the union representing the workers in
your plant waged a successful strike resulting in increased salary and benefits. A
second factor has been imposition of stringent environmental regulations.
Shareholders are concerned about the declining fortunes of the company. Many of the
competitors have moved their operations to less-developed cities, where the
operating costs are less than in City X. ABC Ltd. is a major employer in the city where
it is located, and you know that a plant closure will cause economic dislocation in the
city. Given the situation, answer the following questions:
(a) What are the ethical issues involved in this case?
(b) How can the interests of different stakeholders be reconciled in this case?
Approach:
• After giving a brief introduction of the facts of the case give the ethical issues
involved in the case.
• Suggest a way by which the interest of all parties can be synthesised.
Answer:
An industry is not merely the factory and worker employed. Its working depends
crucially on the social environment where it operates and kind of regulations it is
subject to. The case given presents a situation where because of rising costs due to
multiple factors, relocating the industry is being actively considered. This will lead to
economic distress in the city.
(a) Ethical issues involved
• Corporate Social Responsibility: Company is economic driver of the region as it
gives employment to large number of people. Therefore, relocating it must be
weighed accordingly.
• Profit v/s interest of employees: Production cost may be reduced by relocation to
less developed city, but it will lead to huge unemployment.
• Transfer of burden: Is it ethical to cause environmental damage at some other
place merely to save production cost?
• Public versus organizational interest: Whether relocation is merely for increasing
organizational profit or some public interests would also be served?
(b) Reconciling the interests
The various stakeholders and their interests are:
• The shareholders of the company have interests in higher profits, which the high
costs in the current city are jeopardising.
• The employees are interested in securing better wages and working conditions, and
above all, continuation of employment.
• The local people, as they are economically dependent on the company for their
livelihood.
• The society as a whole, as the operations of the company must be environment
friendly.
Thus, continuing the operation in City X will protect the economic interest of the people
in the region but would be detrimental to the company’s profits, while relocating to a
less developed city would increase the profit margins, but may lead to a collapse of
livelihoods and economy in city X. Also, environment regulations may not be as strict at
the new location, thereby increasing pollution.

DELHI JAIPUR PUNE HYDERABAD AHMEDABAD LUCKNOW CHANDIGARH


25 www.visionias.in # 8468022022 ©Vision IAS
Google it:- https://upscpdf.com
https://t.me/UPSC_PDF Download From > https://upscpdf.com https://t.me/UPSC_PDF

So, to reconcile these different interests, following steps may be taken: Student Notes:
i. Recognition of the social responsibility which the company has and putting across
its interests to all the stakeholders in a transparent manner. The company must
make all efforts to minimise costs without relocations and by taking into confidence
employees, shareholders and local people.
ii. If not, then partial shifting of the company to less-developed cities: The Company
may restructure its units and determine if shifting some of them to smaller cities
may help it to lower the total operating cost. If profitable, it may also consider
outsourcing some of the work to other companies who are already operating in
small cities.
iii. Giving different options to existing employees – such as continuing in marketing or
servicing center as per qualification, shifting to the new location or lump sum salary
for 2 or 3 months as termination amount.
iv. Adopting latest technology to reduce cost of production: This will increase
productivity and reduce the environmental damage. Though initial cost of adopting
may be high but it will be beneficial to the company as well as the society overall in
the longer run.
3. Big firms often undertake sub-contracting to complete large infrastructure projects in
a timely manner. You recently joined one such firm as a manager responsible for
awarding these contracts. Looking at the past records, you find that all contracts for
the past few years have been awarded to a particular firm, X. Your superior has asked
you to award an upcoming contract to the same firm. Although, not binding,
company procedure maintains that sub-contracting work should be offered after
competitive bids. This is to ensure that the firm most suitable for the project in terms
of operations and finance gets the contract. When you discuss this with your superior,
he insists that hiring of the firm X has been done as per legal norms of the company
and no official rule has been violated. You decide to contact the owner of the firm X.
After doing so, you realize that he is the nephew of your superior, who is also a
shareholder in the company.
Approach:
• State some of the ethical issues in the case.
• Evaluate if a situation of interest is generated due to the relationship of your
superior with the owner of firm X and the knowledge of the superior being a
shareholder in the company.
• Suggest ways of awarding contracts in the situation, along with their merits and
demerits.
• State your eventual course of action in this scenario.
Answer:
Basic Facts: In this situation, the company where I work is repeatedly sub-contracting
work to another private firm, X, without holding competitive bids, which should be
carried out according to company procedures. The case highlights the issue of conflict
of interest, as the owner of firm X is the nephew of my superior and is also a
shareholder of the firm.
Stakeholders:
• Myself, as I am the manager of the company, with the responsibility of awarding
contracts.
• My superior who is a shareholder of firm X as well as an employee of the company
where I work.
• Owner of firm X as the decisions made by our company affect his business
interests.

DELHI JAIPUR PUNE HYDERABAD AHMEDABAD LUCKNOW CHANDIGARH


26 www.visionias.in # 8468022022 ©Vision IAS
Google it:- https://upscpdf.com
• Owners of other firms who vie for company contracts as they are not getting a fair Student Notes:
chance in the bidding process.
• Shareholders of the company who can suffer due to poor decisions.
• Consumers who will be affected by substandard infrastructure.
(a) Some of the ethical issues involved in this case are:
• Nepotism and compromise on professional integrity: Subcontracts are being
granted on the basis of personal relations, without competitive bids. Due to this,
there is no guarantee that the firm most suitable in terms of efficiency and financial
aspects gets the contracts.
• Lack of objectivity in decision-making: The interests of my superior and his
nephew are guiding the decision-making process of the company. There is inherent
subjective bias in this scenario.
• Legal vs Ethical: It may be legally correct but not ethical as no official rule of the
company has been violated, but, it goes against established company procedure.
(b) Transaction between two private parties generates a conflict of interest since the
owner of firm X is the nephew of my superior, who has repeatedly directed that the
contract be awarded to firm X. Since my superior is also a shareholder in firm X, he is
invariably taking business decisions on the basis of personal relations and perceived
financial opportunities. He would not be affected much due to poor performance of the
company as his family is benefitting, but other shareholders and employees will be
affected. Thus, personal interests are directly in conflict with interest of shareholders.
Also, they are in conflict with the interest of consumers who expect quality
infrastructure.
(c) Possible ways of awarding contracts in such a situation are:
• Granting the contract to firm X as suggested by my superior.
• Merits: I will remain in good terms with him, may get quick promotion and
increment.
• Demerits: I will be unethical in my work conduct as a manager. I will also obstruct
competitive bidding and play a role in eschewing probable offers that could save
the company more money and get the work done more efficiently. I would be
compromising my integrity and commitment to interest of shareholders, colleagues
and consumers.
• Refuting my superior and revealing his interest in firm X, while simultaneously
organizing the competitive bid.
• Merits: I will follow company procedure and my professional integrity will be
upheld.
• Demerits: I will go directly against the order of my superior, who can hinder my
chances of promotion in the company. I will also violate the established work
hierarchy in the company.
• The most suitable action will be to talk to my superior about the generated conflict
of interest and argue in favour of holding competitive bids. However, if he still
refuses to acknowledge the gravity of the situation, I will approach the company
head/board regarding the issue. This will ensure that other managers and
subordinates are not in a similar position in the future. Meanwhile, I will also
accept competitive bid offers and award the contract to the firm most suitable for
the job. It will ensure my professional integrity and uphold the interest of all
stakeholders.

Copyright © by Vision IAS


All rights are reserved. No part of this document may be reproduced, stored in a retrieval system or
transmitted in any form or by any means, electronic, mechanical, photocopying, recording or
otherwise, without prior permission of Vision IAS.

DELHI JAIPUR PUNE HYDERABAD AHMEDABAD LUCKNOW CHANDIGARH


27 www.visionias.in # 8468022022 ©Vision IAS
PROBITY IN GOVERNANCE: CONCEPTS, Student Notes:

PRINCIPLES AND CHALLENGES


Contents
1. Probity: Definition and Overview ............................................................................................... 2
1.1. Philosophical Basis of Probity in Governance ..................................................................... 2
2. Need for Probity in Governance................................................................................................. 3
3. Principles of Probity ................................................................................................................... 4
4. Transparency: Definition and Overview ..................................................................................... 4
4.1. Transparency: A Pillar of Ethical Governance...................................................................... 5
4.2. Features of Transparent Governance .................................................................................. 6
4.2.1. Information Sharing ..................................................................................................... 6
4.2.2. Participative Governance ............................................................................................. 7
5. Accountability and Responsibility .............................................................................................. 8
5.1. Meaning of Accountability .................................................................................................. 8
5.2. Why is it needed? ................................................................................................................ 9
5.3. How can it be ensured?..................................................................................................... 10
5.4. Meaning of Responsibility ................................................................................................. 11
6. Challenge of Corruption ........................................................................................................... 12
6.1. Understanding Corruption: Unethical Basis ...................................................................... 13
6.2. Impacts of Corruption ....................................................................................................... 14
6.3. Corruption: Analysis and Indian Perspective .................................................................... 15
6.4. Ways to Tackle Corruption ................................................................................................ 17
6.5. Conclusion ......................................................................................................................... 19
7. Other challenges associated with Probity in Governance ....................................................... 19
8. Previous Years UPSC GS Mains Questions ................................................................................ 20
9. Previous Years UPSC Mains Questions: Case Studies ............................................................... 20
10. Previous Years Vision IAS GS Mains Questions ...................................................................... 21
11. Previous Years Vision IAS GS Mains Questions: Case Studies ................................................ 29

Copyright © by Vision IAS


All rights are reserved. No part of this document may be reproduced, stored in a retrieval system or
transmitted in any form or by any means, electronic, mechanical, photocopying, recording or
otherwise, without prior permission of Vision IAS.

DELHI JAIPUR PUNE HYDERABAD AHMEDABAD LUCKNOW CHANDIGARH


1 www.visionias.in # 8468022022 ©Vision IAS
https://t.me/UPSC_PDF Download From > https://upscpdf.com https://t.me/UPSC_PDF

Note- This document shall cover the following sections from the syllabus- Student Notes:
• Probity in Governance- Philosophical Basis of Governance and Probity; Information Sharing
and Transparency in Government; Accountability and Ethical Governance; Challenges of
Corruption

1. Probity: Definition and Overview


Probity originates from the latin word ‘probitas’, meaning good. Probity is confirmed integrity.
It is usually regarded as being incorruptible. It is the quality of having strong moral principles
and strictly following them, such as honesty, uprightness, transparency and incorruptibility.
Probity in Governance is concerned with the propriety and character of various organs of the
government as to whether these uphold the procedural uprightness, regardless of the
individuals manning these institutions.
It involves adopting an ethical and transparent approach, allowing the process to withstand
scrutiny. Probity goes further than the avoidance of being dishonest because it is determined
by intangibles like personal and societal values.

1.1. Philosophical Basis of Probity in Governance


The gist of wisdom on administrative ethics is that the public administrators are guardians of
the administrative state. Two crucial questions raised in this context are-
Why should guardians be guarded? and Who guards the guardian?
Historical perspective:
The ethical concerns of governance have been emphasized broadly in Indian scriptures and
other treatises such as Ramayana, Mahabharata, Bhagvad Gita, Buddha Charita,
Arthashastra, Panchatantra, Manusmriti, Kural, Shukra Niti, Kadambari, Raja Tarangani, and
Hitopadesh. Chinese philosophers such as Lao Tse, Confucius and Mencius also contributed
the theoretical dogmas on ethical governance.
Perspective of various other philosophers:
Western Philosophy identifies 3 eminent school of ethics-
• First, inspired by Aristotle holds virtues (such as justice, charity and generosity) are
dispositions to act in ways that benefit the possessor of these virtues and the society that
he is a part.
• The second by Immanuel Kant, makes the concept of duty central to morality i.e. human
beings are bound, from knowledge of their duty as rational beings, to obey the categorical
imperative to respect other rational beings with whom they interact. Kant supposed that a
rigorous application of the same methods of reasoning would yield an equal success in
dealing with the problems of moral philosophy.
• The third being, the utilitarian standpoint that proclaims that the guiding principle of
conduct should be the greatest happiness (or benefit) of the greatest number.
Utilitarianism is a theory in normative ethics maintained that the moral action is the one
that maximizes utility. Utilitarianism is a form of consequentialism, which implies that the
“end justifies the means”.
Another philosopher who developed ethical theory of governance was David Hume. Hume
upholds that moral distinctions are derived from feelings of pleasure and pain of a special sort,
and not as held by many Western philosophers since Socrates from reason. Working from the
empiricist principle that the mind is essentially passive, Hume claims that reason by itself can
never prevent or produce any action or affection. But since morals concerns actions and
affections, it cannot be based on reason.

DELHI JAIPUR PUNE HYDERABAD AHMEDABAD LUCKNOW CHANDIGARH


2 www.visionias.in # 8468022022 ©Vision IAS
Google it:- https://upscpdf.com
https://t.me/UPSC_PDF Download From > https://upscpdf.com https://t.me/UPSC_PDF

In recent times the movement of good governance initiated by World Bank in 1992 broadly Student Notes:
lays down stress on ethical and moral conduct of administrators.

2. Need for Probity in Governance


In a democracy, probity espouses the principles of equality before law and a respect for the
rights and duties of leaders towards their citizens. Conversely, probity is a societal expectation
which citizens demand from decision makers and all those who function as a part of the state’s
apparatus.
It is a shared belief that the adoption of standards like “accountability”, “transparency” and
“responsiveness” will lead to clean and efficient governance. However, standards do not, by
themselves, ensure ethical behaviour: which requires a robust culture of integrity and probity
in public life. The crux of ethical behaviour does not lie only in standards, but in their
adoption in action and in issuing sanctions against their violation.
The need for probity in governance can be understood as follows:
• Legitimacy of the system: Foremost, it helps build up the legitimacy of the system, i.e. the
state. It builds trusts in the institutions of the
state and a belief that the actions of the state In this regard, the Scandinavian economist-
sociologist, Gunnar Myrdal in his book
will be for welfare of the beneficiaries.
‘Asian Drama’, describes India as a ‘soft
• Trust building through ethical outcomes: It society’. It is a society where there is a:
leads to prudent and ethical outcomes and • Lack of will to enact laws necessary
building of trust over time. for progress and development
• Objectivity: It provides for an objective and • Lack of will to implement even the
independent view on the fairness of the existing laws
process • Lack of discipline at all levels –
• Checks and balances: It helps in checking the including the administration and
abuse and misuse of power by various organs structures of governance
of government such as magistracy, police and
all other providers of public service e.g. PWD, health, education, etc.
• Equitable and sustainable development: It is an essential and vital requirement for an
efficient and effective system of governance and for socio-economic development.
• To serve the constitutional cause: Probity in Governance is required to serve the motto of
Constitution. i.e. to provide Social, Political and economic justice to all. It enhances faith in
the governance.
• Reduced politicization of bureaucracy: It helps address nepotism, Favoritism, Political
partisanship. Public reposes more trust in governance and therefore it facilitates
participatory governance. It leads to avoidance of sub-optimal outcomes, corruption and
poor perception.
The famous economist Mehboob-Ul-Haq, remarked that corruption existed everywhere but it
is a greater cause of worry in South Asia because it is exploitative and feeds on the helpless
poor citizen.
Though instilling a sense of morality in the people and its representatives is a long-term
process which needs an immediate attention, some legislatives measures can be designed with
an utmost urgency to ensure probity in governance.
According to the National Commission to Review the working of Constitution’s (NCRWC,
2002) report on ‘Probity in governance’:
• ‘An important requisite for ensuring probity in governance is absence of corruption. The
other requirements are effective laws, rules and regulations governing every aspect of
public life and, more important, an effective and fair implementation of those laws, etc.

DELHI JAIPUR PUNE HYDERABAD AHMEDABAD LUCKNOW CHANDIGARH


3 www.visionias.in # 8468022022 ©Vision IAS
Google it:- https://upscpdf.com
https://t.me/UPSC_PDF Download From > https://upscpdf.com https://t.me/UPSC_PDF

• The NCRWC made suggestions on ways to enforce probity in governance, specifically on Student Notes:
legal and procedural aspects to control the menace of corruption.
Thus, Probity in governance is an essential and vital requirement for an efficient and effective
system of governance and for socio economic development.

3. Principles of Probity
While the doctrine and the principles of probity are clear and well defined, giving a concrete
shape to implement them is the task of the government.

4. Transparency: Definition and Overview


Transparency has been defined by the Second Administrative Reforms Commission as, the
‘availability of information to the general public and clarity about functioning of governmental
institutions.’
Further, Vishwanath and Kaufmann (1999) have defined transparency as the “increased flow
of timely and reliable information, which is accessible to all relevant stakeholders”. This
perspective emphasizes not only the availability of information, but also its reliability and
accessibility.
It is not a new concept as shown in the timeline depicting the evolution of ‘transparency and
right to information’ which can be charted out as follows:

DELHI JAIPUR PUNE HYDERABAD AHMEDABAD LUCKNOW CHANDIGARH


4 www.visionias.in # 8468022022 ©Vision IAS
Google it:- https://upscpdf.com
https://t.me/UPSC_PDF Download From > https://upscpdf.com https://t.me/UPSC_PDF

• Resolution 59 of the UN General Assembly adopted in 1946 recognised Freedom of Student Notes:
Information an integral part of the fundamental right of freedom of expression.
• International Covenant on Civil and Political Rights (1966) and the American Convention
on Human Rights (1969) also recognized the Right to Information in one way or the other.
• Supreme Court Case in Mr. Kulwal v/s Jaipur Municipal Corporation, 1982 emphasized
that without Right to Information, the freedom of speech and expression cannot be fully
used by the citizens.
• MKSS (Mazdoor Kisan Shakti Sangathan), a grassroots organization led by activists Aruna
Roy and Nikhil Dey that started the movement of Right to Information regarding
development projects in rural Rajasthan
• NCPRI (National Campaign for People’s Right to Information),1959 was formed with an
objective of getting the legislation on Right to Information passed.
• Tamil Nadu was the first state to enact RTI legislation in 1997, Rajasthan enacted the Right
to Information Act in 2000
• The RTI Act 2005 overcame these lacunae and provided a legal framework for the working
of Right to Information
Why is Transparency and Information Sharing essential?
Transparency and information sharing form the two most important pillars of the good
governance in a mature democracy.
• Transparency and information sharing are key tools to build accountability and trust in
public offices. They are indispensable in the quest for good governance – be it timely
execution of projects, or fight against corruption or arbitrary exercise of power.
• These empower citizens, foster their participation in public affairs, strengthen
participatory democracy and usher in people centric governance.
• In transparent governance system, the criteria, procedures and systems of decision-making
are openly known to all. Information sharing is sine-qua-non for transparency.
• Further, transparency of information is also seen as significant for motivating citizens to
exercise ‘voice power’. Voice power is defined as the capacity of citizens to pressurize the
frontline officials in ensuring effective delivery of services. This voice power results in
greater accountability.
Therefore, we can say that in a broad sense, Transparency refers not only to the level of
openness, accessibility and reliability of information but also to style of functioning and the
priorities accorded by the government in policy making.

4.1. Transparency: A Pillar of Ethical Governance


It can be contended that transparency and accountability is the fundamental requirement for
preventing the abuse of power. It is necessary condition for ensuring that power is directed
towards the achievement of efficiency, effectiveness, responsiveness and transparency.
Open, transparent and accountable government is an imperative prerequisite for community-
oriented public service delivery because without it covert unethical behaviour will result.
Transparency is not just a means to fulfill certain functions but an end in itself. Belief in the
openness of government to regular inspection is so firmly ingrained in our collective
consciousness that transparency has innate value.

DELHI JAIPUR PUNE HYDERABAD AHMEDABAD LUCKNOW CHANDIGARH


5 www.visionias.in # 8468022022 ©Vision IAS
Google it:- https://upscpdf.com
https://t.me/UPSC_PDF Download From > https://upscpdf.com https://t.me/UPSC_PDF

Student Notes:

4.2. Features of Transparent Governance


Over time, India has made consistent efforts to imbibe Transparency in the government by
working on the following three focus areas:
• Right to information laws – to establish the constitutional/legal right for a citizen to access
the information that they want;
• Proactive transparency – to commit governments to publishing as much information as
possible in an accessible form;
• Open data approach – to enable all the stakeholders to reconfigure the government data
into inter-operable and accessible formats.
4.2.1. Information Sharing
Information sharing refers to the exchange of data among various governing bodies,
organizations and the general public. Access to information is a crucial, and perhaps the largest,
subset of it.
Indian Parliament has also institutionalized this right by legislating Right to Information Act
2005 which mandates timely response to citizen requests for government information (refer
later sections of this document for the detailed analysis of the RTI Act).
Further, parliamentary debates are televised in India, government audit reports are published
and the government positions on various policy issues are widely advertised through websites,
radio and social media. Most recently, the government has launched data.gov.in website to
promote Open Data Government (ODG).

DELHI JAIPUR PUNE HYDERABAD AHMEDABAD LUCKNOW CHANDIGARH


6 www.visionias.in # 8468022022 ©Vision IAS
Google it:- https://upscpdf.com
https://t.me/UPSC_PDF Download From > https://upscpdf.com https://t.me/UPSC_PDF

Student Notes:

4.2.2. Participative Governance


Participative governance is one of the means to achieve transparency in governance through
the bottom up approach (73rd and 74th Constitutional Amendment Acts, by giving
constitutional status to Panchayats and Urban Local bodies). It entails participation of people
in decision-making at the grass root level through decentralization of powers to the local self-
governing bodies.
Following are some other initiatives towards participative governance:
• Social Audit –It is a potent tool in the hands of citizens to monitor and evaluate the
working of various governance bodies. Initiated by the Mazdoor Kisan Shakti Sangathan
(MKSS) as ‘jansunwai’ (public hearing), in 1990s, now social audits have been made
mandatory in statutes such that the Mahatma Gandhi National Rural Employment
Guarantee Act. Such audits have increased citizen participation and have helped in
ensuring checks and balance in system. For instance, in Andhra Pradesh, state-wide social
audits found fraud of large amounts resulting in administrative or criminal charges
against nearly 7,000 officials. This has also ushered in environmental democracy and
governance.

DELHI JAIPUR PUNE HYDERABAD AHMEDABAD LUCKNOW CHANDIGARH


7 www.visionias.in # 8468022022 ©Vision IAS
Google it:- https://upscpdf.com
https://t.me/UPSC_PDF Download From > https://upscpdf.com https://t.me/UPSC_PDF

• Resident Welfare Associations –These include state-specific incentives to incorporate the Student Notes:
views of society or to devolve the decision-making power for basic civic functions to the
citizens. These initiatives have been applauded for their implementation and intent of
engaging common citizens to promote ‘janbhagidari’ (public participation).
• The Bhagidari scheme of the Delhi Government directly involves citizens through RWAs in
monthly meetings in order to mobilize the community to take responsibility for their
respective zones and areas. These RWAs are also the first point of contact for state utilities
while planning development-related and other changes.
• People’s Plan Campaign: (PPC) in Kerala was an experiment in decentralized planning
(1996). Local governments were required to make their own Five-Year Plans with direct
engagement of public. Adequate funds from the State’s development budget were also
devolved for the process. However, this scheme has since been scrapped.

5. Accountability and Responsibility


5.1. Meaning of Accountability
Accountability means making the public officials answerable for their behavior and responsive
to the entity from which they derive their authority. Holders of public office are accountable for
their decisions and actions and must submit themselves to the scrutiny necessary to ensure this.
Accountability also means establishing criteria to measure the performance of public officials,
as well as oversight mechanisms to ensure that standards are met.

Answerability refers to the obligation of the government, its agencies and public officials to
provide information about their decisions and actions and to justify them to the public and
those institutions of accountability tasked with providing oversight.
Enforcement suggests that the public or the institution responsible for accountability can
sanction the offending party or remedy the contravening behavior. As such, different
institutions of accountability might be responsible for either or both of these stages.
Accountability has little meaning without transparency. Without complete and correct
information in public domain, accountability will be of little value. Accountability is not merely
‘to whom’ and ‘how’, it is also ‘for what’. The ‘what’ aspect is provided through transparency.
This is the reason that transparency and accountability are mostly used in the same breath.

DELHI JAIPUR PUNE HYDERABAD AHMEDABAD LUCKNOW CHANDIGARH


8 www.visionias.in # 8468022022 ©Vision IAS
Google it:- https://upscpdf.com
https://t.me/UPSC_PDF Download From > https://upscpdf.com https://t.me/UPSC_PDF

Student Notes:

Institutions for extracting accountability of public servants for their action and inaction-
1. Head of the organization at the organizational level.
2. Departmental head at departmental level.
3. Concerned ministry at the ministerial level.
4. Parliamentary proceedings at the national level.
5. Public opinion at popular level.
6. Media and civil society at societal level.
7. Courts at judicial level.
There is thus both a routine accountability which is mostly departmental in nature and
externally enforced accountability which is assessed only when called into question.

5.2. Why is it needed?


Evaluating the ongoing effectiveness of public officials or public bodies ensures that they are
performing to their full potential, providing value for money in the provision of public services,
instilling confidence in the government and being responsive to the community they are meant
to be serving.
1. It prevents the public services from turning into tyrant as they are held answerable to their
deeds and mis-deeds.
2. Avoids conflict of interests-Setting accountability clearly demarcates area of one’s actions
where he or she is required to act.
3. The first and last beneficiary of public service is public, as the public services are required to
act in interest of public and they are answerable for their actions.
4. Promotes justice, equality, and egalitarianism because public servants are required to
realize these constitutional ideals and at the same time, they are answerable for their
actions.
5. It brings legitimacy to public services- Accountability promotes the loyalty to service as
actions are carefully calculated and keeps a check on arbitrary and ill-conceived actions and
policies.

DELHI JAIPUR PUNE HYDERABAD AHMEDABAD LUCKNOW CHANDIGARH


9 www.visionias.in # 8468022022 ©Vision IAS
Google it:- https://upscpdf.com
https://t.me/UPSC_PDF Download From > https://upscpdf.com https://t.me/UPSC_PDF

6. Be it fear of legal consequences or an outcome of one’s morality, owing accountability for Student Notes:
their actions motivates the public servants to discharge their duty with honesty, integrity
and efficiency.

5.3. How can it be ensured?


The accountability mechanisms in any country are broadly categorized as those that are located
within the State and those outside it. Accountability of the executive arm of government to
Parliament and to the citizens of the country is of course the fundamental feature of a
democracy.
Analysts have categorized these accountability mechanisms into “horizontal” accountability
mechanisms which refer to those located within the State as against ‘vertical’ accountability
mechanisms which are those outside the State and include the media, civil society and citizens.

Further, by promoting transparency, through effective implementation of RTI in full earnest,


especially the pro-active disclosure, then public officials would be more careful to function

DELHI JAIPUR PUNE HYDERABAD AHMEDABAD LUCKNOW CHANDIGARH


10 www.visionias.in # 8468022022 ©Vision IAS
Google it:- https://upscpdf.com
https://t.me/UPSC_PDF Download From > https://upscpdf.com https://t.me/UPSC_PDF

strictly according to the spirit of the law and constitution. Secondly, it would educate the Student Notes:
people about their rights, entitlements and how to question their fulfillment.
There should be capacity building of the stakeholders to question the public officials about
their discharging of official duties. For example, the concept of Social Audit under MGNREGA.
Also, effective implementation of laws like Prevention of Corruption Act should be ensured.
In India, constitutional and statutory bodies such as the office of the Comptroller & Auditor
General, the Election Commission, and the Central Vigilance Commission (CVC) are examples of
other oversight mechanisms that are autonomous but lie within the framework of the State. It
should be ensured that these bodies are strengthened and empowered in both letter and spirit.
An independent judiciary embodies the constitutional doctrine of separation of powers and is
another important element in the system of checks and balances that exists in any democratic
country. The independence and effective role of judiciary is crucial for upholding the rule of law
in India, thus various steps like Memorandum of Procedure, e-courts among other should be
promoted.
The final expression of accountability in a democracy is through the medium of periodic
elections which is an instrument for punishing and rewarding the Government of the day, and
therefore, serves as an ultimate instrument of accountability. The electoral reforms and
democratic maturity of people is the ultimate bulwark of accountability.

5.4. Meaning of Responsibility


It means accountability to oneself, i.e. when the accountability turns inward. It is a moral
concept, where a person feels the answerable to oneself for all his actions, even if it is not
covered by any law.
It is more enduring than accountability, because it is based on ethical reasoning, and the person
would always do the right thing, even if nobody is there to watch his action, as he holds himself
answerable to oneself. Here the person takes ownership of one’s actions and decisions.

DELHI JAIPUR PUNE HYDERABAD AHMEDABAD LUCKNOW CHANDIGARH


11 www.visionias.in # 8468022022 ©Vision IAS
Google it:- https://upscpdf.com
https://t.me/UPSC_PDF Download From > https://upscpdf.com https://t.me/UPSC_PDF

Though, these terms are used interchangeably, there is a subtle difference between the two. Student Notes:
Accountability makes the person accountable for the consequences of the actions or decisions
made by him/her. As against this, consequences are not necessarily attached to the
responsibility. Further, accountability requires a person to be liable and answerable for the
things, he/she does. Conversely, responsibility expects a person to be reliable and dependable
to complete the tasks assigned to him.

Responsibility is said to be attached with ethical maturity, therefore, to inculcate responsibility


we need to impart the moral, ethical values through training, role playing etc.

6. Challenge of Corruption
The word ‘corrupt’ is derived from the latin word corruptus, meaning ‘to break or destroy’.
One of the most persuasive definition given by the Word bank- “Corruption is the abuse of
public funds and or office for private or political gains.”
This confirms the contention that corruption is a failure of governance. Corruption is an
important manifestation of the failure of ethics.
There are majorly two types of Corruption: Coercive and Collaborative.
Coercive Corruption:
• Coercive Corruption is where a person is forced to give bribe to get a job done to which he
is otherwise entitled, such as getting a ration card, a driving license, a passport, an electric
connection, or sanction for constructing a building.
• Such corruption is endemic to how government carries out its routine activities such as
issue of license, permit, policing and revenue collection.
• Payments are generally of petty nature, to lower level inspectors, babus and clerks, and
payoff facilitates benefits to which one is otherwise legally entitled
Collaborative/ Big ticket Corruption:
• Collaborative Corruption is one involving high officials and ministers that often implicate
multinationals and large domestic firms, in which both parties gain substantial pecuniary
benefits, though the public is the ultimate loser.
• They mostly relate to mega-projects, large value contracts, concessions and other favors
and difficult to prove nexus, as both bribe giver and takers are beneficiaries.
The countries which are rated as ‘very corrupt’ have a high incidence of coercive corruption,
where corruption is institutionalized and citizens have to pay bribe for even basics.

DELHI JAIPUR PUNE HYDERABAD AHMEDABAD LUCKNOW CHANDIGARH


12 www.visionias.in # 8468022022 ©Vision IAS
Google it:- https://upscpdf.com
https://t.me/UPSC_PDF Download From > https://upscpdf.com https://t.me/UPSC_PDF

6.1. Understanding Corruption: Unethical Basis Student Notes:

The ethical and moral values of a person decrease with steep rise in power. A Person who has
absolute authority may misuse his position.
“Power tends to corrupt and absolute power corrupts absolutely”-Lord Acton
The focus on acts of corruption may mainly be placed 'before the act' by deontologists and
'after the act' by utilitarians.
Utilitarians maintain that people must act morally in ways that will produce the best
consequences.
Deontologists rely on adherence to certain rules in order to militate against acts of corruption.
Deontology as a rule-based normative ethical theory illuminates the moral dimension of
human activities, especially those that concern human obligations and responsibilities.
Deontologists see punishment for criminal deeds as a means of deterrence, whilst
utilitarianism views punishment from the retributive angle. Both these approaches demand
special attention in a study of corruption.

Honorable late President Abdul Kalam while terming corruption as a dreadful malady afflicting
the society observed that Erosion of Conscience and lack of righteousness arise from the
never-ending attitude of greed of ‘what can I take?’ rather than ‘What can I give?’. He further
added that Failure of inculcation of virtue ethics in individual character ethics,
work/professional ethics, parental ethics at all levels of society is a reason for increased
corrupt acts and laws not effective as deterrents.

DELHI JAIPUR PUNE HYDERABAD AHMEDABAD LUCKNOW CHANDIGARH


13 www.visionias.in # 8468022022 ©Vision IAS
Google it:- https://upscpdf.com
https://t.me/UPSC_PDF Download From > https://upscpdf.com https://t.me/UPSC_PDF

Student Notes:

6.2. Impacts of Corruption


Irrespective of the causes of corruption which may be economic, social, legal administrative
or political, and the logic given by some ‘pragmatic’ experts in favour of the short-term benefits
of speed money, corruption slowly and surely rots away the credentials of an administration,
person and even the society as a whole in the long term.
Recently, India has been ranked at the 80th position among 180 countries and territories in the
Corruption Perception Index (CPI) prepared by Transparency International.
Ethical:
• Corruption generates black money, which impacts development. Moreover, when the
corrupt gets scot free and prosper, it leads to the development of an unethical culture
wherein means may not necessarily justify the end. Corruption is considered as a given
and is no more abhorred.
• As a corollary to this honesty no more remains a virtue to vouch for and practiced. Once
a society accepts corruption as a given its journey to the abyss initiates.
• It sets in motion a cycle of vengeance wherein a person tries to make good the money he
loses out due to corruption by indulging in corrupt acts himself. For example, a
businessmen forced to pay bribery to an government official make up for the loss by selling
sub-standard products/services or by inflating their prices, which forces others to do the
same and in the long run everyone loses out including the corrupt government official as
he is the consumer of such products and services too, e.g. today it’s a challenge in India to
find unadulterated milk, paneer and other food items.
Political:
• Implications for democracy: EIU's Democracy Index, 2020 has placed India at 51st position
(out of 167 countries) and categorizes it as a flawed democracy citing erosion of civil
liberties primarily due to lingering issues of corruption and integrity. Corruption eats into
the vitals of an administrative setup thus threatening public faith in democracy itself.
• It leads to a general disinterest in political process and may even encourage the growth of
radical trends in marginalized communities, e.g. Naxalism

DELHI JAIPUR PUNE HYDERABAD AHMEDABAD LUCKNOW CHANDIGARH


14 www.visionias.in # 8468022022 ©Vision IAS
Google it:- https://upscpdf.com
https://t.me/UPSC_PDF Download From > https://upscpdf.com https://t.me/UPSC_PDF

Social: Student Notes:


• Corruption erodes trust in government and undermines the social contract. This is cause
for concern across the globe, but particularly in contexts of fragility and violence, as
corruption fuels and perpetuates the inequalities and discontent that lead to fragility,
violent extremism, and conflict.
• Honest are victimized and the competent / honest are not attracted to public services. It
impacts morale and generates a culture of lethargy, non-participation and lack of focus on
public-service delivery.
• Culture of lethargy (‘aaraam’ and ‘chalta hai’) leads to slow pace of development.
Economic:
• Cost overruns- Corrupt workforce is focused on self-aggrandizement than on finding
solutions to felt problems.
• Promotions / postings determined on the basis of criteria other than merit.
• Effectiveness of government programmes decline, impacts the quality of public
infrastructure.
• Corruption impedes investment, with consequent effects on growth and jobs. Countries
capable of confronting corruption use their human and financial resources more efficiently,
attract more investment, and grow more rapidly.
• Corruption has adverse effects on human development (indicated by HDI indicators), as
it’s both the cause of poverty and a barrier in reducing it.
• Corruption hurts the Poor the most, e.g. by fueling inflation, increases the cost of basic
services for common man, shifts government expenditure from priority areas to those
areas where rent seeking is greater.

6.3. Corruption: Analysis and Indian Perspective


It is often said that in India, corruption thrives because it is a low risk, high profit business.
There are so many safeguards and protection in the system in which a public servant operates
that it is very difficult to catch and punish an official indulging in corruption.
Some of the key observations that can be understood is as follows:
1. The weak legal framework for punitive action:
• Because of ill-defined policies, weak regulatory frameworks and large indiscretion
among ministers and high public officials, India has witnessed large cases of crony
capitalism that may derail the entire economy. E.g. The CBI alone has hundreds of
cases pending in various courts under the PCA, some of them as old as 25 years.
• However, the judicial process in India is slow and time-consuming. Cushions of safety
have been built in the legal system on the principles that everybody is innocent till
proved guilty. The legal provisions are exploited by the corrupt to escape punishment.
E.g. The conviction rate of criminal cases in India is hardly 6 per cent. The PCA, 1988
hardly serves as a deterrent.
• There is also a statutory bar that a public servant of the rank of Joint Secretary and
above can’t be prosecuted, without prior government permission; the government
often delays or prevaricates sanction, thus effectively barring trial of the guilty official.
2. Political Corruption: The Electoral Process and Reform:
• It is widely recognized that huge money required to fight elections is the foundation of
political corruption. Due to the vast geographical area of a constituency, with more
than two million voters in many cases, a candidate has to spend huge money to
contest the elections.

DELHI JAIPUR PUNE HYDERABAD AHMEDABAD LUCKNOW CHANDIGARH


15 www.visionias.in # 8468022022 ©Vision IAS
Google it:- https://upscpdf.com
https://t.me/UPSC_PDF Download From > https://upscpdf.com https://t.me/UPSC_PDF

• A good part of this money comes from business houses, who expect quid pro quo, in Student Notes:
the form of opportunities to make black money and other favors. There are several
other problems with our electoral system, such as flaws in the electoral rolls, lack of
voters’ education, booth capturing, and intimidation of voters.
• The most serious problem relates to persons with criminal background getting elected.
In 2004, 24% of members of Parliament (MPs) had criminal cases pending against
them. In 2009, that went up to 30%, in 2014 to 34%, and in 2019 as many as 43% of
MPs had criminal cases pending against them.
3. The economic policy and the controlled economy:
• License Permit Raj and The Rise of Corruption This regulatory regime gave
Monster: India adopted a socialist model of government extensive power of
economic development with the state occupying patronage, as also of delay and
commanding heights since the time Five Year Plans extortion. And they were fully
were launched in 1950s. This model of economic exploited by the politicians on
the pretext of raising party funds,
development led to the government doing almost
the bureaucrats had their own
everything and placing vast discretionary powers in share of the loot, and bribery
the hand of public officials in what has been called became a pervasive phenomenon
‘license permit Raj’. at all levels of the government.
• Bureaucratic Red-Tapism and Abetting Corruption: (S.S Gill, Former Civil Servant)
This has been highlighted suitably by Robert
Merton as, “When the rule becomes more important than the game itself”.
• While we have liberalized the economy, there has been practically no reform in archaic
rules, procedures and systems of delivering public
services. Departments such as police,
municipal corporations, land
• This has built in provision for delay and records, sales tax, income tax,
prevarication giving opportunity to officials to excise and customs are known to
indulge in corruption and harass the hapless be corrupt and even routine work
citizens. cannot be done without giving
• Over-regulation combined with a weak state bribe-
fosters corruption. Cumbersome rules that make (OP Tiwari’s analysis on
market entry difficult or impose costs on Corruption in India)
legitimate economic activity, if well-enforced, tend
to hamper growth because they create high barriers for new projects, investments and
entrepreneurial ventures, and this hurts economic activity.

DELHI JAIPUR PUNE HYDERABAD AHMEDABAD LUCKNOW CHANDIGARH


16 www.visionias.in # 8468022022 ©Vision IAS
Google it:- https://upscpdf.com
https://t.me/UPSC_PDF Download From > https://upscpdf.com https://t.me/UPSC_PDF

4. Rise of the Elites and the post liberalization Corruption: Student Notes:
• The mega corruption thrives because of a nexus between big business, politician and
bureaucrat as highlighted by Transparency International.
• Irrespective of coercion involved, the fact remains that bribery fosters a culture of
impunity and repeat corruption, undermines the functioning of public institutions
and fuels a perception that governments and bureaucracies are up for sale to the
highest bidder.
• Post-economic liberalization opportunity of corruption has increased many-fold due
to policies of privatization, public–private Combating Corruption.
• As per Global Financial Integrity, Deregulation and trade liberalization are the main
drivers of illicit flow of money abroad. (out of the illicit flow of USD 462 billion from
the country since 1948, 68 per cent occurred during the post-reform period of 1991–
2008)
5. MNC’s and Lobbying: Difference between Bribery and
Lobbying:
Lobbying means any activity of a group or individual Bribery is illegal because it prioritizes
with special interest designed to influence the personal gains over public interest
legislative or regulatory actions of a public authority. while lobbying without the use of
extortive and corrupt means further
Lobbying by MNCs in nexus with the incumbent erodes democratic ideals of
government is employed as means to further vested participation.
interests to get the clearances of projects, diversions,
bypassing of laws, land acquisition working in their favour.

Case Study: Kalikesh Narayan Singh Deo, a Lok Sabha MP from Orissa introduced a Private
Members Bill to regulate lobbying activities. The Bill required lobbyists to register themselves
and defined lobbying as communication with and payment to a public servant with the aim of
influencing a legislation or securing an award of a contract. Such a definition however blurs
the distinction between lobbying and bribery. It may even have the unintended consequence
of legitimizing bribery prohibited under the Prevention of Corruption Act, 1988.

6.4. Ways to Tackle Corruption


1. Ingrain ethical organizational culture through regular ethical training modules and
workshops:
The 2nd Administrative Reforms Commission states that poor organizational culture has
led to degradation of values and corruption in administration in India. Also, if the work of
the public servants is not driven by values oriented to public service, it may lose the trust,
confidence and respect of the people who rely on it.
Thus, an ethical organizational culture through regular ethical training modules,
workshops that encourages effective employee participation and transmission of positive
values must be employed that leads to better motivation and higher levels of self-esteem
for public servants.
Ethical and spiritual training sessions need to be organized in various training centers like
LBSNAA for ethical orientation of public officials to bring about an attitudinal change. Use
of Yoga and meditation for development of the inner self of the officials is required.
2. Strict adherence of Code of Conduct and Code of Ethics:
The code of conduct established for various officials needs to be backed by a Code of ethics
which needs to be imbibed internally by the officials. Code of conduct has its limitations
but code of ethics will help in imbibing values like courtesy to public, honesty and
dedication towards work, responsiveness towards needs of public. This will ensure that
there will be no need of coerce public officials to show ethical behaviour.

DELHI JAIPUR PUNE HYDERABAD AHMEDABAD LUCKNOW CHANDIGARH


17 www.visionias.in # 8468022022 ©Vision IAS
Google it:- https://upscpdf.com
https://t.me/UPSC_PDF Download From > https://upscpdf.com https://t.me/UPSC_PDF

3. Power of the people: Enlightened Citizens as part of the system Student Notes:
Citizens also need to undergo attitudinal change and must refrain from bribing officials
through ‘speed money’ to get their work done faster. They must be educated about the
social costs of corruption.
Similar ethical and spiritual training sessions could be organized by the government to
bring about this change. Lessons from the lives of great personalities like Swami
Vivekananda, Mahatma Gandhi, Pandit Jawaharlal Nehru should be propagated among
the masses so as to inspire them to live their lives according to the values followed by these
legends.
4. Effective implementation of Institutional measures:
Strict and evolved implementation of the accountability mechanisms be it effective laws,
mechanisms, procedures and tools in accordance with the changing dynamics of the
requirements of the society is crucial.
Bring together formal and informal processes (this means working with the government as
well as non-governmental groups) to change behavior and monitor progress will cut red-
tapism.
5. Power of Technology:
The ambition of digital government is to transform the analogue, paper-based, legacy
systems used to interact with citizens and make public services open, simple and citizen-
centric. Eg: E-auctions of Coal, telecom spectrum, digitization of services like passport,
DBT, cashless transactions etc.
Use of e-governance and ICT tools to build dynamic and continuous exchanges between
key stakeholders: government, citizens, business, civil society groups, media, academia
thus ensuring transparency and accountability in governance. For example: The Indian
Government’s Unique Identification Number AADHAR initiative of gathering biometric
and demographic data of all Indian residents and linking it with social benefits like public
distribution system entitlements is a good example to plug such leakages.
The Bhoomi online management, and delivery of land records in Karnataka need to be
replicated across other services with rigor.
6. Act globally and locally:
Corruption is a global problem that requires global solutions. Keep citizens engaged on
corruption at local, national, international and global levels – in line with the scale and
scope of corruption. Make use of the architecture that has been developed and the
platforms that exist for engagement. Eg: UN Anti-corruption working group and WB anti-
corruption initiatives
Some global success stories are:
• The success story of M-Pesa/M-Paisa in African countries and Afghanistan is widely
known. It not only made monetary transfers easier and more economical but also
fought corruption.
• Chile’s e-commerce public procurement system ChileCompra has gained immense
popularity around the world by bringing transparency in public spending.
Lastly, the fight against corruption must be turned into a mass movement like the way
Mahatma Gandhi turned the struggle for independence into a mass movement. We must
instill a sense of patriotism among the officials and larger public that by following an ethical
path they are contributing towards nation building. They must be given a sense of ownership
regarding the country they are living in and they should be made stakeholders in this process.

DELHI JAIPUR PUNE HYDERABAD AHMEDABAD LUCKNOW CHANDIGARH


18 www.visionias.in # 8468022022 ©Vision IAS
Google it:- https://upscpdf.com
https://t.me/UPSC_PDF Download From > https://upscpdf.com https://t.me/UPSC_PDF

This will ensure that not only corruption will be eradicated from the country and an ethical Student Notes:
perspective towards life will be inculcated among the people. It will also mark the coming of
the second independence and rebirth of our nation

6.5. Conclusion
If corruption levels in India are reduced to those in Scandinavian countries, GDP would
increase by 1.5% and investment would increase by 12%. If all overseas black money is
brought back, India can pay back its debt. As many philosophers like Rabbi Haim of
Romshishok have pointed out, the difference between heaven and hell lies in the fact whether
the people in a given society have empathy for and trust on each other. ‘Sanskritization of
corruption’ will only push our society towards a living hell.

7. Other challenges associated with Probity in Governance


Probity in governance is an essential and vital requirement for an efficient and effective system
of governance and for socio-economic development.
Some of the important issues/ challenges that hinders the path of ensuring probity in
governance is as discussed below:

• Lack of fair and effective implementation of laws: The other requirements are effective
laws, rules and regulations governing every aspect of public life and, more important, an
effective and fair implementation of those laws, etc. Indeed, a proper, fair and effective
enforcement of law is a facet of discipline.
• Colonial legacy: Due to ingrained culture of indiscipline resulting out of colonial legacy of
bureaucracy/executive functioning, that those in
authority can exercise power in an unchallenged Unfortunately for India, discipline is
way. This culture of impunity amounts to a culture disappearing fast from public life and
of indiscipline and untruth; morality, both public without discipline, as the
and private, is at a premium. Discipline implies inter Scandinavian economist- sociologist,
Gunnar Myrdal, has pointed out, no
alia public and private morality and a sense of
real progress is possible. While in the
honesty. West a man who rises to positions of
• Historical and social asymmetries of powers in higher authority develops greater
society: Nearly 90% of the people work in the respect for laws, the opposite is true
unorganized sector. Two thirds of the remaining in our country.
work in the organized sector having job security and
regular monthly wages and are employees of the state either directly or indirectly. Such
asymmetry of power reduces societal pressure to conform to ethical behaviour.
• Erosion of Values and Institutions: Gross perversion of the Constitution and democratic
institutions amounting to willful violation of the oath of office and this is so deeply
entrenched in the system that most people regard corruption as inevitable and any effort to
fight it as futile. This cynicism is spreading so fast that it bodes ill for our democratic system
itself.
Surveillance Society
Surveillance involves paying close and sustained attention to another person. The term encompasses not
only visual observation but also the scrutiny of all behaviour, speech, and actions. Prominent examples of
surveillance include surveillance cameras, wiretaps, GPS tracking, and Internet surveillance.
When this activity includes an entire or a substantial fraction of a population, it is said that there is a
normalisation of a surveillance society. The recent decision of the Government of Delhi to install CCTV
cameras in different parts of the capital has generated a debate on sanctity and normalisation of a
surveillance society.
Ethical Issues in Surveillance
• What are the areas and events when people do not mind being surveyed?
o We do not have issues with the security checks at airports and railway stations. In fact, we

DELHI JAIPUR PUNE HYDERABAD AHMEDABAD LUCKNOW CHANDIGARH


19 www.visionias.in # 8468022022 ©Vision IAS
Google it:- https://upscpdf.com
https://t.me/UPSC_PDF Download From > https://upscpdf.com https://t.me/UPSC_PDF

demand more and more surveillance. o Whereas on the other hand, even a small intrusion in Student Notes:
our phone is resisted forcefully.
• What powers of surveillance states should have over their citizens?
o Can the state have a universal access and unlimited powers to survey a particular person, if it is
convinced of its efficacy? Or there has be a set of limitations on its use.
• What powers of surveillance private companies should have over clients?
o The companies in the guise of service delivery and research may be use misusing the data of
citizens. E.g. as revealed in the recent case of Facebook where the users personal data and
activity was leaked to other agencies.
• Which technologies should and shouldn’t be used?
o Today there are number of advanced technologies available which can intrude into the private
space of citizens, such as phone tapping, spyglasses etc. There needs to be a rationing of the
technologies available to the government agencies, which should be mandated by law.

8. Previous Years UPSC GS Mains Questions


2013
• The good of an individual is contained in the good of all. What do you understand by this
statement? How can this principle be implemented in public life?
2014
• What does ‘accountability’ mean in the context of public service? What measures can be
adopted to ensure individual and collective accountability of public servants?
• What do you understand by ‘probity’ in public life? What are the difficulties in practicing it
in the present times? How can these difficulties be overcome?
• It is often said that poverty leads to corruption. However, there is no dearth of instances
where affluent and powerful people indulge in corruption in a big way. What are the basic
causes of corruption among people? Support your answer with examples.
2016
• "Corruption causes misuse of government treasury, administrative inefficiency and
obstruction in the path of national development. " Discuss Kautilya's Views.
2018
• What is mean by public interest? What are the principles and procedures to be followed by
the civil servants in public interest?
2019
• What do you understand by probity in governance? Based on your understanding of the
term, suggest measures for ensuring probity in government.
• What are the basic principles of public life? Illustrate any three of these with suitable
examples.

9. Previous Years UPSC Mains Questions: Case Studies


1. You are the head of the Human Resources department of an organisation. One day one of
the workers died on duty. His family was demanding compensation. However, the company
denied compensation because it was revealed in investigation that he was drunk at the
time of the accident. The workers of the company went to strike demanding compensation
for the family of the deceased. The Chairman of the management board has asked for your
recommendation.
What recommendation would you provide the management?
Discuss the merits and demerits of each of the recommendations

2. In a modern democratic polity, there is a concept of political executive and permanent


executive elected people’s representatives forms the political executive and bureaucracy
forms the permanent executive. Ministers frame policy decisions and bureaucrats execute
these. In the initial decades after independence, relationship between the permanent
executives and the political executives were characterized by mutual understanding,

DELHI JAIPUR PUNE HYDERABAD AHMEDABAD LUCKNOW CHANDIGARH


20 www.visionias.in # 8468022022 ©Vision IAS
Google it:- https://upscpdf.com
https://t.me/UPSC_PDF Download From > https://upscpdf.com https://t.me/UPSC_PDF

respect, and cooperation, without encroaching upon each other’s domain. However, in the Student Notes:
subsequent decades the situation has changed. There are instances of the political
executive insisting upon the permanent executives to follow its agenda. Respect for an
appreciation of an upright bureaucrats has declined. There is an increasing tendency
among the political executive to get involved in routine administrative matters such as
transfers, posting etc. Under this scenario, there is a definitive trend towards ‘politicization
of bureaucracy’. The raising materialism and acquisitiveness in social life has also adversely
impacted upon the ethical values of both the permanent executive and the political
executive.
What are the consequences of this ‘politicization of bureaucracy? Discuss

10. Previous Years Vision IAS GS Mains Questions


1. Probity is an essential condition of good governance. Explain.
Approach:
• Define Probity.
• Provide arguments to bring out the significance of probity for good governance.
• Conclude answer.
Answer:
Probity is the quality of adhering to strong moral principles such as honesty and
integrity as well as uprightness, good character and decency. It is the act of following
the highest principles and ideals rather than merely avoiding corrupt or dishonest
conduct. It balances service to the community against the self-interest of individuals.
Probity and Good Governance
Governance is the act and manner of managing public office. A working paper of the
National Commission to Review the Working of the Constitution noted that probity in
governance is an essential and vital requirement for an efficient and effective system of
governance and for socio-economic development. While the constitution and laws
provide legal framework for governance, probity is part of the ethical framework that
determine the nature of governance and the relationship between the government and
the governed.
Probity is essential for good governance as it:
• Enhances the credibility of the state apparatus: Since probity is concerned with
procedures, processes and systems rather than outcomes, it ensures procedural
integrity of the institutions. Thus, efficient and effective system of governance leads
to socio-economic development.
• Ensures institutional integrity: Probity maintains ethicality and legality of
institutions regardless of the individuals manning them. It involves adopting an
ethical and transparent approach, allowing the process to withstand scrutiny.
• Minimizes individual discretions: Strict adherence to the highest moral standards
allows institutions as well as the individuals to deal with everyone impartially.
• Curbs corrupt behaviour: Aspects of probity such as - accountability and
transparency - prevents abuse of public resources or position in public life for
private gain.
• Upholds public confidence: Probity in governance preserves public confidence in
the government and governmental processes. It eases the way authority is
exercised by the public officials.
Creation of a strong moral framework in governance is essential for good governance.
However, procedural probity should not be at the cost of humane aspect of the

DELHI JAIPUR PUNE HYDERABAD AHMEDABAD LUCKNOW CHANDIGARH


21 www.visionias.in # 8468022022 ©Vision IAS
Google it:- https://upscpdf.com
https://t.me/UPSC_PDF Download From > https://upscpdf.com https://t.me/UPSC_PDF

administration. To ensure probity in public life, a robust culture of integrity and moral Student Notes:
standards needs to be cultivated.

2. What do you understand by the terms transparency and accountability in


administration? Bring out the relationship between the two.
Approach:
• Define and explain both transparency and accountability.
• Explain the relationship between transparency and accountability.
• Conclude appropriately.
Answer:
Transparency in administration means the availability of information in the public
domain as permissible under the law. It aims to bring about clarity in the functioning of
government institutions. It brings out clear communication on the process of decision-
making and the reasons taken without distorting facts i.e. sanctity of procedure should
be reflected in decision-making. For example, e-auction of any public procurement
project where all the details and parameters are made available in public domain.
Accountability in administration means the answerability of the decision makers for
their decisions and actions to the designated superior authority. It means they must
submit themselves to the scrutiny necessary to ensure this. Further, it also includes the
enforcement of the sanctions, if actions or justifications are judged unsatisfactory. For
e.g. the government is accountable to the Parliament for its various actions and
parliamentarians are accountable to their constituencies for their performance.
Relationship between transparency and accountability: 
Transparency and
accountability share a complementary relationship. On one hand, accountability
defines the type of transparency needed and on the other hand, the quality of
information decides the type of accountability possible. It can be seen as follows:
• Effect of transparency on accountability: If there is transparency in the working of
an authority, it will aid people to uphold accountability. For e.g the disclosure of
information under Right to Information Act, aids the citizens to uphold the
accountability of public officials.
• Effect of accountability on transparency: If there is an accountability mechanism in
place, it will force the public authority to maintain transparency in the decision-
making. For e.g. the audit and scrutiny conducted by the Comptroller and Auditor
General of India makes way for the departments and authorities to maintain
transparency in their record keeping.
To ensure a positive influence, transparency and accountability systems should be
designed to support each other. Both the concepts reinforce each other and contribute
towards good governance, rule of law and improving the citizen’s trust in the
government.

3. Examine the role of media in promoting probity in governance.


Approach:
• Define the term ‘probity in governance’.
• Assess the role of the media in ensuring probity in governance.
• Mention a few issues with the media and suggest a way forward.

DELHI JAIPUR PUNE HYDERABAD AHMEDABAD LUCKNOW CHANDIGARH


22 www.visionias.in # 8468022022 ©Vision IAS
Google it:- https://upscpdf.com
https://t.me/UPSC_PDF Download From > https://upscpdf.com https://t.me/UPSC_PDF

Answer: Student Notes:


Probity is the quality of having strong moral principles and strictly following them. It
includes principles such as - honesty, integrity, fairness, uprightness, transparency and
incorruptibility. Probity in Governance is concerned with the propriety and character of
various organs of the government as to whether these uphold the procedural
uprightness, regardless of the individuals manning these institutions. An important
requisite for ensuring probity in governance is absence of corruption. Media plays
following role in ensuring it:
• Transparency: Media is essential to safeguarding transparency of democratic
processes. This is often called its ‘watchdog’ role. For example, media presence at
voting and counting centres is critical to fair and transparent elections.
• Corruption: Investigative reporting by media or reporting of instances of
corruption may prompt public bodies to launch formal investigation into allegations
of corruption. For example, investigations into alleged mishandling of funds during
CWG games.
• Institutional Integrity: Media disseminate the findings of public anti-corruption
bodies, thus reinforcing the legitimacy of these bodies as well as insulating them
from undue pressures from vested interests. For example, bringing politicians to
account for violations of code of conduct during elections.
• Accountability: Sometimes, mere inquiries by journalists, in the absence of a story’s
publication or of conclusive proof of wrongdoing, can lead to a tangible response
from authorities eager to protect their reputations and those of the institutions
they represent.
• Participative democracy: It brings public concerns and voices into the open and
works to strengthen government responsiveness to various social problems
including corruption.
• Feedback mechanism: It can also expose flaws in policy, laws or regulation that
foster a climate ripe for corruption, thus creating pressure for reform.
• However, the role of media in ensuring probity is affected by several challenges
such as monopolization of media by powerful interests, lack of protection of
journalists who investigate corruption, inadequate press freedom, less media
accountability whereby profits take precedence over integrity, misuse of self-
regulation provisions etc.
For the media to effectively help in ensuring probity in governance, it needs to be
independent from governmental, political or economic control. As well as, media
should be pluralistic with the existence of the greatest possible number of newspapers,
periodicals and broadcasting stations. Such a media will reflect the widest possible
range of opinions within a community and play its role of the fourth pillar of democracy
in true form.

4. Explain the importance of probity in governance. What measures have been


undertaken for ensuring probity in governance in India?
Approach:
• Briefly explain the term probity.
• Discuss the importance of probity in governance.
• Mention the measures taken by government to ensure probity in governance.
Answer:
Probity is the quality of having strong moral principles and strictly following them. It
includes principles such as - honesty, integrity, uprightness, transparency and

DELHI JAIPUR PUNE HYDERABAD AHMEDABAD LUCKNOW CHANDIGARH


23 www.visionias.in # 8468022022 ©Vision IAS
Google it:- https://upscpdf.com
https://t.me/UPSC_PDF Download From > https://upscpdf.com https://t.me/UPSC_PDF

incorruptibility. Probity is confirmed integrity. It is usually regarded as being Student Notes:


incorruptible.
Probity in Governance is concerned with the propriety and character of various organs
of the government as to whether these uphold the procedural uprightness, regardless
of the individuals manning these institutions. It involves adopting an ethical and
transparent approach, allowing the process to withstand scrutiny. The objective of
having probity is that it ensures procedural integrity. It is concerned with processes,
procedures, and systems. It includes principles such as honesty, integrity, uprightness,
transparency and incorruptibility.
According to NCRCW, absence of corruption is a prerequisite for ensuring probity in
governance. The other requirements are effective laws, rules and regulations to govern
every aspect of public life and fair implementation of those laws. Probity in governance
ensures that the system remains transparent, accountable, responsive and open to
scrutiny.
Importance of probity in governance:
• It helps in building legitimacy of the state and trust in its institutions. It instills a
belief that actions of the state are for the welfare of the beneficiaries.
• It leads to prudent and ethical outcomes and building of trust over time.
• It leads to avoidance of sub-optimal outcomes, corruption and poor perception.
• It provides for an objective and independent view on the fairness of the process.
• It helps in checking abuse and misuse of power by various organs of the
government.
• It is vital for the efficient and effective system of governance and for socio-
economic development.
Measures taken by Government of India to ensure probity in governance are:
• Prevention of Corruption Act, 1988: The Act defines a public servant and crimes
which come within corruption/bribery.
o Amendments in 2018 included the act of offering gifts/bribes to the public
servants as punishable offence.
o It also amended the act to protect honest bureaucrats through bringing more
clarity on criminal misconduct and by making the prior sanction by competent
authority necessary before starting inquiry investigation against them.
• Right to Information Act, 2005: It helps the cause of probity in governance through
citizen activism. The 2nd ARC considers RTI as the master key to good governance.
• Whistle-blowers Protection Act, 2014: The act provides a mechanism for receiving
and inquiring into public interests disclosure against act of corruption, misuse of
power/discretion or criminal offences by public servants.
• Prohibition of Benami Property Transaction Act, 1988 and amendments in 2016:
Now, Benami transactions have been clearly defined, and confiscation of such
properties without payment of compensation has been established as law.
• Central Vigilance Commission: It advices government in matters related to
maintenance of integrity in administration.
• Lokpal and Lokayukta Act, 2013: It envisages an institution of ombudsman which
enquires into allegations of corruption against certain public functionaries and
matters related to them.
There must be a legislation to check misfeasance in public offices, confiscation of
illegally acquired assets by public servants, and a code of ethics for government. A
strong criminal judicial system is also required to ensure probity. As a society, one must
evolve to a level where probity becomes a way of life and honesty becomes a routine

DELHI JAIPUR PUNE HYDERABAD AHMEDABAD LUCKNOW CHANDIGARH


24 www.visionias.in # 8468022022 ©Vision IAS
Google it:- https://upscpdf.com
https://t.me/UPSC_PDF Download From > https://upscpdf.com https://t.me/UPSC_PDF

expectation. The value of integrity, impartiality and merit must become the guiding Student Notes:
principle to be followed by bureaucrats.

5. Transparency in government organisations is an essential pre-condition for good


governance. Elucidate.
Approach:
• Briefly define transparency and its role in a democracy.
• State the tools of transparency in India and discuss how they lead to objective
decision-making, increased efficiency etc.
• Mention the current issues regarding transparency in India.
• Conclude suitably.
Answer:
Transparency refers to the availability of information to the general public and clarity
about functioning of governmental institutions. Governance is the manner in which
decisions are taken and implemented, particularly by the lawful authority. ‘Good’
signifies that governance system adheres to certain well-accepted principles which
make the system of governance ethical, just and effective. Some of these principles
include adherence to rule of law, participation, accountability, transparency, equity,
efficiency & effectiveness and inclusivity.
Transparency as a tool for good-governance
• In a functional democracy, the government is obliged to keep its citizens informed.
This helps citizens to hold their public officials accountable, which is key to just and
good governance.
• As per the 2nd Administrative Reforms Commission, transparency in a democracy
allows bi-directional information flow, which allows citizens to participate in
governance.
• It empowers the citizens to demand and get information about public policies and
programmes, keep themselves informed about their rights as well as duties. Thus,
it promotes efficiency, effectiveness and responsiveness in public administration.
Tools for transparency
• In India, the Right to Information Act, 2005 (RTI Act), Lokpal and Lokayuktas Act,
2013, Citizen’s Charter, social audit, digitization of records, etc. are tools of
transparency and accountability.
• When the process of decision-making is transparent, decisions are taken
objectively in a fair manner. This ensures equality before law, just allocation of
resources, cutting down on discretionary and arbitrary decisions etc.
• It not only leads to increased transparency but also reduction in corruption. For
example, e-auctions of coal blocks by Coal India in 2018-19 made the auction
process more transparent and fetched 44% higher prices.
Despite these measures, India has not fared well on the global parameters related to
transparency in governance.
• According to Global Corruption Index, 2018 developed by Transparency
International corruption is all pervasive Indian administration. The index ranks
India 78 out of 180 countries.
• Despite its positive impacts, there are several issues in the implementation of the
RTI Act such as low level of awareness about the act especially among women,

DELHI JAIPUR PUNE HYDERABAD AHMEDABAD LUCKNOW CHANDIGARH


25 www.visionias.in # 8468022022 ©Vision IAS
Google it:- https://upscpdf.com
https://t.me/UPSC_PDF Download From > https://upscpdf.com https://t.me/UPSC_PDF

rural population, SC/ST/OBCs etc., procedural constraints in filing RTI applications, Student Notes:
poor quality of information, increased attacks on RTI activists etc.
• Institutions such as the judiciary, political parties etc. remain outside the purview
of the RTI Act, hence, it is difficult to monitor their functioning.
• Apathetic government officials who are secretive in their workings. This
undermines both transparency and accountability in public functioning.
Measures should be taken to improve transparency in government organizations,
accountability of public servants and involve more citizens in the governance process.
The government should proactively and voluntarily make information public as per
Section 4 of RTI act to promote transparency in governance. It will ensure just and good
governance.

6. Bring out the significance of probity in public life. What are the requisites for ensuring
probity in governance? Pointing out the key concerns in India in this context, suggest
certain remedial measures.
Approach:
• Define probity, with special focus on your own understanding.
• List what constitutes probity in governance and the relevant issues plaguing the
country in this context.
• Finally, provide a multipronged approach to remedy the issues.
Answer:
Probity can be defined as strict adherence to your moral principles based on
undeviating honesty and a quality of being incorruptible.
According to the Nolan Principles of Public Life, maintaining a high standard of probity
in public life means following the principles of selflessness, integrity, objectivity,
accountability, openness, honesty and leadership.
In the modern world, various areas of public life are plagued by corruption, dishonesty
and the lack of integrity.
Without these principles discipline cannot be maintained, which according to Gunnar
Myrdal is necessary for progress to be made. Corruption has an adverse impact on all
aspects of our lives, to tackle which probity in public life is a must.
To ensure probity in governance, absence of corruption is a must along with effective
institutions to oversee and enforce laws, rules and regulations which govern various
aspects of public life.
In this context the key concerns in India with regards to ensuring probity in governance
are:
• Despite laws being present to combat corruption, there is lack of will to enforce
them in letter and spirit.
• Deficiency in rule of law as there is asymmetry of power and information. This
opens channels for exploitation of dependents on state by those who form part of
the executive arm of the state.
• Acceptance of corruption as a necessary evil in large sections of the society.
• A culture of non-transparency in government which discourages attainment of
outcomes from outlays.
In order to remedy this situation, two-pronged approach:
From legal-administrative perspective, the following steps can be taken:

DELHI JAIPUR PUNE HYDERABAD AHMEDABAD LUCKNOW CHANDIGARH


26 www.visionias.in # 8468022022 ©Vision IAS
Google it:- https://upscpdf.com
• Enacting a comprehensive law to check mala fide actions of public servants. Student Notes:
• Providing a law for confiscation of illegally acquired assets of public servants.
• Strengthening the Whistle-blower Act to protect whistle-blowers.
• Implementation of RTI Act in letter and spirit.
• Strengthening the Institution of Lokpal
• Strengthening the criminal judicial system i.e. reforms in all aspects which include
police/investigating agency, the prosecuting agency, the advocates, witnesses and
finally the judiciary.
• Undertaking measures like social audit to promote social accountability
From a general perspective, we as a society need to impart value education at home
and school. As the children learn a lot from the elders, we need to be better role
models and citizens to ensure that the younger generation grow up to adopt probity in
public life.

7. While discretion is necessary for effective discharge of duties, it is also a major factor
responsible for corruption in administration at all levels. Comment. In this context,
suggest some ways in which smooth execution of responsibilities can be ascertained
while minimizing corruption.
Approach:
• First explain the meaning of discretion in administration.
• Then explain why discretion is required for effective discharge of duties.
• Explain how discretion could led to corruption.
• Finally explain measures to minimise corruption while ensuring smooth execution
of responsibility.
Answer:
Discretion means the power to decide or act according to one’s judgment. Indian law
grants some discretionary powers to administrative authorities. Such exercise is not to
be arbitrary, vague and fanciful, but legal and regular.
Discretionary powers bestowed on the administrative authorities are of a vast range.
Their power serve the purpose of simple ministerial tasks like maintenance of birth and
death register as well as those which seriously affect the rights of an individual, e.g.
acquisition of property, regulation of trade, industry or business, inquiry, seizer,
confiscation and destruction of property, detention of a person on subjective
satisfaction of an executive authority and many more. The list of their functions is
exhaustive in nature.
The problem of administrative discretion is complex. There has been a constant conflict
between the claims of the administration to an absolute discretion and the claims of
subjects to a reasonable exercise of it. Now a question is raised that how it can be
control.
It can be control with two types, first judicial and other one is non- judicial. There are so
many ways to control it.
Judicial Control
Judiciary must concentrate on two points. Firstly, it should direct the legislative that
they do not confer wide and unlimited discretion to executive. And other is that every
discretionary act must come under the power of judicial review. Judiciary can thus play
a good role to control abuse of discretionary powers.
Supreme Court of India in its various judgements has held that:

DELHI JAIPUR PUNE HYDERABAD AHMEDABAD LUCKNOW CHANDIGARH


27 www.visionias.in # 8468022022 ©Vision IAS
• The administrative discretion should be used according to rules of reason and Student Notes:
justice and not according to private opinion, according to law and not humor.
• It is not to be arbitrary, vague and fanciful but legal and regular.
• It must be exercised within limit to which an honest man competent to the
discharge of his office ought to confine himself.
Non-judicial control
We must incorporate such rules which will be mandatory for the authority who will
exercise discretion to adhere to. If legislative is fails to maintain such norms in that
statute to control discretion, those norms must be incorporated by administration with
the help of delegated legislation. Mechanisms like code of conduct and code of ethics
ensure that self-discipline is observed while exercising discretion. Various institutions
like CVC, CBI, Lokpal etc. ensure that abuse of discretion is caught and punished which
acts as deterrent in the future.
Hence, through the above mechanism, abuse of discretion can be curbed while at the
same time flexibility is maintained while taking administrative decisions.

8. Open government is an even more comprehensive concept than transparency and


freedom of information. Elaborate.
Approach:
First, explain the concept of transparency. Then discuss the essential elements of open
government. The last part of the answer should focus on how open government is even
more comprehensive than transparency.
Answer:
Transparency is an essential feature of open government. Transparency means that
information about the activities of public bodies is created and is available to the public,
with limited exceptions, in a timely manner, in open data formats and without
restrictions on reuse. Transparency mechanisms must include the disclosure of
information in response to requests from the public and proactive publication by public
bodies. Key information about private bodies should be available either directly or via
public bodies.
But open government has two other essential elements. They are participation and
accountability. Participation means that the public can engage directly in the
consideration of policy options and decision making, and can contribute ideas and
evidence that lead to policies, laws, and decisions which best serve society and broad
democratic interests. Governments should actively seek to mobilize citizens to engage
in public debate. Mechanisms should exist which permit the public to participate at
their own initiative and to trigger policy debates on matters of concern.
An accountable government is one which makes itself answerable to the public,
upholding standards of behavior and integrity, and both explaining and taking
responsibility for its decisions and actions. Accountability requires that rules,
regulations and mechanisms be in place governing the exercise of public power and the
spending of public funds. Specific and detailed measures are required to reduce
corruption risks, to identify and prevent potential conflicts of interest, and to guard
against illicit enrichment. There should be protections for those who expose
wrongdoing.
Thus in an open government openness through transparency becomes a means to
greater civic participation in an enabled environment, where there is effective free flow
of information both ways to see through the working of government; and to verify

DELHI JAIPUR PUNE HYDERABAD AHMEDABAD LUCKNOW CHANDIGARH


28 www.visionias.in # 8468022022 ©Vision IAS
whether or not public servants are meeting their obligation to expectations of citizen; Student Notes:
All four component of accountability i.e. answerability, sanction, redress and system
improvement ensure responsiveness of government and finally civic engagements in
the process of governance, in the form of people’s planning, participatory budgeting,
corruption watch by citizen audit etc. makes it (open government) the new democratic
culture of an open society toward which every liberal democracy is moving.

9. Ethics is the first line of defense against corruption while law enforcement is remedial
and reactive. Examine the statement with suitable examples.
Approach:
• Describe the importance of ethics and laws in preventing corruption.
• Elaborate the statement while taking a stand on it and justify with examples.
Answer:
Ethics refer to well-founded standards of right and wrong that prescribe what humans
ought to do, usually in terms of rights, obligations, fairness or specific virtues. Laws are
usually based on an ethical framework and aim to bring social order while controlling
the immoral or unethical behaviour of individuals in the society.
Laws act as an external set of standards and rules imposed on an individual by the
government. An individual disobeying the law is liable to face the prescribed penalties
and punishments and thus deter non-compliance. Laws have societal sanctions and
approvals and thus have an important role in smooth functioning of the society. The
fact that laws are grossly violated often by the wealthy and the powerful suggest a
fundamental lack of ethics in the society.
Ethics act as a moral compass in guiding an individual even when the law is silent on
the action which ought to be taken. A typical example is the role of an administrator
where discretion has to be exercised. Discretion provides ample opportunities for
corruption and a person with weak ethics may easily fall prey to money-making even at
the cost of society’s wellbeing. Therefore, ethics is the first line of defense, even if there
is no law on the subject.
As another example, a government may not frame laws to dictate whether a company
should make its products more environmentally safe or easier to recycle, but doing so
may be the ethical thing to do. But a responsible and ethically upright organisation will
consider these measures even in the absence of laws mandating the same.
Hence, it is ethics that act as a strong defence mechanism for all societies to prevent
corruption whereas laws are reactive and only a remedy. A permanent solution for
wiping corruption from its roots shall only come with the internalization and not from
the external forces. External mechanisms remain effective till they tend to have some
reward or punishments while inherited value system shall accompany lifelong.

11. Previous Years Vision IAS GS Mains Questions: Case


Studies
1. Sandhya recently completed her B. Com. and was extremely excited to be hired for her
dream job working for a Public Sector Bank. During her initial days, she began to
notice that funds from grants were being mismanaged and misallocated. Some of her
co-workers were also using bank property materials, including cars, for personal
business.
However, Sandhya was most shocked by the hiring practices she witnessed at the
office. Applicants to the jobs were supposed to take exams that were invigilated by
bank employees. Sandhya began to notice that the invigilators were allowing

DELHI JAIPUR PUNE HYDERABAD AHMEDABAD LUCKNOW CHANDIGARH


29 www.visionias.in # 8468022022 ©Vision IAS
applicants to cheat on the tests because the applicants had already been chosen for Student Notes:
the job. Many of these pre-chosen applicants were friends of current employees.
Sandhya reported what she witnessed to Mahesh, the Branch Assistant Manager,
who was second-in-command to the Branch Manager. Mahesh told her, “You heard
nothing, you saw nothing, and you say nothing.” Sandhya was absolutely shocked;
not only by the corruption, but that it was deliberately being swept under the rug.
Sandhya I was in a dilemma. She really needed the job to pay off loans, and she loved
the actual content of the work she was doing. She was also concerned that it would
look bad to leave her first job out in less than a year, as well as tarnish future chances
to work in a government organization. On the other hand, she felt extremely
uncomfortable in her work environment due to the culture of corruption.
(a) What are the options available to Sandhya?
(b) Evaluate each of these options and choose the option you would adopt, giving
reasons.
Answer:
Sandhya is caught in a typical dilemma where she doesn’t want to leave the job but
cannot see such a work environment embroiled in corruption too.One should point out
how overlooking this matter would mean a lot of loss to public exchequer and
inefficiency in the working of the bank. As Sandhya was unsuccessful in her attempt to
bring out the matter to Mahesh, her approaching the Branch Manager to report the
issue seems the most appropriate option in such a situation.
Some of the options available to Sandhya are:
a) She can ignore the matter and proceed in her usual way as this does not impact her
directly and any action taken would adversely impact her personally
b) She should report the matter to Branch manager and bring to light how she was
told to keep the matter under wraps.
c) She can resign from the job as taking either of the first steps would not leave her
satisfied and in a comfortable situation
d) Become a whistleblower. She can bring the matter to the notice of someone in the
vigilance department overseeing the functioning of the bank as it is public money
after all that is being squandered through such activities.
Option a) As per the rights/duty, fairness and common based approach it is Sandhya’s
duty to uphold the integrity of the banking system. The money that is being
squandered away is public money and therefore any such activity is hurting the public
interest in the long term. Additionally, the faulty hiring practices would greatly impact
the working of the bank and bring down its efficiency and effectiveness further hurting
the interests of the public and the society at large. It would be unethical to ignore the
matter for the dire consequences it might have on the organization in future.
Even though ignoring the matter would save Sandhya from the trouble of getting into
any controversy and help save her job but as discussed above it would not be healthy in
the long run for either the organization or even Sandhya.
Option b) because of the following reasons: Virtue approach: integrity and honor are
two virtues that one should embody and help promote. One must ask myself what is
the highest state of character one can aspire to? One obviously recognizes the virtue of
honesty and merit. Likewise, one should push himself/herself to think about the values
one should live up to, those being integrity and promotion of merit in this particular
case. Therefore, the most appropriate option seems to reporting the matter to the
branch manager.
Option c) Resigning from the job does not seem a feasible option considering the fact
that Sandhya has to pay off her loan and any such action would not show well on her

DELHI JAIPUR PUNE HYDERABAD AHMEDABAD LUCKNOW CHANDIGARH


30 www.visionias.in # 8468022022 ©Vision IAS
CV and have an adverse impact on her career. It would save Sandhya the trouble of Student Notes:
getting into any kind of problem with regards to the conscience but it would put her in
a great trouble as she would be out of job and would put her and her family’s livelihood
at stake. Morover, such an action would mean running away from the situation and
punishing oneself for the fault of others.
Option d) Even option D seems like a possible option but whistleblowing is a double
edged sword and should only be explored once all the possible options have been
exploited. It may be possible that if the matter is brought to branch manager’s
attention, he/she may take a strict action against those embroiled in such activities.
Proceeding to blow whistle on the matter without exploring this option may be taking
an action in haste and puts the reputation of the bank in a danger.

2. Big firms often undertake sub-contracting to complete large infrastructure projects in


a timely manner. You recently joined one such firm as a manager responsible for
awarding these contracts. Looking at the past records, you find that all contracts for
the past few years have been awarded to a particular firm, X. Your superior has asked
you to award an upcoming contract to the same firm. Although, not binding,
company procedure maintains that sub-contracting work should be offered after
competitive bids. This is to ensure that the firm most suitable for the project in terms
of operations and finance gets the contract. When you discuss this with your superior,
he insists that hiring of the firm X has been done as per legal norms of the company
and no official rule has been violated. You decide to contact the owner of the firm X.
After doing so, you realize that he is the nephew of your superior, who is also a
shareholder in the company.
(a) State the ethical issues involved in the case.
(b) Does this form of transaction between two private parties constitute a conflict of
interest? Justify.
(c) Evaluate the possible ways of awarding contracts in such a situation with their
merits and demerits. Also state which method would be more suitable in each.
Approach:
• State some of the ethical issues in the case.
• Evaluate if a situation of interest is generated due to the relationship of your
superior with the owner of firm X and the knowledge of the superior being a
shareholder in the company.
• Suggest ways of awarding contracts in the situation, along with their merits and
demerits.
• State your eventual course of action in this scenario.
Answer:
Basic Facts: In this situation, the company where I work is repeatedly sub-contracting
work to another private firm, X, without holding competitive bids, which should be
carried out according to company procedures. The case highlights the issue of conflict
of interest, as the owner of firm X is the nephew of my superior and is also a
shareholder of the firm.
Stakeholders:
• Myself, as I am the manager of the company, with the responsibility of awarding
contracts.
• My superior who is a shareholder of firm X as well as an employee of the company
where I work.
• Owner of firm X as the decisions made by our company affect his business interests.

DELHI JAIPUR PUNE HYDERABAD AHMEDABAD LUCKNOW CHANDIGARH


31 www.visionias.in # 8468022022 ©Vision IAS
• Owners of other firms who vie for company contracts as they are not getting a fair Student Notes:
chance in the bidding process.
• Shareholders of the company who can suffer due to poor decisions.
• Consumers who will be affected by substandard infrastructure.
(a) Some of the ethical issues involved in this case are:
• Nepotism and compromise on professional integrity: Subcontracts are being
granted on the basis of personal relations, without competitive bids. Due to this,
there is no guarantee that the firm most suitable in terms of efficiency and
financial aspects gets the contracts.
• Lack of objectivity in decision-making: The interests of my superior and his
nephew are guiding the decision-making process of the company. There is
inherent subjective bias in this scenario.
• Legal vs Ethical: It may be legally correct but not ethical as no official rule of the
company has been violated, but, it goes against established company
procedure.
(b) Transaction between two private parties generates a conflict of interest since the
owner of firm X is the nephew of my superior, who has repeatedly directed that the
contract be awarded to firm X. Since my superior is also a shareholder in firm X, he is
invariably taking business decisions on the basis of personal relations and perceived
financial opportunities. He would not be affected much due to poor performance of the
company as his family is benefitting, but other shareholders and employees will be
affected. Thus, personal interests are directly in conflict with interest of shareholders.
Also, they are in conflict with the interest of consumers who expect quality
infrastructure.
(c) Possible ways of awarding contracts in such a situation are:
• Granting the contract to firm X as suggested by my superior.
• Merits: I will remain in good terms with him, may get quick promotion and
increment.
• Demerits: I will be unethical in my work conduct as a manager. I will also obstruct
competitive bidding and play a role in eschewing probable offers that could save
the company more money and get the work done more efficiently. I would be
compromising my integrity and commitment to interest of shareholders, colleagues
and consumers.
• Refuting my superior and revealing his interest in firm X, while simultaneously
organizing the competitive bid.
• Merits: I will follow company procedure and my professional integrity will be
upheld.
• Demerits: I will go directly against the order of my superior, who can hinder my
chances of promotion in the company. I will also violate the established work
hierarchy in the company.
• The most suitable action will be to talk to my superior about the generated conflict
of interest and argue in favour of holding competitive bids. However, if he still
refuses to acknowledge the gravity of the situation, I will approach the company
head/board regarding the issue. This will ensure that other managers and
subordinates are not in a similar position in the future. Meanwhile, I will also
accept competitive bid offers and award the contract to the firm most suitable for
the job. It will ensure my professional integrity and uphold the interest of all
stakeholders.

DELHI JAIPUR PUNE HYDERABAD AHMEDABAD LUCKNOW CHANDIGARH


32 www.visionias.in # 8468022022 ©Vision IAS
3. You are representing India in an international bidding for oil exploration in a country. Student Notes:
Other, richer countries are also bidding for the project. You are sure that your bid of
exploration is better as well as cheaper than that of others, and that you will
definitely win the bid. A day before the auction, you come to know that other
countries are employing every means, including bribing the authorities for being
successful. Some of the officials of the home country have also contacted you and
made some demands in exchange for assurance of India winning the bid. You are
aware of the criticality of this bid in terms of domestic economic and strategic
implications.
Based on above information, answer the following questions.
(a) Specify the ethical dilemma(s) that you face in this situation.
(b) Do ethical concerns really matter in international transactions or are they
secondary to domestic interests?
(c) What will be your course of action in the above situation? Justify with merits and
demerits.
Approach:
• Identify the ethical dilemmas that you face.
• Highlight the importance of ethical concerns in international transactions vis a vis
domestic interests.
• Then mention the course of action that you would follow. Justify it by taking into
account the merits and demerits of the decision.
Answer:
a) The situation in this case study presents the following ethical dilemma:
The dilemma is whether to pay the bribe vs being upright and avoid the temptation to
pay.
The former action may help India win the bid, but it will be an unethical course of
action and may spoil the image of the country in the long run when the truth comes
out in the public. It will also have adverse consequences for India’s relations with the
countries involved in bidding process. Moreover, this is inimical to a healthy
competition, level playing field and innovation. This action will also set a wrong
example to others. The action is not only unethical but also illegal as regards to Indian
laws. It may not bring in me a sense of accomplishment or content.
The latter course of action may lead to a possible defeat in the bidding process, setback
for my career and economic and strategic implications for the country. But it is the right
path to follow.
b) Advocates of national interest in international relations argue that national interests
are paramount. As Henry Kissinger has said- “there are no permanent ally or
permanent enemies, only interests are permanent”. These arguments are based on the
fact that the government of a county primarily works on the behalf of its citizens and
thus it is bound to uphold their interests. The political party in power has to face
general elections regularly and its report card of performance evaluates not only
domestic but international actions as well. Hence, national interest alone should be
paramount in international relations.
However, these arguments suffer from certain inconsistencies. If the national interest
alone is taken into account then wrong doings like colonization, regime change, arm
twisting of weaker nations etc. will be justified. Further, there exists a wide inequality
internationally and if strong nations justify their actions solely based on the national
interests than this gap will further widen. Moreover, the global commons will not
survive and sustainable development will remain a distant dream.

DELHI JAIPUR PUNE HYDERABAD AHMEDABAD LUCKNOW CHANDIGARH


33 www.visionias.in # 8468022022 ©Vision IAS
Thus, fairness, justice, apathy, sustainable development of whole world, equity etc. are Student Notes:
ethical principles which are as important as national interests and really matter in
international relations.
c) In such situation, I will pursue the following course of action:
a. Verifying, at my own level, the correctness of information related to bribery
activities in the auctioning process.
b. Informing my seniors, seeking their advice as they might have faced similar
situation earlier.
c. Approach the head of the authorities handling the whole process of auction
and inform them about inconsistencies which has come to notice and demand
a fair and transparent bidding process.
d. If grievances are not addressed at that level then, after taking my seniors into
confidence, we can approach other higher authorities of home country like
judiciary for intervention.
I will also demand that the officials involved in bribe-seeking activities must be
punished which will deter such malpractices in future. Those nations who are involved
in unfair practices must also be punished (by way of fines, blacklisting them or
cancelling their bids).
Justification of such course of action

In international transactions, sometimes unetical actions like bribery are also sought to
be justified in the name of national interest. However, on a closer look, such actions are
clearly against the national interests in reality. The revelation about involvement in
bribery would jeopardize the international relations of future generations of our
country.
Further, a single contract cannot be so important to our country that we sacrifice our
moral standards and higher values maintained for so long. Moreover, corruption can
never be the true foundation of prosperity. The gains obtained from it corrupt the
whole society.
By following the stated course of action, I will display faith in the governance of home
country, uphold our moral values and there will be higher chances of fair bidding
process. As India’s bid is better and cheaper, it will ensure India’s success. It will
generate the good will for our nation among the people of that country; set a right
example against corruption in international transaction. Overall, it will be a right step
towards the righteousness which we expect in international relations.

Copyright © by Vision IAS


All rights are reserved. No part of this document may be reproduced, stored in a retrieval system or
transmitted in any form or by any means, electronic, mechanical, photocopying, recording or
otherwise, without prior permission of Vision IAS.

DELHI JAIPUR PUNE HYDERABAD AHMEDABAD LUCKNOW CHANDIGARH


34 www.visionias.in # 8468022022 ©Vision IAS

You might also like